0% found this document useful (0 votes)
390 views

Aaos 2011

Uploaded by

Yusufa Ardy
Copyright
© © All Rights Reserved
We take content rights seriously. If you suspect this is your content, claim it here.
Available Formats
Download as PDF, TXT or read online on Scribd
0% found this document useful (0 votes)
390 views

Aaos 2011

Uploaded by

Yusufa Ardy
Copyright
© © All Rights Reserved
We take content rights seriously. If you suspect this is your content, claim it here.
Available Formats
Download as PDF, TXT or read online on Scribd
You are on page 1/ 334

Online 2011 Orthopaedic Self-Assessment Examination by Dr.

Dhahirortho

1
1
Online 2011 Orthopaedic Self-Assessment Examination by Dr.Dhahirortho

2
2
Online 2011 Orthopaedic Self-Assessment Examination by Dr.Dhahirortho
Question 1Which of the following methods of treating a vertically oriented (eg, Pauwels
III) femoral neck fracture is mechanically optimal?
1- Two parallel fully threaded screws
2- Three parallel partially threaded screws
3- Three parallel fully threaded screws
4- Four parallel partially threaded screws
5- Sliding hip screw and side plate

DISCUSSION: Vertical fractures have a higher rate of displacement and nonunion because
of shearing forces across the fracture. Biomechanical and clinical studies indicate that for
the vertically oriented fracture of the femoral neck, the most stable fixation construct is a
sliding hip screw and side plate. Antirotation screws may be used as well. Nonsurgical
management carries a high risk of early displacement because of shear forces. Three
screws are loaded as a cantilever and have less resistance to displacement compared
with a fixed-angle device with a side plate. Fully threaded screws will not allow any
compression and have the same drawbacks as partially threaded screws. The addition of
a fourth screw has not been shown to be of benefit. The Preferred Response # 1 is 5.

Question 2 Figures 2a and 2b are the MR arthrograms of a 19-year-old college baseball


pitcher who injured his throwing elbow during a game 5 days ago when he felt a pop.
Immediately after the throw he reported significant discomfort with pitching and noted
that he could not achieve his normal velocity or accuracy in location with his
subsequent pitches. On further questioning, he admits to increasing medial elbow pain
over the last few seasons with pitching. Examination reveals medial elbow swelling and
somewhat diffuse tenderness to palpation medially. Valgus stress at 30 degrees of
flexion and resisted wrist flexion produced discomfort. He notes some tingling in his
fourth and fifth fingers but Tinel's test posterior to the medial epicondyle is
unremarkable. Radiographs of the elbow show no fracture. Because the patient wishes
to return to competitive throwing, what is the next step in management?
1- Ulnar nerve transposition
2- Ulnar collateral ligament reconstruction
3- Long arm cast for a medial epicondyle fracture
4- Open reduction and internal fixation of the medial epicondyle
5- Elbow arthroscopy and excision of a posteromedial olecranon osteophyte

3
3
Online 2011 Orthopaedic Self-Assessment Examination by Dr.Dhahirortho

DISCUSSION: This high level throwing athlete has a full-thickness injury to the ulnar
collateral ligament and is most likely to be able to return to competitive throwing with an
ulnar collateral ligament reconstruction. There is no radiographic evidence of a medial
epicondyle fracture. The clinical presentation and lack of a posteromedial olecranon
osteophyte makes valgus extension overload unlikely, and therefore, makes arthroscopic
osteophyte excision a suboptimal choice. Whereas ulnar nerve pathology can coexist with
an ulnar collateral ligament injury, isolated ulnar nerve transposition without addressing
the ligament injury is not warranted in this patient. Initial nonsurgical management with
activity modification and physical therapy is appropriate for partial-thickness injury to the
ulnar collateral ligament in a non-throwing athlete, and in athletes whose sporting
activity places them at low risk. The Preferred Response to Question # 2 is 2.

Question 3 Figures 3a and 3b are the radiographs of an active 59-year-old woman who
has had a 5-year history of right great toe pain. Nonsurgical management, consisting of
shoe modifications, an orthotic with a Morton's extension, injections, and medications,
has failed to provide relief. The range of motion is 30 degrees of dorsiflexion to 10
degrees of plantar flexion with pain at each end point, but not through the midrange of
motion. What is the most appropriate management?

1- Cheilectomy
2- Keller resection arthroplasty
3- Silastic implant arthroplasty with titanium grommets
4- Arthrodesis of the first metatarsophalangeal joint
5- Total metatarsophalangeal joint arthroplasty

4
4
Online 2011 Orthopaedic Self-Assessment Examination by Dr.Dhahirortho

DISCUSSION: The patient exhibits significant arthrosis of the first metatarsophalangeal


joint but does not have pain at the midrange arc of motion and is, therefore, a good
candidate for a cheilectomy. Easley and associates and Coughlin and associates have
shown excellent mid-term and long-term results with a cheilectomy, especially in patients
without preoperative pain at the midrange arc of motion. An arthrodesis of the first
metatarsophalangeal joint is an acceptable choice for achieving pain relief but will
somewhat limit her shoe wear choice. A Keller resection arthroplasty is only
recommended for older and low-demand patients. Silastic implant or total
metatarsophalangeal joint arthroplasty has not been shown to be durable in active
patients.
The Preferred Response to Question # 3 is 1.

Question 4 If an orthopaedic surgeon receives royalties from a company for his or her
participation in the design and development of a product, and uses that same product
for the care of his or her patients, what is the orthopaedic surgeon's obligation?
1- Obligated to disclose only the fact that he or she was involved in the design and
development
2- Obligated to disclose only the company relationship if there is a state law requiring it
3- Obligated to disclose his or her full relationship with the company, including the fact
that he or she receives royalties
4- No obligation to disclose this private matter to the patient
5- Avoid this situation because it should not exist since he or she cannot use such a
product

DISCUSSION: The AAOS has a specific code of ethics and professionalism that addresses
this issue: "When an orthopaedic surgeon receives anything of value, including royalties,
from a manufacturer, the orthopaedic surgeon must disclose this fact to the patient." It is
derived from a broader document developed by the American Medical Association, and is
applicable to all physicians. At present, this is an ethical issue receiving greater federal
scrutiny. This issue has had a greater effect on the public's perception of the integrity of
the orthopaedic profession.

The Preferred Response to Question # 4 is 3.

5
5
Online 2011 Orthopaedic Self-Assessment Examination by Dr.Dhahirortho
Question 6 Range of motion after total knee arthroplasty is best described by which of
the following statements?
1- The principle predictive factor of the postoperative range of motion is the
preoperative range of motion.
2- Intraoperative range of motion is not correlated with the postoperative range of
motion.
3- Postoperative stiffness rarely impairs function.
4- Excess distal femoral resection with a thick tibial polyethylene is associated with a
flexion contracture.
5- Inadequate distal femoral resection and a tight posterior capsule are associated with
loss of flexion.

DISCUSSION: The cause of postoperative stiffness after total knee arthroplasty is


multifactorial. Whereas there is no universally accepted definition of stiffness, 90 degrees
of flexion is needed to perform tasks such as stair climbing and getting out of a chair and
nearly full extension is necessary for efficient gait. Predictors of postoperative range of
motion include preoperative and intraoperative range of motion. Capsule release,
ligament release, osteophyte removal, and properly sized components are often
necessary to optimize range of motion. Excess distal femoral resection with a thick
polyethylene will cause a tight flexion gap and loss of flexion. Inadequate distal femoral
resection with retained osteophytes and a tight posterior capsule will lead to a flexion
contracture. The Preferred Response to Question # 6 is 1.

Question 7 What is the proper location for a trochanteric nail starting point?
1- At thetip of the greater trochanter
2- Just medial to the tip of the trochanter
3- Just lateral to the tip of the trochanter
4- Dependent on the position and obliquity of the fracture
5- Dependent on the relative position of the trochanter to the axis of the femoral shaft

DISCUSSION: Contrary to popular belief, the tip of the greater trochanter is not
necessarily the proper starting location for insertion of a trochanteric femoral nail. The
relative position of the tip of the trochanter and the long axis of the femoral canal varies
substantially between patients. Also, the proximal lateral bend varies substantially
between different nails. Therefore, the relative position of the trochanter to the axis of
6
6
Online 2011 Orthopaedic Self-Assessment Examination by Dr.Dhahirortho
the femoral shaft and the particular geometry of the selected nail must be considered.
The Preferred Response to Question # 7 is 5.

Question 8Which of the following statements best describes the 2-year outcome of
workers' compensation patients who received surgical treatment for lumbar
intervertebral disk herniation compared with those who received nonsurgical
management?
1- Decreased pain
2- Decreased disability
3- Improved return to work
4- No improvement with surgical treatment
5- No added benefit associated with surgical treatment

DISCUSSION: Workers' compensation patients demonstrated no added benefit associated


with surgical treatment at 2-year follow-up, in contrast with the non-workers'
compensation patients who had significantly greater improvement with surgery. Both
groups of patients were shown to improve substantially during the study. However, the
workers' compensation group demonstrated similar improvement with surgical and
nonsurgical treatment at 2-year follow-up. Additionally, surgical treatment did not
improve work or disability outcomes at 2 years in the workers' compensation group. The
Preferred Response to Question # 8 is 5.

Question 9Figures 9a through 9c are the MRI scans of a 65-year-old woman on dialysis
who has thoracic back pain, malaise, and an elevated erythrocyte sedimentation rate
(ESR). The clinical history and imaging findings are most consistent with

1- lymphoma.
2- renal osteodystrophy.
3- osteomyelitis and diskitis.
4- metastatic breast carcinoma.
5- osteoporotic compression fracture.

7
7
Online 2011 Orthopaedic Self-Assessment Examination by Dr.Dhahirortho
DISCUSSION: The sagittal MRI scans are pathognomonic for diskitis and osteomyelitis
with fluid signal and destructive changes in the disk on T2 (Figure 9a), low signal with
blurring of the disk margins on T1 (Figure 9b), and on the T1 gadolinium image (Figure 9c)
vertebral body enhancement on either side of the affected disk with dark signals within
the disk corresponding to the bright fluid signal from the T2 image. Metastatic carcinoma
tends to affect the vertebral body with relative disk sparing, and lymphoma can affect the
vertebral body but often has soft tissue extending within the spinal canal. Osteoporotic
fractures are contained with the vertebral body. Renal osteodystrophy can result in a
diskitis picture with disk destruction but one would not expect an elevated ESR or
malaise, and this is much rarer than diskitis in dialysis patients. The Pr Resp# 9 is 3.

Question 10 A 6-month-old child has the deformity seen in Figure 10. There are no
other known associated problems. What is the etiology of this condition?

1- Exposure to teratogens
2- Multifactorily inherited
3- A defect of the apical ectodermal ridge
- A defect in fibroblast growth factor
5- Inherited as an autosomal dominant

DISCUSSION: The radiograph demonstrates a type IV radial clubhand (radial dysplasia)


with complete absence of the radius. This is a pre-axial deficiency usually with complete
absence of the thumb. The condition is thought to be caused by an injury to the
formation of the apical ectodermal ridge early in embryology. It is not an inherited
condition unless it is associated with other syndromic problems. It is not known to be
associated with specific teratogens. Fibroblast growth factor is involved in angiogenesis,
wound healing, and embryonic development, but is not known to be associated with
radial clubhand. The Preferred Response to Question # 10 is 3.

Question 11 A 52-year-old man who dislocated his dominant shoulder has it reduced in
the emergency department and he is placed in a sling. At his 5-day follow-up
evaluation, he reports that this is his first shoulder dislocation and that the pain is
mostly gone but he notes difficulty using his arm overhead and away from his body.
Examination reveals minimal pain with passive range of motion, a positive
8
8
Online 2011 Orthopaedic Self-Assessment Examination by Dr.Dhahirortho
apprehension and relocation test, and 3/5 strength with the empty can test and
external rotation at the side compared with 5/5 with those tests on the contralateral
side. Cutaneous sensation over the lateral aspect of the shoulder is intact. Radiographs
show the glenohumeral joint is reduced with no fractures or degenerative changes.
What is the next step in management?

1- CT of the shoulder
2- MRI of the shoulder
3- Application of a sling for 6 weeks
4- Surgery for diagnostic shoulder arthroscopy
5- Physical therapy to work on range of motion and strengthening

DISCUSSION: Obtaining an MRI scan to evaluate for a rotator cuff tear is a reasonable
next step. The patient sustained a first-time shoulder dislocation, and given his age and
clinical presentation, it is likely that he injured the rotator cuff. Large, full-thickness
rotator cuff tears following dislocation in young individuals warrants early surgical
intervention. Delay of surgical repair for large, full-thickness tears may lead to irreversible
changes, including atrophy and retraction of the tendon. As a result, clinical outcomes
may be compromised. CT will demonstrate bony changes, but it is not as effective as MRI
for soft-tissue pathology. While in the short term a sling for comfort might be helpful, 6
weeks of immobilization is unnecessary because recurrent instability is rarely an issue.
Physical therapy can be beneficial but could potentially delay identification of an acute
rotator cuff tear. In the event the MRI does not reveal a large, full-thickness rotator cuff
tear, physical therapy would be an appropriate next step. There is no indication for
urgent shoulder arthroscopy. The Preferred Response to Question # 11 is 2.

Question 12A 22-year-old woman sustains the injury seen in Figure 12 as a result of a
motor vehicle crash. What factor is most closely associated with development of
osteonecrosis?

1- Reduction quality
2- Time from injury to surgery
3- Presence or absence of a capsulotomy
4- Type of implant used for internal fixation
5- Location of the fracture within the femoral neck
9
9
Online 2011 Orthopaedic Self-Assessment Examination by Dr.Dhahirortho

DISCUSSION: A displaced femoral neck fracture in a young patient is considered a surgical


urgency and prompt anatomic reduction and internal fixation is recommended. There are
a few studies that have specifically looked at the rate of osteonecrosis in this patient
population. A review of femoral neck fractures in patients ages 15 to 50 years revealed
that the incidence of osteonecrosis in displaced fractures was 27% compared with 14% in
nondisplaced fractures. The quality of the reduction also influenced the rate of
osteonecrosis. Time to reduction, type of implant, presence or absence of capsulotomy,
and location of the fracture are not associated with osteonecrosis risk.
The Preferred Response to Question # 12 is 1.

Question 13Figure 13 shows the radiograph of a 2-year-old boy who underwent closed
reduction of a forearm fracture 1 week ago. The parents noted the arm appeared
crooked after a trip to the playground but the child did not report pain. The opposite
forearm appears normal. He has been recently diagnosed with which of the following
conditions?

1- Neurofibromatosis
2- Osteopetrosis
3- Ulnar dysplasia
4- Congenital radial-ulnar synostosis
5- Vitamin D resistant rickets

DISCUSSION: This is a case of a `pathologic fracture` in the forearm of a very young child.
All of the presentation details reveal a deformity in the forearm with little outward signs
of trauma, and the forearm bones do not appear normal on the radiograph. The
medullary canal disappears in the distal third of both bones and there is an associated
bowing deformity. Whereas much less common than congenital pseudarthrosis of the
tibia, congenital pseudarthrosis of the forearm has been well documented and is
associated with neurofibromatosis in about 50% of cases. This is a typical case
presentation. All of the other conditions are not associated with this forearm deformity.
The Preferred Response to Question # 13 is 1.
10
10
Online 2011 Orthopaedic Self-Assessment Examination by Dr.Dhahirortho
Question 14 Which of the following postoperative rehabilitation techniques causes
minimal rotator cuff muscle activation?
1- Active forward flexion
2- Passive forward flexion
3- Active-assisted forward flexion
4- Overhead pulley-assisted passive forward flexion
5- Isometric strengthening

DISCUSSION: Electromyography (EMG) studies have shown that the rotator cuff is least
active with passive range of motion and hence this is allowed early in most postoperative
rotator cuff rehabilitation protocols. Active forward flexion, active-assisted motion, and
isometric strengthening all cause activation of the rotator cuff muscles (as measured by
EMG) and therefore should be introduced later in rehabilitation when the repair can
withstand these forces. Whereas some authors have felt that pulley-assisted range of
motion exercises are safe, EMG analysis has demonstrated that these exercises do cause
activation of the rotator cuff musculature and probably should be avoided early in the
rehabilitation protocol. The Preferred Response to Question # 14 is 2.

Question 15A minimally invasive plate osteosynthesis is seen in Figure 15. The resultant
fracture healing can best be attributed to a fixation construct that was

1- stiff and stable.


2- flexible and stable.
3- facilitating direct osteonal healing.
4- inhibitory to endochondral ossification.
5- stimulatory to intramembranous ossification.

DISCUSSION: Locked plating constructs with long-working


lengths provide flexible but stable constructs that promote
(not inhibit) endochondral ossification. Because of the longer
working length they are not stiff, and these fractures do not heal with intramembranous
ossification which occurs in bones like the calvarium. Direct osteonal healing is usually
seen with constructs where absolute stability is achieved through interfragmentary
compression, unlike in this case. The Preferred Response # 15 is 2.

11
11
Online 2011 Orthopaedic Self-Assessment Examination by Dr.Dhahirortho
Question 16Figure 16 shows the CT scan of a 44-year-old woman who sustained a direct
blow to the head after falling while snowboarding. She is unable to move her upper or
lower extremities and has diffuse numbness. Examination reveals normal strength in
the deltoid muscles bilaterally but 0/5 strength in the remaining upper or lower
extremity muscle groups. She is absent light touch, pinprick, and proprioceptive
function in her upper and lower extremities. She has decreased rectal tone and intact
perirectal sensation with an intact bulbocavernosus reflex. The patient's spinal cord
injury is best classified as

1- complete, ASIA A.
2- complete, ASIA B.
3- incomplete, ASIA B.
4- incomplete, ASIA C.
5- incomplete, ASIA D.

DISCUSSION: The patient has sustained a C5 tear-drop


fracture with spinal cord injury. Examination demonstrated
sacral sparing with perirectal sensation; therefore, this is an incomplete injury. Given her
absent motor function, she would be classified as an ASIA (American Spinal Injury
Association) B. ASIA A represents a complete spinal cord injury with no motor or sensory
sparing below the level of injury. ASIA B is an incomplete spinal cord injury with sacral
sparing (preservation of sacral sensation). ASIA C and ASIA D injuries reveal some motor
function in the lower extremities. ASIA C injuries result in grade 3/5 or less strength,
while ASIA D injuries show greater than 3/5 strength.
The Preferred Response to Question # 16 is 3.

17A 20-year-old collegiate pitcher has had a 5-month history of shoulder pain while
throwing, decreased velocity, and difficulty with location of his pitches despite multiple
attempts at rest. He reports no traumatic event. Examination with his throwing arm
abducted at 90 degrees reveals external rotation to 110 degrees and internal rotation
to 70 degrees when compared with his nonthrowing shoulder which has external
rotation to 95 degrees and internal rotation to 85 degrees. He has a positive O'Brien's
sign, positive modified Jobe's relocation test, full rotator cuff strength, no obvious
muscular atrophy, and no scapular winging. Radiographs of the affected shoulder show
no abnormalities. What is the next most appropriate step in management?
12
12
Online 2011 Orthopaedic Self-Assessment Examination by Dr.Dhahirortho
1- Dynamic ultrasound examination of the rotator cuff
2- Electrodiagnostic testing of the throwing shoulder
3- MR arthrogram of the throwing shoulder
4- Referral to a physical therapist to concentrate on range of motion
5- Laboratory studies to evaluate C-reactive protein and erythrocyte sedimentation rate

DISCUSSION: The study of choice to evaluate the superior labrum is an MR arthrogram.


The patient has symptoms suspicious for superior labral pathology (ie, positive O'Brien's
test, Jobe's relocation test, pain with throwing, loss of velocity and location). Whereas he
does have increased external rotation and decreased internal rotation of his throwing
arm compared with his non-throwing arm, the total arc of motion is 180 degrees and this
is considered a normal adaptive change in the overhead throwing athlete; therefore,
ultrasound is not considered appropriate management. There are no signs of weakness or
rotator cuff pathology to suggest suprascapular nerve compression or a full-thickness
rotator cuff tear; therefore, electrodiagnostic testing or physical therapy are
inappropriate. There are also no signs or symptoms suggesting infection or rheumatologic
issues; therefore, laboratory studies are unnecessary. If the MR arthrogram shows a
labral tear, the initial management would include posterior capsular stretching and
rotator cuff strengthening. The Preferred Response to Question # 17 is 3.

Question 18A patient has an elbow injury that includes a coronoid fracture, medial
collateral ligament injury, and a radial head fracture. When is excision of the radial
head without replacement indicated as definitive treatment for the radial head injury?
1- When the elbow is stable after fixation of the coronoid and medial collateral
ligament
2- When the elbow is unstable after fixation of the coronoid and medial collateral
ligament
3- When the fracture is comminuted and therefore stable internal fixation is
unobtainable
4- When there is preexisting radiocapitellar arthritis
5- Excision is generally not indicated in this clinical scenario

DISCUSSION: The injury likely represents a terrible triad injury. Restoration of the lateral
column is required to restore valgus stability. A repaired or replaced radial head is also
thought to be protective of the coronoid fracture repair. Therefore, excision is not
13
13
Online 2011 Orthopaedic Self-Assessment Examination by Dr.Dhahirortho
indicated. Either radial head arthroplasty or open reduction and internal fixation would
be indicated. The Preferred Response to Question # 18 is 5.

Question 19 An orthopaedic surgeon makes an incision on a right knee and realizes that
the patient was supposed to have a left total knee arthroplasty. The surgeon should do
which of the following?
1- Leave the wound open and talk to the family immediately.
2- Close the wound, abort the surgery, and talk to the patient and family when the
patient is awake.
3- Close the wound, complete the left knee arthroplasty, and talk to the family after the
surgery is complete.
4- Complete the surgery and talk directly to the patient the following day on rounds.
5- Discuss the problem in the office the next week in a calm reassuring manner.

DISCUSSION: The AAOS recommendation is to complete the correct surgery, repair the
incorrect surgery to as close to normal as possible, and then discuss it openly with the
family after the surgery is complete. Prompt informing is necessary. Aborting the surgery
then results in the patient requiring a second anesthesia and surgical time needlessly. The
Preferred Response to Question # 19 is 3.

Question 20Figure 20 is the radiograph of a patient who underwent total hip


arthroplasty 15 years ago and now reports poorly defined pain in the hip. Which of the
following represents the most appropriate management?

1- Revision total hip arthroplasty


2- Evaluation by a physiatrist
3- Physical therapy for strengthening and gait training
4- Nonsteroidal anti-inflammatory drugs (NSAIDs) and
observation
5- CBC, C-reactive protein, erythrocyte sedimentation rate, and
possibly hip aspiration

DISCUSSION: The patient has a fractured femoral component and requires revision.
Poorly defined hip pain in the absence of mechanical failure may respond to physical
therapy or NSAIDs. In addition, new onset pain after total joint arthroplasty may
14
14
Online 2011 Orthopaedic Self-Assessment Examination by Dr.Dhahirortho
represent infection and workup is appropriate (CBC, C-reactive protein, erythrocyte
sedimentation rate, and possibly hip aspiration). Poorly defined hip region pain may also
represent lumbar spine pathology and when infection and mechanical failure have been
ruled out, evaluation by a physiatrist may be appropriate. The Preferred Respon # 20 is 1.

Question 21 A tall 14-year-old girl with joint laxity has progressive right thoracic
scoliosis and is thought to be a surgical candidate. Her neurologic examination is
normal. Presurgical screening should include which of the following studies?
1- CT of the cervical spine
2- MRI of the entire spine
3- Whole body bone scan
4- Echocardiography of the heart
5- Preoperative somatosensory testing

DISCUSSION: The patient is likely to have Marfan syndrome and cardiac complications are
more likely to occur. Therefore, an echocardiogram would be indicated to assess for
valvular insufficiency or other cardiac abnormalities. MRI of the spine is indicated in
rapidly progressive curves, right-sided curves, those patients with an abnormal neurologic
examination, and younger patients. CT of the spine would be indicated in patients with
torticollis or if evaluating a congenital spine disorder. Preoperative somatosensory testing
is occasionally performed in patients with neurologic conditions in which responses may
not be normal and a baseline is needed. A bone scan is not indicated. Pre Res# 21 is 4.

Question 22Figure 22 is the radiograph of a 55-year-old woman with progressive


deformity of the great toe after undergoing bunion corrective surgery 2 years ago.
What is the most likely factor associated with this deformity?

1- Excessive lateral soft-tissue release


2- Excessive medial eminence resection
3- Inadequate correction of the intermetatarsal 1-2 angle
4- Hypermobility of the first tarsometatarsal joint
5- Failure of pin fixation in the first metatarsal

DISCUSSION: The cause of hallux varus is often multifactorial


with overcorrection occurring often from a combination of excessive lateral release,
15
15
Online 2011 Orthopaedic Self-Assessment Examination by Dr.Dhahirortho
overcorrection of the intramedullary 1-2 angle, excessive medial release, excessive laxity
of the soft tissues, and malalignment of the metatarsal osteotomy. In this patient, there
does not appear to be an excessive medial eminence resection and of the answers
available, the excessive soft-tissue release is the best response. The pins in the metatarsal
have no bearing on the result. Hallux varus is not associated with hypermobility of the
tarsometatarsal joint.
The Preferred Response to Question # 22 is 1.

Question 23 A subtrochanteric femur fracture in which the lesser trochanter is intact is


associated with what deformity?
1- Adduction and extension of the proximal fragment
2- Adduction and flexion of the proximal fragment
3- Abduction and extension of the proximal fragment
4- Abduction and flexion of the proximal fragment
5- Predominantly internal rotation of the proximal fragment

DISCUSSION: The most commonly seen deformity in subtrochanteric femur fractures is


abduction and flexion of the proximal fragment. Subtrochanteric fractures can pose
challenges in reduction because of the muscle attachments proximal and distal to the
fragment. The gluteus medius and gluteus minimus attach to the greater trochanter and
abduct the proximal fragment. The iliopsoas attaches to the lesser trochanter, flexing and
externally rotating the proximal fragment. The short external rotators (piriformis,
superior and inferior gamellus) and the obturator internus also cause external rotation of
the proximal fragment. The Preferred Response to Question # 23 is 4.

Question 24A 20-year-old unrestrained driver sustained a midshaft femur fracture in a


high-speed motor vehicle accident. The femoral neck was evaluated with a CT scan with
2-mm cuts through the hip; no fracture was identified. What additional studies (if any)
should be performed to minimize the risk of having an undiagnosed femoral neck
fracture?
1- Postoperative MRI scan
2- Postoperative bone scan
3- Preoperative AP pelvic radiograph
4- No additional imaging studies are needed
5- Intraoperative fluoroscopic images of the femoral neck
16
16
Online 2011 Orthopaedic Self-Assessment Examination by Dr.Dhahirortho
DISCUSSION: Nondisplaced femoral neck fractures may occur concurrently with high-
energy injuries of the femur. Preferably, these are identified prior to or during surgery so
that the fracture can be stabilized to prevent displacement and minimize the risk of
osteonecrosis. However, the diagnosis of these injuries can be difficult. Tornetta and
associates reported on standardized protocol that involved preoperative radiographs and
CT scans with fine cuts through the femoral head. This protocol improved the detection
of femoral neck fractures compared with situations with no set protocol. Of the 16
fractures detected, 13 were identified preoperatively. Of the three fractures that were
missed by the screening, one was iatrogenic, one of these was detected at the time of
surgery with intraoperative internal/external views of the femoral neck, and one had a
late displacement. The overall rate of nondisplaced femoral neck fractures in this study
was 7.5%, of which 91% were treated at the time of initial surgery having been identified
on preoperative and/or intraoperative radiographs. Care must be taken not to neglect
careful scrutiny of the femoral neck at the time of surgery even if preoperative imaging
studies do not detect a fracture. No one method has been shown to have a 100% success
rate. Postoperative bone scans and MRI scans are not routinely used. The Pr Res# 24 is 5.

Question 25 Performing reconstruction of the anterior cruciate ligament by drilling the


femoral tunnel via an anteromedial portal, in contrast to transtibial drilling, affords
what theoretical benefit?
1- Longer femoral tunnel
2- More anatomic graft placement
3- A more vertically oriented graft
4- Diminished risk of posterior tunnel wall violation ("blowout")
5- Diminished risk to lateral femoral articular cartilage and subchondral bone
posteriorly

DISCUSSION: Recent trends in anterior cruciate ligament reconstruction include an


emphasis on anatomic rather than isometric reconstruction of the ligament. According to
some studies, this more effectively restores knee kinematics and with this, rotatory
stability. Transtibial drilling affords limited access to the lateral intercondylar wall and has
been associated with vertical graft orientation. The anteromedial portal, in contrast,
allows independent femoral tunnel drilling and more anatomic positioning of the graft. A
more anatomically positioned tunnel established via an anteromedial portal may afford
increased tunnel and graft obliquity. This has been suggested to resolve rotatory
17
17
Online 2011 Orthopaedic Self-Assessment Examination by Dr.Dhahirortho
instability. Knee flexion angle during the course of reaming has been studied to assess
favorable and negative tunnel characteristics and hazards to regional anatomic
structures. When compared with transtibial drilling, the anteromedial portal is associated
with shorter femoral tunnels, posterior tunnel wall integrity compromise, and increased
risk to lateral femoral articular cartilage and subchondral bone posteriorly. Pr Re# 25 is 2.

Question 26Figures 26a and 26b are the radiograph and MRI scan of an otherwise
healthy 10-year-old girl with increasing pain in the arm. A biopsy specimen is seen in
Figure 26c. Treatment now should consist of

1- amputation.
2- radiation therapy and chemotherapy.
3- limb-sparing surgery with reconstruction.
4- chemotherapy and limb-sparing surgery with reconstruction.
5- radiation therapy, chemotherapy, and limb-salvage surgery and reconstruction.

DISCUSSION: The girl has osteosarcoma of the upper humerus. The biopsy specimen
shows malignant osteoid formation. Osteosarcoma and Ewing's sarcoma are the two
most common primary malignant bone tumors in children and account for approximately
6% of all childhood malignancies. Histopathology distinguishes between the two because
clinical and radiographic imaging can sometimes be similar. Treatment methods have
seen significant advancements, particularly in regard to chemotherapy and limb-sparing
surgery. These advancements have led to an increased survival rate. With many long-
term survivors, it is important to evaluate long-term patient outcomes following
treatment, including function and health-related quality of life. Osteosarcomas are not
radiosensitive tumors and would, therefore, not be treated with radiation therapy.
Although limb-sparing surgery is feasible and preferred over amputation in most
instances, it is best used when combined with chemotherapy. The Preferred Res# 26 is 4.

18
18
Online 2011 Orthopaedic Self-Assessment Examination by Dr.Dhahirortho
Question 27A total knee arthroplasty is recommended to a mentally competent 68-
year-old woman who has disabling knee pain caused by degenerative arthritis. Her son
has researched the procedure on the internet and prefers the Acme Female Knee for his
mother. You have designed the Axis Woman's Knee, for which you receive royalties,
and use it exclusively. Which of the following ethical principles takes precedence in
guiding her treatment?
1- Informed consent
2- Patient autonomy
3- Fiduciary responsibility
4- Physician paternalism
5- Justice

DISCUSSION: Informed consent incorporates a number of ethical principles relevant to


this case. The fundamentals of medical ethics include nonmaleficence, beneficence,
autonomy, and justice. The patient is competent and capable of exercising her autonomy
in choosing the Acme Female Knee. She also depends on her physician's paternalism and
knowledge in looking out for her best interests, which in his opinion, may be use of the
Axis Woman's Knee. The physician has a fiduciary responsibility to inform the patient that
he has a financial interest in the implant system he recommends. A thorough informed
consent will respect the patient's autonomy, explain the rationale for the physician's
recommendation, and notify the patient that there may be a perceived conflict of
interest. The ethical principle of justice has no relevance in this case. The P Re# 27 is 1.

Question 28Figure 28 is the lateral radiograph of a patient who sustained an intra-


articular fracture of the calcaneus. The structure (*) depicted by the arrows most likely
represents which osseous component of the calcaneus?
1- Middle facet
2- Sustentaculum tali
3- Extruded lateral wall
4- Medial portion of the posterior facet
5- Lateral portion of the posterior facet

DISCUSSION: Fractures of the calcaneus occur as a result of shear and compression


forces. Foot position at the time of impact, the force of the impact, and bone quality all
dictate the degree of comminution and fracture line orientation. Two primary fracture
19
19
Online 2011 Orthopaedic Self-Assessment Examination by Dr.Dhahirortho
lines are consistently observed, one of which divides the calcaneus into medial and lateral
portions. An essential feature of this fracture line is that it creates a fragment
(sustentaculum tali) that remains attached to the talus by the interosseous ligament. This
medial portion (constant fragment) of the posterior facet retains its normal anatomic
position beneath the posterior talus. Its corresponding lateral component (labeled with
an * in the figure), however, can be found displaced inferiorly within the body of the
calcaneus. It is often rotated 90 degrees (as depicted in Figure 28) in relation to the
remainder of the subtalar joint. This gives the appearance of what has been described as
the "double-density" sign. The middle facet is more anterior and less commonly
displaced. The lateral wall is nonarticular. The Preferred Response to Question # 28 is 5.

Question 29Figure 29 is the radiograph of a 3-month-old boy who has pain and swelling
in his left thigh after his mother fell with him in her arms. There are no other injuries
and a skeletal survey is otherwise normal. Treatment should consist of
1- flexible nail fixation.
2- external fixation.
3- a Pavlik harness.
4- growing rod insertion.
5- a hip spica cast.

DISCUSSION: The child has a minimally displaced femur fracture that could be stabilized
by any of the methods mentioned; however, a Pavlik harness is the best choice. Flexible
nails, growing rods, and external fixation would be marked overtreatment. A hip spica
cast could be used in a child this age, but a Pavlik harness treats this fracture easily with
no anesthesia and is easier for the parents to manage. The Preferred Response to
Question # 29 is 3.

Question 30During arthroscopic evaluation of a partial-thickness articular-sided


supraspinatus tendon tear, the medial-lateral width of the tear is noted to be 6 mm.
This represents what percent partial-thickness tear?
1- 10% 2- 25% 3- 50% 4- 75% 5- 90%

DISCUSSION: Partial-thickness rotator cuff tears can be bursal-sided, articular-sided,


and/or intratendinous. Management of partial-thickness tears requires an understanding
of the native anatomy. Dugas and associates and Ruotolo and associates studied
20
20
Online 2011 Orthopaedic Self-Assessment Examination by Dr.Dhahirortho
cadaveric specimens and reported the medial-lateral width of the supraspinatus tendon
averages 12.1 to 12.7 mm. Therefore, a 6- to 7-mm tear represents approximately a 50%
tear of the supraspinatus tendon. Most authors agree that tears representing greater
than 50% of the medial-lateral width of the supraspinatus tendon should be repaired. The
Preferred Response to Question # 30 is 3.

Question 31 Fragment excision and triceps reattachment is ideally indicated for which
of the following situations?
1- A 30-year-old woman with a closed comminuted fracture involving more than 50% of
the joint surface
2- A 30-year-old woman with an open transverse olecranon fracture that is proximal to
the trochlear notch
3- A 55-year-old woman with an oblique olecranon fracture through the coronoid
process
4- A 75-year-old woman with an oblique fracture through the coronoid process
5- An 85-year-old man with a comminuted fracture involving less than 50% of the
proximal joint surface

DISCUSSION: Fragment excision and reattachment of the triceps tendon may be indicated
in a select group of elderly patients with osteoporotic bone in whom the olecranon
fracture fragments involve less than 50% of the joint surface, and are too small or too
comminuted for successful internal fixation. The triceps tendon is reattached with
nonabsorbable sutures that are passed through the drill holes in the proximal ulna. In a
physiologically young patient, internal fixation should be performed. Plate fixation would
be appropriate for comminuted fractures, whereas tension band wiring could be used for
a simple transverse fracture. Oblique fractures passing through the coronoid process are
best treated by plate fixation. The Preferred Response to Question # 31 is 5.

Question 32Figures 32a through 32e show the radiographs and T2-weighted MRI scans
of a 51-year-old man who has had bilateral leg pain for the past 6 months. The pain
radiates down both legs, is worsened by ambulation, and relieved with rest and
bending forward. Management consisting of physical therapy and medications has
failed to provide any improvement in symptoms. Examination reveals normal strength,
sensation, and pulses in the lower extremities. What treatment is most likely to provide
the greatest pain relief and improved function?
21
21
Online 2011 Orthopaedic Self-Assessment Examination by Dr.Dhahirortho

1- Lumbar epidural injections


2- Oral anti-epileptic medications
3- Posterior lumbar arthrodesis L4-5
4- Posterior lumbar decompression L4-5
5- Posterior lumbar interbody arthrodesis L4-5

DISCUSSION: The patient has lumbar spinal stenosis and neurogenic claudication.
Posterior decompression (laminectomy and bilateral lateral recess decompression) at the
L4-5 level is the treatment for this condition when nonsurgical management has failed to
provide relief. Weinstein and associates demonstrated statistically significant
improvements among surgically treated patients compared to nonsurgical treatment in a
prospective (randomized and observational) study. Use of oral anti-epileptic medications
(gabapentin) has been reported in small case series to be effective but has not been
validated. Whereas epidural injections can provide some therapeutic improvement, they
have not demonstrated a proven clinical effect. Lumbar arthrodesis, whether
posterolateral or interbody, without a decompression is not recommended because
neither will address the patient's symptoms. Additionally, the adjunct of an arthrodesis is
not indicated in this patient and would not be beneficial compared with decompression
alone given the lack of significant scoliosis, spondylolisthesis, or instability at the L4-5
segment.

The Preferred Response to Question # 32 is 4.

22
22
Online 2011 Orthopaedic Self-Assessment Examination by Dr.Dhahirortho
Question 33Radiographs of a 7-year-old child show mid-diaphyseal fractures of the
radius and ulna. Closed reduction with sedation in the emergency department is
performed. Postreduction radiographs demonstrate 18 degrees angulation, 30%
translation, and what appears to be 20 degrees of rotational malalignment. Based on
these findings, what is the next most appropriate step in management?

1- Another attempt at closed reduction in the operating room


2- Open reduction with plating of the radius only
3- Open reduction with plating of the ulna only
4- Open reduction with plating of both the radius and ulna
5- Close monitoring with follow-up radiographs in 1 week

DISCUSSION: In children younger than 8 years of age, acceptable reduction parameters


for fractures of the forearm are less than 20 degrees of angulation, 100% translation, and
less than 45 degrees of malrotation. Weekly monitoring for loss of reduction and
unstable fractures requiring further intervention is needed. When acceptable alignment
can be maintained, good outcomes can be expected in this age group. In patients older
than 10 years, angulation of less than 10 degrees, full translation, and malrotation of 30
degrees can be accepted. When surgical treatment is indicated, plating of one or both
bones is acceptable. However, in this patient, the reduction is acceptable so a repeat
closed reduction attempt and surgical treatment are not needed. P R# 33 is 5.

Question 34A 73-year-old man has had severe knee pain and swelling for the past 5
days. There has been no fever. Radiographs are normal in appearance. A knee
aspiration specimen is seen in Figure 34 under polarized light. What is the next best
course of action?

1- Obtain an MRI scan


2- Obtain serum uric acid level
3- Await culture and sensitivity results to start antibiotics
4- Inject a cortisone product, followed by management with oral nonsteroidal anti-
inflammatory drugs (NSAIDs)
5- Start colchicine 0.6 mg three times a day until resolution of symptoms

23
23
Online 2011 Orthopaedic Self-Assessment Examination by Dr.Dhahirortho
DISCUSSION: The aspiration specimen shows crystals that are weakly birefringent and
rhomboid in shape, reflecting the strong likelihood of calcium pyrophosphate crystal
disease. Given the severe pain, a cortisone injection following aspiration will be most
useful. Gout is associated with uric acid crystals that are birefringent yet needle shaped.
Serum uric acids are often normal in an acute gout attack. Colchicine is useful in treating
gout. The treatment of acute pyrophosphate crystalline disorder includes NSAIDs or intra-
articular glucocorticoids. The diagnosis of gout is usually confirmed by the presence of
strongly birefringent needle-shaped monosodium urate crystals in aspirates of the
involved joint. Because monosodium urate crystals often can be found in the first
metatarsophalangeal joint and in knees not acutely involved with gout, arthrocentesis of
these joints between attacks is a useful diagnostic tool. The serum level of uric acid has a
limited role in the diagnosis of gout because it can be normal or low at the time of an
acute attack. The mainstay of treatment during an acute gouty attack is the
administration of colchicine or NSAIDs. The Preferred Response to Question # 34 is 4.

Question 35Figures 35a and 35b are the radiographs of a 59-year-old man who is seen
for follow-up after undergoing primary total knee arthroplasty 7 years ago. He has been
doing well but recently began to report some swelling and knee pain. Laboratory
studies reveal an erythrocyte sedimentation rate of 19 mm/h (normal up to 20 mm/h)
and C-reactive protein of 0.9. What is the most appropriate management?

1- Follow-up as necessary
2- Revision of both components
3- Observation with serial radiographs
4- Debridement and bone grafting with polyethylene exchange
5- Implant resection and antibiotic-impregnated cement spacer

24
24
Online 2011 Orthopaedic Self-Assessment Examination by Dr.Dhahirortho
DISCUSSION: Polyethylene wear debris from total knee arthroplasty can produce
significant periprosthetic osteolysis resulting in bony destruction, undermining of
component fixation, and eventual loosening of the components. The management of
periprosthetic osteolysis is somewhat controversial and depends on the extent of the
lysis, the implant design, the method of polyethylene manufacture and sterilization, and
the patient's symptoms. The onset of pain in this patient is concerning for loosening in
the setting of extensive lysis. The surgeon should be prepared to revise the components
at the time of surgery. There is no evidence of infection in the laboratory results so
resection with placement of a spacer would not be necessary. Observation is the
mainstay of management initially in patients with osteolysis, but when they become
symptomatic or the lytic area is large enough to risk component loosening, intervention
should be strongly considered. Patients with known lysis should be monitored and not
followed as necessary. Significant bone loss can occur in the setting of asymptomatic
components and before components become loose and painful, bone grafting with
polyethylene exchange may be an option. The Preferred Response to Question # 35 is 2.

Question 36Figures 36a through 36c show repeat radiographs of an otherwise healthy
15-year-old boy with continued foot pain following 6 weeks of treatment in a short-leg
cast. Initial radiographs showed a minimally displaced fracture. Treatment should now
consist of

1- use of a hard-sole shoe.


2- continued cast treatment for an additional 6 weeks.
3- percutaneous screw fixation of the fracture.
4- electrical stimulation of the fracture.
5- open reduction and internal fixation of the fracture.

25
25
Online 2011 Orthopaedic Self-Assessment Examination by Dr.Dhahirortho
DISCUSSION: The patient has a delayed union of a proximal metatarsal fracture. With
continued pain and a widening of the fracture line, fixation is required. An intramedullary
screw can be used percutaneously to stabilize the fracture. Open reduction and internal
fixation is not necessary because the fracture can be stabilized and reduced
percutaneously. Continued cast treatment or a hard-soled shoe is not likely to provide
healing as demonstrated by the previous cast treatment. Electrical stimulation can be
used but has not been shown to aid in healing of the fracture when used as the only
treatment. The Preferred Response to Question # 36 is 3.

Question 37The variability of the DASH (disabilities of the arm, shoulder, and hand
questionnaire) score reported by patients after nonsurgical management of a distal
radius fracture has been shown to be affected by which of the following?
1- Neuroticism
2- Pain-escaping behavior
3- Depression
4- Occupation
5- Handedness

DISCUSSION: Wide variability has been seen by Ring and associates in the DASH scores for
patients treated for carpal tunnel syndrome, unilateral de Quervain tendinitis, trigger
finger, unilateral lateral elbow pain, or nonsurgical distal radius fractures. The authors
hypothesized that the large variation in DASH scores could not be accounted for by
physical factors and perhaps could be explained by illness behavior. They found that
neuroticism did not correlate with the DASH score but depression and pain anxiety did.
The study found a correlation between depression and all the upper extremity conditions
looked at in the study. Neuroticism was found not to correlate with the DASH score, pain-
escaping behavior is not measurable, and occupation and handedness have not been
found to be associated with variations in the DASH score. The Preferred Respo# 37 is 3.

Question 38 Figures 38a and 38b are the MRI scans of a 28-year-old man who reports
progressively worsening severe back pain for the past 3 months. He denies fevers,
chills, weakness, or neurologic dysfunction. Examination reveals tenderness to
palpation over the lumbar spine but normal neurologic findings. Laboratory studies
reveal an elevated erythrocyte sedimentation rate and C-reactive protein; blood

26
26
Online 2011 Orthopaedic Self-Assessment Examination by Dr.Dhahirortho
cultures are positive for methicillin-sensitive Staphylococcus aureus. In addition to
intravenous antibiotics, what is the next step in management?

1- CT-guided biopsy
2- Application of lumbar orthosis
3- Repeat MRI within 48 hours
4- Anterior lumbar debridement and fusion
5- Posterior lumbar debridement and fusion

DISCUSSION: The patient's symptoms and MRI findings are consistent with osteomyelitis
and diskitis at L3-4 with a paraspinal fluid collection. Cultures confirm bacterial
involvement. Given that finding, a biopsy of the level is unnecessary. Surgical treatment
for infection is not indicated given the lack of neurologic deficit. Nonsurgical
management is the best option, including both intravenous antibiotics and an external
lumbar orthosis. A repeat MRI scan within a short duration would not impact clinical care.
More important is close clinical follow-up to confirm response to treatment and identify
any potential neurologic deficits that may develop. The Preferred Response # 38 is 2.

Question 39Tension band wire fixation is best indicated for which of the following types
of olecranon fractures?
1- Comminuted fractures
2- Fractures that involve the coronoid process
3- Fractures associated with Monteggia fracture-dislocations
4- Oblique fractures distal to the midpoint of the trochlear notch
5- Transverse fractures through the midpoint of the trochlear notch

DISCUSSION: Tension band wiring may not provide adequate stability to prevent
displacement in a comminuted fracture. Plate fixation is most commonly recommended
for comminuted fractures of the olecranon. Additionally, plate fixation is used for oblique
fractures distal to the midpoint of the trochlear notch, fractures that involve the coronoid
process, and those associated with Monteggia fracture-dislocations. Tension band wiring
is best indicated for simple transverse fractures through the midpoint of the trochlear
notch. The Preferred Response to Question # 39 is 5.
27
27
Online 2011 Orthopaedic Self-Assessment Examination by Dr.Dhahirortho
Question 40A 56-year-old man with multiple skin nodules, seven large cafT-au-lait
spots, and significant scoliosis, has severe fatigue and shortness of breath. He should be
evaluated urgently for which of the following problems?
1- Aortic stenosis
2- Malignant peripheral nerve sheath tumor
3- Lisch nodules
4- Superior mesenteric syndrome
5- Acute chest syndrome

DISCUSSION: The clinical description is of a patient with neurofibromatosis, NF-1.


Although most of the answer choices can occur with NF-1 or sequelae of secondary
malignancy, the new onset of cardiac symptoms should prompt an urgent cardiology
evaluation for aortic stenosis, which occurs in approximately 2% of cases. Superior
mesenteric syndrome is a rare complication after scoliosis surgery but could not be a
source of lethargy prior to scoliosis surgery. Lisch nodules are neurofibromas of the iris
and are not an urgent problem at this time. Acute chest syndrome occurs most
commonly in patients with sickle cell anemia and would not be typical of patients with
NF-1. The Preferred Response to Question # 40 is 1.

Question 41 Decreased risk of shoulder and elbow injury in a throwing athlete has been
demonstrated with which of the following?
1- Rotator cuff strengthening
2- Superior labral repair
3- Posterior capsular stretching
4- Periscapular muscle strengthening
5- Repair of partial-thickness rotator cuff tears

DISCUSSION: Posterior capsular contracture has been demonstrated to significantly


impair the ability of the humeral head to translate anterior and inferiorly during the late
cocking and early acceleration phases of the throwing motion. This results in an
obligatory posterosuperior translation of the humeral head that may contribute to
posterior superior glenohumeral internal impingement with posterosuperior labral and
articular-sided rotator cuff pathology. Posterior capsular stretching in throwing athletes
has been demonstrated to decrease the likelihood of clinically significant shoulder or
elbow injury. Periscapular muscle and rotator cuff strengthening are important for
28
28
Online 2011 Orthopaedic Self-Assessment Examination by Dr.Dhahirortho
optimal scapulothoracic rhythm, stable scapular position for throwing, and rotator cuff
function but less directly established to result in a decreased risk of shoulder and elbow
injury than posterior capsular stretching. Partial-thickness rotator cuff repair and superior
labral repair may be necessary for treatment of symptomatic lesions unresponsive to
nonsurgical managemen The Preferred Response to Question # 41 is 3.t, but these do not
necessarily correlate with decreased shoulder and elbow injury risk.

The Preferred Response to Question # 41 is 3.

Question 42Figure 42 shows the radiograph of a 17-year-old girl who reports a 3-month
history of plantar foot pain at the second metatarsal head. Pain occurs with activity and
at rest. She has not noticed any swelling. Examination reveals only tenderness of the
articular portion of the second metatarsal head. What is the most appropriate
management?

1- Metatarsal pad
2- Corticosteroid injection
3- Second metatarsophalangeal arthrotomy
4- Second metatarsal shortening osteotomy
5- Second metatarsal neck dorsiflexion osteotomy

DISCUSSION: A metatarsal pad to reduce pressure on the second metatarsal head


effectively relieves pain caused by Freiberg's infraction. The symptoms are typically self-
limiting and do not require surgery unless thorough and appropriate nonsurgical
management fails to provide relief. The natural history is articular surface collapse and
degenerative arthritis. Both metatarsal neck dorsiflexion osteotomy and arthrotomy with
joint debridement have been demonstrated to be effective for symptoms in young
patients that persist despite the use of thorough and appropriate nonsurgical
management, and for symptoms in adults with degenerative arthritis. Intra-articular
corticosteroid injection will increase the intra-articular pressure and potentially
exacerbate the presumed osteonecrosis, though it may be helpful in adults with
secondary degenerative arthritis.

The Preferred Response to Question # 42 is 1.

29
29
Online 2011 Orthopaedic Self-Assessment Examination by Dr.Dhahirortho
Question 43Figures 43a through 43d show the MR arthrograms of a 42-year-old man
who has shoulder pain. Initially he reported a sharp pain, but now says it is somewhat
better. He describes the pain as aggravating, and has difficulty with overhead activities.
He reports pain deep within his shoulder and often notes a popping sensation. The
primary care physician sent him to physical therapy, which helped initially, but he still is
not able to perform his activities normally. Examination reveals symmetrical rotator
cuff strength, no increased anterior or posterior translation, and a positive O'Brien's
test. What is the next step in management?

1- Arthroscopic SLAP repair


2- Arthroscopic rotator cuff repair
3- Arthroscopic anterior-inferior capsulolabral plication
4- Arthroscopic subacromial bursectomy and acromioplasty
5- Open anterior-inferior capsulolabral plication

DISCUSSION: The patient has a type II SLAP tear. The MR arthrogram shows extension of
gadolinium beneath the biceps anchor; therefore, the most appropriate management is
arthroscopic SLAP repair. There is no evidence of an anterior-inferior labral tear or
rotator cuff injury, making the other surgical choices incorrect. The Preferred Res# 43 is 1.

Question 44When a patient with a grade II open tibia fracture presents to the
emergency department, which of the following components of treatment would be
considered the most important infection deterrent?
1- The use of bacitracin irrigation
2- Application of negative pressure wound therapy
3- A 6-hour time window to get the patient to the operating room
4- High-pressure pulse lavage as a means of mechanical debridement
5- Surgical wound inspection and debridement of devitalized tissue

DISCUSSION: Surgical inspection and debridement of devitalized tissue are the main
means of decreasing infection in open fractures. The arbitrary 6-hour window has not
30
30
Online 2011 Orthopaedic Self-Assessment Examination by Dr.Dhahirortho
been confirmed in recent studies. The use of bacitracin in the irrigation fluid has not been
shown to decrease infection and may create other wound healing problems. Bulb syringe
or low-pressure irrigation has been shown to have lower rates of rebound contamination
at 48 hours when compared with high-pressure lavage. Negative-pressure wound
therapy, although it has been a major advance in soft-tissue management, is still only an
adjuvant to surgical debridement and not a substitute for excision of devitalized tissue.
The Preferred Response to Question # 44 is 5.

Question 45Figures 45a and 45b show sagittal T1-weighted MRI scans of a 35-year-old
man who has had dominant extremity shoulder pain and weakness for the past 6
months. He denies any history of injury. Examination reveals full range of active and
passive motion, negative Hawkins and Neer impingement signs, 5/5 abduction
strength, 3+/5 external rotation strength with arm adducted at his side, and negative
belly press, Hornblower's sign, Gerber lift-off, and O'Brien's test. Radiographs are
unremarkable. An MR arthrogram shows no rotator cuff or labral tears and no
paralabral cysts. What is the next most appropriate step in management?

1- Electromyography (EMG) and nerve conduction velocity (NCV) studies of the


extremity
2- MRI scan of the cervical spine
3- Corticosteroid injection of the subacromial space
4- Arthroscopic suprascapular nerve release at the suprascapular notch
5- Laboratory evaluation of C-reactive protein, erythrocyte sedimentation rate, and
white blood cell count

DISCUSSION: The clinical history and physical examination are suggestive of weakness of
the infraspinatus. An EMG/NCV study should be obtained to determine the etiology of
the atrophy. In this case, the patient was shown to have suprascapular nerve entrapment
at the suprascapular notch with atrophy of the infraspinatus and early signs of
denervation of the supraspinatus. An MRI scan of the cervical spine would provide
information if the EMG study revealed a cervical nerve compression as the etiology of the
31
31
Online 2011 Orthopaedic Self-Assessment Examination by Dr.Dhahirortho
atrophy. Arthroscopic suprascapular nerve release at the suprascapular notch is the
correct treatment for the lesion; however, the EMG needs to be obtained first to
determine the location of nerve compression. Laboratory evaluation of C-reactive
protein, erythrocyte sedimentation rate, and white blood cell count is unnecessary
because there are no signs or symptoms of an infection. Corticosteroid injection of the
subacromial space would not help the current problem because there are no signs or
symptoms of impingement syndrome. The Preferred Response to Question # 45 is 1.

Question 46Figures 46a and 46b are the radiographs of a 10-year-old boy who has
severe pain in the anterior tibial region of his left leg after sustaining an injury 6 hours
ago. What is the most likely associated problem?

1- Vascular injury
2- Peroneal nerve injury
3- Anterior cruciate injury
4- Medial collateral ligament injury
5- Compartment syndrome

DISCUSSION: The patient has a proximal


tibial tuberosity injury with disruption of
the quadriceps mechanism. Compartment syndrome is associated with this injury.
Bleeding from the fracture enters the anterior compartment of the calf and can cause
elevated pressures. Because the injury occurred 6 hours ago and the patient has severe
pain, elevated compartment pressure should be suspected. Ligament injuries are not
associated with this injury. Peroneal nerve and vascular injuries are associated with
proximal tibial physeal fractures, but not with those involving only the tibial tuberosity.
The Preferred Response to Question # 46 is 5.

Question 47Spindled cells that are surrounded in mature osteoid that connect to other
similar cells via canaliculi are best described as which of the following?
1- Osteoblasts
2- Osteoclasts
3- Osteocytes
4- Histiocytes
5- Megakaryocytes
32
32
Online 2011 Orthopaedic Self-Assessment Examination by Dr.Dhahirortho

DISCUSSION: Osteocyte cell processes travel through canaliculi to interconnect with other
osteocytes and cells on the bone surfaces. Osteoblasts are cells that produce bone matrix
and are seen rimming immature bone. Osteoclasts are large multinucleated cells that
resorb bone and are found in Howship's lacunae. Megakaryocytes and histiocytes are
found in marrow but not mature bone cortex. The Preferred Response # 47 is 3.

Question 48Figure 48 shows the radiograph of a 17-year-old boy who sustained a


gunshot wound to his forearm. There is a small entrance wound on the volar surface.
The exit wound is dorsal and more than 15 cm in size, with loss of skin and an extensive
amount of devitalized muscle hanging out of the wound. Vascular supply to the hand is
excellent, the ulnar and median nerves are intact in the hand, but the radial sensory
nerve function is absent. After repeated surgical debridements of the wound and bone,
definitive treatment for the fracture would most likely be which of the following?

1- Spanning external fixation of the radius


2- Open reduction and internal fixation of the radius with free fibular
flap interposition 3- Open reduction and internal fixation of the
radius with interposed strut allograft
4- Open reduction and internal fixation of the radius with massive
cancellous allografting
5- Open reduction and internal fixation of the radius with massive
cancellous autografting

DISCUSSION: The injury needs a very complex traumatic reconstruction. After repeat
debridements, there will be a very long segmental loss of the radius, with a significant
loss of skin and muscle covering the bone. Spanning external fixation represents a good
temporary fixation tool but will not be a definitive solution. The preferred procedure is a
vascularized fibular graft with associated skin flap from the lateral leg. This surgical option
brings healthy vascularized bone and soft-tissue coverage into an area with significant
bone and soft-tissue loss. Placement of large quantities of allograft material, especially
strut allograft, is generally contraindicated in the setting of open fractures with soft-
tissue compromise because of the risk of infection. Internal fixation and massive
cancellous autografting is usually limited to one defect of less than 5 cm with intact soft-
tissue covering. The Preferred Response to Question # 48 is 2.
33
33
Online 2011 Orthopaedic Self-Assessment Examination by Dr.Dhahirortho
Question 49Figure 49 is the radiograph of a 73-year-old woman who underwent a left
knee revision 9 months ago. She states that she has been unable to extend her knee
since she fell 6 months ago. Treatment should consist of which of the following?
1- Knee fusion
2- Extensor mechanism allograft
3- Patellectomy with primary repair
4- Open reduction and internal fixation
5- Cast immobilization in full extension

DISCUSSION: The patient has a chronic extensor mechanism


disruption. Attempts at primary repair or open reduction and
internal fixation have a low probability of clinical success. Similarly, cast immobilization is
not advised as the patient already has a large diastasis between the superior and inferior
pole of the patella. An extensor mechanism allograft will provide the most predictable
outcome in patients with chronic extensor mechanism disruption following total knee
arthroplasty. A knee fusion remains as a surgical option but this should be considered a
salvage procedure. The Pr Res# 49 is 2.

Question 50A 7-year-old child has shoulder pain after falling off a swing. Radiographs
reveal a Salter II fracture with displacement of over two thirds the width of the shaft
(Neer-Horowitz IV). What is the most appropriate management?
1- Sling, graduated physiotherapy, and close monitoring
2- Closed reduction and pinning
3- Open reduction and internal fixation with plates
4- Open reduction and internal fixation with flexible nails
5- Open reduction and internal fixation and removal of the interposed periosteum
followed by pin fixation

DISCUSSION: Proximal humeral fractures in children are most often treated nonsurgically,
even with displaced patterns. Therefore, treatments that include closed or open
reduction are not indicated. There is little controversy in the treatment of proximal
humerus fractures in this age group and most patients attain good functional outcomes.
The humerus contributes about 80% of the growth of the humerus and has excellent
remodeling potential. Some reports indicate higher complication rates when surgically
treated. The Preferred Response to Question # 50 is 1.
34
34
Online 2011 Orthopaedic Self-Assessment Examination by Dr.Dhahirortho
Question 51 A 17-year-old girl with a history of Scheuermann's kyphosis has a fixed
thoracic deformity of 80 degrees. There was no correction of her deformity on supine
hyperextension radiographs. What is the most appropriate treatment?
1- Posterior arthrodesis
2- Anterior interbody arthrodesis
3- Smith-Petersen osteotomy with posterior arthrodesis
4- Vertebral column resection with posterior arthrodesis
5- Pedicle subtraction osteotomy with posterior arthrodesis

DISCUSSION: The Smith-Petersen osteotomy is most appropriate for long, sweeping,


global kyphosis, such as Scheuermann's kyphosis. It can achieve approximately 10
degrees of correction in the sagittal plane at each spinal level at which it is performed.
The pedicle subtraction osteotomy is the preferred osteotomy for patients with
ankylosing spondylitis, who have a sagittal plane imbalance. It can achieve approximately
30 degrees to 40 degrees of correction in the sagittal plane at each spinal level at which it
is performed. Vertebral column resections are extensive procedures, thus they are most
appropriately applied to pathologies with sharp angular kyphosis, anterior fusions, and
when maximal visualization and decompression of the spinal cord is required. Sagittal
curves were reduced an average of 50 degrees, with a lumbosacral deformity treated via
vertebral column resection. Anterior arthrodesis alone will not provide sufficient
correction and stabilization of the deformity. Posterior arthrodesis alone, while providing
stabilization, will not correct the fixed deformity. The Preferred Response # 51 is 3.

Question 52A 21-year-old minor league pitcher reports decreasing velocity and ability
to target his pitches over the last 2 months. He notes that his arm will start to feel
heavy in the later innings and notes pain in the posterior aspect of his shoulder in the
late cocking phase of his motion. He denies any specific event that initiated his
symptoms. Examination reveals symmetric rotator cuff strength and no increased
anterior or posterior translation of either shoulder. Supine range of motion of the right
shoulder in 90 degrees of abduction reveals external rotation to 100 degrees and
internal rotation to 25 degrees. The left shoulder has 95 degrees of external rotation
and 60 degrees of internal rotation. He has pain with an O'Brien's maneuver and a
negative apprehension sign. What is the next most appropriate step in management?
1- Subacromial corticosteroid injection
2- Use of a sling until the pain resolves
35
35
Online 2011 Orthopaedic Self-Assessment Examination by Dr.Dhahirortho
3- Posterior capsular stretching
4- Arthroscopic SLAP repair
5- Arthroscopic anterior-inferior capsulolabral plication with posterior capsular release

DISCUSSION: The patient has glenohumeral internal rotation deficit with posterior
capsular tightness; therefore, initial management should be directed at physical therapy
and posterior capsular stretching. The total arc of motion (external rotation + internal
rotation) should be equal between the shoulders. He has a deficit of 30 degrees in his
throwing shoulder. A "sleeper stretch" is a common way for patients to stretch the
posterior capsule and involves lying on the involved side with the shoulder abducted 90
degrees, the elbow flexed 90 degrees, and pushing the forearm toward the table.
Subacromial injection is not indicated because the pathology of an internal rotation
contracture is located within the glenohumeral joint space and not the subacromial
space. A sling might be useful for comfort but will not resolve his symptoms. There is no
indication for arthroscopy, SLAP repair, or anterior-inferior capsulolabral plication at this
time. The Preferred Response to Question # 52 is 3.

Question 53 A 48-year-old woman has an open subtrochanteric femur fracture. No


other injuries are reported. After thorough evaluation, it is determined that she will
need emergent surgical
fixation. The patient and family indicate that they are practicing Jehovah's witnesses
and desire adherence to the religious standards with respect to blood product usage.
The patient signs a valid advanced directive confirming these wishes. Which of the
following would be considered acceptable treatment?
1- Whole blood
2- Platelets
3- Plasma
4- Starch product (ie, Hetastarch, Hespan)
5- Donor-directed blood from a family member who is a practicing Jehovah's witness

DISCUSSION: Jehovah's witnesses beliefs regarding blood products stems from direct
interpretation of passages from the bible. The use of crystalloid, starch products such as
Hetastarch and colloids are accepted. Typically Jehovah's witnesses will accept most
medical treatment but refrain from the use of blood products including whole blood,
packed red cells, platelets, white cells, or plasma. Any autologous transfusion, whether
36
36
Online 2011 Orthopaedic Self-Assessment Examination by Dr.Dhahirortho
from the patient themself or donor directed, is forbidden. The use of cell-saver type
processes is a matter of individual choice by the patient. The use of hemoglobin-based
oxygen carriers are now accepted by many patients but it is important to respect the
wishes of each individual patient. It is very important to discuss preoperatively with the
patient and family their wishes and thoughts on what is acceptable to use. Many facilities
have adopted bloodless-surgery protocols and committees that definitively outline the
measures that can be used and take into consideration the many ethical issues involved
in taking care of these patients.
The Preferred Response to Question # 53 is 4.

Question 54 A patient who underwent intramedullary nailing of a femoral shaft


fracture 2 weeks ago now reports groin pain. What is the next most appropriate step in
management?
1- Obtain a radiograph of the hip
2- Obtain radiographs of the lumbar spine
3- Obtain an MRI scan of the lumbar spine
4- Review the radiographic report from the time of injury
5- Reassure the patient that the pain will improve and order physical therapy

DISCUSSION: Whereas ipsilateral fractures of the femoral neck and shaft are uncommon,
it is critical to recognize a femoral neck fracture that may occur in conjunction with a
femoral shaft fracture. The combined injury is seen in 2% to 9% of femoral shaft fractures
and may initially be missed in as many as one third of the cases. Preoperative
examination of a thin cut CT scan and dedicated AP internal rotation views of the femoral
neck can help identify this injury. In addition, the intraoperative AP and lateral hip
fluoroscopic view should be examined, and a dedicated radiograph of the hip obtained at
the conclusion of the surgery. At follow-up, Tornetta and associates has recommended
obtaining a dedicated AP radiograph of the hip with the leg internally rotated 15 to 20
degrees. Because the femoral neck is anteverted, 15 to 20 degrees of internal rotation of
the hip offers the best view of the femoral neck. Whereas associated lumbar spine
pathology may cause groin pain, the presence of a missed femoral neck fracture must
first be ruled out prior to investigating other sources of pain.
The Pre Res# 54 is 1.

37
37
Online 2011 Orthopaedic Self-Assessment Examination by Dr.Dhahirortho
Question 55Figure 55 is the lateral radiograph of a 63-year-old man who underwent
knee arthroplasty 8 years ago and is returning for his annual follow-up examination. He
now reports the development of pain and can walk short distances only. Infection
workup is negative. Management should consist of which of the following?

1- Bone scan
2- Knee revision
3- Bisphosphonate therapy
4- Routine follow-up in 1 year
5- Polyethylene liner exchange and bone grafting

DISCUSSION: The patient has severe periarticular osteolysis. The


tibial and femoral components remain well fixed to the bone.
Consequently, he can be treated by removing the wear generator (polyethylene
exchange) along with bone grafting of the osteolytic defect. Observation for 1 year is not
advised because the amount of osteolysis is extensive. Similarly, bisphosphonate therapy
has not been shown to decrease the amount of osteolysis once generated. A bone scan
may be helpful when assessing aseptic loosening. The patient is not very symptomatic
and loosening is unlikely. Pre Res# 55 is 2.

Question 56The femoral insertion of the lateral collateral ligament maintains what
consistent relationship relative to the lateral epicondyle of the femur?
1- Anterior and distal
2- Anterior and proximal
3- Posterior and distal
4- Posterior and proximal
5- The lateral collateral ligament inserts directly on the lateral epicondyle

DISCUSSION: The femoral insertion of the lateral collateral ligament maintains a proximal
and posterior relationship relative to the lateral femoral epicondyle. In a cadaveric study,
LaPrade and associates described the consistent anatomic relationship between the
lateral collateral ligament insertion and the lateral epicondyle of the femur. On average,
the lateral collateral ligament inserts 1.4 mm proximal and 3.1 mm posterior to the
lateral epicondyle. The lateral collateral ligament inserts proximal and posterior to the

38
38
Online 2011 Orthopaedic Self-Assessment Examination by Dr.Dhahirortho
popliteus insertion on the femur. The average distance between the femoral insertions of
the lateral collateral ligament and popliteus tendon was 18.5 mm. The Pre Res# 56 is 4.

Question 57Figures 57a and 57b are the MRI scans of a 61-year-old man who is unable
to elevate his dominant arm following a golf injury 24 hours ago. He has moderate pain
during attempted arm elevation. Examination reveals significant spinati atrophy and he
is only able to elevate his arm fully overhead while supine. The neurologic examination
is normal. What is the next most appropriate step in management?

1- Lidocaine injection test


2- Supraspinatus strengthening
3- Reverse shoulder arthroplasty
4- Conventional total shoulder arthroplasty
5- Arthroscopic rotator cuff repair/subacromial
decompression

DISCUSSION: The patient unknowingly has a chronic massive rotator cuff tear. Because of
excellent compensation, he remained functional and was without symptoms. This is
evidenced by the significant muscle atrophy. Following even trivial injury, the
compensation process of arm elevation fails and the patient suddenly loses the ability to
elevate the arm. At this time in management, it is critical to recognize that the rotator
cuff had already been torn and that pain now prevents the patient from actively using the
arm. To better ascertain a prognosis of return of function, injecting a local anesthetic
(lidocaine) into the joint is important. If, with an anesthetized joint, the patient can now
elevate the arm, a supine strengthening program will likely return the patient to his pre-
injury state. If there is no improvement in the ability to elevate the arm after the
injection, surgical considerations may become relevant. There is no role for arthroscopic
repair in this chronic, massive rotator cuff tear and decompression would likely lead to
superior escape. A reverse shoulder arthroplasty would be contraindicated in a very
active 61-year-old patient who 2 days ago was functioning normally. Based on the MRI
scan, there is no supraspinatus muscle remaining to strengthen. Total shoulder
arthroplasty is contraindicated in patients with a deficient rotator cuff mechanism.
The Preferred Response to Question # 57 is 1.

39
39
Online 2011 Orthopaedic Self-Assessment Examination by Dr.Dhahirortho
Question 58The radiographic finding in Figure 58 is indicative of what type of
acetabular fracture?
1- Anterior column
2- Posterior column
3- Associated both column
4- Transverse
5- Associated transverse plus posterior wall

DISCUSSION: The radiographic image is an obturator oblique view of the left acetabulum
and demonstrates a "spur" sign. It represents a spike of bone from the intact hemipelvis
and no articular surface remains with the hemipelvis, which defines the associated both
column fracture. The weight-bearing surface of the acetabulum is displaced with the
femoral head. In all other patterns, at least part of the articular surface remains with the
intact hemipelvis.

The Preferred Response to Question # 58 is 3.

Question 59A 4-year-old child has a 3-cm limb-length discrepancy, hemi-hypertrophy,


and a large tongue. Additional tests should include which of the following?
1- Thyroid function studies
2- CT scan of the hip, knee, and ankle to measure torsion
3- Echocardiogram and EKG
4- MRI scan of the spine and CBC with differential
5- Abdominal and pelvic ultrasounds and alpha-fetoprotein levels

DISCUSSION: The child likely has Beckwith-Wiedemann syndrome and up to a 10% chance
for the development of a tumor, especially a Wilm's tumor. Therefore, studies consisting
of surveillance abdominal and pelvic ultrasounds and alpha-fetoprotein levels, three to
four times per year until age 8, are recommended. An echocardiogram is not needed in
this population nor is thyroid function studies, MRI scan of the spine, or a CT scan to
address torsion.

The Preferred Response to Question # 59 is 5.

40
40
Online 2011 Orthopaedic Self-Assessment Examination by Dr.Dhahirortho
Question 60 An elderly woman with osteoporosis falls from a standing height,
sustaining a low-energy fracture of the acetabulum. What structures are most likely
fractured?
1- Posterior column and posterior wall
2- Anterior column and medial wall
3- Anterior column, posterior column, and ischium (T-type fracture)
4- Anterior column and posterior column (transverse fracture)
5- Anterior column, posterior column, and posterior wall (transverse/posterior wall
fracture)

DISCUSSION: Epidemiologic studies suggest that 4,000 acetabular fractures occur in


elderly patients each year in the United States. This accordingly may become the most
common age group to present with this fracture. In elderly patients with considerable
osteoporosis, a typical fracture pattern may present with intrapelvic dislocation of the
femoral head with compromise to the anterior column and "medial wall." The resulting
fractures are often complex fracture patterns with extensive comminution and
displacement. These may present as atypical fracture patterns not always conforming to
classic injury patterns described by Judet and associates. This fracture pattern seen
commonly in geriatric patients results from low-energy falls with force directly applied to
the greater trochanter. Fractures involving the posterior column and/or wall and
transverse fracture patterns involving both the anterior and posterior columns occur
infrequently in this age group. They are, however, more commonly encountered in
younger age groups as a result of higher energy trauma. The Preferred Resp # 60 is 2.

Question 61 The foot orthosis/footwear prescription for correction of a flexible


deformity typically seen in Charcot-Marie-Tooth disease includes which of the following
components?
1- Lateral heel and forefoot posting
2- Medial heel wedge with lateral forefoot posting
3- Metatarsal pad for global metatarsal head offloading
4- 3/8" heel lift with firm heel counter
5- SACH with medial flare

DISCUSSION: The typical Charcot-Marie-Tooth deformity consists of a cavus foot with


plantar flexion of the first ray with compensatory heel varus. The corrective foot orthosis
41
41
Online 2011 Orthopaedic Self-Assessment Examination by Dr.Dhahirortho
for this deformity (if flexible on examination) would include a lateral heel posting (ie,
wedge) to shift the heel into a more neutral position and lateral forefoot posting to
elevate the lateral border of the foot and accommodate and neutralize the varus-
producing effects of the fixed plantar flexion of the first ray. None of the other devices
listed will produce this result. The medial heel wedge will make the deformity worse.
Global metatarsal relief is often added to the Charcot-Marie-Tooth foot orthosis for pain
relief, but will not correct cavus deformity, nor will a 3/8" heel lift. A lateral flare might be
useful on the shoe to help control the lateral thrust on the shoe caused by a varus heel,
but a medial flare will accentuate the deformity. Adding SACH cushioning material to the
heel would soften the heel and not correct hindfoot malalignment. The Pr Res# 61 is 1.

Question 62Figure 62 shows the radiograph of a 46-year-old man who has had
increasing shoulder pain and diminishing motion over the last 10 years. Because his
difficulties are severely impacting his quality of life, he is seeking advice and treatment
options. Twenty five years ago, he underwent a shoulder stabilization procedure for
recurrent shoulder dislocations. Examination reveals he can only elevate his arm to less
than shoulder level and his external rotation is no more than 10 degrees. Management
consisting of nonsteroidal anti-inflammatory drugs and intra-articular steroid injections
has failed to provide relief. What is the most appropriate treatment recommendation?

1- Humeral head arthroplasty


2- Total shoulder arthroplasty
3- Reverse shoulder arthroplasty
4- Arthroscopic debridement/capsular release
5- Corticosteroid injection and physical therapy

DISCUSSION: The patient has classic "arthritis of dislocation." Procedures done years ago
were designed to enhance shoulder stability by limiting external rotation. However, it is
now understood that limiting external rotation results in significant alteration of joint
mechanics and kinematics, thus leading to the development of osteoarthritis. The
average age of patients who develop `arthritis of dislocation` is 45 years old. Despite the
young age of these patients, total shoulder arthroplasty offers the most predictable
improvement in pain and function. However, the patient must be made aware of the
need to protect the arm from excessive loads to protect the glenoid implant. Because
there is complete loss of articular cartilage and incongruent joint surfaces, there is no role
42
42
Online 2011 Orthopaedic Self-Assessment Examination by Dr.Dhahirortho
for arthroscopic debridement and capsular release. Injections offer little, if any, chance of
improvement with the prior history of nonresponse. Physical therapy predictably makes
patients worse because loading the arthritic joint generates more pain. Reverse shoulder
arthroplasty is reserved for elderly patients with severe rotator cuff deficiency. A humeral
head arthroplasty, while potentially more ideal than a total shoulder arthroplasty
because of glenoid concerns, would likely not offer pain relief in the face of the significant
glenoid involvement and incongruity. The Preferred Response to Question # 62 is 2.

Question 63A 10-year-old girl is seen in the emergency department after being involved
in a motor vehicle accident. She has right hip pain and is unable to bear weight. She has
no neurovascular deficits and no other injuries. Radiographs reveal a posterior
dislocation of the right hip without apparent fracture. The acetabulum appears to be
developing normally. What is the best course of treatment?
1- Open reduction under general anesthesia
2- Closed reduction under general anesthesia with fluoroscopy
3- Closed reduction under general anesthesia without fluoroscopy
4- Conscious sedation in the emergency department and closed reduction with
fluoroscopy
5- Conscious sedation in the emergency department and closed reduction without
fluoroscopy

DISCUSSION: Hip dislocation in the pediatric population is a rare event. However, prompt
recognition and rapid care for this injury is imperative to avoid future hip problems
including osteonecrosis of the femoral head (a devastating problem for a pediatric
patient). Reduction maneuvers can create violent impact between the posterior wall of
the (intact) acetabulum and the femoral head, resulting in shearing of the proximal
femoral physis and displacement of the epiphysis from the remainder of the femoral
head in skeletally immature patients. Therefore, deep sedation with good muscle
relaxation, such as that achieved with general anesthetic, is recommended. Reduction is
best accomplished with fluoroscopy for a number of reasons, including assessment of
concentricity of the hip joint after reduction, and to detect any catastrophic femoral head
physeal separation that occurs during the reduction maneuver. Sedation in the
emergency department is often insufficient to achieve acceptable muscle relaxation for
the patient. Open reduction is only indicated if closed reduction fails completely or if the
hip is not concentric after an apparently successful closed reduction. Pre Res# 63 is 2.
43
43
Online 2011 Orthopaedic Self-Assessment Examination by Dr.Dhahirortho
Question 64What is the most effective footwear modification for restoring the gait
pattern of the patient who has undergone an ankle arthrodesis?
1- Rocker sole
2- Lateral sole flare
3- Total contact insert
4- Extended steel shank
5- Solid ankle cushion heel (SACH)

DISCUSSION: When ankle range of motion is decreased, a rocker sole on the shoe helps
to accommodate for the lost motion by creating a more efficient heel-to-toe gait pattern
and allows the patient to "roll off" the foot during the late stance phase of gait using the
rolling action of the sole. The SACH is a soft material added to the heel of the shoe to
reduce the stress of heel strike. Although SACH modification will help to mimic the shock
absorbing action of ankle plantar flexion that occurs during heel strike, it is not as
beneficial to gait as a rocker sole. An extended steel shank stiffens the shoe and is
designed to reduce bending of the sole, but will not accommodate for lost ankle motion
(in fact, it will make ambulating more difficult for patients with decreased ankle motion
unless coupled with a rocker sole). A total contact insert is designed to cushion the foot
and offload certain areas of high stress or correct a flexible foot deformity. A lateral sole
flare is an outrigger attached to the sole of shoe and is used to help correct varus
deformities or compensate for lateral ankle instability. The Preferred Resp# 64 is 1.

Question 66Figures 66a through 66d are the radiographs and CT scans of a 72-year-old
woman with osteoporosis who sustained a fall from standing height. She has pain and
is unable to bear weight on the right knee. Surgical management is considered. Which
of the following best describes the preferred proximal screw fixation construct within a
laterally applied buttress plate?

44
44
Online 2011 Orthopaedic Self-Assessment Examination by Dr.Dhahirortho
1- 3.5-mm locking screws only
2- 3.5-mm nonlocking screws followed by 3.5-mm locking screws
3- 3.5-mm locking screws followed by 3.5-mm nonlocking screws
4- 6.5-mm fully threaded cancellous screws
5- 6.5-mm partially threaded cancellous screws

DISCUSSION: Displaced split depression fractures of the lateral tibial plateau require
articular surface elevation, restoration of anatomic plateau width, and sustained
elevation of the reduced articular components. This is accomplished by introducing
nonlocking lag screws first to compress and narrow the lateral rim thus restoring plateau
width. The introduction of locking screws first would disallow compression and
accordingly prevent reduction of the lateral rim. Locking screws are inserted after the lag
screws if the bone is osteoporotic to maintain articular elevation. Several biomechanical
studies have demonstrated inferior performance of large implants (6.5-mm screws and
4.5-mm plates) with regard to sustaining joint surface elevation. The Pre Res# 66 is 2.

Question 67.The radiograph seen in Figure 67 reveals an ankle fracture in a 65-year-old


woman who slipped on the ice. She has a history of diabetes mellitus for the past 7
years and reports that she maintains fair control of her diabetes; her last HgbA1c was
8%. The patient is a community ambulatory who lives independently. Examination
reveals she has absent sensation with the 5.07 monofilament. When determining
management, the physician must consider which of the following?
1- Supplemental internal fixation
2- Primary ankle arthrodesis
3- Nonsurgical treatment to avoid infection
4- Early bone grafting because of poor bone quality
5- Early mobilization and weight bearing to minimize stiffness

DISCUSSION: Increased immobilization and delayed weight


bearing are indicated in the neuropathic population after
treating an ankle fracture. Patients with diabetes mellitus and
peripheral neuropathy have higher complication rates following
ankle fractures treated surgically or nonsurgically. The elevated HgbA1c and neuropathy
both predict a higher complication rate with this fracture. Outcomes after nonsurgical
management of this fracture are poorer than after surgical treatment. Early bone grafting
45
45
Online 2011 Orthopaedic Self-Assessment Examination by Dr.Dhahirortho
is not recommended in closed fractures, but the use of supplemental internal fixation is
recommended because of the high risk of nonunion. More substantial constructs with
supplemental fixation, locking fixation, fixation through the calcaneus and talus into the
tibia, or external fixation are necessary. Primary arthrodesis is not recommended in this
fracture pattern or in a relatively active patient.
The Preferred Response # 67 is 1.

Question 68 A 17-year-old girl has a 2-year history of progressive, painful hallux valgus
deformity that is limiting her activities. Examination reveals no hypermobility. Weight-
bearing radiographs are shown in Figures 68a through 68c. Surgical correction of the
deformity should include which of the following?

1- Lapidus procedure
2- Akin osteotomy
3- Double metatarsal osteotomy
4- Distal metatarsal osteotomy
5- Distal soft-tissue release and/or
proximal metatarsal osteotomy

DISCUSSION: The patient has a


juvenile hallux valgus deformity, with
an increased distal metatarsal articular angle (DMAA), congruent first
metatarsophalangeal joint, and high
intermetatarsal angle. This constellation of findings is best managed with a closing wedge
or biplanar distal metatarsal osteotomy to correct the increased DMAA, and a proximal
metatarsal osteotomy to correct the high intermetatarsal angle. A Lapidus procedure
would be indicated for treatment of a hypermobile first ray, often manifested
radiographically as a plantar flexion sag through the first tarsometatarsal joint. A distal
soft-tissue release is indicated for an incongruent joint, whereas an Akin osteotomy is
used to treat hallux valgus interphalangeus. Although a distal metatarsal osteotomy alone
would correct the increased DMAA, it has insufficient corrective power to address the
high intermetatarsal angle.
The Preferred Response to Question # 68 is 3.

46
46
Online 2011 Orthopaedic Self-Assessment Examination by Dr.Dhahirortho
Question 69 A patient reports startup pain 3 months after undergoing a primary total
hip arthroplasty. Figures 69a and 69b show postoperative radiographs at 6 weeks and 3
months, respectively. Laboratory studies reveal a normal CBC count, C-reactive protein,
and erythrocyte sedimentation rate. Which of the following options is most
appropriate?

1- Continued observation
2- Revision of the femoral component
3- Hip aspiration for cell count and culture
4- Physical therapy for quadriceps
strengthening
5- Resection arthroplasty, antibiotic spacer,
and intravenous antibiotics

DISCUSSION: The patient has a loose femoral component, which has subsided at least 1
cm. The stem is undersized which is a risk factor for subsidence, especially with tapered
stems.Continued observation is not indicated. Revision total hip arthroplasty is the best
option. With a normal erythrocyte sedimentation rate and C-reactive protein, further
workup and treatment for infection is not indicated. The Preferred Response # 69 is 2.

Question 70 A 15-year-old girl sustained the injury shown in Figures 70a and 70b when
she jumped from the back of a moving truck. She is seen in the emergency department
2 hours after her injury. She has no other injuries. Her foot is warm and she has a
normal motor and sensory examination. Pulses are only evident on Doppler. What is
the most appropriate management?

1- MRI scan of the knee


2- CT scan of the distal femur
3- Application of a long-leg cast
4- Arteriogram of the extremity
5- Reduction and fixation of the fracture

DISCUSSION: The radiographs reveal a distal femoral fracture that is often associated with
a neurovascular injury at the level of the fracture. Initial treatment should be to reduce
the fracture, stabilize it, and then reevaluate the extremity for neurovascular function. A
47
47
Online 2011 Orthopaedic Self-Assessment Examination by Dr.Dhahirortho
CT scan, arteriogram, or MRI scan would not help and would delay treatment. A cast
would not be appropriate because access to the extremity is necessary and it would not
provide stabilization for vascular repair if it is required. The Preferred Resp# 70 is 5.

Question 71 A 54-year-old woman sustains the injury seen in Figures 71a and 71b. The
injury involves her nondominant extremity. What should the patient be told regarding
her expected outcome?

1- She should expect to return to full function and regain full range of elbow motion.
2- Reduction and casting has equivalent outcomes to those of surgical treatment.
3- This type of injury is associated with a high rate of complications.
4- Nerve dysfunction is commonly associated with this injury.
5- Ulnohumeral instability is the major complication seen with this fracture pattern.

DISCUSSION: This is a Bado type 2 (posterior) Monteggia lesion, which is associated with
higher rates of complications than other types of Monteggia lesions. The injury is
associated with indirect high-energy trauma and less often pathologic causes. Of the four
types of Monteggia lesions, the type 2 or posterior type is associated with the worst
prognosis. These injuries are best treated surgically with dorsal plating of the ulna and
reduction with fixation or arthroplasty of the radial head. The major complications seen
with this injury pattern are nonunion and plate failure. Almost all patients have some loss
of elbow range of motion. Satisfactory results based on functional scores for this injury
are not universal. Neurologic injury and ulnohumeral instability are unusual with this type
of injury. Full functional recovery is not expected with nonsurgical management. The
Preferred Response to Question # 71 is 3.

48
48
Online 2011 Orthopaedic Self-Assessment Examination by Dr.Dhahirortho
Question 72In a diagnostic test, the proportion of individuals who are truly free of a
designated disorder identified by the test is known as
1- specificity.
2- sensitivity.
3- accuracy.
4- positive predictive value.
5- negative predictive value.
DISCUSSION: Specificity refers to the proportion of individuals who are truly free of the
designated disorder who are so identified by the test. Sensitivity refers to the proportion
of individuals who truly have the disorder who are so identified by the test. Positive
predictive value refers to the proportion of individuals with a positive test who have the
disorder. Negative predictive value refers to the proportion of individuals with a negative
test who are free of the disorder. Accuracy is the overall ability to identify patients with
the disorder (true positives) and without the disorder (true negatives) in the study
population. The Preferred Response to Question # 72 is 1.

Question 73A 21-year-old throwing athlete has persistent shoulder pain. Figures 73a
and 73b are arthroscopic photographs taken from a posterior viewing portal and an
anterior viewing portal. During which phase of the throwing motion did the injury most
likely occur?
1- Wind-up
2- Early cocking
3- Late cocking
4- Acceleration
5- Deceleration
DISCUSSION: Five distinct phases of the throwing motion have been identified, each of
which places the static and dynamic stabilizers of the shoulder under different stresses. In
the late cocking phase, the throwing arm is abducted and maximally externally rotated.
Rotator cuff tears in throwing athletes may be the result of either tensile or compressive
forces. Tensile failure is believed to be the result of repetitive eccentric contractions.
Compressive failure is thought to result from direct contact of the articular side of the
rotator cuff between the greater tuberosity and posterior glenoid. Compressive failure
results in tearing of the posterior supraspinatus and anterior infraspinatus, in contrast to
the more common partial tearing of the anterior supraspinatus seen in the general
population. In addition to tearing of the articular side of the rotator cuff, compressive
49
49
Online 2011 Orthopaedic Self-Assessment Examination by Dr.Dhahirortho
forces also contribute to the peel-back mechanism and resultant avulsion of the
posterosuperior labrum and biceps anchor. Articular-sided posterior supraspinatus and
infraspinatus tears in combination with posterosuperior labral and biceps anchor
detachment has been termed internal impingement. It is believed to be the primary
result of either posterior capsular contracture (GIRD) or anterior capsular laxity. The
Preferred Response to Question # 73 is 3.
Question 74Figures 74a through 74c show the radiograph, bone scan, and MRI scan of a
17-year-old pre-professional ballet student who injured her ankle 9 months ago and
continues to report posterior pain, weakness, and instability. Which of the following
tendons most commonly can have associated pathology?
1- Peroneus brevis
2- Peroneus longus
3- Flexor hallucis longus
4- Flexor digitorum longus
5- Posterior tibialis tendon

DISCUSSION: Dance, especially ballet, requires frequent plantar flexion to an endpoint


greater than that of the average non-dancer. This may result in inflammation of the
posterior ankle caused by irritation of an os trigonum. An os trigonum is typically an
unfused secondary ossification center of the lateral tubercle of the posterior process of
the talus, but can also be a united stress fracture of the lateral tubercle of the posterior
process of the talus. The flexor hallucis longus runs directly medial to the lateral tubercle
of the posterior process of the talus and can develop coexistent tendinopathy. The
remaining tendons are not in as close proximity and are not associated with os trigonum
pathology. The Preferred Response to Question # 74 is 3.
Question 75A 60-year-old woman sustains the injury shown in Figure 75. Prior to her
injury, she lived independently and was a community ambulator without need for any
assistive devices. What treatment will give her the greatest long-term painless hip
function with the lowest reoperation rate?
1- Total hip arthroplasty
2- Hemiarthroplasty
3- In situ percutaneous pinning
4- Open reduction and percutaneous pinning
5- Open reduction and internal fixation with an intramedullary
device
50
50
Online 2011 Orthopaedic Self-Assessment Examination by Dr.Dhahirortho

DISCUSSION: Cemented hemiarthroplasty is typically used to treat displaced femoral neck


fractures in elderly patients. Recently, however, there has been a growing realization that
many of these patients would be candidates for total hip arthroplasty had they presented
with arthritis rather than fracture. Recent randomized studies have demonstrated
improved outcomes up to 4 years following total hip arthroplasty compared with
hemiarthroplasty in pain and functional scores. The rate of dislocation is higher following
total hip arthroplasty. However, some patients with hemiarthroplasties required later
conversion to total hip arthroplasty because of acetabular wear. In situ pinning is not
recommended for patients with a displaced fracture. Open reduction and internal fixation
of displaced femoral neck fractures in elderly patients is not recommended because of
the risk of nonunion and osteonecrosis. The Preferred Response to Question # 75 is 1.

Question 76Figures 76a and 76b are the sagittal T1-weighted MRI scans of an active 27-
year-old man who has had left dominant extremity shoulder pain and weakness for the
past 5 months. He denies any history of a precipitating event but recalls that the pain
began around the time he started lifting weights after a year off from lifting.
Examination reveals full range of active and passive motion, negative Hawkins and Neer
impingement signs, 5/5 abduction strength, 5/5 external rotation strength with arm
adducted at his side, and a negative belly press, Gerber lift-off, and O'Brien's test. He
does have weakness with resisted external rotation with the arm abducted to 90
degrees. Radiographs are unremarkable. An MRI arthrogram shows no rotator cuff tear
or labral tears. What is the most likely diagnosis?
1- Scapular dyskenisia
2- Quadrilateral space syndrome
3- Subacromial impingement syndrome
4- Suprascapular nerve compression by a
spinoglenoid notch
5- Suprascapular nerve compression at the
suprascapular notch

DISCUSSION: Examination reveals weakness of the teres minor muscle, and the MRI scan
shows moderate isolated atrophy of the teres minor muscle belly. This is consistent with
quadrilateral space syndrome, which is compression of the axillary nerve and posterior
circumflex humeral artery in the quadrilateral space (bounded by the teres minor, teres
51
51
Online 2011 Orthopaedic Self-Assessment Examination by Dr.Dhahirortho
major, long head of triceps and the humerus). This syndrome has been related to
compression of the neurovascular structures by muscle hypertrophy consistent with the
patient's history of lifting weights near the onset of symptoms. The next step in
confirming the diagnosis is a subclavian arteriogram with the arm in adduction as well as
in abduction and external rotation. Suprascapular nerve compression would be
manifested by atrophy and weakness of both the supraspinatus and infraspinatus (if
occurring at the suprascapular notch) or just infraspinatus (if occurring at the
spinoglenoid notch). The patient does not demonstrate signs or symptoms of either
impingement syndrome or scapular dyskenisia. The Preferred Response # 76 is 2.

Question 77A 32-year-old woman jammed her ring finger. Figures 77a and 77b show
radiographs of the finger after a closed reduction. Which of the following interventions,
if done correctly, is likely to result in the best possible final
clinical outcome?
1- Early removal of a splint and application of continuous
passive motion
2- Application of dynamic extension bracing after the first
week
3- Maintaining reduction of the middle phalanx on the
condyles of the proximal phalanx with dynamic external
fixation
4- Open reduction and anatomic restoration of the middle
phalanx articular surface
5- Surgical advancement of the volar plate into the middle
phalanx base

DISCUSSION: The most important determinant in the final clinical outcome in proximal
interphalangeal (PIP) joint fracture locations is the maintenance of the PIP joint alignment
on the lateral view. This can sometimes be done with just extension block splinting,
sometimes the fracture requires dynamic external fixation, and sometimes the fracture
requires open reduction or volar plate arthroplasty. Good function can be the result in
the setting of an incongruent middle phalanx base as long as the PIP joint alignment is
maintained. Continuous passive motion has not been shown to be of benefit. Whereas
dynamic external fixation in a flexed position is a very good treatment, dynamic extension
bracing will just precipitate loss of PIP joint reduction and is therefore not indicated.
52
52
Online 2011 Orthopaedic Self-Assessment Examination by Dr.Dhahirortho
Whereas open reduction of the articular surface is theoretically desirable, it is generally
impossible in the setting of the comminution of the volar middle phalanx base.
Furthermore, open reduction and internal fixation by itself does not guarantee that the
PIP joint alignment will be maintained, and typically it causes finger stiffness given the
extensive surgical approach. Likewise, volar plate arthroplasty is a surgery of last resort
and requires careful attention to PIP joint alignment before joint pinning. In this case,
with characteristics of comminution, dynamic external fixation is the preferred choice.
The Preferred Response to Question # 77 is 3.

Question 78Figures 78a and 78b show the CT scans of a 22-year-old man with back pain
after falling out of a tree. Examination reveals no palpable spinal step-offs, posterior
spinal pain, and normal neurologic function in the lower extremities. Normal perineal
sensation and normal rectal tone are present. What is the best management?
1- Bed rest
2- External orthosis
3- Anterior corpectomy and arthrodesis
4- Posterior instrumented arthrodesis
5- Posterior decompression and instrumented
arthrodesis

DISCUSSION: The patient has a stable L2 burst


fracture. There is no evidence of neurologic injury or disruption of the posterior
ligamentous complex. According to the Thoracolumbar Injury Classification System
(TLICS), the severity score for this injury is 2 and therefore nonsurgical management is
recommended. The TLICS was developed to define injury based on three clinical
characteristics: injury morphology, integrity of the posterior ligamentous complex, and
neurologic status of the patient. Point values are assigned to each major category based
on injury severity. The sum of these points represents the TLICS severity score, which may
be used to guide treatment. The injury scores are totaled to produce a management
grade that is, in turn, used to guide treatment. A score of >4 suggests the need for
surgical treatment because of significant instability, whereas a score of <4 suggests
nonsurgical management. The severity score offers prognostic information and is helpful
in medical decision making. An external orthosis provides enough support to obviate the
need for bed rest and avoid associated complications (deep venous thrombosis,
pulmonary embolism, pneumonia, skin ulceration). Surgical treatment, either through an
53
53
Online 2011 Orthopaedic Self-Assessment Examination by Dr.Dhahirortho
anterior or posterior approach, has been shown by Wood and associates to result in
increased pain and
disability and is therefore not indicated in this setting. Additionally, there is no need for
decompression in the setting of a neurologically intact patient. The Preferred Res# 78 is 2.
Question 79Which of the following conditions routinely requires early surgical
intervention in patients with Marfan syndrome?
1- Kyphosis
2- Ankle instability
3- Protrusio acetabula
4- Progressive scoliosis
5- Pseudarthrosis of the tibia

DISCUSSION: Marfan syndrome is a challenging disease for the orthopaedic surgeon.


Most problems of joint laxity, acetabular protrusio, and minor scoliosis curves are treated
nonsurgically. Pseudarthrosis of the tibia is not seen in Marfan syndrome; it is more
common in patients with neurofibromatosis (NF-1). Treating kyphosis is risky for
vertebral subluxation. Rapidly progressive scoliosis in immature patients is associated
with higher surgical complications, but surgery is indicated. Overcorrection is associated
with significant cardiovascular complications and should be avoided. The Pre Res# 79 is 4.

Question 80A 43-year-old woman has a 2-week history of right shoulder pain. She
denies any injury to initiate her symptoms but states that she has shoulder pain with
range of motion and lifting objects. Examination reveals mild pain with abduction,
empty can testing, and with the Neer and Hawkins impingement tests. Her range of
motion with the right shoulder reveals passive forward flexion to 90 degrees, abduction
to 90 degrees, external rotation at the side to 15 degrees, and internal rotation to her
buttock. The uninvolved left shoulder has forward flexion to 160 degrees, abduction to
150 degrees, external rotation at the side to 60 degrees, and internal rotation to T6.
Radiographs of the shoulder are normal. What is the next most appropriate step in
management?
1- Home exercise program
2- Sling at all times until her pain decreases
3- Closed manipulation under anesthesia
4- Arthroscopic rotator cuff repair
5- Arthroscopic anterior and posterior capsular release
54
54
Online 2011 Orthopaedic Self-Assessment Examination by Dr.Dhahirortho

DISCUSSION: The patient has the recent onset of adhesive capsulitis, which is
characterized by loss of both active and passive range of motion. A home exercise
program is as helpful as organized therapy to improve her range of motion. While a sling
might be appropriate for comfort, continuous use might increase her shoulder stiffness.
Surgical treatments, such as a manipulation under anesthesia or arthroscopic capsular
release, might be necessary if her motion cannot be restored with physical therapy and
home exercises. However, the natural history of idiopathic adhesive capsulitis is self
limited and does not usually require surgery. An arthroscopic rotator cuff repair is not
indicated because she does not have a rotator cuff tear. The Preferred Resp# 80 is 1.

Question 81Figures 81a and 81b are the radiographs of a 44-year-old woman who
reports the development of significant left hip pain over the past 6 months with
symptoms located in the groin and buttock. She notes pain while sleeping and
increased pain with walking up stairs or sitting for prolonged periods. Examination
reveals full range of motion, and internal rotation impingement is absent. The left
lower extremity is shorter than the contralateral leg by 1.5 cm. She denies lumbar spine
symptoms and has a normal neurologic examination. Treatment should consist of
which of the following?
1- Hip resurfacing
2- Total hip arthroplasty
3- Periacetabular osteotomy
4- Trochanteric varus osteotomy
5- Trochanteric valgus osteotomy

DISCUSSION: Periacetabular osteotomy is an excellent reconstructive procedure in


middle-aged adult patients with early hip arthritis and symptoms. The best candidates
have a very low vertical center edge angle of 0 degrees to 15 degrees, a minimum of 2
mm of cartilage joint space remaining, and a concentric articulation throughout the range
of motion. In these cases, rotational repositioning consisting of moving the dome of the
acetabular sourcil both laterally and anteriorly for improved surface area and coverage of
the femoral head during weight bearing can produce a long-term solution for this
condition. Whereas the surgical technique is challenging, complication rates are low with
surgical experience and offer a better alternative than a salvage procedure such as a total
joint arthroplasty or hip resurfacing arthroplasty. Trochanteric osteotomy has been used
55
55
Online 2011 Orthopaedic Self-Assessment Examination by Dr.Dhahirortho
for this condition, but does not offer the ability to significantly improve the surface area
on the acetabular side of the joint. The Preferred Response to Question # 81 is 3.

Question 82 A 20-year-old woman sustained a laceration to her volar forearm 4 cm


proximal to the wrist flexion crease. She has numbness in the thumb, index, and middle
fingers. After microscopic repair of the median nerve, 2 weeks of splinting, and
commencement of a hand therapy program, the patient is most likely to require what
secondary operation 6 months after the injury?
1- Tenolysis of the profundus tendons at the wrist
2- Nerve transfer of the ulnar motor nerve to the median motor nerve
3- Opponensplasty with the extensor indicis
4- Open carpal tunnel release
5- Transfer of the extensor digiti minimi to the first dorsal interosseous tendon

DISCUSSION: The patient sustained a laceration of the median nerve in what would be
considered a low median nerve injury. Standard treatment entails exploration and
microscopic repair of the median nerve. With a good quality nerve repair in a young
adult, return of some sensory function (albeit reduced compared with the normal nerve)
is usual. Return of motor function to the thenar muscles is more unpredictable. If the
patient begins a therapy program within a few weeks after nerve repair, it is unlikely that
tenolysis of the profundus tendons would be required. An open carpal tunnel release
would be unlikely to change functional return. The patient would not be expected to have
lost first dorsal interosseous function after a median nerve laceration because this muscle
is innervated by the ulnar nerve. A neurotization procedure for low median nerve palsy
has been described, but it consists of transfer of the distal anterior interosseous nerve
into the median nerve motor fascicles, not transfer of the ulnar nerve. Therefore, the
most likely secondary procedure required in this scenario is an opponensplasty procedure
to improve thumb opposition. The Preferred Response to Question # 82 is 3.

Question 83Figure 83a shows an axillary radiograph and Figures 83b and 83c show axial
MR arthrograms of a 20-year-old collegiate offensive lineman who has shoulder pain
while pass-blocking. He sustained a shoulder injury 3 months earlier when he "jammed
it." Prior to this injury, he denies any pain or instability in either shoulder. Despite
undergoing rehabilitation with a physical therapist and trainer and abstaining from
playing for 6 weeks, he is currently unable to play because of his symptoms.
56
56
Online 2011 Orthopaedic Self-Assessment Examination by Dr.Dhahirortho
Examination reveals full active range of motion, a positive jerk test which reproduces
his symptoms, and a grade 2 posterior translation of the humeral head with load and
shift testing which also reproduces his symptoms. What is the best management option
to allow him to return to his pre-injury function next season?

1- Arthroscopic posterior capsulolabral repair


2- Thermal capsulorrhaphy
3- Open anterior capsulorrhaphy
4- Intra-articular injection of corticosteroid
5- Immobilization for 6 weeks in external rotation
DISCUSSION: Arthroscopic posterior capsulolabral repair is most likely to return him to
competitive athletics. The patient has symptomatic posterior instability that is preventing
him from performing high-level athletic activities. Posterior subluxation of the humeral
head is seen on the axillary radiograph and a posterior labral tear is seen on the axial MR
arthrograms. Because nonsurgical management has failed to provide relief, treatment
should consist of posterior capsulolabral repair. This can be performed either
arthroscopically or open with similar excellent results. An intra-articular injection may
help his pain but will not likely allow him to return to his pre-injury functional level.
Thermal capsulorrhaphy has limited use in the shoulder because of the high rate of
complications reported, and anterior capsulorrhaphy will not correct the posterior
instability. Whereas a trial of immobilization in external rotation may have benefitted him
with the acute injury, it is unlike to help with this recurrent instability. The Pre Re# 83 is 1.
Question 84What is the greatest benefit of external fixation for treatment of displaced
and unstable pelvic ring injuries with hemodynamic instability?
1- It provides rigid fixation of the pelvis.
2- It helps maintain a stable clot over injured vessels.
3- It stabilizes the visceral injuries.
4- It allows the patient to sit and eat without pain.
5- It is more comfortable than skeletal traction.
57
57
Online 2011 Orthopaedic Self-Assessment Examination by Dr.Dhahirortho
DISCUSSION: External fixation has been shown not to provide rigid fixation of the pelvis
because a long moment arm from the fixator clamps to the posterior pelvis. Even with
elaborate constructs, the fixator alone is inferior to internal fixation of the posterior ring.
The main purpose of acute external fixation is to stabilize the initial clot forming about
the injured pelvic plexus. This initial clot contains innate clotting factors, making it more
stable, if not dislodged. If this clot is dislodged after hemorrhage and factor poor
resuscitation, the ensuing hemorrhage will not have the same ability to form a stable clot
around the injured vessels. The fixator does not stabilize any visceral structures. It
interferes with the ability to sit depending on its application and is no more or less
comfortable than skeletal traction. The Preferred Response to Question # 84 is 2.

Question 85 During right knee anterior cruciate ligament (ACL) reconstruction, after
drilling an appropriately positioned and referenced tibial tunnel, the surgeon finds that
the transtibial guide is placing the femoral tunnel at 11:30 within the intercondylar
notch. Which of the following choices will best enable appropriate graft placement in
this clinical scenario?
1- Revise the tibial tunnel to be more oblique.
2- Revise the tibial tunnel to be more posterior.
3- Convert to a transtibial double-bundle ACL.
4- Prepare the femoral tunnel via an anteromedial portal or two-incision technique.
5- Hyperflex the knee and place the femoral tunnel with the transtibial guide.

DISCUSSION: Anatomic placement of the femoral tunnel is best achieved in this clinical
scenario by drilling the femoral tunnel through the anteromedial portal or via a two-
incision technique. Several recent studies have demonstrated the difficulty that may be
encountered in restoring true ACL anatomy on the femoral side when placing a femoral
tunnel through a transtibial technique. While this is not always the case and this
technique may be reasonable and sufficient, it is important for orthopaedic surgeons to
critically assess tunnel placement intraoperatively and postoperatively to minimize errant
tunnel placement, demonstrated in the literature as the most common cause of ACL
failure and need for revision. In this not uncommon clinical scenario, simply converting to
a two-incision ACL technique or drilling through the anteromedial portal with the knee
hyperflexed will permit accurate femoral tunnel placement and increase the likelihood of
an optimal clinical outcome. Femoral tunnel accuracy with these techniques is enhanced
by a lower starting point in the intercondylar notch. Familiarity with these techniques is
58
58
Online 2011 Orthopaedic Self-Assessment Examination by Dr.Dhahirortho
valuable for surgeons performing ACL reconstruction. Revising the tibial tunnel in this
scenario would likely lead to bone compromise of the proximal tibia and may interfere
with graft fixation and incorporation. Converting to a double-bundle ACL with a
transtibial technique would not correct the vertical femoral tunnel. Hyperflexion of the
knee may improve femoral tunnel placement to some extent, but is unlikely to allow
anatomic placement of a femoral tunnel when the transtibial guide lies in a clearly
excessive vertical position. The Preferred Response to Question # 85 is 4.

Question 86An 11-year-old girl has patellar pain with activity and a knock-knee
deformity. A standing radiograph is seen in Figure 86. Physical therapy has provided
relief for the knee pain. The genu valgum is best treated by which of the following?
1- Observation
2- Brace treatment
3- Osteotomy of the proximal tibia
4- Osteotomy of the distal femur
5- Temporary bilateral distal femoral medial hemiepiphyseodesis

DISCUSSION: The hip-knee-ankle axis falls in the lateral


compartment of the knee. Most patients by age 11 have achieved
the axis they will have as an adult. Bilateral distal femoral medial
hemiepiphyseodesis with staples, plates, or screws that can be
placed and then removed after correction of the valgus is the
appropriate treatment. Observation is not likely to correct the
valgus at this age and hemiepiphyseodesis should be done while there is sufficient
growth remaining. Brace treatment and osteotomies of the tibia or femur are not
indicated in this age group. The Preferred Response to Question # 86 is 5.

Question 87Based on the current available best-evidence, what postoperative activities


should be recommended for patients undergoing first-time lumbar diskectomy for disk
herniation?
1- Bed rest
2- Avoid exercise for 6 to 8 weeks
3- Early return to low-intensity exercise
4- Early return to high-intensity exercise
5- Gradual return to low-intensity exercise after 6 weeks
59
59
Online 2011 Orthopaedic Self-Assessment Examination by Dr.Dhahirortho
DISCUSSION: Early return to high-intensity exercise is safe at 4 weeks. An update
Cochrane review demonstrated that exercise programs starting 4 to 6 weeks after
surgery in patients undergoing first-time lumbar diskectomy for disk herniation lead to a
faster decrease in pain and disability than no rehabilitation. Additionally, high-intensity
exercise programs seem to lead to a faster decrease in pain and disability than low-
intensity programs. In a prospective review of 50 consecutive patients undergoing first-
time lumbar diskectomy for disk herniation, Carragee and associates demonstrated that
lifting of postoperative activity restrictions after limited diskectomy allowed shortened
sick leave without increased complications. He concluded that postoperative precautions
in these patients may not be necessary. The Preferred Response to Question # 87 is 4.

Question 88 Which of the following factors is least likely to have an impact on fracture
healing?
1- Smoking
2- Obesity
3- Vitamin D deficiency
4- Use of bisphosphonates for osteoporosis treatment
5- Use of nonsteroidal anti-inflammatory drugs (NSAIDs)

DISCUSSION: Although the effect of obesity on complication rates has been studied and it
may increase wound complications, it has not been shown to increase nonunion rates.
The negative impact of smoking on bone healing has been shown in animal and human
clinical studies. NSAIDs interfere with the inflammatory phase of bone healing and
bisphosphonates interfere with osteoclast function, negatively impacting the remodeling
phase. Vitamin D deficiency has been identified in up to 70% of nonunion patients.
The Preferred Response to Question # 88 is 2.

Question 89 What is the best indication for prosthetic radial head arthroplasty
following fracture?
1- Mason type I fracture with full range of motion
2- Mason type I fracture with decreased supination
3- Mason type I fracture with decreased pronation
4- Mason type III fracture with associated interosseous membrane injury
5- Mason type III fracture without associated interosseous membrane disruption

60
60
Online 2011 Orthopaedic Self-Assessment Examination by Dr.Dhahirortho
DISCUSSION: The Mason classification differentiates the degree of displacement,
angulation, and mechanical block to motion. Most nondisplaced radial head fractures
(Mason I) in which there is no block to motion can be treated nonsurgically. Mason type
III injuries are severely comminuted radial head fractures. Fragment excision can be
considered in unreconstructable fractures in which the interosseous membrane is intact.
However, if the interosseous membrane has been disrupted, fragment excision can lead
to proximal migration of the radius with associated wrist problems. In this case, radial
head arthroplasty is indicated. Radial head arthroplasty may also be required when the
radial head fracture is associated with other ligamentous injuries as seen following an
elbow dislocation, or with an associated unstable coronoid fracture. Pre Resp# 89 is 4.

Question 90An orthopaedic surgeon in his first year of practice is negotiating with a
private for-profit hospital to be their employed trauma specialist. The state of
employment is known to have a high rate of malpractice claims because of a favorable
plaintiff legal environment. During the course of negotiations, malpractice insurance is
being discussed. The surgeon should ask the hospital to provide which type of
malpractice insurance policy?
1- Claims made with "nose" coverage
2- Claims made without tail coverage
3- No policy because of employed status and sovereign immunity
4- Occurrence coverage
5- Occurrence coverage with "nose" coverage

DISCUSSION: An occurrence policy provides coverage for all claims made during
employment irrespective of when it is filed (during or postemployment) and therefore is
the best option. Claims made policy only covers suits for the time employed. A
prepurchased "tail" is needed to provide coverage for cases that occurred during
employment but filed postemployment. Nose coverage is applicable if the surgeon was
previously employed and did not have tail coverage from previous employment, but this
surgeon just emerged from training where it is not applicable. Claims made without tail
coverage is unwise because the surgeon would be unprotected or have to purchase his
own policy postemployment. Only in certain situations does sovereign immunity exist,
and generally not in a for-profit system. Occurrence coverage with nose coverage is
incorrect because it does not apply to this surgeon with no previous employment or
claims policy lacking tail coverage. The Preferred Response to Question # 90 is 4.
61
61
Online 2011 Orthopaedic Self-Assessment Examination by Dr.Dhahirortho
Question 91A 21-year-old man who reports prior left knee pain recently felt a pop in his
knee and now is not able to ambulate. Examination reveals a well-developed, well-
nourished man with some stiffness around the knee. He has some fullness in the lateral
femoral condylar area and tenderness to palpation on the lateral side. There is no
adenopathy. Radiographs are seen in Figures 91a and 91b. At the time of surgery, open
biopsy specimens are seen in Figures 91c and 91d. What is the most appropriate
management?

1- Irradiation
2- Methylmethacrylate injection
3- Chemotherapy followed by wide resection
4- Amputation above the level of the lesion
5- Lateral condylar resection and allograft reconstruction

DISCUSSION: The patient has a giant cell tumor of the lateral condyle with a pathologic
fracture. The best option would be resection of the lateral condyle and osteoarticular
allograft reconstruction. There is collapse of the subchondral bone in the giant cell tumor,
making curretting and simple cementation difficult. Methylmethacrylate injection for
giant cell tumor is never indicated. Although giant cell tumors can be treated with
irradiation, surgery when possible is a better option. Amputation is almost never
indicated for giant cell tumor of bone. Chemotherapy is not indicated for giant cell tumor
of bone.
The Preferred Response to Question # 91 is 5.

62
62
Online 2011 Orthopaedic Self-Assessment Examination by Dr.Dhahirortho
Question 92Figures 92a and 92b are the radiographs of an elderly patient who
underwent revision total hip arthroplasty and was asymptomatic until falling; the
patient is now unable to bear weight. What is the most appropriate management?
1- Revision of the femoral component with a longer
stem
2- Revision of the femoral component with open
reduction and internal fixation with a plate, screws,
and cables or wires
3- Open reduction and internal fixation of the
fracture with a plate, screws, and cables or wires
4- Intramedullary fixation after revision of the stem
with a cemented device
5- Repair with cortical strut allograft and cerclage
wires

DISCUSSION: The patient has a periprosthetic femur fracture below a well-fixed, long
stem femoral component. Because the patient was asymptomatic prior to the fall and the
radiographs do not indicate loosening of the femoral component, revision of the femoral
component is not indicated. The fracture is a Vancouver type b-1 fracture and repair of
the fracture with plates and screws is indicated. Repair with cortical allograft and cerclage
wires may serve as an augment to plates and screws but if used alone (without a plate
and screw construct), it will not provide adequate rotational control. Pre Resp# 92 is 3.

Question 93What prosthetic factor has the most impact on decreasing the rate of
scapular notching in a Grammont-style reverse total shoulder arthroplasty?
1- Posterior tilt of the glenoid component
2- Inferior tilt of the glenoid component
3- Inferior positioning of the glenoid component
4- Use of a cemented humeral component
5- Use of locking screws in the glenoid component

DISCUSSION: A low position of the glenoid base plate has been shown to have the
greatest effect on decreasing scapular notching with a Grammont-style prosthesis.
Scapular notching is the phenomena seen after reverse total shoulder arthroplasty when
bone along the inferior scapular neck is lost. It is thought to be the result of repeated
63
63
Online 2011 Orthopaedic Self-Assessment Examination by Dr.Dhahirortho
contact between the humeral component and the bone. The Grammont-style reverse
total shoulder arthroplasty has a medialized center of rotation that decreases strain at
the glenoid component but has less space for the humerus to clear the scapula. Scapular
notching was seen least in components that are placed low on the glenoid. Posterior and
inferior tilt has minimal effect on scapular notching and may even increase notching by
bringing the humerus closer to the scapula. The use of locking screws and a cemented
humeral stem had no influence on notching.
The Preferred Response # 93 is 3.

Question 94A 16-year-old competitive female swimmer has a 1-year history of left
shoulder pain. She denies any specific injury to her shoulder. She reports that the pain
is worse with swimming but also has pain with daily activities. She also notes similar
occasional symptoms in her right shoulder. Examination reveals symmetric range of
motion and rotator cuff strength. Examination of the left shoulder reveals 2+ anterior
and posterior translation with pain in both directions and a 2-cm sulcus sign. The right
shoulder also has 2+ anterior and posterior translation and a 2-cm sulcus sign with no
pain. She also has hyperextension of the elbows and the ability to touch the radial
border of her thumb to her forearm. What is the next step in management?
1- Open inferior capsular shift
2- Arthroscopic thermal capsulorrhaphy
3- Sling at all times until the pain resolves
4- Arthroscopic anterior and posterior capsular plication
5- Physical therapy for rotator cuff and scapulothoracic strengthening

DISCUSSION: The patient has symptomatic multidirectional instability. A comprehensive


program involving physical therapy to restore dynamic stability to her shoulder is
indicated as a first-line treatment. Periscapular strengthening focusing on the serratus
anterior and rhomboids and rotator cuff strengthening should be emphasized. A sling
might be used occasionally for comfort but will not provide long-term relief of her
symptoms. Thermal capsulorrhaphy, although widely used in the past for shoulder
instability, has been abandoned because of a high complication rate. Surgical
interventions, such as capsular plications or open capsular shift procedures, might be
indicated if rehabilitation fails to relieve her symptoms.
he Preferred Resp # 94 is 5.

64
64
Online 2011 Orthopaedic Self-Assessment Examination by Dr.Dhahirortho
Question 95A 29-year-old man sustained an injury when he was playing basketball,
landing on his left knee while jumping for a rebound. He had vague pain in the anterior
aspect of the knee for several weeks. The initial radiographs were negative with the
exception of a large traumatic effusion. Examination reveals no apparent ligament
instability but a significant extension lag of 30 degrees. There was a palpable defect
above the superior pole of the patella. What is the most appropriate management?
1- MRI scan
2- Diagnostic arthroscopy
3- Surgical repair of a ruptured quadriceps tendon
4- Knee immobilizer for 6 weeks, followed by a sport brace
5- Limited weight bearing for 3 weeks, followed by physical therapy

DISCUSSION: This is the classic presentation of a tendon disruption in an active athlete


that may represent chronic strain or weakening of the tendon insertion. The factors that
lead to this condition are multitude, including biomechanic and cytologic, but there is
little evidence that inflammation is an active factor. Surgical treatment is straightforward
and logical. Suture anchors have been compared with simple holes made in the patella
for suturing the tendon, with no apparent biomechanic advantage.
The Preferred Response to Question # 95 is 3.
Question 96A 23-year-old woman sustains the injury seen in Figures 96a and 96b.
Treatment should consist of which of the following?

1- Total elbow arthroplasty


2- Closed reduction and casting
3- Open reduction and internal fixation of both the radial head and distal humerus
4- Open reduction and internal fixation of the radial head and excisional arthroplasty of
the distal humerus
5- Hinged external fixation with associated ligamentous reconstruction

65
65
Online 2011 Orthopaedic Self-Assessment Examination by Dr.Dhahirortho
DISCUSSION: This injury represents a complex partial articular fracture of the distal end of
the humerus with an associated radial head fracture. Given this patient's young age,
partial or complete arthroplasty is not an option. Closed reduction will lead to poor elbow
function. Ligamentous repair is not indicated and external fixation will not aid in articular
reduction. The patient requires open reduction and internal fixation of both components
of the intra-articular injury. This is best accomplished through an extensile lateral
approach or an olecranon osteotomy. Headless screws are preferred for articular
reconstruction in these cases. The Preferred Response to Question # 96 is 3.

Question 97A 65-year-old patient who underwent ankle arthrodesis 7 years ago is pain
free, but has difficulty walking. Hindfoot and transverse tarsal motion is painless. What
is the best treatment option?
1- Medial heel wedge
2- Heel-to-toe rocker sole
3- Morton's extension
4- Conversion to pantalar arthrodesis
5- Short polypropylene articulated-ankle foot orthosis

DISCUSSION: The use of a heel-to-toe rocker sole can decrease pressure on heel strike,
increase propulsion at toe-off, dissipate the forces across the arthrodesis site, and
normalize gait. The patient presents after a successful ankle arthrodesis. Extending the
arthrodesis is unnecessary with painless hindfoot and transverse tarsal motion. The use
of a medial heel wedge in a well-aligned arthrodesis is not indicated. A Morton's
extension is indicated for forefoot pain. A short articulated ankle foot orthosis would not
relieve any of the stress on the tibiotalar joint.
The Preferred Response to Question # 97 is 2.
Question 98 A 55-year-old woman has arm pain at rest and at night. Studies include a
positive bone scan in the metaphysis of the proximal humerus and a radiograph that
shows what appears to be a lytic bone lesion. What is the next step in management?
1- Indium scan
2- Skeletal survey
3- CT needle biopsy
4- Obtain a serum lactate dehydrogenase
5- MRI scan of the lesion and CT scan of the chest, abdomen, and pelvis

66
66
Online 2011 Orthopaedic Self-Assessment Examination by Dr.Dhahirortho
DISCUSSION: In patients older than age 40 years, a lytic lesion of bone is most likely
metastatic carcinoma. Local staging is achieved with an MRI scan, which can best identify
and localize any soft-tissue extension of the lesion. Identifying the primary site with a CT
scan of the chest, abdomen, and pelvis is successful 90% of the time. Although some
lesions, such as giant cell tumor of bone, have characteristic appearances on an MRI scan,
this modality is primarily used for staging rather than diagnosis. For malignancies,
systemic staging is required and usually includes a technetium Tc 99m total body bone
scan and noncontrast CT scan of the chest to seek potential sites of metastasis. Biopsies
are best performed by a team prepared to provide definitive treatment. For myeloma,
specifically, a skeletal survey is the preferred method for screening the skeleton.
However, bone scans are notoriously negative or inconclusive in patients with myeloma.
Lactate dehydrogenase is useful only in the setting of possible lymphoma of bone. The
primary function of indium scans is determining infection. The Preferred Resp# 98 is 5.

Question 99An 82-year-old woman underwent cemented right total hip arthroplasty
approximately 15 years ago. She fell and sustained the injury shown in Figure 99. What
is the most appropriate management for this injury?

1- Open reduction and internal fixation of the femur with a plate, screws, and cerclage
wires
2- Open reduction and internal fixation of the femur with a plate, screws, cerclage
wires, and cortical strut allograft
3- Revision of the acetabular component with open reduction and internal fixation of
the femur with a plate, screws, and cortical strut allograft
4- Revision of the acetabular and femoral components
5- Revision of the femoral component with a long cemented stem

DISCUSSION: The radiograph demonstrates a Vancouver type b-3 periprosthetic femur


fracture with a loose femoral stem and acetabular component. Open reduction and
67
67
Online 2011 Orthopaedic Self-Assessment Examination by Dr.Dhahirortho
internal fixation would not be appropriate in the setting of loose components. The femur
has circumferential radiolucent lines in the bone-cement interface and the bone quality
of the proximal femur is poor. In addition, the cemented acetabular component is loose
and has migrated proximally. Appropriate treatment includes acetabular revision and
femoral revision. Options for femoral revision include an allograft-prosthesis complex,
extended osteotomy with cables, or a proximal femoral replacement.
Pre Res# 99 is 4.

Question 100Figures 100a and 100b are the MRI scans of a 45-year-old man who has
had elbow and proximal forearm pain for the past 8 months. He can recall no specific
trauma and symptoms have not lessened despite his adopting job modifications that
limit lifting. He has discomfort with resisted elbow extension and pronation. The biceps
tendon can be easily palpated. Treatment should consist of which of the following?

1- Release of the lacertus and transfer of the biceps to the brachialis tendon
2- Open detachment, debridement, and reattachment of the biceps tendon
3- Anterior exploration and decompression of the posterior interosseous nerve
4- Excision of the anterior intramuscular lipoma
5- Endoscopic debridement of the biceps tendon

DISCUSSION: The MRI findings are most consistent with a partial tear of the biceps
tendon. In the setting of prolonged symptoms that are resistant to nonsurgical
interventions like rest, physical therapy, and modality, surgical treatment is indicated.
Exploration, debridement, and reattachment with one of a variety of techniques are the
standards of care. No lipomatous mass is seen on the MRI scan. There is no weakness in
finger extension to suggest posterior interosseous nerve palsy. Transfer of the biceps
would result in loss of supination strength. Endoscopic biceps tendon surgery is reserved
for long-head pathology.
The Preferred Response to Question # 100 is 2.

68
68
Online 2011 Orthopaedic Self-Assessment Examination by Dr.Dhahirortho
Question 101A 60-year-old woman with a long-standing history of diabetes mellitus
with documented peripheral neuropathy has a plantar ulcer. The ulcer has been
present for 3 months. Her primary care physician has treated her with saline dressing
changes with no success. The ulcer is located on the plantar surface of the foot under
the third metatarsophalangeal joint. On probing the wound, the metatarsal head is
visualized. What is the best diagnostic test to determine the presence of bony
involvement?
1- CBC count
2- C-reactive protein
3- Technetium bone scan
4- Bone biopsy
5- Weight-bearing radiographs
DISCUSSION: The presence or absence of osteomyelitis is difficult to discern. The clinical
finding that has been found to be the most specific for bony involvement is the presence
of an ulcer that probes directly to bone. Bone biopsy from the involved area is the most
accurate method to determine the presence or absence of osteomyelitis. A bone biopsy
with culture not only helps determine the presence of osteomyelitis, it helps in
determining the causative pathogen in chronic osteomyelitis. The standard laboratory
test such as a complete blood count with differential is not very helpful because of the
immunocompromised condition and vascular insufficiency in many of these patients. C-
reactive protein elevation and erythrocyte sedimentation rate can be helpful but are not
diagnostic for bone involvement. Standard radiographs can show erosive changes
consistent with osteomyelitis but in a neuropathic patient, this can be confused with
Charcot neuroarthropathic changes. The Preferred Response to Question # 101 is 4.

Question 102Figures 102a and 102b are the radiographs of a 10-year-old boy who
sustained an injury to his elbow in a fall. He is neurovascularly intact. What is the most
appropriate treatment?
1- Open reduction and internal fixation
2- Closed reduction and percutaneous pinning
3- Closed reduction and casting for 4 weeks in full
pronation
4- Closed reduction and casting for 4 weeks in full
supination
5- Splinting for 2 weeks, followed by early motion
69
69
Online 2011 Orthopaedic Self-Assessment Examination by Dr.Dhahirortho
DISCUSSION: The patient has a medial condyle fracture. These are uncommon injuries
and are often confused with fractures of the medial epicondyle. However, unlike medial
epicondyle fractures, medial condyle fractures involve the articular surface and require
anatomic reduction and fixation. This fracture is rotated radiographically. Open reduction
and internal fixation is likely to be necessary to achieve anatomic restoration of anatomy.
Closed reduction or splinting will not restore the joint surface adequately.
The Preferred Response to Question # 102 is 1.

Question 103Results of a study demonstrating no difference between treatments when


a difference truly exists is an example of which of the following?
1- Statistical insignificance
2- Type I error
3- Type II error
4- Fragile p-values
5- Negative predictive value

DISCUSSION: A type II error (also known as a beta error) occurs when results demonstrate
that two groups are similar when, in reality, they are different (with regard to the statistic
being measured). Type I errors show that a difference exists when, in reality, no
difference exists. A statistically insignificant result may lead an investigator to conclude
that no difference exists between two groups; this may be correct (and therefore not a
type II error). The concept of `fragile` p-values is that small sample sizes may result in
wide variability of p-values with only one change in a data point for a given group. This
singular change could be a chance occurrence, but it still can affect the statistical
significance of the outcomes analysis. Fragility of p-values is limited by increasing sample
sizes. Negative predictive value is the proportion of patients with negative test results
who are correctly diagnosed. The Preferred Response to Question # 103 is 3.

Question 104Among patients with lumbar degenerative disk disease and low back pain,
what factor is most predictive of clinical outcomes after surgical management?
1- Duration of symptoms
2- Workers' compensation
3- Use of disk arthroplasty
4- Severity of disk degeneration
5- Number of spinal segments treated
70
70
Online 2011 Orthopaedic Self-Assessment Examination by Dr.Dhahirortho
DISCUSSION: The treatment of low back pain ranges from nonsurgical management to
surgical management. Whereas many other treatment modalities have been
investigated, lumbar arthrodesis remains the primary surgical treatment of lumbar
diskogenic pain. Outcomes of surgical management vary but are consistently impacted
negatively by workers' compensation status. Neither the radiographic severity of disease,
number of spinal segments, nor duration of disease has been correlated with clinical
outcomes. While total disk arthroplasty was hoped to be an improvement over fusion,
the evidence available to date has shown no significant differences over arthrodesis.
The Preferred Response to Question # 104 is 2.

Question 105A 57-year-old woman sustains a posterolateral elbow dislocation.


Following closed reduction, a displaced radial head fracture of 40% of the joint surface
is noted. At surgery, the fragment is found to be comminuted into four pieces. What is
the best choice for treatment?

1- Partial radial head excision


2- Radial head excision
3- Radial head replacement
4- Total elbow arthroplasty
5- Open reduction and internal fixation with prolonged postoperative immobilization

DISCUSSION: The injury described is a ligamentous injury because of the dislocation with
the radial head fracture. Therefore, the surgical goals are to restore stability to the elbow
and allow early range of motion. Only radial head replacement will restore stability and
allow early range of motion of the elbow. Radial head excision is not recommended in the
setting of any instability because the radial capitellar joint is an important secondary
stabilizer of the elbow. Total elbow arthroplasty is not needed because the ulnohumeral
joint is normal. Partial excision of fragments over 30% will likely cause degeneration of
the capitellum and will not restore the secondary stabilizing effect of the radial head.
Attempts to repair the radial head that cannot achieve rigid fixation are not
recommended because they do not restore stability or allow early range of motion.

The Preferred Response to Question # 105 is 3.

71
71
Online 2011 Orthopaedic Self-Assessment Examination by Dr.Dhahirortho
Question 106A patient with a transverse femur fracture undergoes statically locked
antegrade intramedullary nailing. Postoperatively, the patient appears to have a
rotational deformity of greater than 25 degrees. The surgeon informs the patient, who
chooses to undergo corrective treatment with removal of distal interlocking screws,
rotational correction, and relocking of the screws. The patient goes on to heal but has
persistent hip pain and a limp that does not improve completely after extensive
rehabilitation. There is complete healing, no evidence of infection, no hardware issues,
no ectopic bone, and rotational studies indicate less than 2 degrees of malrotation.
Functional capacity testing reveals the affected abductor and quadriceps function to be
about 85% of the uninjured side and the patient returns to work and most of his
recreational activities except rock climbing. Two days before the statute of limitations,
the patient files a malpractice suit alleging negligence of surgery, loss of function,
consortium, and pain and suffering due to the surgeon's efforts. What action should the
surgeon and the defense team take?
1- Settle the case because the surgeon made an error that resulted in unnecessary
surgery, and thus the case is indefensible.
2- Settle the case because they are likely to lose the case, and it would be cheaper to
settle than to defend.
3- Defend the case alleging that there was no error, and no damages, and that the
patient is malingering.
4- Defend the case because despite there being an error, the error was corrected and
there were little or no damages compared with expected outcomes.
5- Contact the patient directly to discuss why he is suing and attempt an amicable
resolution.

DISCUSSION: To establish negligence, certain criteria must be met. 1) A duty was owed by
the surgeon (in this case, yes, a relationship was established). 2) The duty was breached,
where the provider failed to meet the standard of care (there was a technical error, but it
was corrected). 3) The breach caused an injury. In this case, the patient had an outcome
that was very acceptable, as documented with outcome studies, for femur fractures.
Also, the rotational error and locking distally would have had little impact on the hip,
whereas antegrade nailing itself is expected to result in some objective impairment of the
hip in some patients. 4) Damages were incurred as a result. In this case, the patient
returned to work and could not rock climb which could be reasonably expected with a
femur fracture in some patients, and cannot be causally linked to the corrective surgery.
72
72
Online 2011 Orthopaedic Self-Assessment Examination by Dr.Dhahirortho
For all practical purposes, the patient had a very acceptable outcome. Thus, settling the
case for an error would be rather permissive and the important issue is that the surgeon
recognized the problem, addressed it, and fulfilled his or her postoperative responsibility.
The case is very defendable, and thus it is unlikely to be lost. Defending the case and
alleging no error is incorrect because there was an error. The surgeon should never
function outside of his or her legal counsel once a suit is filed.

The Preferred Response to Question # 106 is 4.

Question 107 Five weeks following total knee arthroplasty, a woman has intermittent
knee drainage for 1 week. Clear serous drainage is coming from her wound from a small
area in the central portion of her incision. Her medical comorbidities include
hypertension and a BMI of 50. Fluid aspirated from the knee shows a WBC of
11,500/mm3 with 92% polymorphonuclear cells. Methicillin-resistant Staphylococcus
aureus grows from an aspirate on day 2. What is the next step in management?

1- One-stage revision using antibiotic-containing cement


2- Two-stage revision using an antibiotic cement spacer
3- Irrigation and debridement with polyethylene exchange
4- Infectious disease consult and appropriate oral antibiotics for 6 weeks
5- Infectious disease consult and appropriate IV antibiotics for 6 weeks

DISCUSSION: Infections that are diagnosed early have historically been treated with
irrigation and debridement and IV antibiotics. However, published literature shows that
this treatment is associated with success rates of less than 50%. The presence of resistant
bacteria in the setting of morbid obesity and persistent drainage further decreases the
success rate. A recent paper presented at the AAOS in 2010 showed poorer outcomes
following two-stage revision in those patients in whom an attempt at component
retention with irrigation and debridement had been performed first. Therefore, a two-
stage revision with the use of an antibiotic cement spacer is likely to give a better
outcome in this patient.

The Preferred Response to Question # 107 is 2.

73
73
Online 2011 Orthopaedic Self-Assessment Examination by Dr.Dhahirortho
Question 108Figures 108a through 108c are the radiographs of a 38-year-old man who
fell on an outstretched hand 1 week ago and now reports severe left elbow pain.
Examination of the wrist reveals normal range of motion with no tenderness or
swelling. Pain limits examination of his elbow. What is the most appropriate
management to determine if surgery is indicated?

1- CT of the elbow
2- MRI of the elbow
3- Apply a long-arm splint and schedule a follow-up examination in 2 to 3 weeks when
less painful
4- Examination under general anesthesia
5- Intra-articular lidocaine injection followed by repeat examination

DISCUSSION: Whereas there is controversy regarding the best treatment for comminuted
radial head fractures, nondisplaced and minimally displaced fractures in which there is no
block to motion can be treated nonsurgically. At the time of initial examination, it is
important to determine that there is no block to range of motion. If pain limits the ability
to examine the patient's range of motion, local analgesia with intra-articular lidocaine is
most appropriate. Whereas general anesthesia would be useful for open reduction and
internal fixation, the necessity for open reduction and internal fixation is best determined
first before scheduling surgery. Neither a CT or MRI scan is necessary if the patient has no
block to range of motion. Early range of motion is the best treatment for radial head
fractures treated nonsurgically. After immobilization for 2 weeks, it may be difficult to
determine whether there is a block to motion because the patient will likely have
decreased elbow pronation and supination.
The Preferred Response # 108 is 5.

74
74
Online 2011 Orthopaedic Self-Assessment Examination by Dr.Dhahirortho
Question 109You design a research study in which you ask patients who have a
nonunion of the tibia to fill out a questionnaire in which they report on a variety of
medical conditions and social/behavioral practices. You compare these findings to a
similar group who did not develop a nonunion in order to identify medical and/or social
conditions that might be risk factors for the development of tibial nonunions. This
would be an example of what type of study?
1- Case series
2- Meta-analysis
3- Case control study
4- Retrospective cohort study
5- Prospective cohort study

DISCUSSION: A case control series starts with the occurrence of a specific disease or
observation, and then compares data on those individuals to a similar group without the
disease (control group) in order to identify potential risk factors for the development of
the disorder. A case series is an observational study in which an investigator follows a
series of patients who received a specific treatment, recording the results and outcomes
of that treatment. A meta-analysis is the combination of several separate studies that
look at similar hypotheses in an effort to create a larger patient population for analysis. A
cohort study looks for the incidence of a specific outcome in two groups (cohorts) of
patients who are similar with the exception of a particular research variable (risk factor).
The Preferred Response to Question # 109 is 3.
Question 110Figures 110a and 110b are the radiographs of a 13-year-old boy who has
right lower extremity deformity, pain, and is unable to walk. He fell from the back of a
moving pickup truck. What is the most likely complication associated with this injury?
1- Growth arrest
2- Medial collateral ligament injury
3- Anterior cruciate ligament injury
4- Nonunion
5- Loss of knee motion

DISCUSSION: The radiographs show a distal


femoral Salter type II injury. The most likely
complication is a complete or partial growth arrest. Growth arrest is related to the
severity of displacement. This injury can cause growth arrest in 50% to 80% of patients.
75
75
Online 2011 Orthopaedic Self-Assessment Examination by Dr.Dhahirortho
The older the patient, the more likely growth arrest will occur. Medial collateral and
anterior cruciate ligament injuries are not associated with these fractures because the
bone fails at the physis and not at the joint level. The fractures virtually always heal;
therefore, nonunion is not an issue. Loss of motion is only transient and is not a
permanent complication. The Preferred Response to Question # 110 is 1.

Question 111Figures 111a and 111b show axial MRI scans of a 24-year-old man who
injured his right shoulder several years ago and now reports continued difficulty with
the shoulder and has pain with activity. He reports that when the injury occurred, he
felt that his shoulder "popped" but he never required closed reduction. He wore a sling
for about 6 weeks and went through several months of physical therapy. Which of the
following activities is most likely to cause him pain?

1- Reaching back to hit a forehand in tennis


2- External rotating the shoulder to spike a volleyball
3- Performing a bench press with large amounts of weight
4- Performing a biceps curl with large amounts of weights
5- Throwing a baseball at the point of late cocking/early acceleration

DISCUSSION: Performing a bench press with large amounts of weight is most likely to
cause pain for a patient with a posterior labral tear. A patient who sustains a first-time
posterior dislocation is less likely to have recurrent dislocations compared with first-time
anterior dislocations. Patients often do have problems with loading the shoulder in a
forward flexed position, such as during a bench press. The other activities listed here
might be difficult, but are not as likely to be problematic. A biceps curl might bother a
person with a SLAP tear. The late cocking/early acceleration phase of throwing, the
overhead portion of a tennis serve, and spiking a volleyball places the shoulder in an
abduction/external rotation position, which is likely to be problematic for a person with
anterior instability.
The Preferred Response to Question # 111 is 3.
76
76
Online 2011 Orthopaedic Self-Assessment Examination by Dr.Dhahirortho
Question 112A 29-year-old man sustained an open humeral fracture and underwent
surgical fixation 1 year ago. At that time, the radial nerve was transected and repaired
primarily. He now haspersistent wrist drop and is unable to extend his digits. Nerve
conduction velocity studies show no evidence of re-innervation. While discussing
surgical options, the patient states that one of his hobbies is playing football. The most
appropriate surgical reconstruction should include pronator teres transfer to the
extensor carpi radialis brevis
1- alone.
2- and the flexor carpi radialis to the extensor digitorum communis.
3- and the flexor carpi ulnaris to the extensor digitorum communis.
4- and the flexor carpi radialis to the extensor digitorum communis, and the palmaris
longus to the extensor pollicis longus.
5- and the flexor carpi ulnaris to the extensor digitorum communis, and the palmaris
longus to the extensor pollicis longus.

DISCUSSION: The standard transfers for radial nerve palsy involve the pronator teres to
the extensor carpi radialis brevis for central line of pull wrist extension. To power the
extensor digitorum communis, the choice is between the flexor carpi radialis and the
flexor carpi ulnaris. In a patient who needs power in throwing and needs to generate
ulnarly directed flexion, it is important to preserve the flexor carpi ulnaris function;
therefore, the flexor carpi radialis is the better choice. Furthermore, the thumb extension
deficit should be corrected and the palmaris longus makes a good choice. Pr Re# 112 is 4.

Question 113Compared with myodesis, osteomyoplasty offers which of the following


advantages in transtibial amputation?
1- Enhanced end-bearing
2- Early prosthetic fitting
3- Immediate weight bearing
4- Fibular abduction
5- Decreased surgical morbidity

DISCUSSION: Osteomyoplasty in transtibial amputation, originally described by Ertl in


1949, features creation of a bone bridge between the distal tibia and fibula, which is
theorized to enhance bony stability and increase end-bearing of the residual limb, and
may enhance patient-perceived functional outcomes relative to myodesis. Fibular
77
77
Online 2011 Orthopaedic Self-Assessment Examination by Dr.Dhahirortho
abduction is a known complication of traditional myodesis techniques, and is believed to
represent syndesmotic instability. Osteomyoplasty requires additional surgical time and
increased surgical morbidity, and because the success of the procedure is dependent on
achieving bony union, early prosthetic fitting and immediate weight bearing are typically
contraindicated. The Preferred Response to Question # 113 is 1.

Question 114Which of the following factors is considered to be the strongest predictor


of outcome following arthroscopic partial meniscectomy?
1- Patient age
2- Patient body mass index
3- Amount of meniscal resection
4- Location of the meniscal tear
5- Modified Outerbridge cartilage score

DISCUSSION: In a recent evidence-based review of the literature, the only consistent


factor predicting outcome after arthroscopic partial meniscectomy was the extent of
osteoarthritis as classified by the modified Outerbridge cartilage score at the time of
surgery. All other factors listed (ie, location of meniscal tear, patient age, patient BMI,
and amount of meniscal resection) were shown to not predict outcome following partial
meniscectomy. While not provided as an answer choice, female gender was shown to be
a predictor for slower recovery in the short term. The Preferred Respon # 114 is 5.

Question 115A 72-year-old woman has chronic effusions and pain in her right knee. She
has been treated with physical therapy and periodic epidural steroid injections for back
pain for several years. Radiographs are unrevealing and the MRI scans shown in Figures
115a through 115c reveal evidence of osteonecrosis. The patient has been treated
nonsurgically for the past 6 months without benefit and is now confined to limited
ambulation around the home, has chronic night pain, and requires narcotic medications
for comfort. What is the most appropriate management?

1- Observation for an additional 6 months


2- Diagnostic arthroscopy
3- Core biopsy and drilling procedure
4- Total knee arthroplasty
5- Physical therapy
78
78
Online 2011 Orthopaedic Self-Assessment Examination by Dr.Dhahirortho
DISCUSSION: The patient's condition raises several important issues. Although evidence
remains anecdotal, chronic repeated cortisone injections may be a cause of chronic
osteonecrosis in the hip and knee joints. The patient had chronic effusions, joint pain, and
a classic appearance seen on the MRI scans. With extensive involvement as noted on the
MRI scans, diagnostic arthroscopy and core drilling procedures are unlikely to offer much
benefit. For a patient younger than age 50, temporizing for another 6 months would be
beneficial in the hope that some resolution might occur. However, in a patient with
severe pain and functional disability, total knee arthroplasty is the best alternative.
The Preferred Response to Question # 115 is 4.

Question 116A workers' compensation carrier for a local manufacturing company


requests a second opinion on a 59-year-old man who sustained a crush injury to his
foot and leg at work 6 months ago. His leg and foot were pinned between a forklift and
a wall when an employee he was supervising lost control of the forklift. The employer
suspects that the injured worker is malingering because the treating physician released
him to work, but he has not returned to work. Which of the following elements of your
history will best help you determine that the injured worker does not want to return to
work out of fear of a confrontation with the employee he was supervising?
1- Formality
2- Empathy
3- Yes-no questions
4- Taking copious notes
5- Sitting leaning back in a chair

DISCUSSION: Empathy during the interview demonstrates compassion and earns the
patient's trust; which, in turn, enables the patient to discuss any agenda or concerns he
or she may otherwise feel uncomfortable revealing. It is also important to engage the
patient to establish a trusting relationship and thus understand all the factors impacting
the patient. A formal attitude toward the patient makes it difficult to engage the patient
to be "drawn in." An engaged patient is more comfortable, reliable, and thorough when
providing a history. Closed-end, yes-no questions do not allow the patient to detail all of
the subtle nuances of their condition and its effect on their life. Taking copious notes
likewise prevents engagement of the patient and the distraction of taking notes may
cause the physician to miss an important detail. It is better to lean forward in a chair
when interviewing a patient because this suggests the physician is genuinely interested,
79
79
Online 2011 Orthopaedic Self-Assessment Examination by Dr.Dhahirortho
whereas leaning back in a chair suggests the physician is simply waiting for the patient to
finish talking. Avoid interrupting the patient when talking.
The Preferred Res# 116 is 2.

Question 117 Figures 117a through 117e are the radiographs and CT scans of a 32-year-
old man who fell from a height of 8 feet and now reports pain and is unable to bear
weight on his left lower extremity. The limb has no neurovascular impairment and the
soft tissues are soft and intact. The preferred fixation construct should include which of
the following?

1- A medially applied uniaxial locking plate


2- A laterally applied uniaxial locking plate
3- A laterally applied polyaxial locking plate
4- Anterolateral and posteromedially applied plates via two incisions
5- Anterolateral and medially applied plates via a single anterior incision

DISCUSSION: The injury represents a bicondylar tibial plateau fracture with an associated
posteromedial fragment of considerable size. The posteromedial fragment can go
unrecognized and undertreated, resulting in loss of knee motion, instability, and arthritis.
A laterally applied plate is required to treat the bicondylar fracture. It alone, however,
will not address the posteromedial fragment adequately. A second, posteromedial plate
is required to adequately fix this important component of the fracture pattern. This is
preferentially inserted employing a second (posteromedial) incision. The insertion of both
plates through a single midline approach has resulted in unacceptable infection rates.
Any contemporary laterally applied plate (including polyaxial plates) will be insufficient by
itself to address this fracture pattern. A medially applied plate alone will inadequately
manage either the lateral condyle lesion or the posteromedial fragment.
The Preferred Response to Question # 117 is 4.

80
80
Online 2011 Orthopaedic Self-Assessment Examination by Dr.Dhahirortho
Question 118An 18-year-old football player crossing the field to make a catch is hit on
the shoulder and upper chest by the tackler and falls to the ground with immediate
pain throughout the shoulder region. The emergency department physician obtains the
radiographs, CT scan, and 3-dimensional reconstructions seen in Figures 118a through
118e. What is the next step in management?

1- Obtain an MRI scan of the shoulder.


2- Place the arm in a sling for comfort and treat the injury nonsurgically.
3- Perform closed reduction in the emergency department.
4- Perform closed reduction in the operating room.
5- Perform open reduction using Kirschner wires to hold the joint reduced.

DISCUSSION: The player has sustained a posterior sternoclavicular dislocation. The CT


scans show the medial end of the clavicle in close proximity to the aorta. An MRI scan will
add no diagnostic information and might delay treatment. Nonsurgical management of
an anterior sternoclavicular dislocation is often appropriate, but given the proximity of
the clavicle to the aorta and airway, reduction of the dislocation is recommended to
prevent vascular injury. While reduction is indicated, performing the reduction in the
emergency department is not recommended because of the vascular injury or the need
to perform an open reduction. Performing the procedure in the operating room with a
thoracic surgeon available is recommended. Usually a closed reduction is stable, but if
open reduction is necessary, Kirschner wires should be avoided to avoid the chance of
migration of the implants. The Preferred Response to Question # 118 is 4.

Question 119 Internal impingement of the shoulder and posterosuperior labral


pathology in throwers has been most clearly associated with which of the following?
1- Posterior capsular contracture
2- Anterior capsular laxity
3- Coracoacromial arch stenosis
4- Rotator cuff disease
5- Bennet's lesion

81
81
Online 2011 Orthopaedic Self-Assessment Examination by Dr.Dhahirortho
DISCUSSION: Posterior capular contracture has been recognized to be the primary
pathologic process resulting in internal impingement. Internal impingement of the
shoulder describes contact between the posterosuperior glenoid labrum and the
undersurface of the rotator cuff at the level of the posterior supraspinatus when the
shoulder comes into abduction and external rotation. This contact may be physiologic or
pathologic and is frequently seen in overhead throwing athletes, possibly resulting in
articular-sided rotator cuff tears, glenoid labral tears, tendinitis of the long head of the
biceps, anterior instability, glenohumeral internal rotation deficit, and dysfunction of
scapular rhythm. Nonsurgical management is the initial treatment of choice with an
emphasis on increasing range of motion and improving scapular mechanics. Anterior
capsular laxity may be present with internal impingement but is variable and less directly
associated with internal impingement than posterior capsular contracture.
Coracoacromial arch stenosis is associated with subacromial impingement and unrelated
to internal impingement. Bennett's lesion refers to exostosis or calcification at the
posterior capsule and while potentially associated with overhead throwing athletes who
may have internal impingement, a causal link between the two has not been established
and therefore posterior capsular contracture is the preferred response. The Preferred
Response to Question # 119 is 1.

Question 120Which of the following associated diagnoses is more likely to occur in a


young adolescent with a displaced type III tibial tubercle fracture that occurred as a
result of a noncontact basketball injury?
1- Compartment syndrome
2- Peroneal nerve palsy
3- Patella dislocation
4- Anterior cruciate ligament (ACL) and lateral collateral ligament (LCL) injuries
5- ACL and medial collateral ligament (MCL) injuries

DISCUSSION: Most tibial tubercle fractures occur as a result of a noncontact injury often
while a skeletally immature athlete lands from a jump. The resulting zone of soft-tissue
injury often exceeds that of the tibial tubercle itself, leading to the development of a
compartment syndrome. Typically, tibial tubercle fractures are not associated with ACL,
MCL, LCL, or patella injuries. The Preferred Response to Question # 120 is 1.

82
82
Online 2011 Orthopaedic Self-Assessment Examination by Dr.Dhahirortho
Question 121Figures 121a through 121d are the radiographs and CT scans of a 49-year-
old woman with a history of metastatic breast cancer who has progressively severe
right hip pain over the last 4 weeks. She is in a wheelchair and unable to walk.
Examination reveals she is obviously uncomfortable, has severe groin pain with any
motion of the hip joint, and mild back pain. She has no motor or sensory weakness in
her upper or lower extremities. A bone scan shows increased uptake in the femoral
neck. Treatment should consist of

1- hemiarthroplasty.
2- radiation therapy.
3- percutaneous pinning.
4- total hip arthroplasty.
5- cephalomedullary fixation.

DISCUSSION: The radiographs and CT scans reveals the characteristic appearance of


metastatic bone disease. Based on her sudden increase in pain and inability to walk, a
nondisplaced femoral neck fracture has likely occurred. If the primary is unknown, biopsy
should be considered prior to treatment. Radiation therapy can be used to treat local
bony disease, but is not recommended in the setting of an acute fracture. Although the
fracture is minimally displaced, percutaneous pinning is not indicated with diffusely
metastatic disease. Cephalomedullary fixation would be an option, however, but may not
be optimal for the fracture location. Furthermore, a small study comparing
intramedullary fixation with arthroplasty in proximal femoral metastatic disease found a
lower rate of implant failure and reoperation with arthroplasty (8% versus 16%).
Hemiarthroplasty could be an option, given the patient's advanced stage of disease, and
likely limited lifespan; however, the cystic changes in the acetabulum indicate the
presence of metastatic disease there. Additionally, studies have shown that total hip
arthroplasty pain and functional outcomes at 6 months through 2 years are superior to
hemiarthroplasty when the procedure is performed for elderly patients with hip
83
83
Online 2011 Orthopaedic Self-Assessment Examination by Dr.Dhahirortho
fractures. Total hip arthroplasty is the recommended treatment for patients with
metastatic femoral neck fractures. The Preferred Response to Question # 121 is 4.

Question 122 A physician receives a summons that he is being sued. The first step
should be to
1- call the patient and apologize.
2- notify the medical liability carrier.
3- contact an attorney with whom the physician is familiar with and have the attorney
review the records.
4- be sure to discard any handwritten phone messages because they are not
discoverable.
5- find a colleague with a similar subspecialty and have the colleague review the record
before doing anything.

DISCUSSION: The most appropriate first step is to notify the medical liability carrier. The
medical liability carrier will assign an attorney who is likely to be more appropriate. A
review by a colleague may be requested by the defense attorney but that should be at
their discretion. Patient apology is appropriate early on when and if you discover an
error. Records should be reviewed, but never altered. The Preferred Respon# 122 is 2.

Question 123What is the most efficient pressure for use with negative pressure wound
therapy?
1- 25 mm Hg
2- 75 mm Hg
3- 125 mm Hg
4- 300 mm Hg
5- 500 mm Hg

DISCUSSION: In animal and clinical studies, a range of pressures between 50 mm Hg to


500 mm Hg were tested; the most efficient pressure was 125 mm Hg, resulting in a
fourfold increase in blood flow, 63% increase in granulation tissue with continuous
pressure, and 103% increase in granulation tissue with intermittent pressure. When 125
mm Hg pressures were compared with either those less than 50, or those greater than
250, there was a decrease in granulation tissue in swine models. The Pre Res# 123 is 3.

84
84
Online 2011 Orthopaedic Self-Assessment Examination by Dr.Dhahirortho
Question 124Figures 124a and 124b are the radiographs of a 30-year-old man who
sustained an ankle injury and has swelling with lateral tenderness. The patient denies
any previous ankle injuries. After 6 weeks of rest and use of a removable ankle brace,
he continues to have swelling, lateral pain, and popping. An anterior drawer test
reveals a solid end point. Recommended treatment should include which of the
following?

1- Ankle arthroscopy and debridement of an osteochondral lesion


2- Peroneal retinacular reconstruction
3- Brostrom-Gould lateral ligament reconstruction
4- Immobilization in a walker boot in plantar flexion
5- Ankle rehabilitation and physical therapy
DISCUSSION: The radiographs and examination reveal peroneal tendon instability
requiring surgical treatment for persistent symptoms and tendon instability. The
radiographs demonstrate the "fleck sign," which is an avulsion of the posterior distal
fibular ridge, and represents an injury to the superior peroneal retinaculum and probable
peroneal dislocation. Peroneal tendon dislocations are typically present with vague
lateral ankle findings associated with swelling and tenderness over the distal fibula. The
tendons may be palpated as a ridge over the lateral fibula distally. Initial management of
the acute injury with cast immobilization in plantar flexion/inversion with the use of a
pad in the shape of a "U" or "J" is effective in 50% of patients; the rest will require
surgical treatment. The indications for surgical treatment of peroneal
dislocation/subluxation include continued pain and failure of nonsurgical management.
Associated peroneal tendon tears can be found when performing retinacular
reconstruction. Many techniques have been described including soft-tissue
reconstructions, bone block procedures as well as fibular groove-deepening procedures.
Radiographs do not reveal an osteochondral lesion. There is no evidence of lateral ankle

85
85
Online 2011 Orthopaedic Self-Assessment Examination by Dr.Dhahirortho
ligament instability. Ankle rehabilitation and physical therapy may further damage the
unstable tendons. The Preferred Response to Question # 124 is 2.

Question 125 Figures 125a and 125b are the current radiographs of a 52-year-old man
who sustained an injury to his dominant wrist 8 weeks ago. He is an alcoholic and does
not remember the details of how he injured it. Paperwork showing what treatment he
received at an urgent care facility indicates that he was given a splint for his "sprained
wrist." Examination reveals the pain is getting better, but there is persistent swelling
and range of motion is very limited. Recommended treatment at this time should
consist of

1- discontinuation of the splint and commencement of a regimen of hand therapy.


2- casting for an additional 2 weeks and reassessment of the fracture healing at that
time.
3- open reduction and internal fixation of the injury.
4- proximal row carpectomy.
5- wrist arthrodesis.

DISCUSSION: The injury represents a very uncommon presentation of a perilunate injury


pattern. Whereas these injuries are sometimes overlooked on initial radiographic studies,
they are usually recognized much sooner. In this case of a late presenting perilunate
injury in a patient that is not entirely responsible, a proximal row carpectomy represents
the best treatment option. Open reduction and internal fixation is generally not
successful because of cartilage degeneration and contracture that has developed in the
interim. No further splinting or casting is indicated, and neglecting the injury would be
indicated only if the patient refused any further treatment. Wrist arthrodesis is generally
indicated only as a salvage procedure if a proximal row carpectomy is unsuccessful.
The Preferred Response to Question # 125 is 4.

86
86
Online 2011 Orthopaedic Self-Assessment Examination by Dr.Dhahirortho
Question 126A 30-year-old man has had severe knee pain and swelling for 1 week. He
reports he previously had acromioclavicular joint pain that disappeared. He denies any
fever. Aspiration of a cloudy fluid from the knee reveals a WBC count of greater than
50,000 with 90% polymorphonucleocytes. While awaiting culture results, what is the
most appropriate action?
1- Cortisone injection
2- Open surgical debridement
3- Immediate arthroscopic lavage
4- Intravenous vancomycin for presumptive MRSA infection
5- Obtain sexual activity history and select appropriate antibiotic

DISCUSSION: The patient has polyarticular gonococcal arthritis. Acute septic arthritis in
adults can be separated into two major patient groups: young (age 15 to 40 years)
healthy, sexually active patients with gonococcal pyogenic arthritis and elderly or
immunocompromised patients with nongonococcal septic arthritis. In gonococcal septic
arthritis, the infecting organism is Neisseria gonorrhea. It is the most common cause of
acute joint infection in persons 15 to 40 years of age in the U.S. The clinical presentation
is variable, but typically includes migratory polyarthralgias, fever, rash, urethral or vaginal
discharge, and tenosynovitis. A patient with disseminated gonococcal infection may
report few genital symptoms. More than 50% of these infections are polyarticular.
Because patients with gonococcal septic arthritis are healthy, prompt antibiotic
treatment results in a generally good prognosis. MRSA septic arthritis would be
associated with fever, more rapid onset of symptoms, and is rarely polyarticular.
The Preferred Response to Question # 126 is 5.

Question 127A 38-year-old man sustained a complete thoracic spinal cord injury at age
14. An MRI scan of his shoulder, when compared with studies from uninjured controls,
is more likely to show which of the following?
1- Hypertrophied subscapular muscle
2- Rotator cuff tear
3- Posterior glenohumeral subluxation
4- Increased bone density
5- Supraspinatus nerve compression

87
87
Online 2011 Orthopaedic Self-Assessment Examination by Dr.Dhahirortho
DISCUSSION: Children that sustain a spinal cord injury or otherwise use a wheelchair for
mobility, and thus often have more pain and a higher incidence of structural and
functional changes of the shoulder joint as an adult. MRI studies have shown a four-fold
risk of rotator cuff tears in people with long-term paraplegia when compared with age-
matched controls. An MRI scan would not show bone density changes. The other answer
choices have not been demonstrated in higher numbers on MRI in paraplegics.
The Preferred Response to Question # 127 is 2.

Question 128Figures 128a and 128b show the radiograph and MRI scan of a 74-year-old
woman with severe neck pain and upper extremity numbness, tingling, and clumsiness.
She also reports that she has balance problems and sustained a distal radius fracture in
a fall 6 months ago. Examination reveals hyperreflexia in bilateral quadriceps and
Achilles reflexes, bilateral Hoffman's signs, and eight beats of clonus in both lower
extremities. What is the best treatment option?

1- Posterior laminectomy
2- Posterior laminoplasty
3- Posterior laminectomy and fusion
4- Cervical collar and observation
5- Combined anteroposterior decompression and fusion
DISCUSSION: The patient has cervical spondylosis and symptomatic myelopathy. The
radiograph reveals multilevel spinal cord compression and, most importantly, a fixed
kyphosis of the cervical spine. In the setting of cord compression and kyphotic deformity,
a combined anteroposterior approach allows for ventral and dorsal decompression,
kyphosis correction, and stabilization. Observation in the setting of severe myelopathy
will likely lead to further disease progression. In the setting of cervical kyphosis,
posterior-only treatment options will not adequately address cord deformation and,
therefore, not improve symptoms as reliably. The Preferred Response # 128 is 5.

88
88
Online 2011 Orthopaedic Self-Assessment Examination by Dr.Dhahirortho
Question 129Figures 129a and 129b show the six-month follow-up radiographs of a 62-
year-old woman who sustained a hip fracture in a fall. Prior to the fall, the patient was
active and had no difficulty with ambulation. The patient underwent open reduction
and internal fixation with a sliding hip screw device. She has difficulty with ambulation,
continues to walk with a walker, and reports startup pain. What is the most
appropriate management at this time?

1- Valgus osteotomy
2- Removal of the hardware
3- Intramedullary fixation after removal of the
hardware
4- Conversion to total hip arthroplasty with a
long cementless stem
5- Conversion to total hip arthroplasty with a
primary tapered stem

DISCUSSION: The radiographs demonstrate a healed fracture with penetration of the


screw through the femoral head into the acetabulum as well as osteonecrosis and
collapse of the femoral head (Figure 129b). Conversion to total hip arthroplasty with a
long stem is necessary to bypass the femoral cortical defects from the screw holes. A
primary tapered stem is not appropriate because of the proximal femoral deformity and
the stress risers associated with the screw holes. Removal of hardware, valgus
osteotomy, and revision of the internal fixation are not appropriate in the presence of the
femoral head collapse and acetabular penetration.
The Preferred Resp # 129 is 4.

Question 130A 45-year-old man sustained the injury seen in Figure 130a 6 weeks ago.
He denies any prior injury to his shoulder. After treatment of the injury in the
emergency department, he was noted to have significant weakness with empty can
testing and external rotation at the side. He has full passive range of motion with
forward flexion, abduction, and internal and external rotation, but has difficulty
initiating abduction with his arm at his side. He has negative apprehension and
relocation signs. A detailed neurologic examination shows no deficits. A coronal image
from a follow-up MRI scan is seen in Figure 130b. Follow-up radiographs reveal no
fractures. What is the most appropriate next step in his treatment?
89
89
Online 2011 Orthopaedic Self-Assessment Examination by Dr.Dhahirortho

1- Coracoid transfer
2- Rotator cuff repair
3- Reverse total shoulder arthroplasty
4- Arthroscopic anteroinferior labral repair
5- Physical therapy for range of motion and
strength improvements

DISCUSSION: The most likely concern, in a patient older than age 40 having a first-time
shoulder dislocation, is a rotator cuff tear. The MRI scan shows a tear of the
supraspinatus tendon. Recurrent instability is less likely to be a problem, so an external
rotation brace for an extended period of time is unnecessary. The patient already has
good passive range of motion, and with a full-thickness rotator cuff tear, physical therapy
alone is unlikely to return him to full function. The MRI scan shows no labral tear, so
arthroscopic or open repair is not indicated.
The Preferred Response # 130 is 2.

Question 131During a percutaneous plating of a proximal tibia fracture requiring a 13-


hole minimally invasive locking plate system, the placement of the distal most screws
should be done through a small open incision to avoid injury to what structure?

1- Superficial peroneal nerve


2- Saphenous nerve
3- Posterior tibial artery
4- Peroneal artery
5- Peroneal tendons

DISCUSSION: The superficial and deep peroneal nerves are consistently at risk near the
distal holes of long locking proximal tibia plates but can be avoided with a small open
incision for those screws. The peroneal tendons are more posterior at that level. The
saphenous nerve is medial. The peroneal artery runs behind the fibula and is not at risk.
The posterior tibial artery is posterior to the tibia.
The Preferred Response # 131 is 1.

90
90
Online 2011 Orthopaedic Self-Assessment Examination by Dr.Dhahirortho
Question 132Figures 132a and 132b are the lateral and anteroposterior radiographs of
a 15-year-old boy with a 6-month history of recurrent, activity-related posterior elbow
pain when pitching. Two separate 6-week periods of rest have failed to provide relief.
What is the next best step to enable him to return to play?
1- Physiotherapy
2- Long-arm cast
3- Cannulated screw fixation
4- Plate fixation of the ulna
5- Hinged-elbow bracing

DISCUSSION: Intramedullary
screw fixation of the olecranon
stress fracture is most likely to
allow him to return to play. Stress
fractures through a persistent olecranon apophysis have been well described in the
literature. The AP radiograph reveals the other physes of the elbow to be closed. After
patients fail to respond to appropriate periods of rest and cessation from throwing
followed by appropriate physiotherapy, surgical management with cannulated screw
fixation is appropriate and has been demonstrated to have favorable success rates.
Hinged-elbow bracing will not facilitate healing or return to play. Long-arm casting is
likely to result in stiffness and would not be unreasonable for a short duration at the
onset of symptoms, but is less likely to be helpful at this point. Plate fixation is not
indicated for treatment of this injury. The Preferred Response# 132 is 3.

Question 133Currently, what is the most common clinical study type in the orthopaedic
literature?
1- Level 1 (prospective, randomized trial)
2- Level 2 (cohort trial)
3- Level 3 (retrospective case control)
4- Level 4 (retrospective case series)
5- Level 5 (expert opinion)

DISCUSSION: Although a recent push for prospective, randomized trials has been
advocated by multiple orthopaedic journals, many studies published continue to be of
Level 4 evidence (retrospective case series). Case series represented 64% of all studies
91
91
Online 2011 Orthopaedic Self-Assessment Examination by Dr.Dhahirortho
reviewed by Freedman and associates in 2001 from the British and American volumes of
Journal of Bone and Joint Surgery and from Clinical Orthopaedics and Related Research.
Obremskey and associates published that 58.1% of all studies from nine orthopaedic
journals were Level 4 evidence. The Preferred Response to Question # 133 is 4.

Question 134A 47-year-old woman sustained a nondisplaced distal radius fracture 6


months ago and is unable to extend her thumb. When performing reconstruction using
the extensor indicis proprius to the extensor pollicis longus transfer, tension is ideally
determined by securing the tendons in what manner?
1- In maximum tension with the wrist and thumb in extension
2- In maximum tension with the wrist and thumb in neutral
3- In maximum tension with the wrist and thumb in flexion
4- According to the tenodesis effect with wrist flexion and extension
5- According to functional testing with the patient awake under local anesthesia

DISCUSSION: Extensor pollicis longus rupture can result from distal radius fractures.
Synergistic tendon transfer can be achieved using the extensor pollicis longus as the
motor donor. Whereas different schemes for achieving optimal tension are available, the
most reliable method is to tension the repair under local anesthesia while asking the
patient to perform thumb flexion and extension. Tendon transfer tension can be adjusted
accordingly to achieve maximum extension without compromising active flexion range.
Other methods of tensioning are estimates at best, and maximum tensioning in patients
without neuromuscular disease is rarely used in tendon transfers. The Pre Res# 134 is 5.

Question 135During spinal deformity surgery, which of the following is the most
specific early indicator of an intraoperative injury to the spinal cord?
1- Somatosensory-evoked potentials
2- Transcranial motor-evoked potential monitoring
3- Transcutaneous electroencephalogram neuromonitoring
4- Stimulus-evoked transpedicular electromyography (EMG)
5- Brainstem auditory-evoked responses (BAERs)

DISCUSSION: Transcranial motor-evoked potentials provide the most specific early


indicator of an intraoperative spinal cord injury. Somatosensory-evoked potentials are
routinely used but do not have the sensitivity and specificity of motor-evoked potentials.
92
92
Online 2011 Orthopaedic Self-Assessment Examination by Dr.Dhahirortho
EMG evaluations are routinely used for root evaluation following pedicle screw
placement. BAERs are typically used in monitoring brain surgery. The Pre Res# 135 is 2.

Question 136Figure 136 is the radiograph of a 68-year-old man who reports persistent
pain after undergoing total hip arthroplasty. Examination reveals equal limb lengths
and there is minimal discomfort with straight-leg raise or hip rotation. When asked to
ambulate, however, he has discomfort with the first few steps, and then can walk more
comfortably. C-reactive protein and erythrocyte sedimentation rates are normal.
Management should now consist of

1- an indium scan.
2- a three-phase bone scan.
3- revision of the femoral component with a cemented stem.
4- revision of the femoral component with a cementless stem.
5- cortical strut allografting of the femoral stress fracture.

DISCUSSION: The patient has a 100% radiolucent line around the


femoral component as well as a distal pedestal indicating
loosening of the femoral component. With these findings, there is
no need to do further workup for loosening with a bone scan. Infection is unlikely with
the normal laboratory findings so an indium scan is not necessary. Allograft for the femur
may improve pain with a stress fracture but not in the setting of a loose femoral
component. Revision of the femoral component with a cementless stem after removing
the fibrous endosteal tissue and the distal pedestal is associated with the best results.
Cemented revision stems in this setting are associated with early failure.
The Preferred Response to Question # 136 is 4.

Question 137Figures 137a and 137b show MRI scans of a 56-year-old man who fell
down the stairs and injured his elbow. He felt a pop and noted that his elbow had
significant swelling. The primary care physician ordered radiographs that showed no
fracture. Examination reveals moderate elbow swelling and ecchymosis. He has pain
with passive range of motion, but can achieve full extension and flexion to 150 degrees.
He is tender to palpation in the antecubital fossa and states that he would like to avoid
surgery if possible. Which of the following statements best reflects the outcome of
nonsurgical management?
93
93
Online 2011 Orthopaedic Self-Assessment Examination by Dr.Dhahirortho

1- He will have weakness with forearm supination.


2- He will have instability to valgus stress.
3- He will have weakness with elbow flexion.
4- He will have a significant loss of motion.
5- He will develop degenerative arthritis in his elbow.

DISCUSSION: The patient has a distal biceps tendon rupture. The MRI scans show the
tendon avulsed from its insertion and the amount of retraction of the tendon. Surgical
treatment to repair the tendon avulsion is often indicated, but nonsurgical management
can be recommended. Whereas flexion of the elbow is a biceps function and can be
decreased after this injury, the other elbow flexors often compensate adequately.
Significant decreases in forearm supination strength are frequent complaints of patients
with distal biceps injuries. Loss of motion, instability, and degenerative arthritis are not
common outcomes of this injury. The Preferred Response to Question # 137 is 1.

Question 138Figures 138a through 138c are the radiograph and CT scans of a 42-year-
old man who sustained an injury to both of his ankles and underwent surgical repair 2
weeks prior to presentation to your office. One ankle is healing well. On the
contralateral side, he reports pain and restricted ankle range of motion. Management
should consist of

1- the addition of more aggressive physiotherapy.


2- observation and continued non-weight-bearing.
3- addition of an anteriorly directed "syndesmosis screw."
4- loosening the syndesmotic screws from an overtightened position.
5- removal of screws, re-reduction of the syndesmosis, and revision fixation.

94
94
Online 2011 Orthopaedic Self-Assessment Examination by Dr.Dhahirortho
DISCUSSION: This patient has a malreduced syndesmosis. The CT scans clearly show the
fibula to be subluxated posteriorly relative to the incisura; therefore, surgical revision is
warranted. Revision surgery should include either removal of the current screws with
accurate reduction of the syndesmosis and new screw placement or repair of the
posterior malleolar fragment, which will in turn reduce the syndesmosis. Addition of an
anteriorly directed screw to the current construct will not change the malalignment.
Loosening the syndesmotic screws or addition of aggressive physiotherapy will not
correct the malrotation of the distal fibula within the incisura which is seen on the CT
scan. Outcomes after these injuries are related to the reduction of the ankle mortise.

The Preferred Response to Question # 138 is 5.

Question 139Which of the following substances is likely to cause the most soft-tissue
damage in the long term if injected into a fingertip under high pressure?

1- Grease
2- Latex paint
3- Water
4- Oil-based paint
5- Chlorofluorocarbon-based refrigerant

DISCUSSION: This type of injury represents a difficult problem in hand surgery. The
factors that most determine outcome after high-pressure injection injuries into the
fingertip include: involvement of the tendon sheath, extent of proximal spread of the
injected substance, pressure setting, and delay to surgical treatment. The other factor
that likely is most important is the type of substance injected. Water and latex-based
paints are least destructive. Grease and chlorofluorocarbon-based substances are
intermediate, but aggressive surgical debridement can restore reasonable function. Oil-
based paints are highly inflammatory and can cause such chronic inflammation such that
amputation may be the only reasonable treatment option despite early aggressive
surgical treatment.

The Preferred Response to Question # 139 is 4.

95
95
Online 2011 Orthopaedic Self-Assessment Examination by Dr.Dhahirortho
Question 140Which of the constructs seen in the Figures 140a through 140c best
demonstrates buttress plating technique for the fracture shown?

1- Figure 140a with a nonlocked screw


2- Figure 140a with a locked screw
3- Figure 140b with a nonlocked screw
4- Figure 140b with a locked screw
5- Figure 140c with a locked screw

DISCUSSION: Buttress plating technique relies on


an under-contoured plate secured with a
nonlocked screw near the apex of a vertical fracture. This provides an axilla to resist
vertical displacement. Locked screws do not compress plate to bone and are not ideal for
buttress plating technique. The Preferred Response to Question # 140 is 1.

Question 141The risk for remanipulation of a pediatric distal forearm fracture, after
initial reduction and casting, is most closely related to
1- initial immobilization with a short-arm cast.
2- the location of the fracture.
3- initial translation of the fracture.
4- initial angulation of the fracture.
5- a single versus both bone fracture.

DISCUSSION: Initial fracture translation has been shown to be associated with a higher
risk for remanipulation. Fracture reduction is important and if there is residual translation
after reduction, consideration for fixation should be considered. The location of the
fracture or single versus both bone fracture, in itself, is not a risk factor for
redisplacement, nor is the use of long- versus short-arm casts. The Pre Resp# 141 is 3.

Question 142A 72-year-old man has a severe limp 9 months after undergoing a total hip
arthroplasty. He has no pain. His straight lateral incision from an anterolateral
approach healed well without prolonged antibiotics or drainage. His legs feel equal
when he stands, but he ambulates with a severe Trendelenburg limp and is unable to
actively abduct his hip against gravity. What is the most likely cause of his problem?
96
96
Online 2011 Orthopaedic Self-Assessment Examination by Dr.Dhahirortho
1- Component loosening
2- Component impingement
3- Foraminal stenosis at L3-4
4- Detached gluteus medius tendon
5- Neuropathy of the superior gluteal nerve

DISCUSSION: The direct lateral approach to the hip is commonly used for primary total
hip arthroplasty (50% to 65% of cases). The technique requires detachment of a portion
of the gluteus medius tendon and then reattachment during closure. In a small
percentage of patients the repair will fail, resulting in significant abductor weakness and a
Trendelenburg limp. This is often painless after the initial surgical healing time.
Component impingement can lead to early wear or dislocation but would not cause a
limp. It is usually painless. Foraminal stenosis could cause isolated weakness but is much
more likely to cause radicular type symptoms of pain with or without numbness or
weakness. Any weakness would be in a nerve distribution pattern and because the
superior gluteal nerve has components from L4, L5, and S1, weakness from root
compression would be subtle and incomplete. Dissection of more than 3 cm to 4 cm from
the greater trochanter can injure the superior gluteal nerve and result in weakness, but
this is much less reported and has been shown to be transient in most cases. Component
loosening can cause a limp but is painful and would produce weakness.
The Preferred Response to Question # 142 is 4.

Question 143A 22-year-old woman underwent closed reduction and percutaneous


pinning with casting of a displaced extra-articular distal radius fracture. The surgery
was completed with a supraclavicular regional anesthesia. After the block wears off,
she reports new onset dense numbness in the palmar aspect of the thumb, index, and
middle fingers as well as severe pain in the hand. What is the next step in
management?
1- Bivalve the cast and follow up in 1 week
2- Return to the operating room for open carpal tunnel release
3- Compartment pressure monitoring of the hand
4- Emergent nerve conduction velocity studies
5- Exploration of the supraclavicular brachial plexus

97
97
Online 2011 Orthopaedic Self-Assessment Examination by Dr.Dhahirortho
DISCUSSION: The injury represents a somewhat uncommon problem after surgical
treatment of distal radius fractures; however, vigilance is required to detect the acute
presentation of a carpal tunnel syndrome. In this case, urgent release of the tunnel is
recommended. Bivaling the cast alone is indicated when the pain is less severe, and only
when the numbness is very minimal and more generalized. Compartment syndrome of
the hand is almost unheard of in the setting of a distal radius fracture; rather it is more
commonly associated with a crush injury to the hand. There is no role for emergent nerve
conduction velocity studies or brachial plexus exploration. The Pre Res# 143 is 2.

Question 144Patients with multiple hereditary exostoses have a greater risk of


developing what kind of mesenchymal tumor?
1- Hemangioma
2- Enchondroma
3- Chondrosarcoma
4- Extra-abdominal desmoid tumor
5- Neurofibroma

DISCUSSION: Patients with multiple hereditary exostosis (MHE) have an increased risk of
secondary chondrosarcomas in an area of a prior exostosis. This risk is probably 1 in
10,000 MHE patients and typically is a low-grade chondrosarcoma. Mafucci's syndrome is
a different disorder and is associated with hemangiomas. Ollier's patients have multiple
enchondromas. Extra-abdominal desmoids are associated with Gardner's syndrome, and
von Recklinghausen's disease is associated with plexiform neurofibromas.
The Preferred Response to Question # 144 is 3.

Question 145A 50-year-old woman with a history of type 1 diabetes has a 2-month
history of pain and swelling in her left foot. Initial radiographs are seen in Figures 145a
and 145b. She has been treated in a cast and has been non-weight-bearing for 2
months. Her skin is intact but her foot is swollen, warm, and erythematous. She is
afebrile. Laboratory studies show a uric acid level of 4.0 mg/dL (normal 2.5-7.0 mg/dL),
white blood cell count of 9,700/mm3 (normal 3,500-10,500/mm3), erythrocyte
sedimentation rate of 65 mm/h (normal up to 20 mm/h), and a glucose level of 166 mg.
Current radiographs are seen in Figures 145c and 145d. What is the best treatment
option at this time?

98
98
Online 2011 Orthopaedic Self-Assessment Examination by Dr.Dhahirortho

1- Continued total contact cast immobilization


2- Medical management for gouty arthritis
3- Physical therapy to work on range of motion and strengthening
4- Immediate open reduction and internal fixation of the navicular fracture
5- Open biopsy of the left midfoot for deep cultures and a referral to infectious disease
for antibiotic therapy

DISCUSSION: The radiographs show progressive disruption of the talonavicular joint


consistent with a neuropathic arthropathy. This is clinically in the development-
fragmentation stage (Eichenholtz stage 1) with the triad of erythema, warmth, and
swelling, and is best treated in a non-weight-bearing cast. Cast immobilization for stage 1
has been shown to be effective in multiple studies; however, the non-weight-bearing
status has not been conclusively shown to be necessary. Whereas infection is always a
possibility, the lack of an open wound and signs of sepsis: fever, leukocytosis, or elevated
serum glucose, make an infectious process doubtful. The uric acid level is normal and
gout does not cause significant rapid bony destruction. Physical therapy would potentially
aggravate the neuropathic process and is not indicated during stage 1. While a few
authors have advocated early surgical intervention, there is not enough significant
scientific evidence to recommend surgical management during stage 1 and thus it is
usually reserved for significant deformity and impending skin breakdown.

The Preferred Response to Question # 145 is 1.

99
99
Online 2011 Orthopaedic Self-Assessment Examination by Dr.Dhahirortho
Question 146Figure 146 is the radiograph of a 72-year-old woman with a history of
Parkinson's disease and a multiply revised right total hip arthroplasty with a
constrained implant. She is seen in
the emergency department, reporting pain. Treatment should consist of which of the
following?
1- Closed reduction
2- Open reduction
3- Open reduction with soft-tissue repair
4- Acetabular revision with a constrained implant
5- Acetabular revision with an unconstrained implant

DISCUSSION: The patient has a hip dislocation with a previously placed constrained
acetabular component. The ring around the femoral neck is the locking ring of a
constrained implant that has dissociated. The acetabular component demonstrates
increased vertical inclination and retroversion. The acetabular component malposition
contributed to the dislocation along with the patient's deficient abductor musculature.
The appropriate treatment would be to perform an acetabular revision to improve the
component position along with a constrained liner due to the deficient abductors. A
closed reduction will be extremely difficult to achieve because of the presence of a
constrained liner, whereas an open reduction is not advised because of the persistent
problem of component malposition. The Preferred Response to Question # 146 is 4.

Question 147Figure 147 is an MRI scan of a 72-year-old woman admitted to the hospital
7 days ago with persistent and worsening back pain. A repeat vertebral augmentation
was performed at L2 three days ago. Today she became diaphoretic, reported severe
dyspnea, and collapsed during physical therapy. Examination reveals a pulse of
128/min, blood pressure of 98/55 mm Hg, and temperature of 100 degrees F (37.7
degrees C). Jugular venous distention is noted. What is the most likely complication?

1- Spinal shock
2- Neurogenic shock
3- Hemorrhagic shock
4- Pulmonary embolism
5- Autonomic dysreflexia

100
100
Online 2011 Orthopaedic Self-Assessment Examination by Dr.Dhahirortho
DISCUSSION: The patient has the classic symptoms of a pulmonary embolism. Symptoms
of pulmonary embolism of polymethylmethacrylate (PMMA) following vertebral
augmentation may occur with a delay. A symptomatic pulmonary embolism following
vertebroplasty can occur either by migration of acrylic or the migration of fat and bone
marrow cells. The MRI scan reveals a new superior endplate fracture involving L2. With
this now being the third consecutive vertebral compression fracture in 2 months, one
must be suspicious that these represent pathologic fractures, rather than osteoporosis.
Risk factors for venous thromboembolic disease include increasing age, prolonged
immobility, surgery, trauma, malignancy, pregnancy, estrogenic medications (eg, oral
contraceptive pills, hormone therapy, tamoxifen [Nolvadex]), congestive heart failure,
hyperhomocystinemia, diseases that alter blood viscosity (eg, polycythemia, sickle cell
disease, multiple myeloma), and inherited thrombophilias. In addition to the risk
associated with embolization of PMMA, the patient has been immobile for 7 days and
was ultimately diagnosed with multiple myeloma. The Preferred Response # 147 is 4.

DISCUSSION: Patients with multiple hereditary exostosis (MHE) have an increased risk of
secondary chondrosarcomas in an area of a prior exostosis. This risk is probably 1 in
10,000 MHE patients and typically is a low-grade chondrosarcoma. Mafucci's syndrome is
a different disorder and is associated with hemangiomas. Ollier's patients have multiple
enchondromas. Extra-abdominal desmoids are associated with Gardner's syndrome, and
von Recklinghausen's disease is associated with plexiform neurofibromas.

Question 148Which key factor that induces osteoclastogenesis is secreted by


osteoblasts in response to inflammatory stimuli?
1- Osteoprotegerin (OPG)
2- Tumor necrosis factor (TNF)
3- Insulin growth factor-1 (IGF)
4- Bone morphogenetic protein (BMP)
5- Receptor activator of nuclear factor kappa-B ligand (RANKL)

DISCUSSION: Osteoclasts are derived from cells of the monocyte/macrophage lineage.


They are multinucleated and develop by fusion of mononuclear precursors, a process that
requires receptor activator for nuclear factor kappa-B ligand (RANKL) and macrophage-
colony stimulating factor (M-CSF). RANKL is secreted by osteoblasts in response to
101
101
Online 2011 Orthopaedic Self-Assessment Examination by Dr.Dhahirortho
inflammatory signals and is a key component of inflammation-mediated osteolysis. OPG
binds to and sequesters RANKL, thus inhibiting osteoclast differentiation and activity.
BMP and IGF-1 are potent regulators of osteoblast differentiation and activation. TNF is a
cytokine secreted by macrophages and degranulating platelets infiltrated in the fracture
site and impacts a variety of cells, not osteoclasts. The Preferred Response # 148 is 5.

Question 149A 3-year-old child has the deformity seen in Figures 149a and 149b. In
discussing the condition with the family, it is important to inform them that this
problem is associated with

1- osteogenesis imperfecta.
2- neurofibromatosis.
3- limb-length discrepancy.
4- congenital pseudarthrosis.
5- renal anomalies.

DISCUSSION: The radiographs demonstrate congenital posterior medial bow of the tibia.
It is associated with limb-length discrepancy in the older child and calcaneovalgus foot in
the newborn. The bowing slowly diminishes, although a considerable limb-length
discrepancy can develop (3-8 cm). It is important to differentiate this condition from
anterior lateral bow of the tibia, which is associated with congenital pseudarthrosis of the
tibia and neurofibromatosis. Osteogenesis imperfecta can present with various long-bone
deformities secondary to fracture, but the bone quality in the figure appears normal.
Renal anomalies are not associated with posterior medial or anterior lateral bow of the
tibia. The Preferred Response to Question # 149 is 3.

Question 150Figures 150a and 150b are the MRI scan and biopsy specimen of a 53-year-
old man who has had right knee pain and swelling for the past 9 months. What is the
most likely diagnosis?
1- Liposarcoma
2- Biphasic synovial sarcoma
3- Ganglion cyst
4- Pigmented villonodular synovitis
5- Myxoma

102
102
Online 2011 Orthopaedic Self-Assessment Examination by Dr.Dhahirortho
DISCUSSION: Soft-tissue sarcomas found near joints are very rare. The MRI scan is clearly
not benign fat (lipoma) but could be consistent with any sarcoma, myxoma, or ganglion
cyst. The biopsy specimen, however, is not a cyst or myxoma (generally very acellular).
This is a high-grade liposarcoma because it has high-grade cellular morphology and is not
a biphasic picture. Furthermore, there are lipoblasts in the biopsy specimen. High-grade
liposarcomas may have very little recognizable fat cells on the biopsy specimen. A
biphasic synovial sarcoma has slit-like areas that look almost like glands and other more
solid cellular areas. The name, synovial sarcoma, implies that it is found in the synovium,
but that is not true. It is a misnomer concerning the pathologic appearance. A myxoma
would have this MRI appearance but would be much less cellular on the biopsy specimen.
The Preferred Response to Question # 150 is 1.

Question 151 Figures 151a and 151b are the radiographs of a 15-year-old boy who has
had swelling and knee pain for several weeks. He has pain both at rest and with
activity. What is the next step in management?

1- Through-knee amputation
2- MRI scan of the entire bone as soon as possible
3- Biopsy of the lesion with referral to a tumor specialist if malignant
4- Continued observation with repeated radiographs in 3 months if still painful
5- Symptomatic treatment, including rest and nonsteroidal anti-inflammatory
medication, with follow-up as needed

DISCUSSION: The radiographs reveal a bone-forming tumor with indistinct margins that
most likely represents a malignant sarcoma such as an osteosarcoma. Evaluation requires
a full workup, including an MRI scan of the entire involved bone. Symptomatic treatment
103
103
Online 2011 Orthopaedic Self-Assessment Examination by Dr.Dhahirortho
or continued observation has no role in this treatment. A biopsy should be performed
after the evaluation is complete and preferably by the surgeon that will do the definitive
treatment. Most tumors, even if malignant, can be treated with limb-sparing surgery.
The Preferred Response to Question # 151 is 2.

Question 152Figures 152a and 152b are the radiographs of an otherwise healthy 75-
year-old woman who underwent open reduction and internal fixation of a tibial plateau
fracture 1 year ago. The patient now reports chronic pain that leaves her unable to
walk any more than just about the home and she has great difficulty going up and
down stairs. Laboratory studies show an erythrocyte sedimentation rate of 18 mm/h
(normal up to 20 mm/h). She has no other lower extremity involvement. The valgus
deformity of the knee measures 18 degrees. What is the best option for this patient?

1- Varus osteotomy
2- Knee arthrodesis
3- Standard total knee arthroplasty with hardware removal
4- Removal of hardware and lateral unicondylar arthroplasty
5- Total knee arthroplasty with removal of hardware and a hinged-knee prosthesis

DISCUSSION: The patient is an excellent candidate for a reconstructive procedure, but


this is a difficult procedure for a number of issues and potential problems. The hardware
is lateral and the best option for approaching hardware removal which is needed for
placement is from the lateral side. The lateral approach to the knee has other benefits
including the ability to directly approach lateral ligaments for balancing and to avoid
disruption of the patella blood supply which may have been violated by prior procedures.
The surgeon should prepare for the possibility of needing augments or stems on the
components. The Preferred Response to Question # 152 is 4.
104
104
Online 2011 Orthopaedic Self-Assessment Examination by Dr.Dhahirortho
Question 153 A 31-year-old high school football coach has right medial knee pain that is
made worse with prolonged standing. His knee is minimally painful in the morning but
by the end of the school day, he must sit down. The pain often makes sleeping difficult.
He states that several years ago he underwent a surgical procedure to "clean out" the
cartilage of the knee; however, he only had several months of pain relief. He is noted to
be an athletic male (BMI of less than 30). Knee examination is unremarkable except for
medial joint line pain that is exacerbated with standing and walking. Radiographs,
including a long-leg view, and MRI scans are seen in Figures 153a through 153d. He
wishes to remain active and asks whether he would be a candidate for allograft
meniscus transplantation. You advise him that

1- the current literature does not support allograft meniscus transplantation.


2- allograft meniscus transplantation is a surgical option; however, he is beyond the age
where the procedure will provide much lasting benefit.
3- you would recommend a course of viscosupplementation.
4- based on his age and limb alignment, you would not recommend an allograft
meniscus transplant but would recommend a high tibial osteotomy.
5- based on his age and limb alignment, you would recommend a high tibial osteotomy
and a staged allograft meniscal transplant after the osteotomy has healed.

DISCUSSION: The patient's history, physical findings, and MRI scans indicate that a
complete medial meniscectomy was performed. The meniscus provides an essential
function in dissipating forces to the adjacent articular cartilage. Complete or partial
meniscectomy has been shown to result in more rapid clinical and radiographic arthritis
105
105
Online 2011 Orthopaedic Self-Assessment Examination by Dr.Dhahirortho
than if the meniscus is preserved. Allograft meniscal transplantation has been shown to
be effective in the young patient with an absent meniscus, no or correctable limb
malalignment, and minimal or correctable articular cartilage damage. His age would be
appropriate for an allograft meniscus transplant. Based on the patient's long-leg
radiograph, a valgus-producing high tibial osteotomy would be appropriate but alone
would not address the absent meniscus in this young patient. Viscosupplementation may
provide some temporary relief but is not an
appropriate long-term solution. A staged valgus-producing osteotomy followed by an
allograft meniscus transplant would be the most appropriate treatment. The Preferred
Response to Question # 153 is 5.

Question 154A 45-year-old woman undergoes an uncomplicated total knee


arthroplasty. Nine months later she has not yet returned to work because of pain and
stiffness. Her range of motion is 5 to 80 degrees. She has no instability, is unable to
climb stairs normally, sitting in low chairs is uncomfortable, and she no longer
participates in physical therapy. She has pain with prolonged standing. Radiographs
show a well-aligned, cruciate-retaining implant. Work-up for infection, including joint
aspiration, is negative. What is the next step in management?
1- Dynamic splinting
2- Open release of adhesions
3- Manipulation under anesthesia
4- Revision to a cruciate-sacrificing implant
5- Arthroscopic release of adhesions and manipulation

DISCUSSION: Arthrofibrosis after total knee arthroplasty (TKA) affects 1% of patients.


Ninety degrees of motion allows for most activities, including ascending and descending
stairs. Risk factors for postoperative stiffness include preoperative stiffness, younger age,
posttraumatic arthritis, and multiple prior surgeries. Whether range of motion is affected
by the choice of cruciate retaining versus sacrificing implants is subject to much debate,
but has not been shown to be related to arthrofibrosis following TKA. At earlier time
points, physical therapy, use of dynamic splinting, and manipulation under anesthesia
may be beneficial in restoring motion but at 9 months is unlikely to prove successful.
However, the patient has both pain and motion loss and is 9 months from her original
surgery. Late manipulation may have an increased risk of complications such as fracture
or tendon rupture. Moreover, arthroscopy would allow for lysis of adhesions and
106
106
Online 2011 Orthopaedic Self-Assessment Examination by Dr.Dhahirortho
assessment for other causes of pain and has been shown to be safe and effective
following TKA; however, it does carry the risk of infection. Care must be taken not to
scratch or otherwise damage the implants during surgery. The Preferred Resp # 154 is 5.

Question 155 A 30-year-old accountant and recreational softball player, who is seen at
the end of his baseball season, reports a several month history of pain along the medial
side of his dominant elbow. He cannot identify a specific injury and notes it only hurts
when he throws the ball in from the outfield. Besides the pain, he remarks that his
speed and distance while throwing have diminished considerably. Examination reveals
tenderness along the medial elbow but no weakness or gross instability is found.
Radiographs are normal. Based on the history, what is the most likely diagnosis?
1- Ulnar neuritis
2- Pronator syndrome
3- Medial epicondylitis
4- Medial collateral ligament sprain
5- Varus extension overload

DISCUSSION: Throwing athletes frequently develop medial collateral ligament sprain


related to the repeated valgus stress that occurs on the medial elbow during the
acceleration phase of throwing. This has the effect of not only causing pain, but also
resulting in loss of velocity and distance during the throwing activity. The injury is
generally well tolerated in most activities of daily living and only becomes problematic
during the vigorous, stressful act of throwing. Absence of neurologic signs or symptoms
makes ulnar nerve pathology unlikely. Pronator syndrome causes pain on the volar aspect
of the forearm during resisted forearm pronation and is not associated with the throwing
motion in particular. Valgus extension overload may mimic medial collateral ligament
injury, not varus extension injuries. Medial epicondylitis may be confused with ligament
insufficiency but the examination and a history of pain only while throwing make this an
unlikely diagnosis. The Preferred Response to Question # 155 is 4.

156 A prospective outcome study is performed at a single institution to analyze the


potential differences in treating intertrochanteric hip fractures with a plate/screw
device versus an intramedullary device. No specific randomization is performed
because an equal number of surgeons have preferences for the use of one of these
devices and they are allowed to continue their preferred method. Hip-specific and
107
107
Online 2011 Orthopaedic Self-Assessment Examination by Dr.Dhahirortho
general health-related outcome measures are used, an excellent follow-up rate of 85%
of the patients at 2 years is accomplished, and there appears to be results that favor
the intramedullary device but the confidence intervals are wide. This study would be
considered to carry what level of evidence?

1- I 2- II 3- III 4- IV 5- V

DISCUSSION: This is a prospective comparative study but is not randomized or blinded


and is therefore a Level II therapeutic study. To qualify as Level I, it would need to be a
high-quality randomized trial with narrow confidence intervals regardless of a significant
difference or no difference in outcomes. Level III would be case-control studies or
retrospective comparisons. Level IV is case series and Level V is expert opinion.
The Preferred Response to Question # 156 is 2.

Question 157 Displaced olecranon apophyseal fractures in children are commonly


associated with which of the following?

1- Child abuse
2- Multiple trauma
3- Noonan syndrome
4- Osteogenesis imperfecta
5- Vitamin D deficiency

DISCUSSION: Children with osteogenesis imperfecta often sustain numerous fractures


and the differentiation between this and child abuse can be difficult. Although most
fracture patterns can occur in both, olecranon apophyseal fractures do occur commonly
in children with osteogenesis imperfecta, and are treated most commonly with open
reduction and fixation. Children with osteogenesis imperfecta who have this fracture are
at high risk for a similar fracture on the contralateral side. The fracture is not specifically
associated with child abuse, multiple trauma, or Noonan's syndrome. Vitamin D
deficiency is now being recognized as extremely common in children, especially those
with disabilities, but it is not associated with this fracture pattern.
The Preferred Response to Question # 157 is 4.

108
108
Online 2011 Orthopaedic Self-Assessment Examination by Dr.Dhahirortho
Question 158 A 19-year-old college pitcher reports posterior shoulder discomfort that
started recently with pitching. He is able to throw with normal velocity and control, but
his pain in the early acceleration phase of throwing is getting worse. Examination
reveals symmetric rotator cuff strength and no increased anterior or posterior
translation of either shoulder. He has some discomfort with his shoulder in abduction
and external rotation. Supine range of motion of the right shoulder in 90 degrees of
abduction reveals external rotation to 100 degrees and internal rotation to 25 degrees.
His left shoulder has 95 degrees of external rotation and 45 degrees of internal
rotation. He is not playing the next 2 weeks and requests some exercises that he can do
on his own. Which of the following exercises will most likely improve his shoulder
symptoms?

1- Standard and low rowing exercises


2- Lying on his side with the shoulder abducted 90 degrees, elbow flexed 90 degrees,
and pushing his forearm toward the table
3- Humeral head depressions while holding a ball against a wall
4- Scapular `punches` in many directions
5- Putting a rolled towel between his shoulder blades while lying supine and having a
teammate push posteriorly on the shoulders

DISCUSSION: The patient has a glenohumeral internal rotation deficit of 20 degrees.


Posterior capsular stretching would be beneficial. A sleeper stretch is a common way for
patients to stretch the posterior capsule on their own. It involves lying on the side with
the shoulder abducted 90 degrees and the elbow flexed 90 degrees and trying to push
the forearm toward the table. Closed-chain rotator cuff exercises, such as humeral head
depressions while holding a ball against a wall, pectoralis minor stretching, such as lying
on a rolled towel and pushing posteriorly on the shoulders, scapular protraction, such as
punches, and scapular retraction, such as row exercises, can all be helpful for the disabled
throwing shoulder, but they will not restore the decreased internal rotation.

The Preferred Response to Question # 158 is 2.

109
109
Online 2011 Orthopaedic Self-Assessment Examination by Dr.Dhahirortho
Question 159 Figures 159a and 159b are the radiographs of a 40-year-old woman who
sustained a twisting injury to her lower extremity. What additional information or
studies are important in determining treatment options?

1- Full-length tibia-fibula radiographs


2- Inability to bear weight
3- History of recurrent ankle sprains
4- Presence or absence of medial tenderness
5- MRI scan

DISCUSSION: The radiographs reveal a medial


ankle injury with a widened medial clear
space. No fibula fracture is visualized on this view; therefore, full-length radiographs
looking for a proximal fibula fracture are required to determine treatment. The presence
or absence of medial tenderness has been shown to not be a good predictor of unstable
injuries. A history of previous injuries or ankle instability is typically lateral instability,
which would not present with this radiographic appearance. An MRI scan can be used to
evaluate subtle syndesmotic injuries, but there is a clear widening of the medial clear
space in this case. The inability to bear weight is not helpful in determining the treatment
options. The Preferred Response to Question # 159 is 1.

Question 160 An athletic 35-year-old man participates in competitive sports for


recreation. During a weight-lifting workout, he described striding forward with his left
foot on his flexed right hip. He heard an audible pop and immediately experienced pain
in his right hip. Since the injury, he has had difficulty with movement and pain in the
right hip. After a week of continued symptoms, he consulted an orthopaedic surgeon
who ordered an MRI scan which indicated a partial tear of the right tensor fascia lata
and physical therapy was recommended. Specific instructions for the therapist in the
initial phase of rehabilitation to prevent hip arthrofibrosis, yet not injure the muscle
further, should include which of the following?
1- Immediate active hip internal and external rotation, active hip flexion and extension,
with hip strengthening beginning in 4 weeks
2- Immediate active hip internal and external rotation, active hip flexion, and passive
hip extension with hip strengthening beginning in 4 weeks

110
110
Online 2011 Orthopaedic Self-Assessment Examination by Dr.Dhahirortho
3- Immediate active hip internal and external rotation, passive hip flexion and
extension, with hip strengthening beginning in 4 weeks
4- Immediate passive hip flexion and internal rotation, active hip external rotation and
hip extension, with hip strengthening beginning in 4 weeks
5- Immediate active hip internal and external rotation, and active hip flexion and
extension with immediate hip strengthening

DISCUSSION: The patient injured his tensor fascia lata based on his symptoms, the
mechanism of injury, and the MRI findings. Although various authors have described
different functions of the tensor fascia lata, it has generally been agreed on that it
functions as a hip flexor, hip internal rotator, and to a lesser degree, hip external rotator.
Initial therapy to facilitate healing of a muscle begins with ice, compression, and initial
passive range of motion. After this initial phase, active motion can commence, followed
by strengthening and functional rehabilitation. Initial passive range of motion of the
injured tensor fascia lata would include hip internal rotation and flexion. Answer choice 1
is incorrect because active hip internal rotation and flexion would potentially injure the
tensor fascia lata before it had healed. Answer choice 2 is incorrect because immediate
active hip flexion would injure the tensor fascia lata. Answer choice 3 is incorrect because
active hip internal rotation would injure the tensor fascia lata before healing. Answer
choice 4 is correct for the previously mentioned explanation. Answer choice 5 is incorrect
because active hip flexion and internal rotation would injure the tensor fascia lata.
The Preferred Response to Question # 160 is 4.

Question 161Figures 161a and 161b are the AP and lateral radiographs of a 10-year-old
boy with a painful left distal tibia. An MRI scan is shown in Figure 161c. Figures 161d
and 161e show biopsy specimens. What is the most likely diagnosis?

1- Trauma
2- Osteosarcoma
3- Osteomyelitis
4- Ewing's sarcoma
5- Eosinophilic granuloma

111
111
Online 2011 Orthopaedic Self-Assessment Examination by Dr.Dhahirortho
DISCUSSION: The patient has a distal tibial destructive lesion with a Codman's triangle
and an extensive soft-tissue mass as seen on the radiographs, which is an osteosarcoma.
The pathology shows a malignant osseous-forming lesion. Eosinophilic granuloma would
be characterized by a lytic lesion with variable periosteal response, but rarely ever has a
soft-tissue mass. The pathology would have large histiocytes and scattered eosinophils
with a variable amount of acute inflammatory cells. Osteomyelitis would not present with
a soft-tissue mass, although abcesses are rarely seen. The pathology should not show
malignant cells, but rather acute and chronic inflammatory cells and variable amounts of
dead bone. Trauma such as stress fractures can be difficult to assess on plain radiographs,
but the MRI scan should show a fracture line, best seen on a T1-weighted image. The
amount of soft-tissue mass seen on the MRI scan would not be seen in a fracture, nor
would the malignant cells be seen on biopsy. Ewing's sarcoma is typical of this boy's age
and can be seen in the metadiaphysis of the distal tibia, but the biopsy should have
shown malignant small blue round cells with indistinct cytoplasm. Osteoid seen on the
biopsy specimen can be seen in small areas of Ewing's, where reactive bone occurs but
should not be the prominent feature as it is in this case. The Preferred Respon# 161 is 2.

Question 162 A 37-year-old woman has right-hand numbness and tingling. Based on the
history and examination, carpal tunnel syndrome is suspected, and electrodiagnostic
tests also point to the same diagnosis. The patient has worn night splints for the last 8
weeks with continued persistent symptoms. What is the next most appropriate step in
management?
1- Continue the night splinting for 1 additional month.
2- Continue the night splinting for 3 more months.
3- Switch to full-time splinting and reevaluate in 1 month.
4- Switch to full-time splinting for 3 more months.
5- Perform carpal tunnel release.

DISCUSSION: Various nonsurgical management options exist for carpal tunnel syndrome
(local and oral steroids, splinting, and ultrasound). All effective or potentially effective
nonsurgical forms of management have measureable effects on symptoms within 2 to 7
weeks of the initiation of treatment. If a treatment is not effective within that time
frame, a different treatment option should be chosen. In this case, continued splinting is
unlikely to improve symptoms and steroid injection or surgery is indicated.
The Preferred Response to Question # 162 is 5.
112
112
Online 2011 Orthopaedic Self-Assessment Examination by Dr.Dhahirortho
Question 163Figures 163a through 163c show the radiograph and MRI scans of a 45-
year-old woman with severe right arm pain. She has had symptoms for 6 months
without resolution despite multiple nonsurgical treatments. Examination reveals
weakness in the right triceps and wrist flexors with decreased sensation in the middle
finger and a positive Spurling's sign. What is the most appropriate treatment for the
patient's symptoms?

1- Posterior laminoplasty
2- Posterior cervical foraminotomy
3- Anterior cervical foraminotomy
4- Anterior cervical diskectomy and arthrodesis
5- Anterior corpectomy and arthrodesis

DISCUSSION: The patient has symptoms and signs of cervical radiculopathy despite a long
course of nonsurgical management. Therefore, surgical decompression is indicated and is
best performed through an anterior cervical diskectomy and arthrodesis. Single level
anterior cervical diskectomy and arthrodesis have been shown to produce significant
improvements in arm pain and neurologic function. Anterior cervical foraminotomy,
while reported, has insufficient data to support its use and it places the vertebral artery
at significant risk. Posterior cervical foraminotomy is contraindicated given the ventral
spinal cord compression; foraminotomy places the patient at risk for spinal cord injury.
The patient has one-level cervical disease, therefore a corpectomy is unnecessary.
Posterior laminoplasty is used to treat myelopathy, not radiculopathy.
The Preferred Response to Question # 163 is 4.

113
113
Online 2011 Orthopaedic Self-Assessment Examination by Dr.Dhahirortho
Question 164 A 5-month-old girl sustained an isolated midshaft left femur fracture
when her father tripped and fell while carrying her. She has no other injuries. In
addition to verifying that this was not a case of child abuse, treatment should consist of
1- application of a Pavlik harness.
2- application of a one-and-one-half spica cast.
3- flexible intramedullary nailing.
4- percutaneous submuscular plating.
5- open reduction and internal fixation with a locked plating construct.
DISCUSSION: Femur fractures in children who are not yet walking are rare, but they do
occur. Any child who is not yet walking who sustains a femur fracture should be
considered a victim of abuse until demonstrated otherwise. However, occasionally femur
fractures that are not related to abuse do occur in this patient population. Femur
fractures in the prewalking child heal reliably and rapidly; immobilization in a position
that minimizes deforming forces of surrounding muscles yields comfort and allows for
simpler maintenance of alignment. Pavlik harnesses are well tolerated in children
younger than 6 months of age, and allow for easy diapering for parents. Spica casting is a
reasonable alternative treatment, but diapering is more difficult than with a Pavlik
harness. Surgical methods of fixation for femur fractures in children younger than 6
months of age are rarely used, if ever. The Preferred Response to Question # 164 is 1.

Question 165 A 43-year-old man who works as a plumber has a painful stiff elbow in his
dominant arm. He notes that while he recalls no single event of injury, he thinks the
many years of pulling wrenches and soldering pipes have resulted in his problem. He
reports that he has pain with any motion in bending his arm and can no longer
straighten his elbow. Examination reveals generalized swelling of the elbow, both
medial and lateral with a range of motion that lacks 45 degrees of extension and flexes
only to 110 degrees. Pronation and supination are also limited to 45 degrees. Audible
crepitus is perceived but there is no instability. Radiographs reveal advanced
osteoarthritis at the radiocapitellar and ulnohumeral joints with complete loss of
articular cartilage. What is the most appropriate initial treatment option?
1- Elbow fusion
2- Radial head resection
3- Total elbow arthroplasty
4- Osteophyte resection and capsular release
5- Physical therapy with dynamic extension and flexion splints
114
114
Online 2011 Orthopaedic Self-Assessment Examination by Dr.Dhahirortho
DISCUSSION: Osteoarthritis of the elbow is more common in the middle-age laborer such
as this plumber, whereas rheumatoid arthritis is more common in older females.
Treatment must respect the physical demands of the patient while trying to preserve
joint motion and function with tolerable symptoms. Osteophyte resection and capsular
release have offered many patients significant improvement in their symptoms while
allowing them to return to most activities. The osteophyte resection and releases can be
done effectively by an open or arthroscopic approach. Whereas total elbow arthroplasty
would likely result in better and more thorough pain relief, it would not tolerate the
occupational demands of this individual. There is no role for physical therapy initially in
the face of advanced, painful arthritis associated with long-standing fixed joint
contractures. Elbow fusion results in severe loss of function and its indication is rare and
usually considered in the face of unmanageable sepsis. Radial head resection may
improve symptoms related to the radial capitellar arthritis but would not improve range
of motion or end range impingement pain. Also, radial head resection should be avoided
in heavy laborers with elbow arthritis because it would lead to increased loads across the
arthritic ulnohumeral joint. The Preferred Response to Question # 165 is 4.

Question 166 At what age does the lateral epicondyle normally ossify in males?
1- 2 to 4 years 2- 5 to 6 years 3- 7 to 8 years 4- 9 to 11 years 5- 12 to 14 years

DISCUSSION: The lateral epicondylar epiphysis is the last to ossify in the elbow at age 12
to 14 years in males. The first secondary ossification center to ossify is the capitellum,
which ossifies during the first 6 months of life. Next is the radial head, ossifying between
age 3 and 6 years. The medial epicondyle appears between 5 and 7 years; the trochlea
and olecranon at 8 and 10 years, respectively. In females, the appearance of ossification
centers is about a year earlier than males. The Preferred Response to Question # 166 is 5.

Question 167 Which of the following factors is predictive of a poor patient outcome
after antegrade intramedullary nailing of a femoral shaft fracture?

1- Gait assessment
2- Manual muscle testing
3- "Time to tire" walking trial
4- Visual analog pain scale for hip
5- Radiographic appearance of fracture
115
115
Online 2011 Orthopaedic Self-Assessment Examination by Dr.Dhahirortho
DISCUSSION: Patients who have undergone an antegrade intramedullary nailing of a
femur fracture will commonly have hip abductor weakness and a Trendelenburg gait
following surgery. Patients with a short stride length and an ipsilateral trunk lean are
likely to be dissatisfied with their outcomes. They will lean their trunk toward their
affected side as a result of hip abductor weakness. This may result from abductor damage
during surgery, prominent hardware, and/or inadequate rehabilitation. Archdeacon and
associates examined eight nonconsecutive femoral shaft fractures treated with an
antegrade nail. All patients were enrolled in a standardized postoperative outpatient
protocol as described in the article by Paterno and associates. The authors used hip
kinematics (hip and trunk coronal plane motion) and hip kinetics (hip abductor moment)
and found that patients improved over time. They also found that a patient reported a
dysfunction score at about 2 years postoperative correlated with the presence of an
abnormal ipsilateral trunk lean at the time of initial independent ambulation as well as
ambulation after complete healing had occurred. The authors commented that the
clinical assessment of a shortened stride length and a lateral trunk lean may be predictive
of a poorer functional outcome, and can be used at follow-up visits to assess dynamic hip
abductor function. "Time to tire" is not an existing outcomes test. Visual analog hip pain,
manual muscle testing, and fracture consolidation are not predictive of outcome.
The Preferred Response to Question # 167 is 1.

Question 168 Figures 168a and 168b are the radiograph and CT scan of a 15-year-old
patient who reports a 6-month history of intermittent ankle pain that worsens with
activity. The pain was temporarily relieved with 8 weeks in a walking cast. What is the
next most appropriate step in
management?

1- Repeat use of the walking cast


2- Surgical resection
3- Custom foot orthotic
4- Non-weight-bearing casting
5- Subtalar arthrodesis

DISCUSSION: Pain due to tarsal coalition that recurs after appropriate nonsurgical
management is best treated surgically. In this patient, the talocalcaneal coalition is
fibrocartilaginous, occupies less than 50% of the joint surface, and there are no
116
116
Online 2011 Orthopaedic Self-Assessment Examination by Dr.Dhahirortho
degenerative changes. Therefore, surgical resection is indicated. Even if initially effective,
nonsurgical management such as another walking cast, restricted weight bearing, and
orthotics are rarely effective for recurrent pain. Subtalar arthrodesis is indicated when
the coalition comprises greater than 50% of the talocalcaneal joint or degenerative
arthritis is present. Nonsurgical management such as custom foot orthoses, activity
restrictions, and over-the-counter removable boots can be helpful for the initial
treatment of mild pain, but a walking cast for 4 to 6 weeks has a very high rate of
successful symptom relief of any magnitude and is the most appropriate treatment for
the first presentation of pain. The Preferred Response to Question # 168 is 2.

Question 169 Figures 169a through 169c show the radiograph and MRI scans of a 74-
year-old woman who has had back and bilateral leg pain for the past 6 months.
Nonsurgical management has failed to provide relief. What is the best option for
surgical treatment?

1- Posterior decompression
2- Posterior interbody arthrodesis
3- Posterior decompression and in situ arthrodesis
4- Posterior decompression and instrumented arthrodesis
5- Anterior and posterior arthrodesis

DISCUSSION: The patient has symptoms of lumbar spinal stenosis and radiographic
evidence of a grade I degenerative spondylolisthesis at L4-5. Surgical treatment has been
shown to provide better clinical outcomes than nonsurgical management. Treatment for
spondylolisthesis remains somewhat controversial but posterior lumbar instrumented
arthrodesis is best supported in the literature. Decompression alone places the patient at
risk for recurrent stenosis and progression of deformity. Noninstrumented arthrodesis for
117
117
Online 2011 Orthopaedic Self-Assessment Examination by Dr.Dhahirortho
this condition results in high rates of nonunion and worsened long-term outcomes. There
is insufficient evidence to support the role for interbody arthrodesis (either through an
anterior or posterior approach) compared with posterior decompression and arthrodesis.
The Preferred Response to Question # 169 is 4.

Question 170 Figures 170a through 170d show the radiograph, axial MRI scans, and
sagittal MRI scan of a 60-year-old man who sustained a seizure 12 weeks ago. Since
that time he has had shoulder pain and is unable to use his arm. Examination reveals
pain with any motion and he has no active or passive external rotation of the arm.
What is the most appropriate next step in management?

1-Sling immobilization with gentle passive range of motion starting in 2 weeks


2-Humeral head hemiarthroplasty with subscapularis repair
3-Arthroscopic posterior labral repair
4-Open reduction and transfer of the lesser tuberosity into the defect
5-Closed reduction and early active range of motion

DISCUSSION: Humeral head arthroplasty with subscapularis repair is the most reliable
way to fill this large anterior humeral head defect and achieve joint stability. He has a
chronic locked posterior dislocation with a large reverse Hill-Sachs deformity and a
displaced lesser tuberosity fracture. It has likely been dislocated for 12 weeks since his
seizure. At this point, sling immobilization is not appropriate because this will not provide
reduction of the joint. Closed reduction should not be attempted 12 weeks following the
injury because it is highly unlikely to succeed. Arthroscopic posterior labral repair will not
be successful with a large reverse Hill-Sachs deformity. Transfer of the lesser tuberosity
into the defect may be successful for smaller lesions, but will be unlikely to provide
enough bone to fill this large defect. In a younger patient with similar findings, an
osteochondral allograft to restore humeral head deficiency with subscapularis repair is an
appropriate option. The Preferred Response to Question # 170 is 2.

118
118
Online 2011 Orthopaedic Self-Assessment Examination by Dr.Dhahirortho
Question 171 Postoperative radiographs following a total hip arthroplasty performed
through a posterior approach demonstrate that the cup has been placed in about 35
degrees of abduction. Compared with the ideal placement of 45 degrees of abduction,
this more horizontal cup placement is likely to give what functional result?
1- Increased dislocation rate
2- Increased range of motion
3- Increased risk of iliopsoas impingement when using a large metal head
4- Elevated risk of squeaking in ceramic-ceramic bearing surfaces
5- No adverse effect on wear
DISCUSSION: Decreased abduction angles have no adverse effect on wear rates. The
optimal placement of the acetabular cup is accepted to be 45 degrees of abduction and
20 degrees of anteversion. However, outliers to these positions are common. Increased
abduction angle has been shown to markedly increase wear rates. Decreased anteversion
may contribute to posterior hip dislocation, whereas increased anteversion may increase
impingement. However, placing the cup at 35 degrees of abduction may decrease range
of motion, especially if there is insufficient anteversion of the femoral and acetabular
components. Squeaking in ceramic hips is associated with more vertical rather than
horizontal cup placement. The Preferred Response to Question # 171 is 5.

Question 172 Randomized controlled trials can be designed in several ways. Which of
the following study designs refers to a randomized controlled trial in which two
interventions are compared within the same study group?

1- Parallel 2- Case control 3- Case series 4- Factorial 5- Crossover

DISCUSSION: A factorial randomized control trial design is more easily represented in a


two by two table. Practically, patients are randomized to either treatment A and B,
treatment A or control, treatment B or control, or no treatment. The strength of this trial
design is that two interventions can be assessed with the same study population. Also,
any interaction between the treatments can be determined (for example, does treatment
A work differentially when combined with treatment B). The parallel design trial is the
simplest and most classic design for a randomized controlled trial. In this trial design,
participants are randomized to two or more groups of different treatments and each
group is exposed to a different intervention and only that intervention. In the crossover
design trial, both groups receive both interventions over a randomly allocated time
119
119
Online 2011 Orthopaedic Self-Assessment Examination by Dr.Dhahirortho
period. Group A can receive the treatment, and after a suitable washout period, can
receive the placebo. Group B can receive the placebo and later can receive the
treatment; this produces within-participant comparisons. The crossover trial design has a
limited role in surgical interventions because it is difficult or impossible for patients to
receive both treatment interventions, such as plate and nail fixation, or a cemented
versus a cementless total hip arthroplasty. Case control and case series are not
randomized trials, but observational studies. The Preferred Response # 172 is 4.

Question 173 A 46-year-old man sustains an injury to his left index finger while cleaning
his paint gun with paint thinner. Examination reveals a small puncture wound at the
pulp. The finger is swollen. What is the next most appropriate step in management?

1- Elevation and observation


2- Surgical debridement and lavage
3- Infiltration with corticosteroids
4- Infiltration with a neutralizing agent
5- Administration of antibiotics

DISCUSSION: High-pressure injection injuries are associated with a high risk of


amputation. The risk of amputation is highest with organic solvents. The presence of
infection and the use of steroids do not impact the amputation rate. Amputation risk is
lower if surgical debridement is performed within 6 hours. Elevation and observation
would delay necessary care. Neutralizing agents may be used in specific situations, such
as hydrofluoric acid exposure or chemotherapeutic agent extravasation, but in high
pressure paint thinner injection, the best outcome is achieved through early surgical
lavage. The Preferred Response to Question # 173 is 2.

Question 174Figures 174a through 174c are the MRI scans of a 16-year-old football
player who dislocated his dominant left shoulder 3 weeks ago while landing on his
outstretched arm. The dislocation was reduced in the emergency department. He has
since had two episodes where he felt like his shoulder slipped partially out of place.
Which of the following statements to the athlete and his parents is most accurate
regarding treatment options?

120
120
Online 2011 Orthopaedic Self-Assessment Examination by Dr.Dhahirortho

1- Physical therapy should allow him to return to football with recurrent dislocations
unlikely.
2- Immobilization of his shoulder in an external rotation brace will eliminate the chance
of further dislocations.
3- Arthroscopic capsulolabral repair is a reasonable option if he wishes to undergo this
procedure, despite this being a first-time dislocation.
4- Arthroscopic capsular and labral repair will likely fail in this situation.
5- Open repair definitely provides a better outcome.

DISCUSSION: Arthroscopic capsulolabral repair is a reasonable option despite this being a


first-time dislocation. The patient has had recurrent instability episodes with two
subluxations since his dislocation. Outcome studies have shown up to 90% recurrent
instability rates in young, active populations. Capsulolabral repair has the best chance to
reduce the risk of recurrent instability. Physical therapy is unlikely to significantly reduce
the high likelihood of recurrence. While there is controversy regarding immobilization in
internal or external rotation, studies have shown that immobilization may not reduce the
risk of recurrent instability. While older studies did show that open repairs had lower
recurrence rates than arthroscopic repairs, more recent studies have shown similar rates
for arthroscopic capsulolabral plication with modern suture anchor techniques and no
glenoid bone loss or engaging Hill-Sachs lesion. The Preferred Response # 174 is 3.

Question 175 When performing a right proximal humeral hemiarthroplasty, the relative
placements of the lesser tuberosity relative to the biceps tendon is best depicted, in
Figure 175, by the
1- lesser tuberosity at A, biceps at B.
2- lesser tuberosity at B, biceps at C.
3- lesser tuberosity at C, biceps at B.
4- lesser tuberosity at A, biceps at C.
5- lesser tuberosity at C, biceps at D.

121
121
Online 2011 Orthopaedic Self-Assessment Examination by Dr.Dhahirortho
DISCUSSION: The lesser tuberosity should be placed at position A, and the biceps tendon
at position B. One of the most common errors during proximal humeral arthroplasty is
the use of the lateral keel of the prosthesis as the landmark, around which the
tuberosities are reconstructed. If this is done, the anterior soft tissue/bone element is
stretched, while the posterior soft tissue/bone element is lax, with a resultant loss of
external rotation of the arm. The biceps should be used as the proper landmark for
tuberosity reconstruction and in its absence, the anterior aspect of the prosthesis, where
the bicipital groove would have been, should be used as the central juncture of tuberosity
reconstruction. The upper border of the pectoralis is best used to gauge appropriate
height but knowing that the biceps tendon runs directly underneath the tendon insertion
can also aid in estimating the proper location. The Preferred Response # 175 is 1.

Question 176 Which of the following rehabilitation techniques is appropriate for initial
nonsurgical management of an isolated grade 2 posterior cruciate ligament injury?
1- Immobilization in full extension for 4 weeks
2- Immobilization in 30 degrees of flexion for 4 weeks
3- Relative protection for 10 to 14 days, then range of motion with progressive
plyometric exercises
4- Relative protection for 10 to 14 days, then range of motion with gentle open-chain
hamstring strengthening
5- Relative protection for 10 to 14 days, then range of motion with gentle closed-chain
quadriceps strengthening

DISCUSSION: Treatment should consist of relative protection for 10 to 14 days followed


by early range of motion and gentle closed-chain quadriceps strengthening. Isolated
grade 1 and grade 2 posterior cruciate ligament injuries can be successfully managed
nonsurgically. Progression to global knee strengthening can begin 4 to 6 weeks after the
injury, with return to functional activity when full range of motion and strength is
established. Plyometric exercises involve rapid alteration of contraction and loading of a
muscle and should not be used in the early rehabilitation of a ligament injury of the knee
because it risks further injury to the ligament. Hamstring strengthening should be
avoided until the ligament has healed (4to 6 weeks) because the posterior force on the
tibia will stress the injured posterior cruciate ligament. Immobilization may be used for a
short time to allow swelling and pain to subside, but early range of motion is preferred to
avoid unnecessary stiffness following the stable injury. The Preferred Respons # 176 is 5.
122
122
Online 2011 Orthopaedic Self-Assessment Examination by Dr.Dhahirortho
Question 177 Figure 177 is an intra-articular photograph taken while viewing from the
anterior superior portal during arthroscopy of a right shoulder. Which of the following
findings identified at the time of surgery would be the most predictive for recurrence
following arthroscopic repair of the demonstrated pathology?
1- Nonengaging Hill-Sachs deformity
2- Intra-articular loose body
3- Anterior glenoid bone deficiency of 35%
4- Subacromial bursitis
5- 10% partial-thickness, articular side tear of the supraspinatous

DISCUSSION: Anterior glenoid bone deficiency of 35% is most predictive of recurrence.


Figure 177 shows an acute tear of the anterior inferior glenoid labrum consistent with a
Bankart lesion. It has been clearly shown that there is a direct relationship between
failure (ie, recurrent dislocation) of arthroscopic Bankart repair and anterior glenoid bone
loss. Anterior glenoid bone loss of greater than 25% in the setting of anterior
glenohumeral instability is a relative contraindication to performing arthroscopic
stabilization and instead is an indication to perform a bony glenoid augmentation
procedure to address the articular arc deficit. Therefore, an anterior bony defect of 35%
is the most predictive finding at the time of surgery for recurrent dislocation. An engaging
Hill-Sachs deformity has a significant effect on the rate of redislocation, but a
nonengaging one should not. An intra-articular loose body, subacromial bursitis, and a
partial-thickness articular-sided supraspinatous tear should not lead to an increased risk
of recurrent dislocation following Bankart repair. The Preferred Response # 177 is 3.

Question 178 A 12-year-old boy has severe left hip pain that is worse at night and
dramatically improves with the use of nonsteroidal anti-inflammatory drugs. A
radiograph and CT scan are seen in Figures 178a and 178b. Which of the following
options is associated with the most rapid resolution of symptoms with the least long-
term morbidity?
1- Steroid injection
2- Radiofrequency ablation
3- En bloc excision with osteoarticular allograft
4- Continued use of parenteral naproxen sodium 500
mg bid
5- Open curettage of the lesion with careful dislocation of the femoral head
123
123
Online 2011 Orthopaedic Self-Assessment Examination by Dr.Dhahirortho
DISCUSSION: These studies are characteristic of an osteoid osteoma. Radiofrequency
ablation is the least invasive and highly successful procedure for osteoid osteomas.
Ninety-five percent of lesions are destroyed completely with one procedure. Open
curettage or en bloc excisions are associated with significant late joint morbidity. Steroid
injections have been reported as successful for unicameral bone cysts and eosinophilic
granulomas, but not osteoid osteomas. The Preferred Response to Question # 178 is 2.

Question 179 A 56-year-old woman undergoes an arthroscopic rotator cuff repair for a
two-tendon retracted tear (supraspinatus and infraspinatus), requiring the use of four
suture anchors placed in a double row technique. At her 1 month follow-up visit, what
is the appropriate recommendation for her continued rehabilitation program?
1- Initiate isometric external rotation strengthening and continue passive range of
motion.
2- Initiate eccentric supraspinatus strengthening and continue passive range of motion.
3- Initiate light resistance training to minimize atrophy and continue passive range of
motion.
4- Continue passive range of motion and initiate concentric deltoid strengthening.
5- Continue passive range of motion with no active strengthening of the shoulder
muscles.

DISCUSSION: Regardless of the technique of rotator cuff repair, the biology of tendon
healing remains the same. Therefore, the repaired muscle tendon(s) must be protected
from stress for a minimum of 6 weeks and more likely 8 weeks in a large two-tendon tear
such as this patient had repaired. Therefore, at the 1 month follow-up visit, the patient
should continue strict passive motion exercises and should perform no strengthening
activities. Deltoid strengthening cannot be isolated from rotator cuff strengthening;
therefore, deltoid strengthening is inappropriate as well. Because the infraspinatus is the
primary shoulder external rotator, it should not be strengthened for 6 to 8 weeks.
Supraspinatus strengthening at this time frame would likely ensure its disruption and
result in failure of the surgery. Any resistance training at 1 month from surgery would
likely result in tendon failure at the tendon-bone interface. The obligatory need to
protect the muscles during healing will predictably result in atrophy but it is easier to
strengthen healed muscles than it is to strengthen muscle/tendon units that have failed
to heal. The Preferred Response to Question # 179 is 5.

124
124
Online 2011 Orthopaedic Self-Assessment Examination by Dr.Dhahirortho
Question 180 A 54-year-old woman who has a history of undergoing trapezium excision
with ligament reconstruction and tendon interposition using the entire flexor carpi
radialis performed by another surgeon, now reports left basilar thumb pain.
Examination reveals pain and subluxation of the carpometacarpal joint with axial
loading. The metacarpophalangeal joint hyperextends to 60 degrees, but radiographs
show intact joint space. What is the best option to improve function?
1- Bracing with a hand-based thumb spica splint
2- Pinning of the carpometacarpal joint
3- Pinning of the carpometacarpal and metacarpophalangeal joints
4- Carpometacarpal revision stabilization
5- Carpometacarpal revision stabilization and metacarpophalangeal joint fusion

DISCUSSION: The patient previously underwent ligament reconstruction and tendon


interposition. However, the previous surgeon failed to address metacarpophalangeal
joint hyperextension, which leads to adduction contracture and collapse of the basilar
joint. With the basilar joint causing pain and instability, repeat ligament reconstruction
should be performed. Splinting alone is unlikely to resolve instability problems. Because
the flexor carpi radialis was used, the next option is to use the abductor pollicis longus.
Additionally, the severe metacarpophalangeal joint hyperextension should be corrected
by fusion. Simple pinning is unlikely to provide long-term stability when this degree of
hyperextension exists. The Preferred Response to Question # 180 is 5.

Question 181 A 16-year-old boy has had knee pain for the past 6 months, and activity
restrictions have not provided relief. An MRI scan reveals a stable 1.5 cm by 1 cm
osteochondritis dissecans on the weight-bearing surface of the lateral femoral condyle.
What is the best course of treatment?
1- Continued activity restrictions for 6 more months or until asymptomatic
2- An aggressive physical therapy program that includes closed chain quadriceps
strengthening
3- Arthroscopic drilling of the subchondral bone
4- Open debridement and screw fixation
5- Osteochondral autograft transplant procedure

DISCUSSION: As a child approaches skeletal maturity, osteochondritis dissecans lesions


are unlikely to heal with continued nonsurgical management. Drilling of the lesion has a
125
125
Online 2011 Orthopaedic Self-Assessment Examination by Dr.Dhahirortho
high success rate. The lesion is stable and an open repair or osteochondral transplant is
not needed. The Preferred Response to Question # 181 is 3.
Question 182 Paget's disease of bone is considered an osteoclastic abnormality
resulting in which of the following?
1- Decreased vascularity of bone
2- Decreased osteoblast bone formation
3- Decreased resorption of bone by osteoclasts
4- Focally increased remodeling of bone
5- Sclerotic bone replacing normal marrow

DISCUSSION: Paget's disease of bone, first described in 1877, is an osteoclastic


abnormality marked by focally increased skeletal remodeling within the axial or
appendicular skeleton. There is an initial wave of osteoclast-mediated bone resorption,
followed by the second phase of disorganized skeletal repair. This process leads to
excessively disorganized woven bone and lamellar bone, characterized by osteosclerosis
and hyperostosis, respectively, and results in the characteristic findings of cement lines
seen histologically. The disorganized bone is weaker and prone to fractures. The final
phase of the disease is the quiescent phase in which there is little bone turnover. Because
of the increased bone remodeling, there is usually an associated increased vascularity
which should be taken in account when surgery is performed. There is no bony
replacement of the bone marrow. The Preferred Response to Question # 182 is 4.

Question 183Figures 183a and 183b are the radiographs of an otherwise healthy 62-
year-old man with a history of a total knee arthroplasty followed 1 year later by a
periprosthetic fracture treated with open reduction and internal fixation. The surgery
was complicated by multiple wound infections with a sensitive organism. He eventually
had hardware and implant removal and placement of an antibiotic spacer that was
subsequently removed. After a full course of antibiotics, retesting reveals persistent
infection and he is referred for further treatment. His subsequent treatment should be
1- knee fusion.
2- above-knee amputation.
3- antibiotic suppression.
4- arthroscopic irrigation and debridement.
5- repeat debridement and placement of an
antibiotic spacer.
126
126
Online 2011 Orthopaedic Self-Assessment Examination by Dr.Dhahirortho
DISCUSSION: Two-stage resection with placement of an antibiotic impregnated spacer
followed by reimplantation has been demonstrated to have success rates as high as 80%
and has become the standard treatment for an infected total joint arthroplasty in the
United States. Failure to eradicate the infection can be due to the virulence or drug
resistance of the organism, the appropriateness of the antibiotic selection, or the
adequacy of the debridement. Retained metal fragments, cement, or devitalized bone
can result in failure to clear the infection. Special attention should be made to the patella
because the exposure for a total knee arthroplasty can remove the majority of its blood
supply. A lateral release (or lateral dissection as in this case) can compromise the primary
remaining vessel to the patella (the superior lateral genicular) and result in osteonecrosis.
In the setting of infection, the devitalized patella may become a large sequestrum and
patellectomy should be considered. Antibiotic suppression should be used rarely and
would not be a viable option in an otherwise healthy 62-year-old that would require
decades of treatment. Above-knee amputation is a last resort, and in most situations at
least a second attempt at two-stage resection and reimplantation should be attempted
first. Knee fusion would not be indicated until the infection was eradicated and is also
considered a last resort. Arthroscopic irrigation and debridement would not allow for
adequate debridement of the joint and should not be used in the treatment of an
infected arthroplasty. The Preferred Response to Question # 183 is 5.

Question 184 Figures 184a and 184b are the weight-bearing radiographs of a 19-year-
old college baseball player who underwent surgery 4 months ago for an unstable ankle
fracture sustained while sliding into a base. Figure 184c is a CT scan of the injured side
and Figure 184d is the normal uninjured side. He now reports medial ankle pain and
"rolling inward" sensations of the ankle. are seen in. Based on these findings, what is
the most appropriate treatment?

127
127
Online 2011 Orthopaedic Self-Assessment Examination by Dr.Dhahirortho
1- Revision open reduction and internal fixation of syndesmosis
2- Removal of syndesmosis screws
3- Multiplanar fibular osteotomy
4- Ankle arthroscopy and debridement
5- Deep deltoid ligament repair

DISCUSSION: The patient has a malalignment of the syndesmosis with persistent


widening, which is best managed by revision open reduction and internal fixation. The
anatomic location of the deep deltoid ligament is such that a true repair is not feasible.
Although syndesmosis screws are frequently removed in competitive athletes, screw
removal alone will not address the widening nor will ankle arthroscopy. Multiplanar
osteotomy of the fibula is indicated in the instance of fibular malunion. In this case,
fibular length and talofibular symmetry are anatomic. The Preferred Respons# 184 is 1.

Question 185 Since the adoption by the American Academy of Orthopaedic Surgeons in
1997 of the presurgical protocol in which the surgeon signs the surgical site and the
mandate for this protocol by the Joint Commission on the Accreditation of Healthcare
Organizations (JCAHO) in 2003, the total number of wrong-site surgeries reported per
year in the United States has
1- increased.
2- decreased.
3- decreased for orthopaedic surgery but stayed the same for other surgeries.
4- remained the same.
5- only improved for hospital-based surgery.

DISCUSSION: Despite the initiatives by the American Academy of Orthopaedic Surgeons


and the JCAHO, the number of reported cases of wrong-site surgery has continued to
increase yearly since 1997. Because reporting of these events is not mandated by JCAHO,
it is possible that the continued increase is due to a greater awareness of the problem
and thereby a greater level of reporting. The U.S. estimates are 12.7 wrong-site surgeries
per million cases performed. Orthopaedic surgery and podiatry are the most common
specialties associated with wrong-site surgery (41%) followed by general surgery (21%),
neurosurgery (14%), and urologic surgery (11%). The Preferred Respons# 185 is 1.

128
128
Online 2011 Orthopaedic Self-Assessment Examination by Dr.Dhahirortho
Question 186 Figures 186a and 186b are the radiographs of a 10-year-old girl who
sustained an injury 2 days ago after jumping off another girl's shoulders while
cheerleading. She is unable to walk and has no other injuries. Examination reveals
swelling below the knee and a palpable defect at the tibial tubercle. The knee is
ligamentously stable medial-lateral and anterior-posterior. What is the next most
appropriate step in management?

1-MRI scan of the knee


2-CT scan of the knee
3-Open reduction and internal fixation
4-Excision of the fragment
5-Application of a long-leg cast

DISCUSSION: The radiographs show the


patella elevated and the patellar
ligament insertion retracted greater than 2 cm. The most appropriate treatment is repair
of the patellar ligament. Excision of the fragment and application of a cast will not restore
quadriceps function. A CT scan will only demonstrate what is evident on the radiographs
and an MRI scan is not needed because the knee is ligamentously stable.
The Preferred Response to Question # 186 is 3.

Question 187 Figure 187 is the radiograph of a 65-year-old woman who underwent
uneventful bipolar hip arthroplasty for a displaced femoral neck fracture 5 years ago.
Although she initially did well and returned to an active lifestyle, recently she reports
increasing pain with ambulation and has become sedentary. Appropriate management
should consist of which of the following?
1- Removal of the trochanteric wires
2- Use of an assistive device for ambulation
3- Physical therapy for abductor strengthening
4- Conversion to a total hip arthroplasty with femoral
revision and acetabular implantation
5- Conversion of the bipolar hip arthroplasty to a total hip
arthroplasty with placement of an acetabular component

129
129
Online 2011 Orthopaedic Self-Assessment Examination by Dr.Dhahirortho
DISCUSSION: The patient has lost acetabular articular cartilage. In addition, the bipolar
component is migrating superiorly and laterally. Revision to a total hip arthroplasty is
necessary. Removal of trochanteric hardware and abductor strengthening is not likely to
improve the patient's symptoms. The use of assistive devices for ambulation may
decrease the discomfort but does not address the proximal migration of the bipolar head
associated with acetabular wear. The femoral component is not loose and does not need
to be revised. The Preferred Response to Question # 187 is 5.

Question 188 Figures 188a and 188b are the radiographs of a 6' 1" 205-lb 22-year-old
female collegiate basketball player who landed awkwardly on her right leg during
practice and collapsed on the ground. She immediately reported severe pain in her
right leg and could not move her right knee. Examination in the emergency department
reveals symmetric dorsalis pedis and posterior tibial pulses in her lower extremities. An
MRI scan reveals anterior cruciate ligament, posterior cruciate ligament, and
posterolateral corner injury. What is the next most appropriate step in management?

1- Admission and observation overnight, followed by closed reduction and examination


under anesthesia in the morning, splinting, discharge, and follow-up in 48 hours for
delayed ligament reconstruction
2- Emergent closed reduction followed by immediate transfer to the vascular suite for
an angiogram
3- Emergent closed reduction and examination under anesthesia, followed by
immediate ligament reconstruction
4- Emergent closed reduction and examination under anesthesia followed by repeat
neurovascular examination, observation overnight, and delayed ligament
reconstruction

130
130
Online 2011 Orthopaedic Self-Assessment Examination by Dr.Dhahirortho
5- Emergent closed reduction and examination under anesthesia followed by repeat
neurovascular examination, discharge, and follow-up in 48 hours for delayed ligament
reconstruction

DISCUSSION: Management should include emergent closed reduction and observation


overnight for neurovascular compromise and compartment syndrome. The radiographs
clearly show an anterior knee dislocation. This condition can result in vascular and/or
neurologic compromise and represents a true emergency. Initial treatment is emergent
closed reduction followed by close observation for 24 to 48 hours. Some controversy
exists regarding the indications for invasive vascular studies out of concern for occult
catastrophic arterial injury. Stannard and associates and Klineberg and associates studied
the incidence of vascular compromise following knee dislocation and concluded that
patients with symmetric lower extremity pulses were at low risk for progression to
vascular compromise. As a result, selective arteriography based on serial physical
examinations is a safe and prudent policy following knee dislocation. Definitive treatment
of the ligamentous injuries is not advocated in the immediate period following injury
because of the need to ensure the vascular integrity of the limb. Repair versus
reconstruction of damaged ligaments at a later time is controversial, with some authors
advocating early repair/reconstruction while others support reconstruction in a staged
fashion. The Preferred Response to Question # 188 is 4.

Question 189 A 68-year-old woman sustains the injury seen in Figure 189 following a
fall. Careful neurologic and vascular examinations reveal no associated injury. What is
the most common complication of surgical fixation with a locked plate and screw
construct through a deltopectoral approach?
1- Infection
2- Axillary nerve palsy
3- Peri-implant fracture
4- Deltoid insertion detachment
5- Screw penetration of the articular surface

DISCUSSION: Several studies have documented screw


penetration through the articular surface as the leading complication with locked plate
and screw fixation for displaced proximal humerus fractures. Axillary nerve palsy is rare
but can be seen with a deltoid split approach. Whereas some loss of range of shoulder
131
131
Online 2011 Orthopaedic Self-Assessment Examination by Dr.Dhahirortho
motion is expected following this injury, infection rates about the shoulder are in the 1%
to 5% range. While a portion of the deltoid insertion is commonly taken down, complete
deltoid insertion detachment has not been described. The Preferred Response to
Question # 189 is 5.

Question 190 When evaluating a patient with suspected purulent flexor tenosynovitis
in the thumb, the distal forearm and little finger are found to be swollen as well. The
most likely anatomic explanation is the existence of a potential space in which of the
following?
1- Through the carpal tunnel
2- Across the midpalmar space
3- Communicating with the subcutaneous tissue
4- Superficial to the distal antebrachial fascia
5- Between the fascia of the pronator quadratus and flexor digitorum profundus
conjoined tendon sheaths

DISCUSSION: Pyogenic flexor tenosynovitis is an infection within the flexor tendon sheath
that can involve the fingers or thumb. The tendon sheaths begin at the metacarpal neck
level and extend to the distal interphalangeal joint. In the little finger and the thumb, the
sheaths usually communicate with the ulnar and radial bursae, respectively. The potential
space of communication, Parona's space, lies between the fascia of the pronator
quadratus muscle and flexor digitorum profundus conjoined tendon sheaths. Infection
tracking through this space presents as a horseshoe abscess. The Pre Resp# 190 is 5.

Question 191What complication is most likely to occur following proximal humeral


fixation with a locked plate-and-screw construct?
1- Screw penetration
2- Rotator cuff injury
3- Axillary nerve damage
4- Fracture of the humeral shaft
5- Impingement
DISCUSSION: Proximal humeral locking plates have been associated with screw
penetration (incidence 23%). The rotator cuff injury is not due to the plate or its
application and is associated with dislocations in the elderly. Axillary nerve damage, while
possible, has a low reported incidence from open reduction and internal fixation of the
132
132
Online 2011 Orthopaedic Self-Assessment Examination by Dr.Dhahirortho
proximal humerus with locking constructs. Impingement and fracture of the humeral
shaft are also unlikely. More likely but not offered as a choice is the problem of varus
reduction which can result in failure. However, penetration of the screws remains the
most commonly reported complication. The Preferred Response to Question # 191 is 1.

Question 192 Which of the following associated diagnoses is more likely to occur in a
child with a Myers and McKeever type II tibial spine fracture?
1- Lower leg compartment syndrome
2- Patellar dislocation
3- Peroneal nerve palsy
4- Meniscal entrapment at the fracture site
5- Proximal tibial growth arrest

DISCUSSION: Tibia eminence, also referred to as a tibial spine fracture, occurs as a result
of stress on the anterior cruciate ligament that results in an avulsion fracture at the
anterior cruciate ligament's proximal tibia footprint. These avulsion injuries have a high
association of meniscal entrapment of the anterior portion of the meniscus underneath
the angulated or displaced tibial spine fracture fragment. Compartment syndrome is
associated with tibial tubercle fractures but not tibial spine fractures. Patellar dislocation,
peroneal nerve palsy, and proximal tibial growth arrest are not associated with this
fracture. The Preferred Response to Question # 192 is 4.

Question 193 The use of evidence-based studies among professions associated with
health care, including purchasing and management, is known as
1- decision analysis.
2- cost-utility analysis.
3- cost-benefit analysis.
4- cost-effectiveness analysis.
5- evidence-based health care.

DISCUSSION: Evidence-based health care extends the application of the principles of


evidence-based medicine to all professions associated with health care. This concept is
becoming more important because data will be used by the different parties for their
decision making (policy makers, health insurances, hospitals, doctors, and the public).
Cost-benefit analysis refers to the conversion of effects into the same monetary terms as
133
133
Online 2011 Orthopaedic Self-Assessment Examination by Dr.Dhahirortho
the costs and compares them. Cost-effectiveness analysis refers to the conversion of
effects into health terms and describes the costs for some additional health gain (eg, cost
per additional event prevented). Cost-utility analysis refers to the conversion of effects
into personal preferences (or utilities) and describes how much it costs for some
additional quality gain (eg, cost per additional quality-adjusted life-year). Decision
analysis refers to the application of explicit, quantitative methods to analyze decisions
under conditions of uncertainty. The Preferred Response to Question # 193 is 5.

Question 194 A 55-year-old woman has had a swollen and painful right knee for 1 year.
Figures 194a and 194b show AP and lateral radiographs, and Figure 194c shows a
biopsy specimen. What is the most likely diagnosis?

1- Pigmented villonodular synovitis


2- Synovial chondromatosis
3- Septic arthritis
4- Loose bodies related to osteoarthritis
5- Chondroblastoma

DISCUSSION: The patient has synovial chondromatosis that is characterized by multiple


calcified masses seen in the radiographs and by benign chondroid masses seen in the
biopsy specimen. This is a benign condition that is typically treated by open, complete
synovectomy of the joint. There have been rare cases reported of late conversion to
chondrosarcoma, but most patients have good local control with synovectomy.
Pigmented villonodular synovitis would not have mineralization and is characterized by
recurrent bloody effusions and low-signal changes of gradient echo images on MRI. Loose
bodies are usually much smaller, completely intra-articular, and can move around on
examination. Septic arthritis is typically more acute in history and would not have
cartilage formation on biopsy. Chondroblastoma is an osseous lesion and almost never
134
134
Online 2011 Orthopaedic Self-Assessment Examination by Dr.Dhahirortho
has a soft-tissue extension. The biopsy specimen would show cobblestone chondroblasts
with occasional giant cell-like osteoclasts.
The Preferred Response to Question # 194 is 2.

Question 195 Which of the following proximal phalanx fractures can most reliably be
treated with a closed reduction and avoidance of surgical measures?

1- Midshaft transverse diaphyseal fracture with 30 degrees of angulation


2- Long spiral diaphyseal fracture with 15 degrees of malrotation
3- Open fracture with skin loss and exposed extensor tendon
4- Distal condylar intra-articular fracture with minimal displacement
5- Proximal metaphyseal fracture location with 30 degrees of dorsal tilting

DISCUSSION: Proximal phalanx fractures are very common, but care must be taken to
understand which injuries are reliably treated with nonsurgical measures, and which ones
are prone to clinically symptomatic malunion without surgical treatment. The proximal
metaphyseal location is a problematic fracture to get reduced with closed measures, and
due to the forces of the extensor apparatus, is prone to collapse into the original
deformity. Imaging is also frequently difficult because of the overlap of the other fingers
and frequently the true angulation is underappreciated. With 30 degrees of angulation,
consideration should be given to surgical treatment. Long oblique/spiral fractures with
malrotation are also most reliably treated with multiple lag screws, because maintaining
the reduction with nonsurgical measures is unreliable, and can lead to significant
functional problems in the form of crossover of the fingers with gripping. Open fractures
with skin loss clearly are treated with surgical measures. Distal condylar fractures with
minimal displacement are another fracture pattern that have a high rate of loss of
reduction when treated nonsurgically. Like most articular fractures, they are best treated
with anatomic reduction and rigid internal fixation. By comparison, closed midshaft
transverse diaphyseal fractures can usually be anatomically reduced and held in this
position with closed measures.

The Preferred Response to Question # 195 is 1.

135
135
Online 2011 Orthopaedic Self-Assessment Examination by Dr.Dhahirortho
Question 196 Figures 196a through 196c are the radiographs of a 52-year-old woman
who reports knee pain after falling from a standing height. Examination reveals a
moderate knee effusion but no obvious instability of the knee in extension. What is the
most appropriate treatment?

1- Long-leg cast
2- Open reduction and internal fixation using percutaneous screws
3- Open reduction and internal fixation using a medial buttress plate
4- Open reduction and internal fixation using a small wire Ilizarov frame
5- Strict non-weight-bearing with active range of motion from the outset for 6 weeks,
followed by gradual weight bearing

DISCUSSION: The patient has a nondisplaced split condyle fracture of the proximal tibia
that importantly does not show displacement of any significance. The conclusion is
particularly clear on the lateral radiograph. Whether or not the fracture is displaced is a
good predictor of eventual outcome. This relates to the damage that occurs to the
cartilage surface as indicated by recent studies if significant disruption of the joint surface
occurs. The patient has an excellent prognosis for recovery with nonsurgical management
consisting of non-weight-bearing and early active range of motion along with careful
clinical monitoring of radiographs.

The Preferred Response to Question # 196 is 5.

136
136
Online 2011 Orthopaedic Self-Assessment Examination by Dr.Dhahirortho
Question 197 Figures 197a through 197c are the radiograph and MRI scans of a 63-year-
old woman who reports the insidious onset of severe right hip pain. Her pain is worse
with weight bearing and alleviated with rest. She takes no medications and is otherwise
healthy. What is the next best step in her treatment?

1- Bone resection and mega-prosthetic reconstruction


2- Radiation therapy
3- Prolonged course of antibiotics
4- Partial weight bearing and observation
5- Core decompression

DISCUSSION: The patient has transient osteoporosis, which most commonly involves the
hips. The etiology is unknown but may be related to an interruption of the intraosseous
blood supply. Patients have joint pain and usually have normal findings on radiographs or
CT scans. The MRI scan shows complete replacement of the marrow on T1-weighted
images and marked hyper-intensity of the marrow on T2-weighted sequences.
Osteonecrosis of bone would show focal marrow changes and a serpentine line of
demarkation. Crescent-shaped bone collapse can later be seen on the radiographs. This
case does not show radiographic changes of osteonecrosis, but does show early
subchondral bone formation in the femoral head. Osteonecrosis would not show early
subchondral bone healing. The findings of transient osteoporosis are commonly mistaken
for metastatic bone disease; however, the MRI scan does not show a focal mass. The
diagnosis of transient osteoporosis can be made by correlating the clinical history of
severe pain with the markedly abnormal MRI scan in the face of a normal radiograph and
CT scan. Transient osteoporosis is a self-limiting disease. Therefore, surgeons should use
a treatment approach based on the clinical symptoms. Current, therapeutic strategies
include partial weight bearing, mild analgesics, and administration of nonsteroidal anti-
inflammatory drugs. Treatment protocols to avoid include bone resection (malignancy),
radiation (malignancy), antibiotics (osteomyelitis), or core decompression
(osteonecrosis). The Preferred Response to Question # 197 is 4.

137
137
Online 2011 Orthopaedic Self-Assessment Examination by Dr.Dhahirortho
Question 198 A 24-month-old boy with clubfoot is not walking independently. What is
the most likely reason he is not walking independently?
1- Neurologic disorder
2- Hip dysplasia
3- In-toeing
4- Limb-length inequality
5- Foot deformity

DISCUSSION: Children should ambulate independently at about 1 year of age and


although this age may vary, if not ambulating by 18 months, an underlying neurologic or
developmental condition should be considered. Clubfoot does not significantly delay
ambulation nor does hip dysplasia or limb-length inequality, although this is a common
misconception and reason for referral. Torsional deformities, such as in-toeing, are
common and do not alter the age at which a child will ambulate. The Pre Res# 198 is 1.

Question 199 Which of the following is the best method of initial pelvic stabilization for
a patient with hemodynamic instability and the pelvic ring injury seen in Figure 199?

1- Symphyseal plating
2- Iliosacral screw fixation
3- Pelvic binder
4- Pelvic C-clamp
5- External fixation

DISCUSSION: For a patient with an unstable pelvic ring injury and hemodyamic instability,
the most appropriate initial treatment method is a pelvic sheet or binder. Symphyseal
plating and iliosacral screw fixation require surgical intervention and may be appropriate
following initial stabilization. External fixation and the pelvic C-clamp can be applied in
the emergency setting, but usually are reserved for patients who do not respond to
simpler less invasive methods initially. The Preferred Response to Question # 199 is 3.

138
138
Online 2011 Orthopaedic Self-Assessment Examination by Dr.Dhahirortho

Question 200 A 20-year-old collegiate volleyball player has vague left, nondominant
elbow pain. Five years ago, he sustained a dislocation of the same joint and, while he
could participate in his sport, he notes that the elbow 'never felt quite right.` The pain
is not severe but prevents him from playing sports and he cannot localize the pain to
any specific location. Occasionally he will perceive a catching when pushing himself out
of a chair but the elbow never locks in one position. Examination reveals full passive
and active range of motion in flexion, extension, supination, and pronation. There is
tenderness of the lateral elbow during elbow extension with the forearm supinated and
a momentary painful `clunk` is noted. Radiographs and MRI scans are normal. What is
the most likely instability?

1- Varus
2- Valgus
3- Longitudinal forearm
4- Posteromedial rotatory
5- Posterolateral rotatory

DISCUSSION: Posterolateral rotatory instability of the elbow is seen in athletes and


frequently follows a previous injury such as a dislocation where the lateral ulnar collateral
ligament becomes weakened and attenuated. The ulna supinates away from the humerus
and the radius subluxates posteriorly on the capitellum with the forearm supinated and
the elbow in extension. Posteromedial rotatory instability is more often seen in
association with fracture of the coronoid process following a varus stress to the elbow.
Valgus instability occurs due to an injury to the medial ulnar collateral ligament seen
most commonly in throwers from overuse. Varus instability is rare but results in lateral
gapping of the elbow. Longitudinal forearm instability is seen after an Essex-Lopresti
injury.

The Preferred Response to Question # 200 is 5.

139
139
2011 Upper Extremity Self-Assessment Exam by Dr.Dhahirortho

1
140
2011 Upper Extremity Self-Assessment Exam by Dr.Dhahirortho

2
141
2011 Upper Extremity Self-Assessment Exam by Dr.Dhahirortho

Question 1A 23-year-old patient with lateral epicondylitis underwent a routine


elbow arthroscopy and an anterolateral portal was used. The patient now has
complications associated with nerve injury in this area. What symptoms will most
likely be present?
1- Loss of digital extension
2- Weakness of the interossei
3- Decreased sensation in the ring and little fingers
4- Decreased sensation in the ulnar dorsal forearm
5- Loss of flexor pollicis longus function

DISCUSSION: The anterolateral portal as originally described puts the radial nerve at
risk because of its close proximity to the portal. The best test to demonstrate radial
nerve function is the ability to extend the metacarpophalangeal joints. Weakness of
the interossei, sensation to the ring and little fingers, and ulnar forearm sensation are
all ulnar nerve functions. The flexor pollicis longus is innervated by the median nerve.
The Preferred Response to Question # 1 is 1.
Question 2..On MRI, what nerve is most likely to demonstrate increased signal
intensity about the elbow in asymptomatic patients?
1- Ulnar 2- Radial 3- Median 4- Anterior interosseous 5- Musculocutaneous
DISCUSSION: The ulnar nerve has been shown to have increased signal intensity in
asymptomatic patients when compared with other nerves about the elbow. It has been
shown to have increased signal in approximately 60% of normal patients compared
with 0% for the median and radial nerves. This suggests that the presence of increased
signal in the ulnar nerve may be of questionable clinical relevance.Pre Res# 2 is 1.
Question 3Figure 3 shows an arthroscopic view of the radiocarpal joint from the 3-4
portal, looking volarly and radially (Sc=scaphoid, R=Radius). What structure is
marked by the asterisk?
1- Radioscaphocapitate ligament
2- Scapholunate ligament
3- Palmar oblique ligament
4- Dorsal intercarpal ligament
5- Triangular fibrocartilage complex (TFCC)

3
142
2011 Upper Extremity Self-Assessment Exam by Dr.Dhahirortho

DISCUSSION: The radioscaphocapitate ligament is a volar capsular structure running


obliquely from the radial styloid to the scaphoid waist, ultimately inserting on the
proximal radial aspect of the capitate. The radioscaphocapitate ligament is important
in preventing ulnar translocation of the carpus. The scapholunate ligament is located
intra-articularly, between the scaphoid and lunate. The dorsal intercarpal ligament is a
dorsal structure, and not visible during routine wrist arthroscopy. The palmar oblique
ligament connects the first and second metacarpal bases. The TFCC is visible during
wrist arthroscopy between the radius and ulna. Preferred Response # 3 is 1.

Question 4A 15-year-old boy sustained an anterior sternoclavicular joint dislocation.


What is the preferred management?
1- Open reduction and internal fixation
2- Observation
3- Closed reduction
4- Closed reduction and percutaneous pinning
5- Figure-of-8 brace

DISCUSSION: The medial clavicular epiphysis is the last to fuse (age 22 to 25 in men)
and sternoclavicular injuries are often Salter-Harris type II fractures in this age group,
with opportunity to remodel. Closed reduction is generally not necessary and has a
high recurrence rate. Closed reduction is necessary with posterior dislocations
associated with compression of the trachea, esophagus, or great vessels. Figure-of-8
bracing has not been shown to secure a sternoclavicular reduction. Pre Resp# 4 is 2.

Question 5A 22-year-old man reports a 2-week history of a burning pain along the
dorsoradial aspect of the distal forearm. The pain radiates to the dorsum of the
thumb. Examination reveals tenderness and reproduction of symptoms with
percussion 8 cm proximal to the radial styloid. Reproduction of symptoms also occurs
with forearm pronation and ulnar deviation of the wrist. No discrete sensory deficit
is noted and electrodiagnostic studies are normal. Nonsurgical management
consisting of rest, splinting, and anti-inflammatory medications for 6 weeks has
failed to provide relief. Treatment should now consist of decompression of the

4
143
2011 Upper Extremity Self-Assessment Exam by Dr.Dhahirortho

1- lateral antebrachial cutaneous nerve in the interval between the abductor pollicis
longus and the extensor pollicis brevis in the forearm.
2- lateral antebrachial cutaneous nerve in the interval between the brachioradialis
and the extensor carpi radialis longus in the distal forearm.
3- radial sensory nerve in the interval between the extensor carpi radialis longus and
the extensor carpi radialis brevis in the distal forearm.
4- radial sensory nerve in the interval between the brachioradialis and the extensor
carpi radialis longus in the distal forearm.
5- radial sensory nerve in the interval between the brachioradialis and the extensor
carpi radialis brevis in the distal forearm.

DISCUSSION: Wartenberg's syndrome, or compression of the sensory branch of the


radial nerve, occurs in the interval between the brachioradialis and the extensor carpi
radialis longus approximately 8 cm proximal to the radial styloid. There may be history
of repetitive wrist/forearm circumduction activity (ie, knitting) or of wearing a tight
wristwatch or jewelry. It can occur in patients who have been handcuffed. Typical
clinical findings are pain, paresthesia, and/or hypesthesia in the dorsoradial aspect of
the wrist and hand in the distribution of the radial sensory nerve. There is often a
positive Tinel's sign over the compression site. Hypesthesia may be present in the
distribution of the radial sensory nerve which is typically on the dorsal aspect of the
first dorsal web space and dorsum of the thumb; however, with overlap in the
distribution of the superficial radial nerve and the lateral cutaneous nerve of the
forearm this may not always be present. Surgical management consists of release of
the nerve as it exits the interval between the brachioradialis and the extensor carpi
radialis longus in the distal forearm. The Preferred Response to Question # 5 is 4.
Question 6A patient has multidirectional instability of the shoulder that has not
responded to nonsurgical management. Successful surgical treatment will most likely
include which of the following?
1- Abrasion arthroplasty of the anterior glenoid rim
2- Posterior capsular advancement into a Hill-Sachs defect
3- Repair of an inferior glenohumeral ligament detachment
4- Repair of an unstable SLAP tear
5- Closure of the rotator interval
5
144
2011 Upper Extremity Self-Assessment Exam by Dr.Dhahirortho

DISCUSSION: Published reports establish the importance of the rotator interval in


shoulder stability and improvements achieved through suture closure of the interval.
Multidirectional instability treated surgically following failure to respond to nonsurgical
management has been shown to be associated with classic Bankart lesions, Hill-Sachs
defects, glenoid chondral lesions, and even SLAP lesions (Werner). However, these
lesions were seen in a lower percentage than that found for unidirectional anterior
dislocations. Likewise, these lesions do not appear to be significant in influencing
treatment in the majority of patients. The Preferred Response to Question # 6 is 5.

Question 7An active 65-year-old man has pain in the left shoulder 5 years after
undergoing a hemiarthroplasty. He has a remote history of two previous instability
operations. Examination reveals that forward elevation is 140 degrees and external
rotation is 40 degrees. Serologic studies for infection are negative. AP and axillary
radiographs are shown in Figures 7a and 7b. What surgical procedure will provide the
most predictable pain relief and function?

1- Conversion to a reverse total shoulder arthroplasty


2- Conversion to a standard total shoulder arthroplasty
3- Conversion to a glenohumeral fusion
4- Resection arthroplasty
5- Biologic resurfacing of the glenoid

DISCUSSION: The radiographs show glenoid arthrosis, which is common after a


hemiarthroplasty. Conversion to a conventional total shoulder arthroplasty with
placement of a glenoid component predictably decreases pain and improves function.
There is no indication for a reverse total shoulder arthroplasty because the patient has
140 degrees of elevation with an intact rotator cuff. Biologic resurfacing has more
6
145
2011 Upper Extremity Self-Assessment Exam by Dr.Dhahirortho

unpredictable results and is usually reserved for younger patients in whom a prosthetic
glenoid component might not be desired. Both resection arthroplasty and arthrodesis
are associated with poor function. The Preferred Response to Question # 7 is 2.

Question 8A 55-year-old woman with rheumatoid arthritis reports that she awoke
with an inability to flex the interphalangeal joint of her thumb. Figure 8 shows an
intraoperative finding. What is the most appropriate surgical treatment?

1- Primary repair of the tendon


2- Tendon reconstruction with the palmaris longus tendon
3- Tendon reconstruction using a transfer of the flexor digitorum profundus (FDP) of
the ring finger
4- Thumb metacarpophalangeal fusion
5- End-to-side repair of the flexor pollicis longus to the FDP of the index finger

DISCUSSION: The patient has sustained a chronic flexor pollicis longus rupture
(Mannerfelt lesion). The injury is most likely a result of tendinopathy and attritional
rupture of the tendon secondary to synovitis and bony osteophytosis at the
scaphotrapeziotrapezoid joint. Because of the attritional injury and inherent
tendinopathy, primary repair is unlikely to be successful. Among the options listed,
tendon graft reconstruction with the palmaris longus tendon is the most appropriate
treatment. Tendon reconstruction is possible with the flexor digitorum profundus of
the index finger, not the flexor digitorum profundus of the ring finger. If osteophytes
are encountered, these should be debrided. Thumb interphalangeal fusion is an
option, but metacarpophalangeal fusion is not beneficial. End-to-side repair of the
flexor pollicis longus to the FDP of the index finger is not appropriate and would
sacrifice needed function of the index finger. Preferred Response to Question # 8 is 2.

7
146
2011 Upper Extremity Self-Assessment Exam by Dr.Dhahirortho

Question 9A 56-year-old man who tripped and fell out of his golf cart onto his right
shoulder 4 days ago now reports mild pain while chipping. Examination reveals mild
bruising over the lateral clavicle but good shoulder range of motion and strength. A
radiograph is shown in Figure 9. Appropriate treatment at this time should include
which of the following?
1- Intramedullary pinning
2- Bone stimulator
3- Sling for comfort, followed by gentle range-of-motion exercises
4- Open reduction and internal fixation with a plate and screws
5- Arthroscopic distal clavicle resection

DISCUSSION: Treatment of this minimally displaced distal clavicle fracture should begin
with nonsurgical management consisting of sling therapy followed by gentle motion
therapy. Any form of surgical intervention at this time is unnecessary because this
fracture pattern has a high incidence of union. A bone stimulator may be used if
healing becomes delayed. The Preferred Response to Question # 9 is 3.

10,Atraumatic suprascapular nerve compression usually occurs at which of the


following anatomic locations if it develops atraumatically?
1- Scalenus anterior
2- Suprascapular and spinoglenoid notches
3- Cervical rib
4- Conjoined tendon
5- Subcoracoid

DISCUSSION: The suprascapular nerve has the potential to be compressed as it passes


through the suprascapular and spinoglenoid notches. If the site of compression occurs
at the suprascapular notch, both the supraspinatus and infraspinatus muscles will be
affected. If the site of compression occurs at the spinoglenoid notch, only the
infraspinatus muscle will be affected. Fascial bands and ganglion cysts often compress
the nerve in these areas. The other anatomic areas are not associated with
suprascapular nerve compression. The Preferred Response to Question # 10 is 2.

8
147
2011 Upper Extremity Self-Assessment Exam by Dr.Dhahirortho

11..Which of the following 50-year-old patients with an irreparable rotator cuff


tendon is the best candidate for an isolated latissimus dorsi muscle transfer?
1- Man with active elevation to 90 degrees
2- Woman with active elevation to 45 degrees
3- Woman with a Hornblower's sign (complete absence of external rotation with
abduction)
4- Man with superior escape of the humeral head
5- Man with full motion and a positive lift-off test

DISCUSSION: Patients with superior escape or a torn subscapularis (demonstrated by a


positive lift-off test) will not benefit from a latissimus dorsi transfer, even if combined
with a pectoralis muscle transfer. In the study by Iannotti and associates, women had
poorer outcomes than men, and patients with preoperative elevation below shoulder
level or 90 degrees also had poorer outcomes. Patients with complete loss of external
rotator function have worse function after latissimus dorsi transfer than patients with
some external rotation function. The Preferred Response to Question # 11 is 1.

12.Figures 12a and 12b show the initial radiographs of a 27-year-old snow boarder
who fell backward onto his left outstretched hand. Which of the following most
accurately describes the sequence of events that occurred during this injury?

1- Lunotriquetral ligament failure followed by distal row dissociation, scaphoid


extension, scaphoid failure, and dorsal dislocation of the carpus
2- Volar dislocation of the lunate followed by scaphoid extension, scaphoid failure,
lunotriquetral failure, and distal row dissociation
3- Dorsal intercarpal ligament failure followed by distal row dissociation, scaphoid
failure, lunotriquetral ligament failure, and dorsal dislocation of the carpus
9
148
2011 Upper Extremity Self-Assessment Exam by Dr.Dhahirortho

4- Short radiolunate ligament failure followed by volar dislocation of the lunate,


lunotriquetral ligament failure, scaphoid failure, and distal row dissociation
5- Scaphoid extension followed by scaphoid failure, distal row dissociation,
lunotriquetral ligament failure, and dorsal dislocation of the carpus

DISCUSSION: As described by Mayfield and associates, the typical sequence of events


referred to as "progressive perilunar instability" that result in a volar perilunate
dislocation are as follows: scaphoid extension, followed by opening of the space of
Poirer, scaphoid failure, and distal row dissociation, which in turn lead to
hyperextension of the triquetrum, lunotriquetral ligament failure, and finally dorsal
dislocation of the carpus. The lunate remains in the lunate fossa in a perilunate
fracture-dislocation but is dislocated in a lunate dislocation. The short radiolunate and
dorsal intercarpal ligaments typically remain intact. Preferred Response # 12 is 5.

13..One week ago a 25-year-old man slipped on the ice and fell, catching himself on a
railing. He sustained an anterior shoulder dislocation that was subsequently reduced
without difficulty in the emergency department, and he was discharged in a sling. He
is now back for follow-up and reports no pain. Examination reveals no weakness on
external rotation strength testing. What is the most appropriate management for
this patient?
1- Arthroscopic Bankart repair
2- MRI for possible rotator cuff tear
3- Physical therapy
4- Sling immobilization for an additional 2 weeks
5- Cortisone injection

DISCUSSION: On the basis of the patient's age, lack of weakness, and the fact that this
is a first-time traumatic shoulder dislocation, he is unlikely to have sustained a rotator
cuff tear. Immobilization should be continued for 2 more weeks. Scheduling a surgical
stabilization procedure at this time is not indicated. Immediate therapy is
contraindicated because of the acuity of the injury. A cortisone injection is not
indicated in an acute traumatic shoulder dislocation. Preferred Response # 13 is 4.

10
149
2011 Upper Extremity Self-Assessment Exam by Dr.Dhahirortho

14..A 75-year-old woman who is right-hand dominant fell from a 2-foot step and
landed on her outstretched right arm, sustaining an injury to the elbow. She reports
no other injuries and is in generally good health. Examination reveals a swollen,
ecchymotic elbow and intact skin, with a normal distal neurovascular examination.
Radiographs are shown in Figures 14a and 14b. Management of the injury should
include which of the following?
1- Total elbow arthroplasty
2- Spanning articulated external fixation
3- Long arm cast for 2 weeks, followed by
progressive mobilization
4- Open reduction and internal fixation
5- Closed reduction and percutaneous screw fixation

DISCUSSION: The patient sustained a displaced,


comminuted, intercondylar distal humerus fracture. In
an otherwise healthy patient older than age 65 years,
randomized clinical trials have demonstrated more predictable outcomes with total
elbow arthroplasty when compared with open reduction and internal fixation. Closed
reduction and percutaneous fixation is biomechanically inadequate to maintain
fracture alignment and allow early mobilization. Spanning external fixation is typically
used to maintain stability in the setting of a complex elbow fracture-dislocation.
Nonsurgical management would be considered when a patient is medically unfit for
surgery. The Preferred Response to Question # 14 is 1.

15Which of the following is the most consistently proposed tendon transfer for radial
nerve palsy?
1- Pronator teres to extensor carpi radialis brevis
2- Brachioradialis to extensor carpi radialis brevis
3- Flexor carpi radialis to extensor digitorum communis
4- Palmaris longus to extensor pollicis longus
5- Flexor digitorum superficialis to abductor pollicis longus and extensor pollicis
brevis

11
150
2011 Upper Extremity Self-Assessment Exam by Dr.Dhahirortho

DISCUSSION: Whereas there are many variations of tendon transfers for radial nerve
palsy, the most consistently proposed tendon transfer is the pronator teres to extensor
carpi radialis brevis. The brachioradialis is innervated by the radial nerve so that is not
an option. The flexor digitorum superficialis, flexor carpi radialis, and flexor carpi
ulnaris are appropriate options to transfer to the extensor digitorum communis. The
palmaris longus is not always present. A transfer to the abductor pollicis longus and
extensor pollicis brevis may not be necessary if the extensor pollicis longus is rerouted
to allow for abduction of the first ray. The Preferred Response to Question # 15 is 1.

16..A 20-year-old man has activity-related deep-seated shoulder pain in his dominant
right shoulder. He has taken 3 months off training as a college javelin thrower, and
management consisting of physical therapy has failed to provide relief. Shoulder
arthroscopic views are shown in Figures 16a through 16c. What is the underlying
association with this condition?

1- Ehlers-Danlos syndrome
2- Traumatic anterior instability
3- Humeral head osteonecrosis
4- Internal impingement
5- Partial-thickness supraspinatus tear

DISCUSSION: The patient is involved in overhead athletics and reports deep-seated


pain. The arthroscopic views show a SLAP tear with posterior extension that is typical
of internal impingement. The history lacks a component of gross instability expected in
traumatic anterior dislocations or multidirectional instability associated with a
connective tissue disorder, and it also lacks risk factors for osteonecrosis. The images
do not show evidence of an unstable humeral cartilage flap or a supraspinatus tear.
The Preferred Response to Question # 16 is 4.

12
151
2011 Upper Extremity Self-Assessment Exam by Dr.Dhahirortho

17.Figure 17 shows the radiograph of an 82-year-old right-hand dominant woman


who fell while weeding her garden. She has severe right shoulder pain. She is
neurovascularly intact. What is the most appropriate treatment?

1- Rest, ice, nonsteroidal anti-inflammatory drugs, activity as tolerated, and follow-


up in 4 weeks
2- Coaptation splinting and follow-up in 4 weeks
3- Surgical replacement with hemiarthroplasty or reverse total shoulder arthroplasty
4- Physical therapy for range-of-motion exercises
5- Closed reduction, splinting, and follow-up in 4 weeks

DISCUSSION: The patient has a displaced four-part proximal humerus fracture. The
humeral head is displaced and if allowed to heal in this position, the patient will likely
have a stiff and painful shoulder. The humerus is at risk for osteonecrosis given the
displacement of the fracture. Given a patient age of 82 years, replacement options of
either hemiarthroplasty or reverse total shoulder arthoplasty, allow maximal
restoration of function. Physical therapy is not indicated in this acute fracture. Closed
reduction techniques will not be successful in this displaced fracture. Preferred
Response to Question # 17 is 3.

18..During the Kocher approach to repair a radial head fracture, care must be taken
not to release what posterior structure lying under the anconeus that may be
inadvertently injured during this common lateral approach to the elbow?
1- Ulnar nerve
2- Annular ligament
3- Anterior band of the medial collateral ligament
4- Lateral ulnar collateral ligament
5- Arcade of Struthers

13
152
2011 Upper Extremity Self-Assessment Exam by Dr.Dhahirortho

DISCUSSION: The lateral ulnar collateral ligament may be iatrogenically injured during
dissection through the internervous plane between the extensor carpi ulnaris and
anconeus (Kocher approach). Dissection posteriorly may compromise this ligament,
leading to pain and rotatory instability of the elbow. The ulnar nerve, annular ligament,
medial collateral ligament, and arcade of Struthers are not anatomically in this area.
The Preferred Response to Question # 18 is 4.

19A 27-year-old man has recurrent anterior shoulder instability following an


arthroscopic Bankart repair 4 years ago. Current CT scans are shown in Figures 19a
and 19b. Deficiency of what mechanism is most likely to contribute to the current
joint instability?

1- Synovial fluid adhesion-cohesion


2- Negative intra-articular pressure
3- Concavity-compression of the humeral head in the glenoid
4- Decreased functional arc of motion as a result of a Hill-Sachs lesion
5- Poor rehabilitation of scapulothoracic rhythm

DISCUSSION: Loss of the anterior glenoid rim can commonly occur as a result of acute
fracture or progressive wear following multiple dislocations. This decreases the
effective depth of the glenoid. The ability of the rotator cuff to stabilize the joint
through production of a joint reactive force is markedly decreased. Synovial fluid
adhesion-cohesion and negative intra-articular pressure are maintained in the closed
capsular space. The Hill-Sachs lesion in this case is not large enough to be a significant
factor in failed Bankart repair. Poor scapulothoracic rhythm can increase the risk of
instability but is not typically the primary factor. The Preferred Response # 19 is 3.

20..A 48-year-old man undergoes arthroscopy to repair a rotator cuff tear. During the
arthroscopy, the tear is characterized and found to involve the entire supraspinatus
and a majority of the infraspinatus tendons. After mobilization, the posterior rotator
cuff can reach the greater tuberosity. However, the supraspinatus tendon cannot
reach its insertion point at the greater tuberosity. What is the most appropriate
treatment?
14
153
2011 Upper Extremity Self-Assessment Exam by Dr.Dhahirortho

1- Conversion to a latissimus dorsi muscle tendon transfer


2- Acromioplasty and coracoacromial ligament release
3- Reverse acromioplasty (tuberoplasty)
4- Reverse total shoulder arthroplasty
5- Partial repair of the rotator cuff

DISCUSSION: If a complete rotator cuff repair is not possible, a partial rotator cuff
repair should still be considered and is the appropriate treatment for this patient. In
patients with an irreparable massive rotator cuff tear, acromioplasty with
coracoacromial ligament release, reverse acromioplasty, and tenotomy of the biceps
tendon may improve shoulder pain. If these procedures fail, then a muscle transfer
procedure can also be considered in select patients. If, however, a portion of the
rotator cuff can be repaired, even partial repair can balance the coronal and axial
forces about the shoulder to restore the kinematics of the joint. Reverse total shoulder
arthroplasty is not appropriate for this relatively young patient. The Preferred
Response to Question # 20 is 5.

21..A 71-year-old woman reports the insidious onset of shoulder pain at night and
when moving her shoulder. She cannot raise her arm above shoulder level. Physical
therapy has failed to provide pain relief or improve function. An injection relieved
her pain in the office, but she could not raise her arm above shoulder level. A
radiograph is shown in Figure 21. What surgical procedure will provide the best
chance of restoring above shoulder function and pain relief?
1- Reverse total shoulder arthroplasty
2- Hemiarthroplasty of the shoulder
3- Arthroscopic biceps tenolysis
4- Open subacromial debridement
5- Total shoulder arthroplasty

DISCUSSION: The radiograph shows complete loss of the


acromiohumeral space. The glenohumeral joint space is also severely narrowed, which
is consistent with rotator cuff tear arthropathy. In patients who have pain that limits
elevation, pain-reducing procedures such as biceps tenolysis, open debridement, or
15
154
2011 Upper Extremity Self-Assessment Exam by Dr.Dhahirortho

hemiarthroplasty may allow the patient to regain the shoulder function. If the patient
cannot elevate the arm after a successful local anesthetic injection, then pain is not the
reason for the patient's loss of elevation. In this situation, a reverse total shoulder
arthroplasty will most reliably restore function and provide pain relief. The Preferred
Response to Question # 21 is 1.

22..A 47-year-old man who works as a carpenter reports a 12-month history of


painful mechanical locking of his dominant elbow in the mid range of movement. He
also has progressive pain at terminal extension that has not responded to
medication, rest, and intra-articular cortisone injection. Active range of movement is
from 35 degrees to 130 degrees, and he has full pronation and supination. The ulnar
nerve is stable, and he has no subjective or objective neurologic dysfunction in the
hand. Radiographs are shown in Figures 22a and 22b. What is the most appropriate
treatment?

1- Oral corticosteroid medication and


changes in job activities
2- Soft-tissue interposition arthroplasty
3- Arthroscopic capsular release, loose body
removal, and osteophyte decompression
4- Radial head arthroplasty
5- Total elbow arthroplasty

DISCUSSION: The most appropriate treatment is arthroscopic capsular release, loose


body removal, and osteophyte decompression. The patient has moderate
osteoarthritis of the dominant elbow, with mechanical symptoms suggestive of loose
osteochondral body formation. Because the patient has failed to respond to the typical
nonsurgical therapeutic options, it is unlikely that further oral medication will be
helpful, and job modification may not be practical at this stage. Soft-tissue arthroplasty
may be reasonable to consider when less invasive methods, such as arthroscopy, fail.
Isolated radial head arthroplasty would not sufficiently address the symptoms. Total
elbow arthroplasty is indicated in cases of more advanced disease in older patients
with lower physical demands. The Preferred Response to Question # 22 is 3.
16
155
2011 Upper Extremity Self-Assessment Exam by Dr.Dhahirortho

23.A healthy 33-year-old man falls from a ladder onto his outstretched arm. He
sustains the injury shown in Figure 23. This is an isolated injury. What is the most
appropriate treatment?
1- Fragment excision
2- Sling for 1 week, followed by early range of motion
3- Open reduction and internal fixation
4- Radial head arthroplasty
5- Capitellar replacement

DISCUSSION: The injury is a coronal plane fracture of the distal humerus. The
radiograph shows the classic "double-bubble" sign. These fractures often include the
capitellum; however, frequently, the fracture extends medially to involve a portion of
the trochlea. Small articular fragments may be amenable to simple fragment excision;
excision of large fragments can result in posttraumatic arthritis or instability if a medial
collateral ligament injury is present. Fractures involving a significant portion of the
articular surface should be treated with reduction and fixation to reestablish a
congruent joint surface. Closed reduction and percutaneous pinning has shown
variable success rates. Open reduction is the treatment of choice because it allows for
precise restoration of the articular surface and more rigid fixation, more safely
permitting early range of motion. Capitellar replacement is not recommended in a
young active patient with a repairable fracture. Preferred Response # 23 is 3.

24..A 74-year-old woman with rheumatoid arthritis has pain in the shoulder that has
failed to respond to nonsurgical management. AP and axillary radiographs are shown
in Figures 24a and 24b. Active forward elevation is 120 degrees and external rotation
is 30 degrees. At the time of surgery, a 1-cm rotator cuff tear is found, which is
repairable. Which of the following treatment options will result in the most
predictable pain relief and function?
1- Total shoulder arthroplasty and rotator cuff repair
2- Rotator cuff repair
3- Reverse total shoulder arthroplasty
4- Interpositional arthroplasty and rotator cuff repair
5- Hemiarthroplasty and rotator cuff repair
17
156
2011 Upper Extremity Self-Assessment Exam by Dr.Dhahirortho

DISCUSSION: Most studies have shown that total shoulder arthroplasties yield better
pain relief and improved forward elevation when compared with hemiarthroplasty in
patients with rheumatoid arthritis. Patients with repairable rotator cuff tears should
undergo repair at the time of surgery because good results have been shown. Reverse
arthroplasties are not indicated with rotator cuff tears that are repairable, and
interpositional arthroplasties are not indicated for elderly patients. Pr Res # 24 is 1.

25..A 66-year-old man who underwent shoulder arthroplasty 7 years ago reports
progressively worsening shoulder pain for the past 4 weeks after hospital discharge
for community-acquired pneumonia. He is afebrile and reports no chills or night
sweats. Laboratory studies show a white blood cell count of 11,200/mm3 and an
erythrocyte sedimentation rate of 25/h. Shoulder radiographs are negative for
fracture, dislocation, or signs of implant loosening. What is the most appropriate
management?
1- Follow-up in 2 weeks with a repeat white blood cell count and erythrocyte
sedimentation rate
2- Shoulder aspiration with Gram stain and culture of fluid
3- Prescription strength nonsteroidal anti-inflammatory drugs
4- Physical therapy for shoulder stretching and modalities
5- Emergent surgical irrigation, debridement, and revision shoulder arthroplasty
DISCUSSION: The patient may have hematologic spread of the pulmonary infection to
the shoulder arthroplasty; however, further work-up is necessary at this point. The
elevated laboratory studies may still be secondary to the pulmonary infection.
Aspiration of the shoulder joint with stat Gram stain and culture of the fluid is
indicated. If the aspirate shows signs of infection and irrigation and debridement is
indicated, complete revision of the well-seated implants may not be necessary.
Physical therapy and nonsteroidal anti-inflammatory drugs are not indicated until the
possibility of a shoulder infection has been ruled out. A wait of 2 weeks to repeat the
laboratory values, in the presence of new shoulder pain, is contraindicated. The
Preferred Response to Question # 25 is 2.

18
157
2011 Upper Extremity Self-Assessment Exam by Dr.Dhahirortho

26.A 25-year-old man has a swollen painful sternoclavicular joint. He denies using
drugs or having any other medical conditions. Examination does not reveal any
evidence of a dislocation. The joint is tender and slightly warm. The chest radiograph
is normal. What is the next most appropriate step in management?
1- CT of the chest
2- Bone scan
3- Irrigation and debridement in the operating room
4- Lidocaine injection of the joint
5- Physical therapy

DISCUSSION: A common cause of a septic sternoclavicular (SC) joint is IV drug use.


Recently, however, there have been case reports of septic SC joints in patients without
a history of drug use; therefore, this history should not be used as a predictor of
severity or extension beyond the SC joint capsule. Because of the risk of extension of
the infection to the retrosternal area and pericardium, it is recommended that a CT
scan be obtained before proceeding with any surgical management. A bone scan
cannot accurately show abscess extension into the retrosternal area. Lidocaine
injection and physical therapy should not be considered until infection is ruled out. The
Preferred Response to Question # 26 is 1.

27.Elbow distraction interposition arthroplasty may be most appropriate treatment


for which of the following patient profiles?
1- 25-year-old woman with destructive juvenile rheumatoid arthritis
2- 41-year-old male laborer with posttraumatic arthritis of the elbow
3- 44-year-old woman with distal humerus osteonecrosis and collapse
4- 65-year-old man with painful primary elbow osteoarthritis
5- 70-year-old sedentary woman with end-stage rheumatoid arthritis

DISCUSSION: Elbow interposition arthroplasty is reserved for younger, active patients


who may otherwise be candidates for prosthetic replacement. Osteoarthritis,
posttraumatic arthritis, and rheumatoid arthritis patients may all be candidates for
interposition arthroplasty if bone stock is preserved and the elbow maintains inherent
stability. Primary osteoarthritis may also be treated with ulnohumeral arthroplasty (ie,
19
158
2011 Upper Extremity Self-Assessment Exam by Dr.Dhahirortho

Outerbridge) or arthroscopic debridement with release. Patients with destructive


juvenile rheumatoid arthritis and distal humerus osteonecrosis would better benefit
from prosthetic replacement because of bone loss issues. The Preferred Respo# 27 is 2.

28.An active 22-year-old man falls onto his outstretched arm, sustaining the fracture
shown in Figures 28a and 28b. Examination is notable for tenderness over the radial
aspect of the elbow, as well as tenderness at the wrist. Radiographs of the wrist
show no fracture or dislocation. What is the most appropriate treatment?
1- Excision of the radial head
2- Silastic replacement of the radial head
3- Metallic replacement of the radial head
4- Open reduction and internal fixation of the
radial head
5- Sling use and early motion

DISCUSSION: In a young patient, the treatment of


choice is open reduction and internal fixation; in
patients with a nonreconstructible radial head,
metallic replacement can be performed. Fractures
of the radial head are classified by Mason into type I, II, and III. Type I fractures are
nondisplaced, and can be treated with a sling and early motion. Type II fractures are
fractures of a single piece with greater than 2 mm of displacement, and can be treated
with a sling and motion if they are not associated with instability or mechanical blocks
to motion. Type III fractures are comminuted, displaced fractures. The fracture shown
in the figures is a type III fracture with less than three fragments. Fractures with
greater than three fragments have been shown to have generally poor outcomes with
open reduction and internal fixation; fractures with three or fewer fragments had
better results with fewer complications. Silastic replacement has been associated with
uniformly poor long-term results. Whereas radial head excision has excellent results in
the treatment of radiocapitellar arthritis, it is contraindicated in this patient because
he has wrist pain, suggesting an injury to the interosseous membrane (Essex-Lopresti
lesion), and radial head excision has a high likelihood of leading to proximal radial
migration and distal radioulnar joint instability. The Preferred Respon # 28 is 4.
20
159
2011 Upper Extremity Self-Assessment Exam by Dr.Dhahirortho

29..Which of the following is considered a contraindication to the use of a reverse


total shoulder arthroplasty?
1- Prior shoulder joint infection
2- Pseudoparalysis
3- Prior partial acromioplasty
4- Absent glenohumeral joint space narrowing
5- Axillary neuropathy

DISCUSSION: The reverse total shoulder arthroplasty depends on a functional deltoid


muscle which is innervated by the axillary nerve to restore elevation for the patient.
Pseudoparalysis is an indication for a reverse shoulder arthroplasty. Acromioplasty has
not been correlated with poor results with a reverse shoulder arthroplasty. As long as
the patient does not have an active infection, prior infections are not a
contraindication. Patients can still have pain and pseudoparalysis from a chronic
rotator cuff tear, despite having normal cartilage, and they will still benefit from a
reverse total shoulder arthroplasty if other treatments have failed. Pr Res# 29 is 5.

30..Figure 30 shows the radiograph of an 82-year-old woman who reports a 1-month


history of shoulder pain. She is able to actively elevate her arm to 150 degrees but is
experiencing discomfort. Her sleep is disrupted because of the shoulder pain. What is
the most appropriate management?
1- Total shoulder arthroplasty
2- Hemiarthroplasty
3- Reverse shoulder arthroplasty
4- Arthroscopic shoulder debridement
5- Trial of anti-inflammatory medication or cortisone injection and/or deltoid
strengthening
DISCUSSION: The patient is experiencing rotator cuff tear arthropathy. Given that this
is the first medical treatment she has sought, a nonsurgical treatment plan of anti-
inflammatory medication or a corticosteroid injection is warranted. Proceeding to the
operating room without a trial of nonsurgical management is not indicated in this
patient population. Surgical procedures may be necessary in the future if nonsurgical
measures fail. The Preferred Response to Question # 30 is 5.
21
160
2011 Upper Extremity Self-Assessment Exam by Dr.Dhahirortho

31..A 53-year-old woman reports a 4-month history of gradual onset diffuse shoulder
pain and limited function. She has had no prior treatment, and her medical history is
unremarkable. Examination reveals globally painful active range of motion to 120
degrees forward elevation, 25 degrees external rotation with the arm at the side,
and internal rotation to the sacrum. Passive range of motion is also limited in
comparison with the contralateral shoulder. Radiographs are shown in Figures 31a
through 31c. What is the most appropriate management?

1- Sling immobilization and rest


2- Physical therapy for aggressive stretching
3- Intra-articular corticosteroid injection and stretching program
4- Manipulation of the shoulder under anesthesia
5- Arthroscopic subacromial decompression and capsular release

DISCUSSION: The patient has stage II adhesive capsulitis. Patients most commonly
affected are women between the ages of 40 and 60, and most cases are considered
idiopathic. The preferred method of treatment is an intra-articular corticosteroid
injection to decrease inflammation in the joint and allow for a gentle stretching
therapy program. Sling immobilization is contraindicated because it likely will promote
further joint contracture and prolonged recovery. Aggressive capsular stretching in the
early stages of the disease is often counterproductive, unless pain can be adequately
controlled with medication or injections. Manipulation under anesthesia and
arthroscopic surgical treatment are used when symptoms remain refractory despite
initial nonsurgical management. The Preferred Response to Question # 31 is 3.

22
161
2011 Upper Extremity Self-Assessment Exam by Dr.Dhahirortho

32.A 65-year-old woman with rheumatoid arthritis is unable to actively extend her
index, middle, ring, and little fingers secondary to tendon rupture. In performing a
flexor digitorum sublimis (FDS) of the middle/ring finger to extensor digitorum
communis (EDC) transfer to restore active metacarpophalangeal (MCP) joint
extension, the FDS should be passed
1- ulnarly, around the ulna in a dorsal direction.
2- radially, around the radius in a dorsal direction.
3- through the interosseous membrane.
4- through the intermetacarpal spaces between the index, middle, ring, and little
fingers.
5- through the lumbrical canals of the index, middle, ring, and little fingers.

DISCUSSION: Although the early use of FDS as a transfer to restore finger extension in
patients with radial nerve palsy was performed by passing the tendon through the
interosseous membrane, Nalebuff and Patel later modified this procedure for the
rheumatoid arthritis patient by passing the FDS radially, around the radius in a dorsal
direction. They felt that this provided a number of advantages, including: 1. technical
ease, 2. avoidance of synovial disease on the dorsum of the wrist, and 3. correction of
ulnar deviation of the fingers through the line of pull from the radial side of the
forearm. The Preferred Response to Question # 32 is 2.

33 A patient sustains a traumatic injury to the right shoulder. An axial CT scan is


shown in Figure 33. Management should include which of the following?
1- Sling and swathe
2- Excision of fragment
3- Closed reduction
4- Open reduction and internal fixation
5- Hemiarthroplasty

DISCUSSION: The 2-D axial CT scan shows a displaced glenoid fracture involving
approximately one third of the articular surface. Anatomic restoration by an open
reduction and internal fixation is necessary to avoid traumatic osteoarthrosis. Removal
of the fragment would likely result in instability of the joint. Closed reduction of the
23
162
2011 Upper Extremity Self-Assessment Exam by Dr.Dhahirortho

fragment is not possible in this injury, and there is no indication for a hemiarthroplasty
because the humerus is not involved. The Preferred Response to Question # 33 is 4.

34 A 47-year-old man undergoes a posterior cervical procedure for a benign tumor.


Postoperatively, severe dysfunction with decreased forward elevation and abduction
develops and he has lateral winging of the scapula. What is the recommended
treatment to best restore motion and function?
1- Rhomboids and levator transfer
2- Split pectoralis major transfer
3- Long head of triceps transfer
4- Scapulothoracic fusion
5- Infraspinatus transfer

DISCUSSION: The patient has sustained a permanent injury to the spinal accessory
nerve and has resultant scapular winging (lateral winging) because of trapezius palsy
with weakness in abduction and forward elevation. The modified Eden-Lange
procedure (transfer of the rhomboid minor, major, and levator scapulae) has been
shown to reliably restore range of motion and function. Split pectoralis major transfer
is performed to restore serratus anterior function. The long head of the triceps and
infraspinatus tendon transfers are rarely used for any shoulder muscle transfer. A
scapulothoracic fusion can also be performed for this problem, but the results are not
as effective as the Eden-Lange procedure. The Preferred Response # 34 is 1.

35 The standard Bankart lesion involves detachment of the labrum along with which
of the following capsular ligaments?
1- Superior glenohumeral ligament and coracohumeral ligament
2- Superior glenohumeral ligament and middle glenohumeral ligament
3- Middle glenohumeral ligament and inferior glenohumeral ligament
4- Inferior glenohumeral ligament
5- Superior glenohumeral ligament, middle glenohumeral ligament, and inferior
glenohumeral ligame

24
163
2011 Upper Extremity Self-Assessment Exam by Dr.Dhahirortho

DISCUSSION: The Bankart lesion involves detachment of the labrum corresponding to


the attachment of the middle and inferior glenohumeral ligaments. The superior
glenohumeral ligament and the coracohumeral ligament are too superior, inserting
near the biceps tendon, and play no role in the Bankart lesion. The Pre Res# 35 is 3.

36,Complications following a reverse shoulder prosthesis occur most frequently


when performed for what diagnosis?
1- Rotator cuff tear arthropathy with superior escape
2- Massive rotator cuff tear with osteoarthritis
3- Fracture-dislocation of the glenohumeral joint
4- Four-part proximal humeral fractures
5- Failed shoulder arthroplasty

DISCUSSION: Revision following failed shoulder arthroplasty is associated with the


highest complication rates, including dislocation, loosening, and decreased function.
However, when performed for rotator cuff tear arthropathy or failed rotator cuff
repairs, the complication rate is reasonably low. The complication rate is unknown
when the reverse total shoulder is used for fracture-dislocation or acute four-part
fractures of the proximal humerus. The Preferred Response to Question # 36 is 5.

37.A patient has severe cubital tunnel syndrome and marked wasting of the intrinsic
muscles of the hand. Why is the little finger held in an abducted position?
1- Accessory slip of the extensor digiti minimi attaching to the abductor digiti minimi
tendon
2- Tetanic contraction of the abductor digiti minimi
3- Radial collateral ligament insufficiency of the fifth metacarpophalangeal (MCP)
joint
4- Unopposed pull of the flexor digitorum profundus
5- Muscle innervation from a Martin-Gruber anastomosis

DISCUSSION: A Wartenberg's sign, where the little finger is held in an abducted


position, is associated with an ulnar nerve palsy. This happens when there is an
accessory slip of the extensor digiti minimi, which is innervated by the radial nerve,
25
164
2011 Upper Extremity Self-Assessment Exam by Dr.Dhahirortho

crossing ulnar to the center of the MCP joint to attach to the tendon of the abductor
digiti minimi and the proximal phalanx. The abductor digiti minimi and the volar
interosseous muscles are both innervated by the ulnar nerve; therefore, there is no
tetanic contraction of the abductor digiti minimi.
Unopposed pull of the flexor digitorum profundus results in excess flexion of the
proximal interphalangeal and distal interphalangeal joints of the hand as seen with a
clawing-type deformity. A Martin-Gruber anastomosis, which is a neural connection
between the ulnar and median nerves in the forearm, cannot explain this finger
position. The Preferred Response to Question # 37 is 1.

38Figure 38 shows the radiograph of a 41-year-old man who reports ulnar palmar
pain, decreased sensibility and tingling in the ring and little fingers, and a grating
sensation in the ulnar fingers with motion. He reports that he sustained a fall on an
outstretched hand 6 months ago. What is the most appropriate treatment option?

1- Ulnar gutter cast


2- Short arm cast
3- Carpal tunnel release
4- Decompression of Guyon's canal
5- Excision of a fractured hook of hamate

DISCUSSION: Excision of a fractured hook of hamate is the most appropriate


management. The patient has a hook of hamate fracture with ulnar nerve compression
and irritation of the flexor tendons by the fracture surfaces; this puts the tendons at
risk for rupture. Cast treatment will most likely not gain union of the fracture and will
not address the nerve or tendon problems. Decompression of Guyon's canal alone will
not address the tendon issue.
The Preferred Response to Question # 38 is 5.

26
165
2011 Upper Extremity Self-Assessment Exam by Dr.Dhahirortho

39 A 22-year-old javelin thrower reports that he has had increasing discomfort in his
right elbow and loss of distance from his throws for the past 3 months. Examination
reveals tenderness over the medial elbow. Application of valgus torque to the elbow
through a passive range of motion elicits pain from 70 degrees to 120 degrees of
flexion, with no pain at the limits of extension. What structure is primarily
responsible for the patient's symptoms?

1- Anterior bundle of the medial collateral ligament (MCL)


2- Posterior bundle of the MCL
3- Annular ligament
4- Triceps insertion
5- Olecranon osteophytes

DISCUSSION: The MCL is divided into anterior and posterior bundles; the anterior
bundle is subdivided into anterior and posterior bands. Sectioning studies showed that
the anterior band of the anterior bundle is the primary restraint to valgus stress at 30
degrees, 60 degrees, and 90 degrees; the posterior band of the anterior bundle is the
primary restraint at 120 degrees. Medial elbow pathology in a throwing athlete can
present with pain, instability, loss of velocity or control, or with ulnar nerve symptoms.
Differentiating between different causes of disability can be largely accomplished
through physical examination. The moving valgus stress test is performed by applying a
valgus stress to a maximally flexed elbow, then passively extending the elbow.
Reproduction of the patient's symptoms in the mid arc of flexion suggests MCL
insufficiency. Pain at the end point of extension suggests posterior compartment
symptoms, which were not present in this patient. The posterior bundle is a secondary
stabilizer at 30 degrees of flexion, and not susceptible to valgus load when the anterior
bundle is intact. The annular ligament and triceps insertion are not involved with
medial instability of the elbow. Olecranon osteophytes likely cause pain in terminal
extension of the elbow.
The Preferred Response to Question # 39 is 1.

27
166
2011 Upper Extremity Self-Assessment Exam by Dr.Dhahirortho

40 Which of the following statements best describes the typical early presentation of
osteochondritis dissecans of the elbow?
1- Often associated with loss of elbow extension
2- Often associated with catching or locking
3- Involves the capitellum or lateral trochlea
4- Presents in boys younger than age 10 years
5- Outlining of the margins of the lesion on MR arthrogram is a good prognostic sign

DISCUSSION: This condition is the result of repetitive valgus overload of the


radiocapitellar joint in the immature elbow. The clinical presentation is of lateral elbow
pain and loss of extension in a juvenile older than age 10 years. Panner's disease
typically affects the capitellum in boys younger than age 10 years. Osteochondritis
dissecans (OCD) of the elbow affects the capitellum and occasionally the radial head.
Fracturing of the OCD region can lead to an unstable fragment with margins outlined
on an MR arthrogram and can progress to loose bodies that cause clinical catching or
locking. These are typically late signs with a poorer prognosis. The Pre Re# 40 is 1.

41 Two years after undergoing a total shoulder arthroplasty, a patient reports


increasing pain, stiffness, and swelling, and has an increased white blood cell count.
Radiographs show lucencies around the glenoid and humeral components. You
suspect infection. Which of the following is the most likely responsible organism?
1- Staphylococcus aureus
2- Staphylococcus epidermidis
3- Propionibacterium acnes
4- Escherichia coli
5- Pseudomonas aeruginosa

DISCUSSION: The most likely organism to cause late infection in shoulder arthroplasty
is Propionibacterium acnes. This is a slow growing organism that is present in over 50%
of chronic infections. Staphylococcus epidermidis is the second most likely organism in
this setting, present in 15% of cases. The other three organisms are unlikely to present
with this clinical picture. The Preferred Response to Question # 41 is 3.

28
167
2011 Upper Extremity Self-Assessment Exam by Dr.Dhahirortho

42 The MRI scan of a patient with symptomatic shoulder pain reveals subacromial
bursitis. What markers have been shown to be significant contributors to this pain?
1- Metalloproteases
2- Alpha fetoprotein
3- Prostate-specific antigen (PSA)
4- Carcinoembryonic antigen (CEA)
5- CA-125

DISCUSSION: Several inflammatory markers have been shown to be elevated in


subacromial bursitis. These include metalloproteases, tumor necrosis factors, and
cyclooxygenase 1 and 2. The other answers provided are all tumor markers and not
typically present in routine subacromial bursitis ("impingement syndrome"). The
Preferred Response to Question # 42 is 1.

44 An 11-year-old boy sustained a fall onto his outstretched right hand while playing
soccer. Examination reveals tenderness in the anatomic snuff box. Wrist radiographs
reveal a scaphoid fracture. This injury most commonly presents with which of the
following?
1- Within the distal one third of the scaphoid
2- Within the middle one third of the scaphoid
3- Within the proximal one third of the scaphoid
4- In association with injury to the scapholunate ligament
5- As a unicortical injury

DISCUSSION: The distal pole of the scaphoid ossifies before the proximal pole, resulting
in an increased incidence of distal one third fractures and avulsions of the distal radial
aspect of the scaphoid (59% to 94%) as compared with adults. Scaphoid fractures in
the pediatric population can be seen in association with distal radius fractures, but are
not commonly associated with ligamentous injury. While 23% of pediatric scaphoid
fractures are unicortical, bicortical injuries still predominate. Most pediatric scaphoid
fractures are nondisplaced and heal with 4 to 6 weeks of immobilization. As in adults,
displaced fractures are treated with open reduction and internal fixation. The
Preferred Response to Question # 44 is 1.
29
168
2011 Upper Extremity Self-Assessment Exam by Dr.Dhahirortho

45 A 25-year-old man was involved in an altercation. Examination reveals loss of


active extension of the middle finger metacarpophalangeal (MCP) joint. A diagnosis
of sagittal band rupture is made. Which of the following is considered the key
diagnostic finding?
1- Extensor lag of 30 degrees
2- Extensor lag of 60 degrees
3- Positive Bunnell intrinsic tightness test
4- Ability to maintain active extension of the interphalangeal joints
5- Ability to maintain MCP extension after passive extension

DISCUSSION: In sagittal band rupture, the extensor tendon may subluxate into the
valley between the metacarpal heads. The patient will not be able to actively extend
the MCP joint from a flexed position with the subluxated tendon, but will be able to
maintain MCP extension after it has been passively extended. Extensor lags can have
other etiologies other than extensor digitorum communis subluxation such as tendon
laceration or rupture, posterior interosseous nerve palsy, but in these conditions,
patients cannot maintain MCP extension. Active interphalangeal extension can be
achieved with the intrinsic muscles that are not affected by sagittal band rupture.
The Preferred Response to Question # 45 is 5.

46 A 45-year-old man has been treated nonsurgically with a fracture brace for a
closed midshaft humeral fracture. At 16 weeks after his injury he has continued pain
and gross motion at the fracture site. A radiograph is shown in Figure 46. What is the
most appropriate and reliable management at this point?
1- Sling immobilization and electrical stimulation
2- Continued functional bracing and repeat radiographs in 6 weeks
3- Closed reduction and intramedullary nailing
4- Open reduction and compression plating with autograft
5- Open reduction with fully locked plating and allograft

DISCUSSION: The radiograph reveals a delayed union of a midshaft


humerus fracture that has been treated appropriately with closed
fracture bracing. The gold standard remains compression plate
30
169
2011 Upper Extremity Self-Assessment Exam by Dr.Dhahirortho

fixation with autograft, most commonly iliac crest bone graft. Intramedullary nailing is
associated with a higher incidence of nonunion and iatrogenic rotator cuff damage.
Fully locked plating is unnecessary along with allograft in this setting. Immobilization
with electrical stimulation offers little success in this atrophic delayed union.
The Preferred Response to Question # 46 is 4.

47 A 45-year-old man reports a history of a popping sensation and pain in the right
shoulder while lifting boxes 6 months ago. The pain has persisted with loss of motion
of the shoulder. Radiographs and MRI scans are shown in Figures 47a through 47d.
Which of the following studies is likely to produce a significant positive result?

1- Rheumatoid factor
2- HLA-B27
3- Synovial fluid analysis
4- MRI of the upper cervical spine
5- Urine screen for tetrahydrocannabinol (THC)

DISCUSSION: The patient has a neuropathic joint secondary to syringomyelia that can
be seen on a cervical MRI scan. The patient sustained minimal trauma that lead to a
chronic anterior glenohumeral dislocation. He did not seek treatment for several
months and has a massive rotator cuff tear and hygroma on MRI in addition to the
chronic dislocation. Rheumatoid arthritis does not present with a neuropathic picture,
except theoretically as the result of numerous intra-articular cortisone injections. This
Charcot picture is inconsistent with ankylosing spondylitis or gout. Cannabis use is not
typically associated with seizures that could produce anterior as well as posterior
shoulder dislocations.
The Preferred Response to Question # 47 is 4.

31
170
2011 Upper Extremity Self-Assessment Exam by Dr.Dhahirortho

48 A 55-year-old patient with rheumatoid arthritis reports increasing elbow pain and
swelling for the past 2 months. She underwent a cemented, semiconstrained elbow
arthroplasty 8 years ago. Laboratory studies show a normal peripheral white blood
cell count; however, the erythrocyte sedimentation rate and C-reactive protein level
are elevated. Radiographs are shown in Figures 48a and 48b. Which of the following
organisms is most difficult to eradicate?

1- Streptococcus viridans
2- Staphylococcus epidermidis
3- Escherichia coli
4- Vibrio parahaemolyticus
5- Clostridium difficile

DISCUSSION: The patient's history and radiographs are suspicious for a relatively
aggressive infection. Staphylococcus epidermidis is difficult to eradicate because of its
encapsulation. The lytic area surrounding both the ulnar and humeral components
suggests that the prosthesis is also loose. This revision will require component
removal, antibiotic spacer placement, and parenteral antibiotics. The Pre Res# 48 is 2.

49What is the most common complication associated with the exposure method
shown in Figure 49?
1- Ulnar nerve injury
2- Symptomatic hardware
3- Lateral instability
4- Medial instability
5- Nonunion

DISCUSSION: Olecranon osteotomy provides excellent exposure of the articular surface


of the distal humerus, and allows for direct visualization of the entirety of the distal
humerus. Complications associated with this exposure include infection, nonunion,
hardware failure, symptomatic hardware, and improperly placed hardware, limiting
forearm rotation. Recent studies have shown a nonunion rate of 0% to 2%; the lack of
32
171
2011 Upper Extremity Self-Assessment Exam by Dr.Dhahirortho

overlying soft tissue makes symptomatic hardware common, with rates from 8% to
33%. Additionally, olecranon fixation is often removed at the time of other subsequent
procedures performed on the elbow. Olecranon osteotomy does not destabilize the
collateral ligaments of the elbow. The Preferred Response to Question # 49 is 2.

50 What is the effect of shortening of metacarpal fractures?


1- Causes the greatest degree of extensor lag in the index finger
2- Causes the greatest degree of extensor lag in the little finger
3- Results in an average extensor lag of 7 degrees for every 2 mm of shortening
4- Results in an average extensor lag of 14 degrees for every 2 mm of shortening
5- Has no effect on grip strength
DISCUSSION: Cadaveric models have demonstrated a 7-degree extensor lag for every 2
mm of metacarpal shortening, with the amount of lag increasing in a linear fashion.
There was no statistical difference in the amount of lag in regard to the digit involved.
Based on muscle length-tension relationships, cadaveric models have also been used
to demonstrate an 8% loss of power secondary to decreased interosseous force
generation with 2 mm of shortening. Because the intrinsic muscles of the hand
contribute anywhere from 40% to 90% of grip strength, decreased interosseous force
generation secondary to metacarpal shortening will invariably cause a decrease in grip
strength. The Preferred Response to Question # 50 is 3.

51 Figures 51a and 51b show the AP and lateral radiographs of the elbow of a 26-
year-old man who fell. Closed reduction was performed in the emergency
department, and management consisted of immobilization for 3 weeks prior to the
initiation of motion. At 12 weeks after injury, he reports continued feelings of
instability and catching in his elbow when using his arms to rise from a chair. Which
of the following procedures needs to be performed, at a minimum, to reestablish
stability of the elbow?
1- Medial collateral ligament repair
2- Medial collateral ligament reconstruction
3- Hinged external fixation
4- Lateral collateral ligament repair
5- Lateral collateral ligament reconstruction
33
172
2011 Upper Extremity Self-Assessment Exam by Dr.Dhahirortho

DISCUSSION: The patient has chronic posterolateral instability of the elbow following
dislocation. The lateral collateral ligament complex is responsible for maintaining
stability of the elbow. Because of the chronicity of the injury, the ligamentous tissues
are frequently attenuated and not amenable to simple repair; while the native
ligament can be imbricated, reconstruction with allograft or autograft is
recommended. Medial collateral ligament reconstruction or hinged external fixation is
needed only if restoration of the lateral ligamentous complex does not restore elbow
stability; however, these procedures are rarely required. Lateral elbow pain when
rising from a chair is equivalent to a positive pivot shift test. Preferred Resp # 51 is 5.
52 What is the most common mode of failure following unconstrained total elbow
arthroplasty?
1- Polyethylene wear
2- Bushing wear
3- Instability
4- Component fracture
5- Loosening of the humeral component
DISCUSSION: Elbow instability after placement of an unconstrained implant is most
often the result of ligamentous insufficiency that can occur late after the index
procedure. Instability can also occur from component malpositioning that creates
undue stress to the collateral ligaments during the life of the prosthesis. Instability
leads to revision surgery in many patients. Polyethylene wear and bushing wear are
more common in linked and semiconstrained elbow arthroplasties. Loosening of
humeral components may occur with aseptic or septic disease. Component fracture is
uncommon. The Preferred Response to Question # 52 is 3.
53 A 41-year-old woman with diabetes mellitus fell onto her outstretched arm and
sustained an injury to the right elbow. Radiographs are shown in Figures 53a and
53b. What is the most appropriate management?
1- Open reduction and internal fixation
2- MRI of the elbow to assess the integrity of the collateral
ligaments
3- Immobilization in a long arm cast for 3 weeks
4- Short-term immobilization in a splint, followed by early motion exercises
5- Radial head replacement
34
173
2011 Upper Extremity Self-Assessment Exam by Dr.Dhahirortho

DISCUSSION: The radiographs reveal a capitellum fracture with anterior displacement.


To regain concentric and stable joint motion, this fragment requires reduction and
stabilization. Without a joint dislocation, the ligaments are unlikely to be damaged and
do not require further assessment with MRI. Closed reduction may be considered, but
is unlikely to be successful. Without anatomic reduction of the fracture fragment,
immobilization in either a long arm cast or a splint will not provide optimal outcomes.
Based on the radiographs, the radial head is intact and does not require replacement.
The Preferred Response to Question # 53 is 1.

54 A 38-year-old man reports a 6-week history of shoulder pain and stiffness after
falling on the stairs and landing onto the affected side. Radiographs are shown in
Figures 54a and 54b. What is the most appropriate treatment?

1- Physical therapy including ultrasound and gentle stretches


2- Closed manipulation of the shoulder
3- MRI and possible rotator cuff repair
4- Open glenohumeral reduction, with possible lesser tuberosity transfer
5- Shoulder hemiarthroplasty

DISCUSSION: The patient has a chronic posterior shoulder dislocation of 6-weeks


duration. A CT scan will provide preoperative information regarding the size of the
McLaughlin or reverse Hill-Sachs lesion. Open glenohumeral reduction with transfer of
the lesser tuberosity and attached subscapularis has been shown to be successful in
stabilizing a posterior dislocation. Closed reduction is highly unlikely to achieve a
reduction and may cause displacement of an unrecognized humeral surgical neck
35
174
2011 Upper Extremity Self-Assessment Exam by Dr.Dhahirortho

fracture. Hemiarthroplasty would be considered for lesions involving more than 50% of
the humeral head or when the joint has been dislocated for several months and late
collapse of the head postreduction is likely. Rotator cuff tears are not commonly
associated with posterior shoulder dislocation. The Preferred Response # 54 is 4.

55 A 22-year-old motorcyclist sustains open fractures to the left radial shaft and
second and third metacarpals with exposed extensor tendon and bone. The fractures
are approached via the dorsal open wounds of the forearm and hand with no
additional incisions made. The radiograph and clinical photograph of the remaining
defect in the hand are shown in Figures 55a and 55b. The remaining wound can be
most appropriately covered with which of the following?
1- Split-thickness skin grafting
2- Posterior interosseous rotational flap
3- Radial forearm rotational flap
4- Groin flap
5- Free lateral arm flap

DISCUSSION: After adequate debridement, there is exposed bone, tendon, and


hardware. Split-thickness skin grafting over exposed tendon will not have a viable bed
to support the graft. The tendons would not have healthy surrounding tissue, resulting
in poor tendon gliding. The dorsal wound has disrupted the posterior interosseous
artery that runs in the septum between the extensor digiti minimi and the extensor
carpi ulnaris. Following the reconstructive ladder, the radial forearm rotational flap
accomplishes wound coverage with a local flap rather than a groin flap (a distant flap)
or a lateral arm flap (microvascular free tissue transfer). The Preferred Resp # 55 is 3.

56 What preoperative patient factor has been shown to most closely correlate with
poor results after a latissiumus dorsi transfer for an irreparable rotator cuff tear?
1- Age of younger than 70 years
2- Positive lift-off test
3- Previous shoulder surgery
4- Loss of passive external rotation
5- Male gender
36
175
2011 Upper Extremity Self-Assessment Exam by Dr.Dhahirortho

DISCUSSION: Patients with a positive lift-off test have a tear of the subscapularis
tendon. Patients with a subscapularis tendon tear did much worse than other patients
in the studies by Gerber and associates and Irlenbusch and associates. Latissimus dorsi
muscle transfer during the primary surgery when a complete rotator cuff repair could
not be performed results in a better outcome than a muscle transfer done as a second
surgery, but other prior surgery was not shown to affect transfer results. Iannotti and
associates found poor results in patients who were female or had external rotation and
forward flexion weakness. The Preferred Response to Question # 56 is 2.

57 A 22-year-old man reports that he initially dislocated his shoulder while playing
basketball 2 years ago and was subsequently treated with an arthroscopic Bankart
repair. Despite appropriate rehabilitation, the patient continues to report recurrent
instability. An axillary view radiograph and CT scan are shown in Figures 57a and 57b.
What is the most appropriate management at this time?

1- Supervised physical therapy


2- Arthroscopic capsulorrhaphy and labral repair
3- Open shoulder capsulorrhaphy and labral repair
4- Open shoulder capsulorrhaphy and bone block
5- Shoulder arthrodesis

DISCUSSION: Although the changes are subtle on the radiograph, an anterior inferior
glenoid bone defect is clearly evident on the CT scan. With loss of greater than 20% to
25% of the glenoid width, patients may experience persistent instability despite
appropriate labral repair and capsulorrhaphy. Therefore, nonsurgical management
with supervised therapy or surgical treatments that do not address the bony defect,
such as arthroscopic or open labral repair and capsulorrhaphy, are not likely to
stabilize the joint. An open shoulder stabilization procedure with a bone block should
address the defect and stabilize the joint. Shoulder arthrodesis is not warranted in this
patient at this time because the shoulder is likely salvageable.
Preferred Resp # 57 is 4.

37
176
2011 Upper Extremity Self-Assessment Exam by Dr.Dhahirortho

58 Scapular notching following reverse shoulder arthroplasty may be minimized by


what technical modification?
1- Horizontal humeral cut
2- Superior inclination of the baseplate
3- Inferior inclination of the baseplate
4- Use of a 36-mm glenosphere
5- Use of a retentive polyethylene liner
DISCUSSION: Biomechanical studies have shown that a 10-degree inferior inclination
may decrease scapular notching; whereas superior inclination may worsen notching.
Scapular notching has been recognized as a complication following reverse shoulder
arthroplasty. Mechanical abutment of the humeral component possibly leads to
erosion of the anteroinferior scapular neck, with progressive vulnerability of the
inferior baseplate screws. A horizontal humeral cut does not affect notching because
the humeral component causes the notching, not the bone on the humerus.
Glenosphere size has not been shown to correlate with scapular notching. The
Preferred Response to Question # 58 is 3.

59 A 42-year-old man sustained a displaced humeral surgical neck fracture that was
well-fixed with proximal humeral plating. Postoperative management consisted of a
sling for 6 weeks, followed by physical therapy. Examination at 4 months after
surgery revealed passive 90 degrees forward elevation, 10 degrees external rotation,
and internal rotation to the greater trochanter. Radiographs show an anatomically
healed fracture and no evidence of loose hardware; the plate is appropriately
positioned. What is the most likely reason for the decreased range of motion?
1- Hardware impingement under the acromion
2- Rotator cuff tear
3- Postoperative scar tissue
4- Fracture malunion
5- Development of posttraumatic glenohumeral arthritis

DISCUSSION: The patient has reduced motion secondary to postoperative scarring. If


excellent stability is obtained in the operating room, immediate passive motion can
begin. Hardware impingement under the acromion will not account for limited external
38
177
2011 Upper Extremity Self-Assessment Exam by Dr.Dhahirortho

rotation to the side. There is no evidence for a rotator cuff tear. A rotational malunion
does not reduce motion in all planes. Four months after surgery it is unlikely that
arthritic changes developed that are affecting his range of motion. The Preferred
Response to Question # 59 is 3.

60 A 74-year-old man underwent a hemiarthroplasty with acromioplasty for rotator


cuff tear arthropathy 2 years ago. Despite continued therapy, he is still unable to
elevate his arm beyond 40 degrees. Attempted elevation is painful and demonstrates
bulging in the anterosuperior aspect of his shoulder. Radiographs show a well-
positioned hemiarthroplasty without signs of loosening. What is the most
appropriate treatment for this patient?
1- Conversion to a total shoulder arthroplasty
2- Conversion to a reverse shoulder arthroplasty
3- Continued physical therapy
4- Cortisone injection
5- Anti-inflammatory medication

DISCUSSION: The patient is experiencing anterosuperior escape with attempted


shoulder elevation. A conversion to a reverse shoulder arthroplasty will provide the
stability to allow active elevation without subluxation. Further physical therapy,
cortisone injection, or anti-inflammatory medication will not resolve this instability. A
total shoulder arthroplasty is contraindicated because of the anterosuperior escape.
The Preferred Response to Question # 60 is 2.

62 What is the name of the structure that is identified by the arrow on the sagittal
T1-weighted MRI scan shown in Figure 62?

1- Infraspinatus
2- Teres minor
3- Subscapularis
4- Long head of the triceps
5- Latissimus dorsi

39
178
2011 Upper Extremity Self-Assessment Exam by Dr.Dhahirortho

DISCUSSION: The sagittal T1-weighted MRI scan is useful for interpreting the quality of
a muscle; the arrow is identifying the teres minor. The Preferred Response# 62 is 2.

64 What is the effect of performing a flexor tenosynovectomy with an open carpal


tunnel release for idiopathic carpal tunnel syndrome?
1- Increased risk of nerve injury
2- Improved postoperative finger flexion
3- No added long-term clinical benefit versus open carpal tunnel release alone
4- Increased postoperative pain
5- Decreased recurrence of carpal tunnel syndrome

DISCUSSION: In patients with idiopathic carpal tunnel syndrome, flexor


tenosynovectomy has not been shown to change the clinical outcome compared with
open carpal tunnel release alone. This has been demonstrated in a randomized clinical
trial of open carpal tunnel release with or without flexor tenosynovectomy. There has
also been no evidence to suggest there is an added risk to performing the flexor
tenosynovectomy. At time of surgery, the gross or histologic appearance of the flexor
tenosynovium does not correlate with preoperative symptoms nor with clinical
outcomes. The histology of the tenosynovium has been shown to be that of fibrosis in
a setting of chronic inflammatory changes and no evidence of an acute inflammatory
process exists. There may be an added role for flexor tenosynovectomy in non-
idiopathic carpal tunnel syndrome such as in patients with renal disease or diabetes.
The Preferred Response to Question # 64 is 3.

65 A 22-year-old professional X-games motocross bike rider is thrown from his ride
during a jump. He lands directly onto the point of his left shoulder and feels sharp
pain. Examination reveals mild deformity over the lateral clavicle and bruising. A
radiograph is shown in Figure 65. What is the most appropriate treatment?
1- Sling and swathe
2- Kenny-Howard brace
3- Percutaneous repair with smooth Kirschner wires
4- Open distal clavicle resection and transfer of the coracoacromial ligament
5- Open reduction and internal fixation
40
179
2011 Upper Extremity Self-Assessment Exam by Dr.Dhahirortho

DISCUSSION: The history and radiograph indicate a traumatic displaced distal clavicle
fracture in a professional athlete. Open reduction and internal fixation provides the
best chance to heal and retain shoulder function. Smooth Kirschner wires are at risk for
migration, and acute acromioclavicular joint reconstruction with coracoacromial
ligament transfer is unnecessary when there is good quality bone stock. Nonsurgical
management may lead to a high chance of nonunion. Kenny-Howard braces may cause
skin irritation and breakdown. The Preferred Response to Question # 65 is 5.

66 What structure is at risk during arthroscopic resection of the capsule just anterior
to the radial head and neck?

1- Radial nerve
2- Median nerve
3- Brachial artery
4- Lateral collateral ligament
5- Extensor carpi radialis brevis (ECRB) tendinous origin

DISCUSSION: Neurovascular injuries during anterior capsular release have been


reported throughout the literature. During arthroscopic release/resection of the
capsule located directly anterior to the radial head, the structure at greatest risk is the
radial nerve, located along the anterolateral capsule just distal to the radiocapitellar
joint. The radial nerve is consistently the neurovascular structure located closest to the
capsule; it lies closer to the capsule distally than proximally. One cadaveric study noted
the radial nerve to lie 9 mm from the capsule proximally, but only 3.5 mm from the
capsule distally. The median nerve and brachial artery are also at risk, but are located
more medially, and are protected by the brachialis. Similarly, the median nerve is
located closer to the capsule distally than proximally; thus, the current
recommendations are to release the anterior capsule from its humeral insertion rather
than its distal insertion. The lateral collateral ligament is located posterior and lateral
to the radial head, as is the ECRB origin, and should not be at risk during resection of
the anterior capsule.
The Preferred Response to Question # 66 is 1.

41
180
2011 Upper Extremity Self-Assessment Exam by Dr.Dhahirortho

67 Which of the follow scenarios is most likely to be amenable to a complete repair


of a massive rotator cuff tear?
1- 42-year-old woman with rheumatoid arthritis
2- 45-year-old man with a tear associated with an anterior shoulder dislocation
3- 49-year-old man who underwent repair of an ipsilateral rotator cuff 3 years ago
4- 56-year-old male laborer with superior humeral migration on radiographs
5- 59-year-old woman with muscular atrophy noted in the supraspinatus fossa

DISCUSSION: Whereas a rotator cuff tear associated with an acute anterior dislocation
in 45-year old patient may be massive, its acute nature typically means that significant
retraction and atrophy of the musculature has not occurred. Therefore, repair is often
complete and tension-free. A massive tear associated with rheumatoid arthritis is likely
one of chronic attrition with poor tendon tissue because of the underlying disease and
chronic corticosteroid use. Repairs of massive chronic rotator cuff tears have been
reported to have a 50% rate of retear and this rate would be expected to be higher in
the revision setting and with evident supraspinatus atrophy on physical examination.
Superior humeral migration on static upright radiographs indicates loss of the superior
glenoid rim, leading to rotator cuff tear arthropathy. The Preferred Respon# 67 is 2.

68 Based on the clinical photograph, radiographs, and biopsy specimen shown in


Figures 68a through 68d, what is the most likely diagnosis?

1- Calcium pyrophosphate deposition disease


2- Bacterial infection
3- Fungal infection
4- Gout
5- Giant cell tumor
42
181
2011 Upper Extremity Self-Assessment Exam by Dr.Dhahirortho

DISCUSSION: The patient has gout. Unfortunately, gout may mimic several conditions
affecting the small joints of the hand, including infection. The histologic specimen
shows negatively birefringent intracellular rods consistent with gout. The histology
rules out giant cell tumor and calcium pyrophosphate deposition disease. The
Preferred Response to Question # 68 is 4.

69 Figures 69a and 69b show the radiographs of a 62-year-old man with severe
radially sided wrist pain. Management has consisted of wrist splinting, nonsteroidal
anti-inflammatory drugs, and activity modification, but he continues to have pain
and reports difficulty sleeping. What is the most appropriate treatment for this
patient?
1- Arthroscopic debridement
2- Open reduction and internal fixation
3- Scaphoid nonvascularized bone graft and screw
fixation
4- Scaphoid vascularized bone graft and screw
fixation
5- Scaphoid excision and 4-corner fusion

DISCUSSION: Scaphoidectomy and 4-bone fusion is


the most appropriate management based on the
choices available. The patient has arthritic changes of SNAC (scaphoid nonunion
advanced collapse) wrist, stage III. Stage I is at the radial styloid, stage II is at the
radioscaphoid joint, and stage III is at the midcarpal joint. Arthroscopic debridement is
not appropriate in patients with arthrosis. Attempting to achieve scaphoid union is
only appropriate if there is no arthrosis or the changes are classified as stage I where
radial styloidectomy can be performed. The Preferred Respo # 69 is 5.

70 A 30-year-old right-hand dominant man has pain in the right side of his upper
torso and right extremity after being involved in a car accident. Examination reveals
local tenderness, intact skin, and no dysphagia. Figure 70 shows an axial 2-D CT scan.
Treatment should include which of the following?

43
182
2011 Upper Extremity Self-Assessment Exam by Dr.Dhahirortho

1- Observation
2- Closed reduction
3- Closed reduction and percutaneous pinning
4- Closed reduction and a figure-of-8 splint
5- Open reduction and internal fixation

DISCUSSION: The CT scan shows a right anterior sternoclavicular joint dislocation.


These injuries are best managed with simple observation. Anterior dislocations rarely
have any long-term negative sequelae and are well tolerated, rarely leading to any
clinically relevant symptoms. Therefore, attempts to reduce the joint are not
necessary. The Preferred Response to Question # 70 is 1.

71 A 50-year-old man fell from a ladder onto his left shoulder and sustained the
injury shown in the radiographs in Figures 71a and 71b. He underwent surgery with
repair of the coracoclavicular ligaments and deltotrapezial fascia with
coracoclavicular screw placement. Which of the following statements regarding
postoperative complications is most accurate?

1- Hardware migration is more likely than with acromioclavicular pinning.


2- Failure of fixation is usually at the level of the clavicle.
3- Hardware removal is avoided to prevent late displacement.
4- Neurologic injury most likely involves the axilllary nerve.
5- Acromioclavicular arthritis is more likely than with nonsurgical management.

DISCUSSION: Whereas pain and functional disturbance may persist with nonsurgical
management, the lack of articular surface contact prevents arthritic symptoms from
developing. Cartilage injury caused by trauma and any persistent joint incongruity
following repair would contribute to posttraumatic arthritis. Pinning across the
44
183
2011 Upper Extremity Self-Assessment Exam by Dr.Dhahirortho

acromioclavicular joint has a high incidence of hardware migration and potential


catastrophic consequences. Most cases of lost fixation of coracoclavicular screws are at
the level of the thread purchase in the coracoid. Routine hardware removal at 8 to 12
weeks is recommended to avoid screw breakage because of natural movement
between the clavicle and scapula. The axillary nerve passes around the inferior edge of
the subscapularis and is anatomically distant to the the coracoid. The
musculocutaneous nerve would have the closest anatomic position to the coracoid.
The Preferred Response to Question # 71 is 5.

72 Following total elbow arthroplasty, patients should be instructed to


1- return to impact activities such as golf or tennis.
2- permanently limit the load bearing of that arm to 5 pounds or less.
3- aggressively strengthen the triceps immediately following surgery.
4- immobilize the wrist and hand for 4 weeks postoperatively to minimize stress on
the surgical site.
5- avoid pronation and supination to reduce torsional stress on the implant.

DISCUSSION: Current recommendations are for a lifetime restriction of load bearing


and avoidance of impact activities following total elbow arthroplasty (TEA). TEA is a
very effective procedure in reducing pain or reconstructing previously
unreconstructable fractures. However, its usage must be tempered with the limitations
of currently available prostheses. Aggressive triceps strengthening must be delayed
following TEA to allow healing of the triceps attachment, regardless of the surgical
approach. Wrist and hand mobilization should begin immediately postoperatively to
prevent stiffness. Pronation and supination should not stress a humeral ulnar
arthroplasty. The Preferred Response to Question # 72 is 2.

73 Which of the following is considered a contraindication to elbow arthroscopy?


1- Osteonecrosis of the elbow (Panner disease)
2- Loose body in the ulnohumeral joint
3- Status post open reduction and internal fixation of a radial head fracture
4- Ulnar neuropathy with prior submuscular ulnar nerve transposition
5- Elbow stiffness
45
184
2011 Upper Extremity Self-Assessment Exam by Dr.Dhahirortho

DISCUSSION: Neurovascular complications are the most common complications


reported with elbow arthroscopy. Any distortion in the anatomy of the elbow,
especially when it involves neurovascular structures, such as a prior ulnar nerve
transposition, increases the risk of neurovascular injury and is generally considered a
contraindication to elbow arthroscopy. The other answers listed are either indications
for arthroscopy or are not contraindications for the procedure.Prefer Respo# 73 is 4.

74 Which of the following pathologic entities is most often encountered in


association with the clinical diagnosis of internal impingement of the shoulder?
1- Bankart lesion
2- SLAP tear
3- Anterior capsular contracture
4- Full-thickness rotator cuff tear
5- Humeral avulsion of glenohumeral ligaments (HAGL) lesion

DISCUSSION: A SLAP tear with posterior extension of the labral detachment is felt to be
an important aspect of pathology in internal impingement. Whether this is the cause of
condition or a result of the altered glenohumeral mechanics is still debated. While
described as instability after repetitive microtrauma, it is not associated with Bankart
or HAGL lesions as with gross dislocations. The current theories do not associate it with
anterior capsular contracture. An articular-sided partial-thickness tear of the posterior
supraspinatus can be seen but full-thickness tears have not been described. The
Preferred Response to Question # 74 is 2.

75 A 17-year-old quarterback reports shoulder pain localized over the anterior aspect
of the shoulder that occurs during the follow through phase of throwing. The pain
worsens toward the end of the game, but becomes asymptomatic the next day. He
denies any pain during the cocking phase of throwing or during normal daily
activities. Examination reveals a negative relocation test and a negative posterior
load and shift test. Motion of the shoulder is normal. An MRI arthrogram is shown in
Figure 75. Based on the history, examination, and MRI findings, what initial
treatment should be recommended?

46
185
2011 Upper Extremity Self-Assessment Exam by Dr.Dhahirortho

1- Labrum repair
2- Capsular release
3- Labrum debridement
4- Physical therapy emphasizing a throwing program
5- Physical therapy emphasizing an internal rotation stretching program

DISCUSSION: The MRI scan shows a small amount of contrast between the posterior
labrum and the glenoid, suggesting a posterior labral tear. The patient's symptoms are
more consistent, however, with rotator cuff deconditioning because of the timing of
his pain during the throwing motion and increased severity at the end of the game.
Treatment should focus on reconditioning of the rotator cuff and scapular stabilizers,
combined with a return to throw program. Posterior labral tears are often found on
MRI scans of asymptomatic throwers, and therefore, should not be considered the
primary cause of a patient's symptoms unless it is supported by the history and
physical examination. Internal rotation contractures can cause a similar pain pattern,
but this patient has full and equal range of motion. The Preferred Response # 75 is 4.

76 A 7-year-old boy is referred to your office 3 months after jamming his finger while
playing basketball. Examination reveals 40 degrees of active and passive motion at
the proximal interphalangeal (PIP) joint. The PIP joint is stable to radial and ulnar
stressing. Radiographs are shown in Figures 76a and 76b. What is the most
appropriate management?
1- Observation
2- Corrective osteotomy
3- Ostectomy
4- Hand therapy for aggressive stretching
5- Dynamic splinting

DISCUSSION: The most appropriate


management is an ostectomy, or resection of
the bone in the subcondylar fossa region. This is a malunion where the subcondylar
fossa is blocked by malaligned bone. Because it is a bony block to motion, stretching or
dynamic splinting will be of no benefit. The physis of the proximal phalanx is proximal,
47
186
2011 Upper Extremity Self-Assessment Exam by Dr.Dhahirortho

making remodeling of a fracture at the distal end very unlikely. A corrective osteotomy
has a risk of osteonecrosis of the very small distal fragment. The Preferred Res# 76 is 3.

77 A 27-year-old man sustains an injury in a fall while downhill skiing. Two days after
injury he is seen by an orthopaedic surgeon and is diagnosed with a clavicle fracture.
Examination and radiographs reveal 3 cm of shortening between the fracture
fragments of the midshaft clavicle fracture. The surgeon has a discussion with the
patient concerning surgical versus nonsurgical treatment. With regards to results, the
patient is informed that they are similar concerning which of the following?
1- Nonunion rates
2- Infection
3- Shoulder range of motion
4- Shoulder strength
5- Shoulder rotational endurance

DISCUSSION: Shoulder range of motion is well maintained for both surgical and
nonsurgical managment. Recent reports suggest that nonsurgical management of this
fracture pattern may result in deficits of shoulder endurance and strength. Nonunion
rates are significantly lower with surgical repair. Patient satisfaction, as determined by
Constant scores, DASH, and patient-specific questionnaires, was higher with surgical
intervention. Shoulder strength and rotational endurance are improved with surgical
repair. The Preferred Response to Question # 77 is 3.

78 Figure 78 shows the clinical photograph of a patient who injured his finger while
playing football. He cannot actively flex the distal interphalangeal joint of the ring
finger. Which of the following is the most accurate statement regarding the injury
shown?
1- The tendon is attached to the avulsed fragment from the distal phalanx.
2- There is no difference in time sensitivity in an acute injury
whether or not the tendon has retracted into the palm.
3- In a chronic (> 3 months) case of flexor digitorum profundus
(FDP) avulsion, the FDP should be tenodesed to the flexor digitorum sublimis (FDS).
4- If the FDP is advanced more than 1.5 cm, there is a risk for quadriga effect.
48
187
2011 Upper Extremity Self-Assessment Exam by Dr.Dhahirortho

5- The method of repair does not affect repair gapping or strength of the tendon
repair.

DISCUSSION: Overadvancement of the FDP tendon is one of the causes of the quadriga
effect. Relative shortening of an FDP tendon decreases the excursion of the
neighboring FDP tendons because they originate from a common muscle belly. The
patient reports a weak grasp. Answer 1 is not correct because there can be a fracture
and the tendon can avulse off of the fracture fragment (Trumble JHS-A 1992). Whether
the tendon has retracted into the palm or not does matter because retraction into the
palm allows pulleys to collapse and contract and it also means that the vinculae have
been stripped off of the tendon. Regarding answer 3, in chronic cases where the FDS is
intact and strong, many patients may be better off with a sublimis finger and no FDP
reconstruction that could, in the worst case scenario, worsen a functional proximal
interphalangeal joint. Regarding the repair method, there is recent research showing
method of repair (button vs anchor), suture type, and method do affect the
biomechanical properties of the repair. The Preferred Response to Question # 78 is 4.

79 What is the most common complication associated with the treatment of the
distal biceps ruptures as shown in Figures 79a and 79b?
1- Re-rupture
2- Radioulnar synostosis
3- Posterior interosseous nerve injury
4- Lateral antebrachial cutaneous nerve irritation
5- Radial fracture

DISCUSSION: The patient shown underwent distal


biceps repair with a button technique. Among the
reports in the literature, the most commonly
noted complication associated with this
technique is lateral antebrachial cutaneous nerve irritation. Re-rupture, radioulnar
synostosis, and posterior interosseous nerve injury can occur, but are not as common
as lateral antebrachial cutaneous nerve injury.The Preferred Response # 79 is 4.

49
188
2011 Upper Extremity Self-Assessment Exam by Dr.Dhahirortho

80 A 16-year-old right-hand dominant male pitcher has had increasing pain in his
dominant shoulder for the past 6 months without treatment. A coronal T2-weighted
MRI scan is shown in Figure 80. What is the most appropriate treatment plan?
1- Decreased pitch count for 4 weeks
2- Continued play with close observation
3- Cessation of all throwing for 6 weeks
4- Arthroscopic repair
5- Mini-open repair

DISCUSSION: The coronal MRI scan shows an undersurface partial-thickness rotator


cuff tear. Initial treatment for this injury should include complete cessation of throwing
(or other overhead activities dependent on the athlete). Despite the duration of
symptoms, he has had no treatment to date; therefore, nonsurgical management
should include activity cessation, a rotator cuff and periscapular strengthening
program, and then a slow and supervised return to throwing with particular attention
to proper pitching mechanics. Decreasing the pitch count or continued play with
observation risks progression of the problem. Surgical intervention is not indicated for
initial treatment. The Preferred Response to Question # 80 is 3.

81 A 36-year-old woman reports vague right shoulder pain. She denies any previous
shoulder problems or any recent trauma. MRI scans are shown in Figures 81a and
81b. Weakness of which of the following is the most likely finding in her physical
examination?

1- Shoulder abduction and internal rotation


2- Shoulder external rotation and scapula protraction
3- Shoulder external rotation with the arm at the side
4- Shoulder internal rotation with the arm at the side
5- Scapula protraction

DISCUSSION: The MRI scans show a cyst formation within the suprascapular notch that
can compress the suprascapular nerve. The suprascapular nerve innervates both the
supraspinatus and the infraspinatus muscles. Therefore, patients with compression of
50
189
2011 Upper Extremity Self-Assessment Exam by Dr.Dhahirortho

this nerve may demonstrate weakness of shoulder abduction and external rotation
with the arm at the side. If the nerve is compressed after its innervation of the
supraspinatus muscle, however, patients will demonstrate weakness of shoulder
external rotation only. Suprascapular nerve does not innervate muscles that control
scapula motion or shoulder internal rotation. The Preferred Response # 81 is 3.

82 A 61-year-old man reports right shoulder pain and loss of external rotation since
having a seizure 5 months ago. MRI scans are shown in Figures 82a and 82b. What is
the most appropriate treatment?

1- Closed reduction and application of a shoulder immobilizer


2- Open reduction and lesser tuberosity transfer
3- Hemiarthroplasty placed in anatomic version
4- Hemiarthroplasty placed in anteversion
5- Total shoulder arthroplasty

DISCUSSION: The patient has a chronic posterior shoulder dislocation with loss of
approximately half of the humeral head. Hemiarthroplasty or osteochondral allograft
to fill the defect would be required. Given the time since injury, the remaining native
head and articular surface may have lost structural integrity, making hemiarthroplasty
the preferred choice. The implant should be placed close to the patient's natural
version, which normally is in the range of 20 to 30 degrees of retroversion. Excessive
anteversion is not recommended to avoid repeat posterior dislocation. Closed
reduction is highly unlikely to achieve a reduction and may cause displacement of an
unrecognized humeral surgical neck fracture. Open reduction and lesser tuberosity
transfer is best suited for smaller head defects and a less chronic dislocation. Glenoid
integrity is not affected, thus a glenoid implant is unnecessary.
The Preferred Response to Question # 82 is 3.

51
190
2011 Upper Extremity Self-Assessment Exam by Dr.Dhahirortho

83 A 17-year-old high school baseball player injured his dominant throwing arm
sliding head first into third base. He has immediate pain and swelling along the
medial aspect of the elbow and forearm, and demonstrates painful apprehension
with any attempt at movement of the elbow. Radiographs of the elbow are shown in
Figures 83a and 83b. What is the most appropriate management?
1- Cast immobilization for 6 weeks followed by
rehabilitation
2- Hinged elbow brace for 6 weeks and initiation
of early motion
3-Open reduction and internal fixation
4- Fragment excision
5- Closed reduction and percutaneous pinning

DISCUSSION: The patient has sustained a significantly displaced fracture of the medial
epicondyle. Nonsurgical management is unlikely to restore valgus stability to the elbow
necessary for overhead throwing. The fragment is large enough that bony stability
should be achieved with rigid internal fixation, thereby allowing early range of motion
and rehabilitation. Closed reduction attempts are unlikely to result in anatomic
reduction, and pinning of a displaced fracture may put the ulnar nerve at risk. Fracture
excision may further destabilize the elbow. The Preferred Response # 83 is 3.

84 A 51-year-old woman with shoulder pain responds transiently to a subacromial


injection and physical therapy exercise program. When her symptoms recur, an
arthroscopic subacromial decompression is recommended. During the surgery, a
partial-thickness articular-sided supraspinatus tear is noted. The supraspinatus
footprint is exposed for 3 mm from the articular margin. The remaining intra-
articular structures are normal. Inspection from the bursal surface reveals the tendon
to be intact. What is the most appropriate course of management?
1- Completion of the tear from the bursal surface and rotator cuff repair
2- Arthroscopic long head biceps tenotomy
3- Arthroscopic glenohumeral synovectomy
4- Arthroscopic tendon debridement and subacromial decompression
5- Transtendinous rotator cuff repair
52
191
2011 Upper Extremity Self-Assessment Exam by Dr.Dhahirortho

DISCUSSION: The patient has a partial articular supraspinatus tendon avulsion (PASTA)
lesion. Outcome studies suggest that articular-sided tears of this magnitude do well
with arthroscopic decompression and debridement alone. Determination of lesion
thickness is important in recommending treatment, and may be done with a variety of
methods. Tears that involve exposure of less than 5 mm of the rotator cuff footprint
likely measure less than half of the tendon thickness. In the absence of other
associated pathology, bicipital tenotomy or synovectomy would be unnecessary.
Completion of the tear or transtendinous tear would be considered for lesions of
greater than 50% thickness. The Preferred Response to Question # 84 is 4.

85 A 40-year-old man sustains a scapular body fracture after an all-terrain vehicle


accident. Which of the following is the most commonly associated injury?
1- Chest injury
2- Clavicle fracture
3- Glenohumeral dislocation
4- Humeral fracture
5- Axillary nerve injury

DISCUSSION: Chest injury (rib fracture, pneumothorax, hemothorax, contusion) is the


most commonly associated injury in patients who have sustained a significant scapular
injury. Chest injury becomes even more commonly found when the scapula has more
than one zone of injury (ie, multiple fractures). Humeral fracture, clavicle fracture, and
axillary nerve injury are not as common as chest injury. The Preferred Respo# 85 is 1.

86 A 44-year-old woman with cubital tunnel syndrome and associated ulnar nerve
subluxation with elbow flexion has failed to respond to nonsurgical management.
Which of the following statements is most acccurate regarding in situ simple
decompression of the nerve compared with subcutaneous anterior transposition?
1- Patients undergoing anterior transposition have improved motor outcomes.
2- Patients undergoing anterior transposition have improved sensory outcomes
3- Patients undergoing simple decompression have improved motor outcomes.
4- Patients undergoing simple decompression have improved sensory outcomes.
5- No differences in outcome are likely between treatment types.
53
192
2011 Upper Extremity Self-Assessment Exam by Dr.Dhahirortho

DISCUSSION: Recent reports comparing outcomes of surgical treatment of ulnar nerve


compression at the elbow have demonstrated no differences in outcome between
simple decompression and anterior transposition. The presence of subluxation of the
ulnar nerve was not a contraindication to in situ decompression in the study by Keiner
and associates. The Preferred Response to Question # 86 is 5.

87 A 25-year-old electrician sustained an injury to his dominant arm while bench


pressing at the gym. He reports that he felt a tearing sensation while extending his
arms. Examination reveals that he has lost the normal contour of the axillary fold
which worsens with resisted adduction. Additionally, there is extensive ecchymosis
down the arm and weakness to adduction and internal rotation. Radiographs are
normal. What is the most appropriate management?
1- Arthroscopic subscapularis repair
2- Repair of the long head of the biceps with tenodesis
3- Open repair of the pectoralis major tendon avulsion
4- Ultrasound and physical therapy to reduce swelling and improve strength
5- Brace immobilization for 6 weeks

DISCUSSION: This description is classic for an acute pectoralis major humeral avulsion.
The loss of contour in the axillary fold confirms this diagnosis. Treatment for a
pectoralis tendon avulsion should be open surgical repair in this young patient.
Therapy may be considered for injuries within the muscle or at the musculotendinous
junction. Examination for subscapularis rupture and biceps injuries would not cause a
change in the axillary fold. Bracing will not improve long-term strength.P R# 87 is 3.

88An otherwise healthy 30-year-old man undergoes right shoulder arthroscopic


Bankart repair under regional anesthesia using an interscalene brachial plexus block.
In the recovery room, he reports mild difficulty breathing and his chest radiograph
shows a high riding diaphragm on the right side. His peripheral oxygenation is 97%
on 2 liters of oxygen by nasal cannula. What is the most appropriate management?
1- Continued observation and monitoring
2- Obtain arterial blood gas measurements
3- Obtain emergent spiral CT scan to assess for pulmonary embolism
54
193
2011 Upper Extremity Self-Assessment Exam by Dr.Dhahirortho

4- Insertion of a chest tube


5- Airway control and, if necessary, endotracheal intubation

DISCUSSION: Because the phrenic nerve lies in close proximity to the site of anesthetic
injection, temporary hemidiaphragmatic paresis is a very common side effect of
interscalene brachial plexus block. Pulmonary function and chest wall mechanics may
be slightly compromised, but can easily be compensated in a healthy patient.
Therefore, with sufficient oxygenation, aggressive assessments or treatments such as
arterial blood gas measurements, emergent spiral CT scans, chest tube insertions, or
endotracheal intubation are not warranted. For this stable patient, continued
monitoring with gradual withdrawal of oxygen is the most appropriate treatment.
The Preferred Response to Question # 88 is 1.

89 What is the interval used during an anterior approach (Henry) for a distal radius
shaft fracture?
1- Flexor digitorum superficialis-flexor carpis ulnaris
2- Flexor carpi radialis-flexor digitorum superficialis
3- Brachioradialis-flexor carpi radialis
4- Flexor pollicis longus-flexor digitorum profundus
5- Flexor pollicis longus-flexor carpi radialis

DISCUSSION: The anterior approach to the radial shaft uses the internervous plane
between the brachioradialis (radial n) and flexor carpi radialis (median n) distally, and
the brachioradialis and pronator teres (median n) proximally. The Preferred Response
to Question # 89 is 3.

90 A 37-year-old man with a nondisplaced radial neck fracture has failed to respond
to 8 months of nonsurgical management. He has undergone extensive physical
therapy and bracing without improvement. Examination reveals that active and
passive range of motion is limited to 50 degrees to 85 degrees, with full
pronosupination. He has mildly diminished sensation in the little and ring fingers.
Radiographs reveal healing of the fracture, no deformity, and no arthrosis or
heterotopic bone formation. What is the most appropriate management?
55
194
2011 Upper Extremity Self-Assessment Exam by Dr.Dhahirortho

1- Radial head resection and release of the anterior capsule


2- Anterior and posterior capsule release, with ulnar nerve transposition
3- Ulnar nerve transposition and release of the posterior capsule
4- Ulnar nerve transposition
5- Intra-articular corticosteroid injection

DISCUSSION: The patient has refractory extra-articular elbow stiffness and ulnar
neuritis following trauma. Important considerations are ruling out failure of fracture
healing, persistent deformity, and heterotopic bone formation. In this patient, further
nonsurgical management is unlikely to provide any benefit; therefore, the treatment of
choice is anterior and posterior capsule release, with ulnar nerve transposition. Radial
head resection is not indicated because of the absence of deformity or arthrosis. There
is restriction of both flexion and extension, so limited capsular release techniques will
not maximize functional restoration. Ulnar nerve transposition alone will not restore
motion. An intra-articular injection is not likely to improve motion 8 months after the
injury. The Preferred Response to Question # 90 is 2.

91 Figure 91 shows the radiograph of a 57-year-old man who fell 6 feet off a ladder.
He is neurovascularly intact but reports shoulder pain. What is the most appropriate
acute treatment for this patient?

1- Physical therapy for range of motion, advancing to strengthening as tolerated


2- Sling immobilization and a recheck in 1 week with radiographs
3- CT scan of the shoulder
4- Open reduction and surgical stabilization with plates and screws
5- Ice, nonsteroidal anti-inflammatory drugs, and activity as tolerated

DISCUSSION: The patient has sustained a traumatic surgical neck fracture of the
humerus. Sling immobilization and a recheck in 1 week with radiographs is appropriate
56
195
2011 Upper Extremity Self-Assessment Exam by Dr.Dhahirortho

to check for maintenance of alignment. The fracture is minimally displaced and


therefore does not require surgical stabilization or further diagnostic imaging. Surgical
reduction and plating is not indicated in this nondisplaced fracture. Physical therapy
and activity as tolerated at this point are contraindicated because of the acuity of the
fracture. The Preferred Response to Question # 91 is 2.

92 What anatomic structure must be excised when performing a volar plate


arthroplasty of the proximal interphalangeal joint?
1- Central slip
2- Collateral ligament
3- Checkrein ligament
4- Triangular ligament
5- Flexor digitorum superficialis insertion

DISCUSSION: The collateral ligament must be excised or released from the proximal
phalanx to allow gliding of the middle phalanx on the articular surface of the proximal
phalanx. Failure to do so may prevent this gliding motion and make the middle phalanx
just hinge on the proximal phalanx. The Preferred Response to Question # 92 is 2.

93A 67-year-old woman with rheumatoid arthritis has had a 3-year history of
gradually progressive right elbow pain and limited function despite intra-articular
injections and medical management. She previously underwent a rheumatoid hand
reconstruction, and has no pain or dysfunction of the ipsilateral shoulder.
Radiographs are shown in Figures 93a and 93b. What is the most appropriate
treatment?
1- Soft-tissue interposition arthroplasty with radial head
resection
2- Arthroscopic synovectomy with radial head resection
3- Elbow arthrodesis
4- Total elbow arthroplasty
5- Resection arthroplasty

57
196
2011 Upper Extremity Self-Assessment Exam by Dr.Dhahirortho

DISCUSSION: Total elbow arthroplasty is the treatment of choice. The patient has end-
stage rheumatoid involvement of the ulnohumeral and radiocapitellar joints. Given the
advanced nature of the disease and evidence of bony erosion, arthroscopic
synovectomy and interposition arthroplasty are unlikely to provide lasting benefit or
functional improvement. Elbow arthrodesis and resection arthroplasty are considered
salvage techniques and are generally not considered as a primary treatment method.
The Preferred Response to Question # 93 is 4.

94A 78-year-old woman undergoes an uneventful semiconstrained total elbow


arthroplasty through a Bryan-Morrey approach. Her immediate postoperative
management should include which of the following?
1- Five days of intravenous antibiotics for perioperative prophylaxis
2- Use of continuous passive motion beginning on postoperative day one
3- Immediate initiation of active flexion and gravity-assisted passive extension
4- Splinting at 60 to 90 degrees of flexion for 5 to 10 days, followed by initiation of
active flexion and gravity-assisted passive extension
5- Splinting at 60 to 90 degrees of flexion until the triceps has healed, followed by
initiation of active flexion and extension

DISCUSSION: Postoperative management of total elbow arthroplasty patients is


directed to avoidance of complications commonly associated with this procedure.
Following total elbow arthroplasty, 24 hours of perioperative antibiotics should be
given, consistent with other arthroplasty procedures. Because of the relatively thin
soft-tissue envelope surrounding the elbow, particularly in patients with rheumatoid
arthritis, consideration must be given to the surrounding soft tissues postoperatively.
The surgical wound should be given several days of quiescence prior to initiation of
motion to minimize wound healing complications. Splinting at 60 to 90 degrees allows
tension to be removed from the soft tissues. Immediate motion places these tissues
under immediate stress; immobilization of the elbow for 6 to 8 weeks until the triceps
has healed would result in significant stiffness. Splinting should not be used more than
10 days to avoid stiffness of the elbow.
The Preferred Response to Question # 94 is 4.

58
197
2011 Upper Extremity Self-Assessment Exam by Dr.Dhahirortho

95 A 55-year-old man who underwent total shoulder arthroplasty 10 years ago


recently reports an increase in shoulder pain. Laboratory studies consisting of a
white blood cell count, erythrocyte sedimentation rate, and C-reactive protein are all
negative, as is joint aspiration. Radiographs are shown in Figures 95a and 95b. If all
intraoperative frozen sections are negative, what is the appropriate treatment during
revision surgery to provide pain relief and improved function?
1- Placement of antibiotic spacer
2- Removal of the glenoid, and possible bone grafting
3- Conversion to reverse shoulder arthroplasty
4- Referral to pain management
5- Shoulder arthrodesis

DISCUSSION: The radiographs reveal a loose glenoid in the


setting of no infection. Glenoid removal may give this
patient the best chance of improved function and pain relief if sufficient bone stock
remains. Bone grafting of defects may allow future glenoid implantation. Conversion to
reverse shoulder arthroplasty would be a salvage procedure in this younger patient.
Shoulder arthrodesis would be difficult and unpredictable after shoulder arthroplasty.
The Preferred Response # 95 is 2.

96 Which of the following clinical tests is used to diagnose medial instability of the
elbow?
1- Posterolateral rotatory drawer test
2- Lateral pivot-shift test
3- Moving valgus stress test
4- Chair test (apprehension or dislocation on terminal extension of the supinated
forearmwhen rising from a seated position)
5- Pushup sign

DISCUSSION: The moving valgus stress test is used in the diagnosis of medial collateral
ligament instability of the elbow. The other tests apply a varus force to the elbow and
are used to diagnose lateral ulnar collateral insufficiency. Preferred Respo# 96 is 3.

59
198
2011 Upper Extremity Self-Assessment Exam by Dr.Dhahirortho

97Figures 97a and 97b show a clinical photograph and radiograph of a patient who
has a history of repeated drainage from the lesion. What is the preferred surgical
treatment?

1- Excision of the lesion alone


2- Removal of the osteophyte alone
3- Distal interphalangeal joint fusion
4- Excision of the mass and osteophyte removal
5- Removal of the mass and skin with skin grafting

DISCUSSION: The patient has a mucoid cyst. Whereas many of these lesions are
associated with osteoarthritis, the best surgical treatment of the lesions in patients
who have little or no pain is typically excision of the mass with osteophyte removal.
Studies have shown that osteophyte excision helps minimize the risk of recurrence.
Distal interphalangeal joint fusion is reserved for patients with pain and more
advanced radiographic arthritis. Excision of the lesion alone is a less favorable option
than excision of the mass and osteophyte removal. The lesion is independent of the
skin and thus, skin removal with the mass is unnecessary.Preferred Respo# 97 is 4.

98 Isolated coronoid fractures are most likely related to what instability pattern?
1- Posterolateral rotary instability
2- Valgus anterolateral instability
3- Posterior instability
4- Varus posteromedial instability
5- Anterior instability

DISCUSSION: Coronoid fractures in the absence of radial head or associated fractures


are often a sign of a varus posteromedial instability. Depending on the size of the
coronoid fragment, fixation and stabilization of the coronoid may be necessary to
restore medial stability of the elbow. Posterolateral instability is related to lateral ulnar
collateral ligament incompetance. Anterior, valgus anterolateral, and posterior
instability are not generally specific to isolated coronoid fractures. Pre Respo # 98 is 4.

60
199
2011 Upper Extremity Self-Assessment Exam by Dr.Dhahirortho

99 A 35-year-old man has pain and swelling of his right, dominant wrist. Radiographs
and MRI scans are shown in Figures 99a through 99d. What is the most appropriate
management?

1- Incisional biopsy
2- Allograft reconstruction
3- Vascularized fibula reconstruction
4- Nonvascularized fibular autograft
5- Intralesional curettage and polymethylmethacrylate (PMMA) packing

DISCUSSION: Whereas the imaging studies show a benign giant cell tumor of bone, an
incisional biopsy is still the first surgery that should be performed. After a tissue
diagnosis is confirmed, then the reconstructive options can be discussed. A malignancy
may present like a benign, aggressive giant cell tumor. Preferred Resp# 99 is 1.

100 A 27-year-old woman underwent shoulder arthroscopy for multidirectional


instability 3 years ago. She was unable to regain shoulder range of motion despite
therapy and has had progressively worsening pain. A current axillary radiograph is
shown in Figure 100. In reviewing the medical records from the index procedure,
what factor may be significant in contributing to her current condition?

1- Subsequent development of a supraspinatus tear


2- Subscapularis tendon dehiscence
3- Coagulation of the anterior humeral circumflex artery
4- Use of monopolar radiofrequency thermal capsulorrhaphy
5- Lack of compliance with postoperative therapy program
61
200
2011 Upper Extremity Self-Assessment Exam by Dr.Dhahirortho

DISCUSSION: Reports from several centers suggest the potential to develop


glenohumeral chondrolysis because of the heat production associated with use of
radiofrequency or laser thermal capsulorrhaphy. A tear of the supraspinatus may lead
to poor function and progression to rotator cuff tear arthropathy with superior
humeral head migration. Subscapularis dehiscence is a risk in open surgery through a
deltopectoral approach and can lead to anterior instability. The anterior humeral
circumflex artery is the main supply to the humeral head and its coagulation can lead
to osteonecrosis. Whereas a lack of postoperative therapy can lead to unresolved pain
and stiffness, chondrolysis is not reported. The Preferred Response # 100 is 4.

101Which of the following structures cannot be seen during standard radiocarpal


arthroscopy?
1- Scapholunate ligament
2- Lunotriquetral ligament
3- Radioscaphocapitate ligament
4- Extensor carpi ulnaris tendon
5- Superficial insertion of the triangular fibrocartilage complex (TFCC)

DISCUSSION: The extensor carpi ulnaris tendon is located in an extra-articular position,


and as such, cannot be seen during arthroscopy. Wrist arthroscopy is a useful
technique for evaluation and treatment of radiocarpal and midcarpal maladies. During
standard radiocarpal arthroscopy, the scapholunate and lunotriquetral ligaments can
be easily visualized. The superficial TFCC is seen overlying the ulnar head. Volarly, the
radioscaphocapitate ligament can be seen as a discrete band of the capsule. The
Preferred Response to Question # 101 is 4.

102An active 66-year-old man who underwent total shoulder arthroplasty 3 years
ago now reports pain. Laboratory studies reveal an elevated erythrocyte
sedimentation rate and C-reactive protein. Intraoperative frozen section reveals
greater than 10 white blood cells per high power field on two slides and the Gram
stain reveals gram-positive cocci in clusters. What is the most appropriate surgical
treatment to eradicate the infection and maintain function?
62
201
2011 Upper Extremity Self-Assessment Exam by Dr.Dhahirortho

1- Removal of the components and placement of an antibiotic spacer


2- Removal of the components, placement of an antibiotic spacer, and bone grafting
of the glenoid defect
3- Resection arthroplasty
4- Exchange of the humeral head and debridement
5- Arthroscopic debridement

DISCUSSION: The prosthesis is grossly infected. Removal of the components and


placement of an antibiotic spacer is necessary to eradicate the infection and allow for a
second stage reimplantation. Resection arthroplasty is an option to treat the infection
but the functional outcome would be limited. Bone grafting with concurrent infection
is not likely to heal and should be delayed until the second stage. Humeral head
exchange and debridement or arthroscopic debridement alone is unlikely to eradicate
the infection. The Preferred Response to Question # 102 is 1.

103 A 77-year-old woman underwent semiconstrained right total elbow arthroplasty


4 weeks ago through a Bryan-Morrey approach. Her recovery was uneventful until 2
days ago when she began her physical therapy session at an outpatient clinic. During
resisted extension exercises, she felt a "pop" in her elbow, accompanied with pain
and inability to extend her elbow against resistance. What is the most likely cause of
her symptoms?
1- Fracture of the ulnar component
2- Disengagement of the axle of the prosthesis
3- Failure of the triceps mechanism repair
4- Periprosthetic fracture of the humerus
5- Periprosthetic fracture of the ulna

DISCUSSION: During a Bryan-Morrey approach for total elbow arthroplasty, the triceps
is dissected free from its ulnar insertion and reflected laterally. At the conclusion of the
procedure, the triceps tendon is reattached to the ulna through drill holes. Whereas
motion can be initiated postoperatively, 6 to 8 weeks of protection are recommended
before initiation of resistance exercises to protect the triceps repair. A periprosthetic

63
202
2011 Upper Extremity Self-Assessment Exam by Dr.Dhahirortho

fracture or component failure is rare in the absence of more significant trauma, and
they are usually late complications. The Preferred Response to Question # 103 is 3.

104 A 47-year-old man who is right-hand dominant reports lateral-sided elbow pain
after playing golf. His symptoms developed gradually and without trauma, and he
has pain with gripping and repetitive movements with the hand and wrist.
Examination reveals his shoulder and wrist to be normal, and the elbow has no
effusion and normal range of movement. He is tender near the lateral epicondyle,
and symptoms are exacerbated with resisted wrist extension. Radiographs are
shown in Figures 104a and 104b. What is the next most appropriate step in
management?

1- Subtendinous epicondylar corticosteroid injection


2- Corticosteroid injection into the radial tunnel
3- MRI of the elbow
4- Percutaneous extensor carpi radialis brevis tenotomy
5- Physical therapy for an eccentric conditioning and strengthening program

DISCUSSION: The patient has lateral epicondylitis of relatively short duration. At this
early stage of disease, nonsurgical management is indicated. An eccentric physical
therapeutic exercise program has been shown to have a beneficial effect on tendon
biology; therefore, it would be the most appropriate initial management. While the
diagnosis of lateral epicondylitis may be confused with radial tunnel syndrome, the
clinical examination and history are most suggestive of the former. Corticosteroid
injection has been shown to help with symptoms in short-term follow-up, but does
little to affect the natural progression of the condition; it is more appropriate as a
64
203
2011 Upper Extremity Self-Assessment Exam by Dr.Dhahirortho

second line of treatment. MRI may be beneficial in patients with refractory disease
and/or when the diagnosis is in question. Percutaneous surgical treatment is indicated
only when nonsurgical measures fail to provide relief. The Preferred Respo# 104 is 5.

105 A 45-year-old woman has had a 4-month history of mild to moderate lateral
shoulder pain that is aggravated with active elevation. Radiographs and MRI scans
are shown in Figures 105a through 105d. Initial treatment should include which of
the following?

1- Moist heat and a stretching program


2- Attempted calcium aspiration
3- Extracorporeal shock wave therapy
4- Low-dose radiation therapy
5- Arthroscopic rotator cuff debridement

DISCUSSION: The images show a well-circumscribed mass consistent with calcific


tendinitis in the formation phase. Pain is not typically as severe as in the resorptive
phase and amenable to nonsurgical management. The calcific deposit in this phase is
granular, making aspiration difficult to achieve. Extracorporeal shock wave therapy has
been studied with numerous protocols of amount of energy and number of
treatments. Its role in the treatment of calcific tendinitis is still poorly defined. Low-
dose radiation therapy has been successfully applied to calcific tendinitis in the past
but is not currently used because of concerns of malignant tissue transformation and
success with lower risk modalities. Arthroscopic debridement of the calcific deposit can
be considered in patients who have not responded to nonsurgical management.
The Preferred Response to Question # 105 is 1.

65
204
2011 Upper Extremity Self-Assessment Exam by Dr.Dhahirortho

106 A 62-year-old man has had worsening pain in the left shoulder for the past 6
weeks without trauma. He participated in physical therapy to "strengthen" his
shoulder; however, it failed to provide relief. On examination, his right shoulder
motion is 180, 60, and T8 (forward flexion, external rotation, and internal rotation).
His left shoulder motion, both active and passive, is 150, 40, and L1. T1- and T2-
weighted MRI scans are shown in Figures 106a and 106b with an official diagnosis of
partial supraspinatus tendon tear. What is the appropriate treatment?

1- Physical therapy for rotator cuff strengthening and scapula stabilization


2- Regimen of stretching exercises for motion
3- Arthroscopic acromioplasty
4- Arthroscopic acromioplasty and rotator cuff repair
5- Open rotator cuff repair

DISCUSSION: The patient lacks both active and passive motion in all planes of shoulder
motion; his primary pathology is adhesive capsulitis. Although the MRI scans reveal a
partial-thickness rotator cuff tear, this is not uncommon in asymptomatic patients
older than age 60 years. Physical therapy for patients with adhesive capulitis should
stress shoulder motion rather than rotator cuff strengthening. Because most cases of
adhesive capsulitis improve without surgical management, surgical treatment options
are not appropriate at this time. The Preferred Response to Question # 106 is 2.

107 A 66-year-old woman with known poorly controlled rheumatoid arthritis reports
that for the past 4 weeks she has been unable to extend the metacarpophalangeal
(MCP) joints of her right hand index, middle, ring and little fingers. She cannot
hyperextend the thumb interphalangeal joint. Active wrist extension is possible, but
shows radial deviation. Examination reveals mild synovitis at the wrist and MCP
joints of the affected hand. There is no ulnar deviation at the MCP joints with normal
66
205
2011 Upper Extremity Self-Assessment Exam by Dr.Dhahirortho

alignment. When the MCP joints are passively extended, the patient is unable to
maintain them in this position. There is no piano key sign at the distal ulna. Passive
wrist motion shows a normal tenodesis effect. Which of the following would most
likely confirm your diagnosis?
1- Radiographs of the hand
2- Radiographs of the cervical spine
3- Electrodiagnostic studies of the affected upper extremity
4- Surgical exploration of the extensor tendon ruptures
5- MRI of the elbow
DISCUSSION: There are many causes of inability to extend the MCP joints in a patient
with rheumatoid arthritis. The most common cause is rupture of the extensor tendons.
An intact tenodesis test suggests that the extensor tendons are intact, thus surgical
exploration is not indicated and would not confirm the diagnosis. The patient has
normal alignment of the fingers without ulnar deviation, suggesting that there are no
MCP dislocations to account for the inability to extend the MCP joints; therefore,
radiographs would not confirm the diagnosis. The most likely cause of inability to
extend the fingers in this patient is posterior interosseous nerve (PIN) palsy.
Electrodiagnostic studies would confirm the presence of PIN palsy. An MRI of the
elbow may show synovitis at the radiocapitellar joint, which can cause the PIN palsy.
This finding however, is nonspecific and many patients without PIN palsy would also
demonstrate synovitis at the radiocapitellar joint. Therefore, although an MRI would
be helpful in localizing a potential cause of PIN compression, it would not in itself
confirm the diagnosis. The Preferred Response to Question # 107 is 3.

108 A 23-year-old woman with recurrent anterior instability undergoes an open


Bankart procedure. Six months after surgery the patient reports shoulder weakness
and is concerned about instability of the shoulder joint. An MRI scan is shown in
Figure 108. What is the most appropriate management?
1- Physical therapy
2- Biceps tenolysis
3- Subscapularis repair
4- Supraspinatus repair
5- Pectoralis major repair
67
206
2011 Upper Extremity Self-Assessment Exam by Dr.Dhahirortho

DISCUSSION: The axial MRI scan shows rupture of the subscapularis tendon with
dislocation of the biceps tendon. Treatment should include a biceps tenotomy or
tenodesis in conjunction with a subscapularis repair. A pectoralis major transfer may
be necessary in chronic cases where the subscapularis is irreparable, but in this patient
the tendon is repairable. As a single operation, biceps tenolysis will not correct the
instability, and would likely result in a cosmetic deformity. Physical therapy will not
restore subscapularis function. The Preferred Response to Question # 108 is 3.

109 A patient has a mass at the base of the middle finger just distal to the distal
palmar flexion crease. The mass is 2 mm in size, firm, round, and does not move with
finger motion. It is painful with gripping activites such as a steering wheel. What is
the most appropriate management?

1- Diagnostic ultrasound
2- MRI
3- Needle aspiration
4- Observation
5- Surgical excision

DISCUSSION: The clinical scenario is of an A2 retinacular cyst. These are firm round
cysts arising from the pulley system so they do not move with tendon motion. Needle
aspiration in the office is highly effective, thus surgery can be avoided. Based on the
clinical diagnosis, ultrasound and MRI are unnecessary. Because the patient has pain
and functional limitations, observation is not recommended. The Prefer Resp# 109 is 3.

110A 72-year-old woman was evaluated with an MRI scan for a shoulder mass that
was confirmed to be a lipoma. Additional MRI findings included a 7-mm full-
thickness tear of the supraspinatus tendon. Therefore, the patient was referred by
her internist for evaluation and management of the rotator cuff tear. The patient
reports mild "stiffness" with certain motion but denies any limitations in her
functional capacity. Examination reveals a slight decrease in internal rotation and
mild weakness with resisted abduction of the shoulder. What is the most appropriate
management?
68
207
2011 Upper Extremity Self-Assessment Exam by Dr.Dhahirortho

1- Observation
2- Arthroscopic rotator cuff debridement
3- Arthroscopic rotator cuff repair with acromioplasty
4- Arthroscopic biceps tendon tenotomy
5- Open rotator cuff repair with bone tunnels

DISCUSSION: In patients older than age 60 years, over 30% of asymptomatic shoulders
show MRI findings of full-thickness rotator cuff tears. Therefore, without significant
symptoms, surgical treatment is not warranted. The Preferred Response# 110 is 1.

111A baseball player reports a dull pain in the posterior aspect of his throwing arm.
Examination reveals decreased internal rotation and prominence of the inferomedial
corner of the scapula. An MRI scan suggests a partial-thickness tear of the posterior
supraspinatus tendon. Successful treatment would most likely include which of the
following?

1- Anti-inflammatory medication, posterior capsular stretching, and rotator cuff


strengthening
2- SLAP repair
3- Debridement of the partial-thickness rotator cuff tear
4- Rotator cuff repair
5- Imbrication of the labrum and anterior capsule

DISCUSSION: Internal impingement is related to an internal rotation contracture (GIRD-


glenohumeral internal rotation deficit) and an increase in external rotation caused by
repetitive overhead throwing. Most patients can be successfully treated with
rehabilitation that focuses on internal rotation stretches along with anti-inflammatory
medication and strengthening as symptoms improve. SLAP repair and rotator cuff
debridement may be considered in refractory cases. Rotator cuff repair is not typically
required, and capsulolabral imbrication is more consistent with the surgical treatment
for multidirectional instability.
The Preferred Response to Question # 111 is 1.

69
208
2011 Upper Extremity Self-Assessment Exam by Dr.Dhahirortho

112Figures 112a and 112b show the radiographs of a 28-year-old motorcyclist who
sustained a closed hand injury in a collision. What is the most appropriate definitive
treatment?

1- Closed reduction and a hand/forearm cast in the intrinsic plus position


2- Closed reduction and a hand splint
3- Primary fusion of the carpometacarpal joints
4- Closed versus open reduction and internal fixation
5- Closed reduction and external fixation

DISCUSSION: Closed versus open reduction and internal fixation is the most
appropriate treatment. The radiographs show fracture-dislocations of all five
carpometacarpal joints. These injuries are extremely unstable and not amenable to
closed (splint or cast) treatment only. External fixation may be warranted in an open,
contaminated injury. Fusion would be an option if this were a chronic, painful
condition on presentation. The Preferred Response to Question # 112 is 4.

113A 32-year-old male hockey player who is right-hand dominant was checked from
behind and landed with full force into the boards. In the emergency department he
reports shortness of breath. Figure 113 shows a 2-D CT scan. What is the best initial
treatment for this injury?
1- Observation
2- Closed reduction with a towel clip
3- Open reduction
4- Open reduction and internal fixation
5- Open reduction and sternoclavicular ligament allograft reconstruction

DISCUSSION: The CT scan shows a posterior sternoclavicular joint dislocation. Initial


management involves attempted closed reduction in the operating room. This can be
performed with a towel clip and anterior translation of the displaced clavicle. However,
70
209
2011 Upper Extremity Self-Assessment Exam by Dr.Dhahirortho

the orthopaedic surgeon should be prepared to open this injury and reconstruct the
joint if necessary. Furthermore, it is recommended that a thoracic surgeon be available
prior to beginning these procedures. Open reduction should be done only if closed
reduction is unsuccessful. The Preferred Response to Question # 113 is 2.

114What additional procedure should be done when performing a


radioscapholunate fusion for posttraumatic arthrosis following a distal radius
fracture?
1- Excision of the triquetrum and distal pole of the scaphoid
2- Anterior interosseous neurectomy
3- Fascial interposition arthroplasty of the capitolunate joint
4- Sectioning of the dorsal intercarpal ligament
5- Ulnar shortening osteotomy

DISCUSSION: Excision of the triquetrum and distal pole of the scaphoid frees up the
mid-carpal joint, improving radial deviation and the flexion-extension arc of motion of
the wrist. This offers an alternative to complete wrist arthrodesis for posttraumatic
arthrosis of the radiocarpal joint. An anterior interosseous neurectomy is believed to
decrease some pain transmission from the wrist but because the fusion is done dorsal,
cutting this volar structure is not routinely done. Fascial interposition is not needed
because the capitolunate should be preserved in posttraumatic radiocarpal arthrosis.
Sectioning of the dorsal intercarpal ligament would provide no benefit. If the
triquetrum is excised, then an ulnar shortening osteotomy is unnecessary. The
Preferred Response to Question # 114 is 1.

115A 72-year-old man who underwent an uncomplicated total shoulder arthroplasty


4 weeks ago now reports injuring his shoulder in a fall on the ice. He attempted to
catch himself on a railing with his operative arm. He continues to feel pain anteriorly
in the shoulder. His range of motion is 140 degrees forward elevation, 90 degrees
external rotation with the arm at the side, and internal rotation up the back to L1.
Radiographs are normal. What is the most likely diagnosis?
1- Deltoid contusion
2- Rupture of the subscapularis repair
71
210
2011 Upper Extremity Self-Assessment Exam by Dr.Dhahirortho

3- Traumatic loosening of the glenoid


4- Locked posterior shoulder dislocation
5- Biceps tendon rupture

DISCUSSION: The patient sustained a rupture of the subscapularis tendon repair. This
can occur in the postoperative period with forced internal rotation or excessive
external rotation beyond the normal 40 to 60 degrees. On examination, the patient has
90 degrees of external rotation at the side; this is not a normal finding for a 72-year-old
man. There is no indication at this time that the glenoid component has loosened or
that the patient has a locked posterior dislocation. Both of these would be evident on
radiographs. A biceps tendon rupture or a deltoid contusion would not explain the
excessive external rotation to 90 degrees as seen on examination. The Preferred
Response to Question # 115 is 2.

116 A 68-year-old man with a history of diabetes and total shoulder arthroplasty 4
years ago, now reports increasing shoulder pain and stiffness. Radiographs show
lucent lines around both the humeral and glenoid components. Laboratory studies
show a white blood cell count of 12,600/mm3, an erythrocyte sedimentation rate of
72 mm/h, and a c-reactive protein of 3.5. The shoulder is aspirated and cultures are
negative at 3 days. What is the most appropriate treatment for this patient?
1- 4-week trial of nonsteroidal anti-inflammatory drugs (NSAIDs)
2- Physical therapy for range-of-motion work
3- Repeat aspiration and culture
4- Open irrigation and debridement with implant removal and possible exchange
arthroplasty
5- Arthroscopic irrigation and debridement

DISCUSSION: The patient has clinical and radiographic signs of infection. Open
debridement, component removal, an antibiotic spacer, and possible exchange
arthroplasty are necessary to resolve the infection. Aspiration and culture can often be
negative at 3 days. NSAIDs, sling immobilization, or physical therapy are not indicated.
With radiographs indicating lucent lines surrounding the prosthetic implants,

72
211
2011 Upper Extremity Self-Assessment Exam by Dr.Dhahirortho

arthroscopic irrigation and debridement will not eradicate the infection. The Preferred
Response to Question # 116 is 4.

117 A 35-year-old construction worker sustained a midshaft clavicle fracture that


developed a hypertrophic nonunion. One year after the injury, it was internally fixed
without bone graft. Four months after the surgery he was asymptomatic and he was
released to full activity. Five months following surgery, the patient was digging a
ditch and he felt pain in the clavicle. The 4-month and 5-month postoperative
radiographs are shown in Figures 117a and 117b. What is the most likely cause of this
failure?

1- Iliac crest bone graft was not used to augment the fixation
2- Infection
3- Inadequate strength of the plate
4- Use of superior plating rather than anterior plating
5- Inadequate medial screw fixation

DISCUSSION: In this patient, the hardware was intact for 5 months without any
evidence of loosening prior to the catastrophic failure. This suggests that the primary
cause of nonunion was poor biology rather than insufficient fixation. Biologic
compromise can be caused by either infection, poor blood supply, or lack of osteogenic
induction cells. Iliac crest bone graft has been used by some for any nonunion of the
clavicle, but two studies have shown that bone graft is not necessary to achieve union.
Rigid fixation is all that is required. Infection will still complicate any fixation technique.
The radiographs show unicortical screw fixation medially, but the construct did not
loosen; therefore, it is not the cause of failure.
The Preferred Response # 117 is 2.

73
212
2011 Upper Extremity Self-Assessment Exam by Dr.Dhahirortho

118 A 60-year-old woman with a history of osteoporosis fell from a standing height
and sustained a supracondylar distal humerus fracture with an intercondylar
extension. Which of the following plate constructs yields the highest stiffness for
fixation of the fracture?
1- Single posterior Y plate
2- Single medial plate with bicortical locking screws
3- Dual plating with medial and posterolateral LC-DCP
4- Dual plating with medial and posterolateral one third tubular plates
5- Dual plating with medial and lateral LC-DCP

DISCUSSION: Optimal treatment of distal humeral fractures relies on reestablishment


of a congruent articular surface with a fixation construct that is stable enough to allow
for early range of motion. Several biomechanical studies have been performed to
evaluate the biomechanical strength of various plating configurations. These studies
have shown that dual plate configurations are more stable than single plates,
regardless of the type of plate used. One third tubular plates have been shown to be
significantly weaker than LC-DCP or reconstruction plates, resulting in weaker
constructs, and clinically higher rates of hardware failure and nonunion. Whereas
traditional teaching has suggested plating in perpendicular planes, recent
biomechanical studies have demonstrated that parallel medial and lateral plates confer
a greater rigidity to the construct than perpendicular plating schemes. The Preferred
Response to Question # 118 is 5.

119 A 35-year-old construction worker continues to have weakness with lifting


overhead 2 years after he was treated with physical therapy for a "chest muscle"
tear. An obvious deformity noted in his axilla worsens with resisted extension and
adduction. A clinical photograph and MRI scan are shown in Figures 119a and 119b.
What is the most appropriate treatment?

74
213
2011 Upper Extremity Self-Assessment Exam by Dr.Dhahirortho

1- Allograft reconstruction with semitendinosis weave to the humerus


2- Latissimus dorsi tendon transfer
3- Electrical stimulation
4- Shoulder arthrodesis
5- Arthroscopic pectoralis major tendon repair

DISCUSSION: This scenario describes a chronic, symptomatic pectoralis major tendon


rupture in a young laborer. Direct repair is difficult at this time; therefore, allograft
reconstruction is a good alternative to recover strength. Tendon transfers, electrical
stimulation, shoulder arthrodesis, and arthroscopy are not indicated in this patient.
They will not offer proper reconstruction of the lost muscle tendon unit and/or
cosmetic repair. The Preferred Response to Question # 119 is 1.

120 A 74-year-old patient is seen for follow-up 6 weeks after undergoing a total
shoulder arthroplasty for glenohumeral osteoarthritis. The patient missed the 2-
week follow-up appointment and is currently wearing a sling. The incision is well
healed with no signs of breakdown. Examination reveals that passive range of
motion is forward elevation of 90 degrees, external rotation at the side 0 degrees,
and internal rotation up the back is to the level of the greater trochanter. A
radiograph shows no signs of fracture or dislocation. What is the next most
appropriate management for this patient?
1- Physical therapy for range-of-motion exercises
2- Aspiration for possible infection
3- MRI to evaluate for possible rotator cuff tear
4- Sling immobilization and reevaluation in 4 weeks
5- Duplex ultrasound for possible upper extremity deep venous thrombosis

DISCUSSION: The patient has a postoperative stiff shoulder. The patient missed follow-
up appointments and has not been participating in physical therapy for stretching.
Based on normal radiographic findings, the shoulder is not dislocated; therefore,
physical therapy should begin immediately. Continued sling immobilization will further
worsen the stiffness. There is no indication of an infection or rotator cuff tear. Deep

75
214
2011 Upper Extremity Self-Assessment Exam by Dr.Dhahirortho

venous thrombosis would present with abnormal swelling and pain. The Preferred
Response to Question # 120 is 1.

121 A 22-year-old man sustained a shoulder dislocation while playing collegiate


football at age 18. Since that time, he has dislocated the shoulder three more times
despite physical therapy. His last dislocation occurred 4 weeks ago while sleeping.
What is the most appropriate management for this patient?
1- Corticosteroid injection
2- Changing the physical therapist to an athletic trainer
3- A 1-month trial of nonsteroidal anti-inflammatory drugs (NSAIDs)
4- Shoulder immobilization for 6 weeks
5- A discussion regarding surgical stabilization procedures

DISCUSSION: The patient sustained a traumatic shoulder dislocation at age 18 that has
subsequently failed to respond to nonsurgical management. Discussion of surgical
stabilization procedures is warranted at this time. A corticosteroid injection or a trial of
NSAIDs will not provide any stabilizing effect. Further immobilization in this patient
population has not been shown to improve stability.Prefer Respons# 121 is 5.

122 While performing a total shoulder arthroplasty, excessive retraction is placed on


the "strap muscles" (short head of biceps and coracobrachialis). Neurovascular
examination would reveal weakness of which of the following?
1- Shoulder abduction
2- Shoulder external rotation
3- Shoulder internal rotation
4- Elbow extension
5- Forearm supination

DISCUSSION: The musculocutaneous nerve can be as close as 3 cm to the coracoid


process; therefore, this relationship is important to keep in mind when performing
surgery in this area. Excessive traction on the musculocutaneous nerve could lead to a
neurapraxia with resultant weakness of elbow flexion and forearm supinaton because
of the loss of biceps function. The Preferred Response to Question # 122 is 5.
76
215
2011 Upper Extremity Self-Assessment Exam by Dr.Dhahirortho

123With the arm abducted 90 degrees and fully externally rotated, which of the
following glenohumeral ligaments resists anterior translation of the humerus?
1- Coracohumeral ligament
2- Superior glenohumeral ligament
3- Middle glenohumeral ligament
4- Anterior band of the inferior glenohumeral ligament complex
5- Posterior band of the inferior glenohumeral ligament complex

DISCUSSION: With the arm in the abducted, externally rotated position, the anterior
band of the inferior glenohumeral ligament complex moves anteriorly, preventing
anterior humeral head translation. Both the coracohumeral ligament and the superior
glenohumeral ligament restrain the humeral head to inferior translation of the
adducted arm, and to external rotation in the adducted position. The middle
glenohumeral ligament is a primary stabilizer to anterior translation with the arm
abducted to 45 degrees. The posterior band of the inferior glenohumeral ligament
complex resists posterior translation of the humeral head when the arm is internally
rotated. The Preferred Response to Question # 123 is 4.

124 Which of the following statements regarding the use of thermal shrinkage during
arthroscopic shoulder surgery is most accurate?
1- The amount of shrinkage is fixed for a given peak temperature, irrespective of the
time of application.
2- Denatured capsular tissue does not undergo a healing response.
3- The capsule is typically found to be thick and fibrotic in revision cases following
thermal shrinkage.
4- Patients with good results at 1 year are unlikely to develop recurrent instability in
the future.
5- High failure rates have been reported in its use for anterior, posterior, and
multidirectional instability.

DISCUSSION: Reports of clinical results at 2- and 5-year follow-up indicate much higher
failure rates than traditional stabilization techniques for all common instability
patterns. The degree of capsular shrinkage is dependent on the total amount of
77
216
2011 Upper Extremity Self-Assessment Exam by Dr.Dhahirortho

thermal energy delivered, as well as the rate of delivery. Denatured tissue undergoes a
healing response. The capsule typically encountered in revision cases is thin and
patulous, rather than thick and fibrotic. The Preferred Response to Question # 124 is 5.

125 A 54-year-old man with a history of diabetes mellitus underwent internal


fixation for a humeral shaft nonunion 8 months ago. His postoperative course had
been unremarkable. However, over the past few weeks, he reports mild pain with
activity. At rest, he has no pain. He denies any recent fevers or chills. Radiographs
are shown in Figures 125a and 125b. What is the next most appropriate step in
management?

1- CT scan of the humerus to confirm the nonunion


2- Application of a functional fracture brace
3- Laboratory evaluation
4- Removal of hardware and intramedullary fixation
5- Revision internal fixation with a plate
DISCUSSION: Radiographs showing broken hardware (screw head) and the clinical
history are consistent with fracture nonunion; therefore, a CT scan is not required.
Treatment for this nonunion may include various options including functional fracture
bracing, intramedullary fixation, or revision internal fixation. However, prior to any
treatment, infection must be eliminated as a cause for the nonunion. Evaluation for
infection can include laboratory studies such as erythrocyte sedimentation rate and C-
reactive protein level. The Preferred Response to Question # 125 is 3.

78
217
2011 Musculoskeletal Tumors Self-Assessment Exam by Dr.Dhahirortho

1
218
2011 Musculoskeletal Tumors Self-Assessment Exam by Dr.Dhahirortho

2
219
2011 Musculoskeletal Tumors Self-Assessment Exam by Dr.Dhahirortho

Question 1 A 30-year-old man has a painful ankle mass. Radiographs are seen in
Figures 1a and 1b, and MRI scans are seen in Figures 1c and 1d. What is the most
likely diagnosis?

1- Synovial hemangioma
2- Synovial sarcoma
3- Lipoma arborescens
4- Synovial chondromatosis
5- Pigmented villonodular synovitis

DISCUSSION: The calcified loose bodies shown on the imaging studies are
characteristic of synovial chondromatosis. Pigmented villonodular synovitis and
lipoma arborescens do not show calcifications. Synovial sarcoma and synovial
hemangioma can mineralize but they tend to be more diffuse, and not associated
with loose bodies. Synovial sarcoma is rarely intra-articular. Preferred Res # 1 is 4.

Question 2 Figures 2a through 2d show the radiographs, MRI scan, and bone scan
of a 44-year-old man who twisted his knee and felt a pop. Following the injury he
had swelling; however, both the pain and swelling have now resolved. What is the
next most appropriate step in the management of the patient's distal femoral
lesion?

1- Radiographic follow-up
2- Biopsy
3- Surgical resection
4- Radiation therapy
5- Chemotherapy

3
220
2011 Musculoskeletal Tumors Self-Assessment Exam by Dr.Dhahirortho

DISCUSSION: The lesion seen on the radiographs and MRI scan is consistent with an
enchondroma. Enchondromas are commonly encountered by orthopaedic surgeons
as an incidental finding during evaluation of a patient for pain of other causes.
Enchondromas are painless, show no aggressive bone destruction, and commonly
exhibit radiotracer uptake on a bone scan. Surgery for enchondromas is not
generally necessary, but when clinical and radiographic features suggest a potential
chondrosarcoma, action is necessary. Chondrosarcoma almost always occurs in
adults. Patients usually present with pain and have characteristic findings on staging
studies. Lucencies developing within the calcification, periosteal reaction, and an
associated soft-tissue mass all are characteristic features of chondrosarcoma.
Biopsies are not typically indicated because of the histologic similarity of benign and
malignant cartilage lesions. Radiographic follow-up is appropriate when the clinical
suspicion of chondrosarcoma is low. In this patient, the bone lesion was discovered
incidentally because of unrelated trauma and the patient is essentially
asymptomatic with benign-appearing radiographic features.Preferred Res # 2 is 1.

Question 3 Figures 3a through 3e show the radiographs and MRI scans of a 16-
year-old boy. Which of the following best describes features of the patient's
tumor?

1- Telomere translocations
2- Supernumerary ring chromosomes
3- A translocation involving genes 11 and 22
4- A rearrangement involving the X and 18 chromosomes
5- Genetic alterations in the retinoblastoma gene and p53 tumor suppressor gene

DISCUSSION: The imaging studies show a classic case of conventional high-grade


osteosarcoma. Genetic alterations in the retinoblastoma gene and p53 tumor
suppressor gene are common in this tumor. A translocation involving chromosomes
11 and 22 is characteristic of Ewing's sarcoma. Supernumerary ring chromosomes
4
221
2011 Musculoskeletal Tumors Self-Assessment Exam by Dr.Dhahirortho

are seen in parosteal osteosarcoma. Telomere translocations have been described in


giant cell tumor of bone. A translocation of chromosomes X and 18 is typical of
synovial sarcoma. The Preferred Response to Question # 3 is 5.

Question 4 A 12-year-old girl has a 6-month history of a painful expanding mass in


the right lateral ankle. A clinical photograph, radiograph, MRI scans, and histology
specimens are seen in Figures 4a through 4f. What is the most appropriate
management?

1- Resection alone
2- Resection and chemotherapy
3- Resection and radiation therapy
4- Chemotherapy and radiation therapy
5- Observation

DISCUSSION: The radiograph shows an expansile radiolucent lesion without matrix


production or a periosteal reaction. On close inspection, there are subtle internal
septations on the mortise view. The MRI scans show heterogeneous septated areas
with prominent fluid-fluid levels. The histopathology on low power shows villous
projections and prominent clefts, whereas the high power view reveals prominent
spindled stromal cells and interspersed multinucleated giant cells. This is an
aneurysmal bone cyst. The treatment of aneurysmal bone cyst is generally
intralesional surgery (eg, curettage). However, for patients with large destructive
lesions in expendable bones such as the fibula, excision is preferred. The main
differential diagnosis in this patient is telangiectatic osteosarcoma. Radiographs of
telangiectatic osteosarcoma show geographic bone lysis, a wide zone of transition,
and matrix mineralization. Under the microscope, telangiectatic osteosarcoma
exhibits pleomorphic cells with at least focal malignant osteoid production.
Telangiectatic osteosarcoma is treated with chemotherapy and wide excision. The
Preferred Response to Question # 4 is 1.

5
222
2011 Musculoskeletal Tumors Self-Assessment Exam by Dr.Dhahirortho

Question 5 Figures 5a and 5b show the AP radiograph and axial CT scan of an 18-
year-old woman who has proximal thigh pain. What is the most appropriate
treatment?
1- Observation
2- Curettage and grafting
3- Radiofrequency ablation
4- Wide resection
5- Wide resection and chemotherapy

DISCUSSION: The imaging studies are classic for osteoid


osteoma, a benign lesion. Conventional treatment has
involved burring down to find the lesion, followed by
curettage. More recent experience with
radiofrequency ablation under CT guidance has shown
equivalent results with few complications. Curettage in
this location would carry significant morbidity and risk for fracture, making
radiofrequency ablation the most appropriate treatment. More aggressive
resections are not indicated because the lesion has a low chance of recurrence.
Observation and/or use of nonsteroidal anti-inflammatory drugs would be
reasonable if the patient is willing to live with prolonged pain. The Preferred
Response to Question # 5 is 3.

Question 6 What are the five soft-tissue sarcomas that can frequently metastasize
to the lymph nodes?
1- Synovial sarcoma, angiosarcoma, osteosarcoma, chondrosarcoma, Ewing's
sarcoma
2- Chondrosarcoma, osteosarcoma, fibrosarcoma, Ewing's sarcoma,
adamantinoma
3- Rhabdomyosarcoma, synovial sarcoma, epithelioid sarcoma, clear cell sarcoma,
angiosarcoma
4- Myxoid chondrosarcoma, fibrosarcoma, malignant peripheral nerve sheath
tumor, liposarcoma, malignant fibrous histiocytoma
5- Rhabdomyosarcoma, myxoid liposarcoma, acral mxyoinflammatory fibroblastic
sarcoma, synovial sarcoma, epithelioid sarcoma

DISCUSSION: When sarcomas spread, they classically metastasize to the lung. Their
second most common site to metastasize to is either another bone or soft tissue,
depending on whether it is a primary bone or soft-tissue sarcoma. Regional
6
223
2011 Musculoskeletal Tumors Self-Assessment Exam by Dr.Dhahirortho

metastases are relatively a rare occurrence - only about 5% of patients with soft-
tissue sarcomas. The incidence is slightly higher in patients with
rhabdomyosarcoma, synovial sarcoma, epithelioid sarcoma, clear cell sarcoma, or
angiosarcoma. In these patients, consideration for sentinel node biopsy should be
given. The Preferred Response to Question # 6 is 3.

Question 7 A 36-year-old man has had an enlarging left posterior thigh soft-tissue
mass for the past month, and he now reports numbness and tingling in his sciatic
nerve distribution. Based on the MRI scan and biopsy specimen shown in Figures
7a and 7b, what is the most likely diagnosis?

1- Myxoid liposarcoma
2- Synovial sarcoma
3- Malignant fibrous histiocytoma
4- Fibrosarcoma
5- Well-differentiated liposarcoma

DISCUSSION: Most soft-tissue sarcomas have a similar MRI appearance and cannot
be differentiated by signal intensity characteristics. Histologic and often
immunohistochemical analysis is necessary to subtype soft-tissue sarcomas. Myxoid
liposarcomas account for one half of all liposarcomas, with a peak incidence during
the fifth decade. Myxoid liposarcomas have a characteristic histologic appearance
with myxoid background and an interlacing network of fine vessels. The delicate
plexiform capillary vascular network is present throughout these tumors and
provides an important clue for distinguishing them from myxomas. Treatment
consists of wide surgical resection. Synovial sarcomas have a monophasic or biphasic
histologic pattern. Malignant fibrous histiocytoma (undifferentiated pleomorphic
sarcoma) is composed of pleomorphic cells in a storiform pattern. Fibrosarcoma has
a typical "herringbone" pattern on low power histology. Well-differentiated
liposarcoma has bland histology of adipocytes with scattered lipoblasts. The
Preferred Response to Question # 7 is 1.

Question 8 An otherwise healthy 15-year-old boy is seen in the


emergency department for an injury sustained while playing
dodge ball. A radiograph is shown in Figure 8. What is the most
appropriate treatment for this patient?

1- Aspiration and injection


7
224
2011 Musculoskeletal Tumors Self-Assessment Exam by Dr.Dhahirortho

2- Resection and reconstruction


3- Sling and pain medication
4- Curettage and bone grafting
5- Intramedullary nail fixation

DISCUSSION: The patient has a pathologic fracture through a benign-appearing


eccentric bone lesion with a soap-bubble geographic appearance, most suggestive of
a nonossifying fibroma. The best initial treatment is to provide comfort measures
and allow the fracture to heal. Aspiration and steroid injection has been advocated
for a unicameral bone cyst, though it should be done after any associated fracture
has been allowed to heal. Wide resection is not indicated for this benign tumor
Extended curettage and grafting may be appropriate, but only after fracture healing
has occurred unless open reduction and internal fixation is indicated for the fracture
itself. Intramedullary nail fixation is unnecessary and generally should not be
performed if the histopathologic diagnosis is not known. The Preferred Resp # 8 is 3.

Question 9 A 7-year-old boy has a 3-week history of neck pain. History and
physical examination reveal no neurologic symptoms. A radiograph, CT scan, and
MRI scans are seen in Figures 9a through 9d. Figure 9e shows a needle biopsy
specimen. Based on these findings, what is the most appropriate management?

1- Cervical collar and observation


2- Intravenous antibiotics and cervical collar
3- Chemotherapy and radiation therapy
4- Neoadjuvant chemotherapy alone
5- Neoadjuvant chemotherapy and surgical resection

DISCUSSION: The findings are consistent with eosinophilic granuloma. Eosinophilic


granuloma is a focal destructive lesion of bone commonly seen in children younger
than age 10 years. Despite the very aggressive appearance of these lesions, they

8
225
2011 Musculoskeletal Tumors Self-Assessment Exam by Dr.Dhahirortho

typically will heal spontaneously or following biopsy. Local corticosteroid injection


has been advocated by some. Surgical decompression is indicated in rare cases of
cord compression. Chemotherapy is indicated only in disseminated forms of
histiocytosis. The Preferred Response to Question # 9 is 1.

Question 10 A 12-year-old boy is diagnosed with osteosarcoma of the left distal


femur. The lesion measures 7 cm in its greatest dimension and is associated with a
soft-tissue mass that extends into the vastus lateralis. A CT scan of the chest is
normal. A bone scan reveals another lesion in the contralateral right proximal
femur. Biopsy of the right proximal femur lesion also reveals osteosarcoma.
According to the American Joint Committee on Cancer (AJCC), what is the stage of
this tumor?
1- II-A 2- II-B 3- III 4- IV-A 5- IV-B

DISCUSSION: The AJCC staging system for bone sarcomas is based on tumor grade,
size, and the presence as well as location of metastases. Stage I tumors are low
grade. Stage II tumors are high grade. Stages I and II are subdivided based on size.
Stages I-A and II-A are less than or equal to 8 cm in their greatest linear
measurement. Stages I-B and II-B are greater than 8 cm in size. Stage III tumors are
those that have "skip metastases" which are defined as discontinuous lesions within
the same bone. Stage IV-A involves pulmonary metastases, whereas stage IV-B
includes patients with non-pulmonary metastases; therefore, the stage of this tumor
would be IV-B. The Preferred Response to Question # 10 is 5.

Question 11 A 28-year-old woman has a painless mass on the dorsum of the foot
that has been rapidly increasing in size over the last 3 months. It measures 5.7 cm
in maximum diameter. A lateral radiograph of the foot is shown in Figure 11a.
Sagittal T1-weighted and coronal T2-weighted MRI scans are shown in Figures 11b
and 11c. A photomicrograph from the biopsy specimen is shown in Figure 11d. The
most appropriate surgical treatment would likely include which of the following?

9
226
2011 Musculoskeletal Tumors Self-Assessment Exam by Dr.Dhahirortho

1- Intralesional excision
2- Marginal excision and split-thickness skin grafting
3- Aspiration and antibiotics
4- Wide resection and flap coverage
5- Below-knee amputation

DISCUSSION: The clinical presentation, imaging studies, and histology are consistent
with a high-grade soft-tissue sarcoma. The MRI scans show a heterogeneous lesion.
Histology demonstrates a spindle cell lesion with pleomorphism, atypica, and large
bizarre mitoses. Treatment of soft-tissue sarcomas should include at least a wide
resection. Intralesional or marginal excision is not advised for high-grade sarcoma.
Below-knee amputation is an option for this patient, but most likely limb-sparing
procedures for this particular sarcoma would be possible and also more functional
while also allowing proper oncologic surgical resection. On this area of the dorsum
of the foot, split-thickness skin grafting would likely have a higher failure rate than a
fasciocutaneous flap over the tendons. Whereas aspiration of a fluid-containing
cystic lesion (ganglion cyst) would be appropriate, it is not appropriate for this solid,
heterogeneous lesion. The Preferred Response to Question # 11 is 4.

Question 12 Figures 12a and 12b show the radiographs of a 22-year-old man who
reports left knee pain. An MRI scan is seen in Figure 12c, and a photomicrograph is
seen in Figure 12d. What stage is this lesion according to the Enneking staging
system?

1- I
2- II
3- III
4- 1
5- 2

DISCUSSION: The radiographs show a purely radiolucent lesion in the distal femur
with no cortical destruction. There is no soft-tissue extension noted on the MRI
10
227
2011 Musculoskeletal Tumors Self-Assessment Exam by Dr.Dhahirortho

scan. The photomicrograph demonstrates multiple giant cells with a background of


mononuclear cells, confirming the diagnosis of giant cell tumor. This lesion is
classified as a stage 2 (benign active) tumor according to the Enneking staging
system. Using this system, malignant tumors are staged using Roman numerals, and
benign tumors are staged using Arabic numerals. Stage 1 tumors are "latent." Stage
2 tumors are "active," and stage 3 tumors are "aggressive." Whereas some giant cell
tumors are stage 3, the lesion in this patient does not demonstrate particularly
aggressive imaging features, such as cortical breakthrough and soft-tissue extension.
The Preferred Response to Question # 12 is 5.

Question 13 A 9-year-old boy with a history of unilateral retinoblastoma has a 2-


week history of left arm pain. His mother, who had bilateral retinoblastoma, noted
a mass on his left proximal arm 2 days prior to the office visit. A radiograph, MRI
scan, and biopsy specimen are seen in Figures 13a through 13c. What is the most
appropriate treatment?

1- Observation
2- Wide resection
3- Radiotherapy and wide resection
4- Chemotherapy and wide resection
5- Extended curettage and bone
grafting

DISCUSSION: The radiograph and MRI scan show an aggressive lesion in the left
proximal humerus in a child with a history of retinoblastoma. The biopsy specimen
shows pleomorphic spindled cells with focal osteoid. There is a well-documented
association between retinoblastoma and osteosarcoma because it has been found
that most, if not all, such tumors have defects in their RB1 pathway through genetic
lesions in the RB1 gene itself or other genes in the pathway. Linkage analysis at the
retinoblastoma gene (RB1) locus is required for identification of individuals at risk of
developing retinoblastoma and osteosarcoma. Identification of disease-causing
mutations is necessary for accurate risk prediction. The treatment for osteosarcoma
is chemotherapy and surgery, either wide local resection or amputation.
Radiotherapy is not a useful adjunct in the treatment of osteosarcoma.
Chemotherapy and wide resection is the preferred treatment. The Preferred
Response to Question # 13 is 4.

11
228
2011 Musculoskeletal Tumors Self-Assessment Exam by Dr.Dhahirortho

Question 14 An otherwise healthy teenager has an isolated lower extremity mass.


The biopsy of the mass reveals the diagnosis of rhabdomyosarcoma. Management
of the patient's mass should consist of which of the following?
1- Hemipelvectomy amputation
2- Wide resection, chemotherapy, and radiation therapy
3- Wide resection and radiation therapy
4- Marginal excision
5- Observation

DISCUSSION: Treatment of rhabdomyosarcomas includes wide resection of the


primary tumor, chemotherapy, and radiation therapy. Rhabdomyosarcomas are one
of the soft-tissue sarcomas where chemotherapy has been shown to have a
substantial survival benefit. In all cases without a radical margin, radiation therapy
should also be added to the treatment plan. Amputation is rarely needed for local
tumor control and is typically selected when an amputation provides better
functional results than a limb-sparing procedure. Observation of these tumors is
rarely, if ever, indicated. Sentinal node biopsy may be indicated in these patients if
there is clinical or radiographic node enlargement. The Preferred Respon # 14 is 2.

Question 15 A 28-year-old man has had left shoulder pain for 1 year. The pain is
constant and has steadily worsened over time. History reveals that he underwent
curettage for a "noncancerous bone tumor" 5 years ago. A radiograph, bone scan,
MRI scan, and biopsy specimens are shown in Figures 15a through 15e. What is the
most appropriate treatment for this patient?

1- Wide resection alone


2- Resection and chemotherapy
3- Intralesional curettage and grafting
4- Forequarter amputation
5- Radiation therapy and prophylactic internal fixation
12
229
2011 Musculoskeletal Tumors Self-Assessment Exam by Dr.Dhahirortho

DISCUSSION: The radiographs and histology are consistent with a clear cell
chondrosarcoma. The treatment is wide resection. No chemotherapy is indicated
because this is a locally aggressive tumor with minimal risk of metastasis.
Intralesional curettage may be appropriate for a benign bone tumor but not for a
clear cell chondrosarcoma. Amputation and radiation therapy are not indicated to
achieve local disease control. This case illustrates the occasional problems with
diagnosis of this tumor. The Preferred Response to Question # 15 is 1.

Question 16 A 45-year-old man has a 6-month history of a leg mass and recent
ulceration of the skin. The clinical photograph and biopsy specimen are seen in
Figures 16a and 16b. What is the most likely diagnosis?

1- Ewing's sarcoma/peripheral primitive neuroectodermal tumor


2- Extraskeletal myxoid chondrosarcoma
3- Dermatofibrosarcoma
4- Myxoid/round cell liposarcoma
5- Synovial sarcoma

DISCUSSION: Dermatofibrosarcoma is a rare, monoclonal, cutaneous sarcoma arising


in the dermis in the trunk, 47%; lower extremity, 20%; upper extremity, 18%; and
head and neck, 14%. Symptoms may be present for a duration of 6 months to 30
years. It may occur at any age, with a peak incidence in the fourth decade; it is more
common in men than women (3:2). Most tumors are superficial and less than 5 cm,
but 3% are larger than 10 cm. They present early as pink or violet-red plaques
surrounded by telangiectatic skin. A nodular growth pattern with ulceration and
attachment to deeper structures is observed in advanced and/or recurrent cases.
Most cases (85% to 90%) are low grade but (5% to 15%) contain focal, high-grade
fibrosarcomatous areas with intermediate-grade tumor (DFSP-FS). MRI is useful in
ascertaining tumor extent and depth of invasion. Tumor cells exhibit a storiform
growth pattern and infiltrate adjacent adnexal structures and adipose tissue.
Fibrosarcomatous transformation can occur over time and is recognized by
13
230
2011 Musculoskeletal Tumors Self-Assessment Exam by Dr.Dhahirortho

increased mitotic activity. Because some tumors express platelet derived growth
factor, they may be responsive to imatinib. Ewing's sarcoma, extraskeletal myxoid
chondrosarcoma, liposarcoma, and synovial sarcoma are generally deep-seated
tumors that can but very rarely cause skin ulceration. The Preferred Resp# 16 is 3.

Question 17 An otherwise healthy 52-year-old man has had a several year history
of a slowly enlarging, symptomatic left shoulder, axillary, and chest wall mass.
Radiographs are only remarkable for a large soft-tissue mass. Selected sequences
of MRI scans are shown in Figure 17a (T1), Figure 17b (T2 fat saturated), and Figure
17c (T1 fat saturated post-gadolinium). Management of this symptomatic mass
should consist of which of the following?

1- Forequarter amputation
2- Wide resection, chemotherapy, and radiation therapy
3- Wide resection and radiation therapy
4- Marginal excision
5- Incisional biopsy

DISCUSSION: The MRI scans show a large soft-tissue mass that is iso-intense with
subcutaneous fat of all sequences. This is diagnostic of a lipocytic (fatty) tumor-
either lipoma, atypical lipomatous tumor, or low-grade liposarcoma. As such, no
biopsy is necessary because biopsies of these fatty tumors can be fraught with
sampling error. The most appropriate treatment of this symptomatic lesion is
simple, marginal excision without radiation therapy or chemotherapy. Local
recurrence can occur in 25% to 50% of patients at 10 years. Dedifferentiation is rare
with subsequent recurrent disease; but when it occurs, it may result in metastases.
The Preferred Response to Question # 17 is 4.

14
231
2011 Musculoskeletal Tumors Self-Assessment Exam by Dr.Dhahirortho

Question 18 Tumoral calcinosis is a hereditary disease that involves which of the


following?
1- Calcium metabolism
2- Phosphate metabolism
3- Oxalate metabolism
4- Chloride metabolism
5- Renal tubular insufficiency

DISCUSSION: Although not completely understood, the phosphate metabolic


dysfunction seen in tumoral calcinosis is a hereditary disease. This is characterized
by soft-tissue lesions that are calcified, lobulated, well-demarcated lesions usually
found over the extensor surfaces of large joints. It is most commonly found around
the hip, elbow, shoulder, foot, and wrist joints. Renal tubular insufficiency is
associated with acidosis, glucosuria, phosphaturia, aminoaciduria, and mild
proteinuria. Oxalate metabolism can contribute to calcium oxalate crystals and
kidney stones. Oxalate is found in many food products. Calcium metabolism, which
can be abnormal in renal insufficiency, hypervitaminosis D, and milk-alkali
syndrome, is normal in tumoral calcinosis. Metabolic alkalosis is often associated
with a alteration in chloride metabolism. The Preferred Response # 18 is 2.

Question 19 A 25-year-old man has a painful mass in a web space of his foot. MRI
scans are seen in Figures 19a (T2 STIR) and 19b (T1), a representative gross
specimen is seen in Figure 19c, and a H&E stain is seen in Figure 19d. What is the
most likely diagnosis?

1- Melanoma
2- Synovial sarcoma
3- Interdigital neuroma
4- Epithelioid sarcoma
5- Giant cell tumor of tendon sheath
15
232
2011 Musculoskeletal Tumors Self-Assessment Exam by Dr.Dhahirortho

DISCUSSION: Giant cell tumors of tendon sheath are common in the hands and feet.
Because of significant hemosiderin deposition, they commonly appear hypointense
to skeletal muscle on both T1 and T2 pulse-weighted sequences. The hemosiderin is
manifested in the brownish discoloration in the gross specimen. The
photomicrograph shows bland spindled stromal cells and abundant multinuclear
giant cells. Treatment is marginal excision with relatively low rates of tumor
recurrence. Although the foot is not an infrequent site of melanoma and there are
some shared radiologic features with giant cell tumor of tendon sheath,
histologically melanoma is composed of cells both spindled and epithelioid arranged
in nests or clusters. Synovial sarcoma is the most common sarcoma of the foot
which radiographically has mineralizations in 30% of cases. It is typically
heterogeneous on both MR pulse sequences. Microscopically, monophasic synovial
sarcoma contains spindled cells that are arranged in short intersecting fascicles
similar to fibrosarcoma. Pseudoglandular areas can be observed in biphasic cases.
Epithelioid sarcoma, though common in the hand, is relatively rare in the foot and is
histologically distinct from giant cell tumor of tendon sheath. When this tumor
secondarily involves bone, it may be confused with osteomyelitis.

Pref Res# 19 is 5.

Question 20 Osteofibrous dysplasia and adamantinoma are both rare tumors of


bone generally occurring in the tibia. Which of the following histologic findings is
typically seen only in adamantinoma?

1- Biphasic pattern of glandular epithelial cells surrounded by spindle cells


2- Bone trabeculae with osteoblastic rimming and fibrous stroma
3- Bone trabeculae without osteoblastic rimming within fibrous stroma
4- Multinucleated giant cells with background stromal cells with similar nuclei
5- Herringbone pattern with pleomorphic cells

DISCUSSION: A biphasic pattern of glandular epithelial cells surrounded by fibrous


spindle cells is the distinguishing histologic feature of an adamantinoma. The other
choices are the histologic patterns seen with osteofibrous dysplasia, fibrous
dysplasia, giant cell tumor, and fibrosarcoma, respectively.

Preferred Res# 20 is 1.

16
233
2011 Musculoskeletal Tumors Self-Assessment Exam by Dr.Dhahirortho

Question 21 A 7-year-old boy has multiple firm, fixed masses about his knees and
extremities. Occasionally he has pain when he bumps his knee or around his
proximal legs when he is playing soccer. Radiographs are shown in Figures 21a and
21b. A CT scan of the distal femurs is shown in Figure 21c. What is the next most
appropriate step in management?

1- Observation
2- Biopsy of the largest lesions
3- Whole body PET scan
4- Removal of all lesions about the knee
5- Bisphosphonate therapy

DISCUSSION: The patient's clinical presentation and imaging studies are diagnostic
of multiple hereditary exostoses (MHE). Whereas removal of symptomatic
osteochondromas is indicated if symptoms are severe enough, biopsy of the lesions
or removal of all of the lesions is not indicated. Currently, a PET scan does not have a
defined role in the evaluation of MHE patients. Bisphosphonate therapy currently
has no defined role in the treatment of MHE patients. These patients are best
observed if asymptomatic, and the development of symptoms or masses that grow
after skeletal maturity should be evaluated for possible malignant degeneration of
these lesions.

The Preferred Response to Question # 21 is 1.

17
234
2011 Musculoskeletal Tumors Self-Assessment Exam by Dr.Dhahirortho

Question 22 A 24-year-old woman has had a mass on her right shoulder for the
past 3 months. The mass is intermittently painful, warm, and swollen, particularly
after periods of activity. Rest and ice relieve her symptoms. Radiographs are
normal. T1-, T2-weighted, and contrast-enhanced MRI images are shown in Figures
22a through 22c. What is the most likely diagnosis?

1- Synovial cell sarcoma


2- Carcinoma metastatic to soft tissue
3- Muscle tear with hematoma formation
4- Abscess
5- Intramuscular hemangioma

DISCUSSION: The lesion is an intramuscular hemangioma. Imaging characteristics


can appear quite aggressive, and sarcoma is often in the differential. Fat is a
prominent feature in diagnosing these tumors. In addition contrast aids in
demonstrating the vascular channels, particularly in cavernous subtype. No necrosis
or rim enhancement is demonstrated and no appreciable mass effect is noted,
making malignancy or abscess less likely. A muscle tear with hematoma formation
could have a similar appearance, but the time course should have allowed for partial
or complete hematoma resolution. Observation is generally the treatment of choice,
but if symptoms are not relieved by noninvasive means, surgery, sclerotherapy, and
sometimes embolization may have a role. The Preferred Response # 22 is 5.

Question 23 What syndrome is associated with the presence of enchondromas and


hemangiomas?
1- Maffucci's syndrome
2- Hunter's syndrome
3- Multiple hereditary exostoses
4- Ollier's disease
5- Trevor's disease

DISCUSSION: Maffucci's syndrome is characterized by the presence of


enchondromas and hemangiomas. Ollier's disease is multiple enchondromatosis.
Multiple hereditary exostoses is characterized by the presence of multiple exostoses
18
235
2011 Musculoskeletal Tumors Self-Assessment Exam by Dr.Dhahirortho

not associated with skin lesions or endocrine abnormalities. Hunter's syndrome is a


mucopolysaccharidosis and lysosomal storage disease that affects all large joints
with pain and decreased range of motion. Trevor's disease is an epiphyseal
osteochondroma. The Preferred Response to Question # 23 is 1.

Question 24 A 4-year-old boy has a 3-month history of limping and pain in the
right tibia. According to his parents, he has frequent night pain. There is no history
of weight loss, fevers, or night sweats. Examination reveals that the left tibia is
moderately tender to palpation. There is no palpable mass or lymphadenopathy.
Radiographs, MRI scans, and biopsy specimens are seen in Figures 24a through
24f. What is the fusion protein frequently associated with this disorder?

1- SYT-SSX
2- ASPL-TFE3
3- EWS-ATF1
4- EWS-FLI1
5- TLS-CHOP

DISCUSSION: The permeative changes and "onion skin" periosteal reaction with no
observable matrix production on radiographs are suspicious for Ewing's sarcoma.
The cells are small, round, and monotonous on low power and have large
hyperchromatic nuclei and indistinct cytoplasmic borders on high power, supporting
the diagnosis of Ewing's sarcoma. Molecular analysis confirmed the EWS-FLI1
mutation. The radiographic differential diagnosis includes: osteomyelitis, Langerhans
cell histiocytosis, and lymphoma. EWS-ATF1 is associated with clear cell sarcoma,
while ASPL-TFE3 is associated with alveolar soft part sarcoma. SYT-SSX and TLS-
CHOP are associated with synovial sarcoma and liposarcoma, respectively. The
Preferred Response to Question # 24 is 4.
19
236
2011 Musculoskeletal Tumors Self-Assessment Exam by Dr.Dhahirortho

Question 25 A 15-year-old girl is referred for evaluation of a tibial lesion noted on


radiographs performed after a twisting injury of the left knee. She was
asymptomatic prior to the injury that occurred 3 weeks ago. The pain from the
injury has now resolved. Radiographs of the left knee are seen in Figures 25a and
25b. What is the next step in management of this patient?
1- CT
2- MRI
3- Bone scan
4- Needle biopsy
5- Observation

DISCUSSION: The radiographs reveal a


benign-appearing lesion of the
proximal tibia metaphysis with a
narrow zone of transition, an eccentric position, and a thin rim of reactive bone. The
imaging characteristics are most consistent with nonossifying fibroma. These lesions
may occur multiply in 8% of patients. Most lesions are eccentric with a "soap
bubble" appearance and may have significant cortical thinning. Pathologic fractures
may occur. This lesion was discovered as an incidental finding. Because the patient is
asymptomatic, no further work-up or treatment is indicated except for observation.
Most lesions resolve spontaneously by adulthood. The Preferred Response # 25 is 5.

Question 26 A 13-year-old girl injured her left shoulder playing volleyball. The
shoulder was previously asymptomatic. She was referred for evaluation of a left
humeral lesion noted on radiographs obtained after the injury. Currently she has
returned to playing volleyball and is asymptomatic again. AP and lateral
radiographs of the shoulder are seen in Figures 26a and 26b. What is the next most
appropriate step in management of this patient?

1- MRI
2- CT
3- Bone scan
4- Needle biopsy
5- Observation

20
237
2011 Musculoskeletal Tumors Self-Assessment Exam by Dr.Dhahirortho

DISCUSSION: The radiographs reveal an osteochondroma of the left humerus;


therefore, no further work-up is needed. The lesion was found as an incidental
finding after an injury and since it is asymptomatic, no further treatment is needed
except for observation. An MRI scan or a CT scan could be obtained to evaluate the
thickness of the cartilaginous cap if a secondary chondrosarcoma is suspected in an
enlarging lesion in an adult. These studies could also be obtained to define anatomic
relationships if surgery is planned. A bone scan could be used to identify other
lesions but would not likely provide useful information for this patient. Needle
biopsy would not provide any useful information. The Preferred Response # 26 is 5.

Question 27 A 55-year-old woman with a history of lung cancer who underwent


resection 1 year ago now reports thigh pain. Radiographs are shown in Figures 27a
and 27b. A CT scan of the chest, abdomen, and pelvis and a bone scan show no
other lesions. What is the most appropriate next step in management?

1- Chemotherapy
2- External fixation
3- Reamed intramedullary nail with reamings
sent to pathology
4- Biopsy
5- External beam radiation followed by
placement of an intramedullary nail

DISCUSSION: The radiographs show a lytic lesion that is highly suggestive of


metastatic cancer in this clinical setting. However, in this patient with no other bone
lesion as demonstrated by bone scan, a primary sarcoma or myeloma cannot be
excluded. Therefore, biopsy should be done prior to prophylactic stabilization to
avoid inadvertently contaminating the entire femur. Sending reamings during the
intramedullary nailing is not an acceptable method of performing a biopsy on an
indeterminate lesion. Chemotherapy, radiation, nailing, or external fixation should
not be performed prior to establishing a diagnosis. The Preferred Resp # 27 is 4.

21
238
2011 Musculoskeletal Tumors Self-Assessment Exam by Dr.Dhahirortho

Question 28 Which of the following is the most common soft-tissue sarcoma of the
hand?
1- Chondrosarcoma
2- Rhabdomyosarcoma
3- Epithelioid sarcoma
4- Myxoid liposarcoma
5- Hemangiopericytoma

DISCUSSION: The hand is a rare location for soft-tissue sarcomas. The most common
histologic subtypes seen there are epithelioid sarcoma, malignant fibrous
histiocytoma, synovial sarcoma, and clear cell sarcoma. The Preferred Res# 28 is 3.
REFERENCES: Pradhan A, Cheung YC, Grimer RJ, Peake D, Al-Muderis OA, Thomas

Question 29 A 70-year-old man has a painful leg mass. The mass has been present
for many years and has not changed in size. It is exquisitely tender to palpation.
MRI scans are seen in Figures 29a through 29c, and a biopsy specimen is seen in
Figure 29d. What is the most likely diagnosis?

1- Lipoma
2- Fibrosarcoma
3- Neurofibroma
4- Synovial sarcoma
5- Giant cell tumor of tendon sheath

DISCUSSION: Exquisite tenderness is a hallmark of benign nerve sheath tumors. The


MRI scans show a well-defined soft-tissue mass within the posterior tibial nerve. The
histology is consistent with a neurofibroma and characterized by hypocellularity and
"rope-like" or wavy collagen bundles. The MRI scan findings are not consistent with
a lipoma. The histology is not consistent with fibrosarcoma (characterized by
hypercellularity, atypia, and "herringbone" growth pattern), giant cell tumor of
22
239
2011 Musculoskeletal Tumors Self-Assessment Exam by Dr.Dhahirortho

tendon sheath (monomorphic stromal cells, hemosiderin-laden macrophanges, and


giant cells), synovial sarcoma (hypercellular, monophasic or biphasic epithelioid
cells), or lipoma (bland lipocytes). The Preferred Response to Question # 29 is 3.

Question 30A 10-year-old boy has had increasing left knee pain for the past 4
months. AP and lateral radiographs of the left proximal tibia are seen in Figures
30a and 30b. An MRI scan and biopsy specimen are seen in Figures 30c and 30d.
What is the most appropriate treatment for this patient?

1- Surgery and chemotherapy


2- Surgery and radiation therapy
3- Radiation therapy and chemotherapy
4- Surgery alone
5- Radiation therapy alone

DISCUSSION: The imaging studies demonstrate an aggressive-appearing blastic


lesion of the proximal tibial metaphysis. This is highly suggestive of osteosarcoma.
The biopsy specimen reveals malignant spindle cells that produce osteoid, thus
confirming the diagnosis of osteosarcoma. Treatment of osteosarcoma is
multimodal including multi-agent chemotherapy and surgery (wide resection or
amputation). Radiation therapy is not used in the initial curative treatment of
osteosarcoma. The Preferred Response to Question # 30 is 1.

Question 31 A 72-year-old woman reports pain with overhead activity.


Examination reveals a prominent mass near the inferior angle of the scapula. A CT
scan is shown in Figure 31a and a biopsy specimen is shown in Figure 31b. What is
the most likely diagnosis?
1- Lipoma
2- Lymphoma
3- Hemangioma
4- Elastofibroma
5- Giant cell tumor of tendon sheath
23
240
2011 Musculoskeletal Tumors Self-Assessment Exam by Dr.Dhahirortho

DISCUSSION: Although elastofibromas can occur anywhere in the body, the most
common location is the ventral aspect of the scapula. The CT scan shows a mass
anterior to the left scapula and overlying the chest wall with strands of low
attenuation representing fat. The biopsy specimen shows a pauci-cellular disease
process with dense collagen and elastin fibers. The dark staining areas represent
elastin. Lipomas generally are homogeneous with low attenuation on CT. Lymphoma
has a fairly uniform small round cell morphology microscopically. Hemangiomas may
have mineralizations (phleboliths) on radiographs and CT and microscopically reveal
multiple capillary or cavernous vascular spaces with intervening fat. On histologic
sections, giant cell tumor of tendon sheath is a spindle cell proliferation with
interspersed giant cells and hemosiderin-laden macrophages.
Pre Res# 31 is 4.

Question 32 A 46-year-old man has a 2-month history of an enlarging medial arm


mass. Representative MRI scans and a biopsy specimen are shown in Figures 32a
through 32c. What is the genetic abnormality associated with this type of soft-
tissue tumor?
1- t(11;22)
2- t(X;18)
3- t(12;16)
4- t(2;13)
5- t(12;22)

DISCUSSION: The images demonstrate a deep, heterogenous mass with histology


showing a typical biphasic pattern seen with synovial sarcoma. Synovial sarcoma is
characterized by t(X;18), which leads to the formation of a fusion gene between the
SYT on chromosome 18 and one of the adjacent genes on the X chromosome, SSX1
or SSX2. Using RT-PCR, it is possible to detect the SYT-SSX fusion transcript. A
characteristic t(11;22) occurs in Ewing's sarcoma. This is the most studied solid
tumor chromosome translocation. This rearrangement leads to juxtaposition of a
gene, designated EWSR1, on chromosome 22 with FLI1 gene on chromosome 11.
Myxoid liposarcoma is characterized by the presence of t(12;16). Most alveolar
rhabdomyosarcoma cells exhibit t(2;13). t(12;22) has been associated with clear cell
sarcoma.
The Preferred Response to Question # 32 is 2.

24
241
2011 Musculoskeletal Tumors Self-Assessment Exam by Dr.Dhahirortho

Question 33 Figures 33a through 33c show the MRI scans and biopsy specimen of a
9-year-old girl who has had progressive swelling and a mass on her medial elbow
for 1 month. The area is increasingly painful to touch and with range of motion.
The remainder of her examination is unremarkable. What is the next most
appropriate step in management?
1- Chemotherapy
2- Radiation therapy
3- Antibiotic therapy
4- Observation
5- Physical therapy

DISCUSSION: Cat scratch disease (CSD) is typically a benign and self-limited illness
lasting 6 to 12 weeks in the absence of antibiotic therapy. Regional
lymphadenopathy (axillary, epitrochlear, inguinal) is the predominant clinical feature
of CSD; affected nodes are often tender and occasionally suppurate. Between 25%
and 60% of patients report a primary cutaneous inoculation lesion (0.5- to 1-cm
papule or pustule) at the site of a cat scratch or bite. The skin lesions typically
develop 3 to 10 days after injury and precede the onset of lymphadenopathy by 1 to
2 weeks. Bartonella henselae is now regarded as the etiologic agent of CSD. For
many years, CSD has been clinically diagnosed when three of the following four
criteria are met in a patient: 1) history of traumatic cat contact; 2) positive skin-test
response to CSD skin-test antigen; 3) characteristic lymph node lesions; and 4)
negative laboratory investigation for unexplained lymphadenopathy. Treatment
consists of azithromycin, ciprofloxacin, doxycycline, or multiple other antibiotics, all
of which have been used successfully. Radiation therapy and chemotherapy would
be reserved for malignant diseases and would not be appropriate in this setting.
Treatment is necessary for this infectious entity; therefore, observation or physical
therapy is not indicated. The Preferred Response to Question # 33 is 3.

Question 34 Figures 34a through 34d show the AP and lateral radiographs, MRI
scan, and biopsy specimen of a 45-year-old man who has had pain for 3 months.
Immumohistochemistry shows CD99 negativity. What is the most likely diagnosis?
1- Lymphoma of bone
2- Ewing's sarcoma
3- Chondrosarcoma
4- Metastatic adenocarcinoma
5- Osteogenic sarcoma
25
242
2011 Musculoskeletal Tumors Self-Assessment Exam by Dr.Dhahirortho

DISCUSSION: Lymphoma is a destructive lesion of bone, often with a large soft-tissue


mass. Histology shows basophilic cells of varying size with little background stroma.
Ewing's sarcoma histology also shows uniform sheets of small blue round cells, but
typically occurs in childhood and shows CD99 positivity on immunohistochemistry.
Chondrosarcoma demonstrates malignant cartilage, adenocarcinoma demonstrates
a glandular arrangement of epithelial cells, and osteogenic sarcoma is a mixed
lytic/blastic lesion demonstrating malignant cells producing immature osteoid. The
Preferred Response to Question # 34 is 1.

Question 35 A 45-year-old woman has a slowly enlarging mass over the radial
aspect of her middle finger at the level of the proximal interphalangeal joint. It is
associated with decreased flexion of the joint and clinically is fixed to the
underlying bone. Radiographs reveal erosion of the lateral cortex of the proximal
phalanx. Gross observation at the time of surgery reveals that the mass has a
yellowish-brown tint and lobulated areas. Histology demonstrates bland fibrous
stroma with scattered histiocytes, giant cells, and hemosiderin. What is the most
likely diagnosis?

1- Epithelioid sarcoma
2- Giant cell tumor of tendon sheath
3- Gouty tophus
4- Hemangioma
5- Epithelial inclusion cyst

DISCUSSION: The clinical and pathologic description is typical of a giant cell tumor of
tendon sheath. Epithelioid sarcoma is the most common soft-tissue sarcoma in the
hand and is composed of a nodular arrangement of tumor cells with epithelioid
appearance and eosinophilia with a tendency to undergo central degeneration and
26
243
2011 Musculoskeletal Tumors Self-Assessment Exam by Dr.Dhahirortho

ulceration. Gouty tophi have a characteristic white, chalky gross appearance and will
demonstrate negatively birefringent crystals on polarized light microscopy.
Hemangiomas are composed of a variable amount of fat and vessels. Epithelial
inclusion cysts are filled with keratin from desquamation of the hyperkeratotic,
stratified squamous epithelial cells that line the cysts. The Preferred Resp# 35 is 2.

Question 36 Figures 36a through 36e show the AP and lateral radiographs, axial CT
scan, sagittal MRI scan, and biopsy specimen of an 18-year-old man with knee
pain. What is the most likely diagnosis?

1- Fibrous dysplasia
2- Osteoblastoma
3- Osteosarcoma
4- Nonossifying fibroma
5- Osteomyelitis

DISCUSSION: The most likely diagnosis is osteoblastoma, which has a widely varied
radiographic appearance. It can often best be described as a large osteoid osteoma
(> 1 to 2 cm). It can be lytic or blastic, but is usually well-marginated, indicating its
benign nature. Histology shows trabecula with intermixed nonmalignant
osteoblasts. Osteoid osteoma is a small (< 1 to 2 cm) lesion with thick sclerotic bone
surrounding a small lytic nidus. Histology is very similar to osteoblastoma.
Osteosarcoma has malignant cells seen on histology. Nonossifying fibroma is an
eccentric lytic lesion occurring in the metaphysis and has a bland fibrous background
arranged in a storiform pattern with scattered giant cells under the microscope.
Osteomyelitis tends to be partially lytic often with a draining sinus or overlying skin
changes. Fibrous dysplasia is characterized by ground glass calcification and a
chinese character pattern on histology. The Preferred Response # 36 is 2.

Question 37 Which of the following is associated with local recurrence following


excision of the lesion shown in Figures 37a through 37c?
1- Age of patient
2- Effectiveness of chemotherapy
3- Tumor necrosis
4- Sensitivity to radiation
5- Type of graft material
27
244
2011 Musculoskeletal Tumors Self-Assessment Exam by Dr.Dhahirortho

DISCUSSION: The lesion shown in the images is an aneurysmal bone cyst. These
lesions are known to have a local recurrence rate of 5% to 50%. Young age, open
physes, stage, and type of surgical removal and resulting margin have all been
shown to affect the recurrence rate. Chemotherapy and radiation are not used in
the treatment of aneurysmal bone cysts. The percentage of necrosis of the lesion is
prognostic in osteosarcoma. The type of graft material does not affect local
recurrence. The Preferred Response to Question # 37 is 1.

Question 38 Figures 38a through 38c show the radiographs and CT scan of a 24-
year-old man who reports tightness in the left knee and decreased range of
motion. A biopsy specimen is shown in Figure 38d. What is the most appropriate
treatment plan?

1- Radiation therapy and surgical resection


2- Chemotherapy and surgical resection
3- Chemotherapy, radiation therapy, and surgical resection
4- Surgical resection only
5- Observation

DISCUSSION: Parosteal osteosarcoma is a well-differentiated malignant tumor


arising on the surface of the bone. It comprises less than 5% of all osteogenic
sarcomas. It is amenable to surgical resection alone. A number of reports have
confirmed that parosteal osteosarcoma has a better prognosis than other
osteosarcomas. Histologic features are a bland fibroblastic stroma intermixed with
well-developed bony trabeculae. Radiation therapy and chemotherapy have no role
in the treatment of low-grade parosteal osteosarcoma. Observation is not
appropriate for this lesion. The Preferred Response to Question # 38 is 4.

28
245
2011 Musculoskeletal Tumors Self-Assessment Exam by Dr.Dhahirortho

Question 39 A 63-year-old man has had increasing left leg pain over the last
several months. History reveals that he has had recurring cyclic pain in the leg for
the past several years. Radiographs show an enlarged, sclerotic tibia, with
thickened coarse trabeculae and varus bowing. What is the most appropriate
management for this patient?
1- Vitamin D
2- Calcium supplement
3- Methotrexate
4- Nonsteroidal antiinflammatory drugs (NSAIDs)
5- Bisphosphonate therapy

DISCUSSION: Based on the signs and symptoms, Paget's disease is the most likely
diagnosis. In Paget's disease, an elevated alkaline phosphatase level and high output
heart failure may be seen. Hearing loss can be seen when there is involvement of
the skull, and malignant degeneration is uncommon but recognized as a risk.
Patients are often treated with bisphosphonate medications during the active
disease process to help control osteoclastic activity and pain. Vitamin D and calcium
are more appropriate for treatment of osteoporosis. Methotrexate is not indicated
for the treatment of Paget's disease. NSAIDs may be helpful to treat pain associated
with Paget's disease but will not alter the clinical course. The Preferred Res # 39 is 5.

Question 40 Radiographs of the right knee of a 21-year-old man are seen in Figures
40a and 40b. What is the inheritance pattern of this disorder?

1- Sporadic
2- Autosomal dominant
3- Autosomal recessive
4- X-linked dominant
5- X-linked recessive

DISCUSSION: The radiographs show


multiple osteochondromas and are
thus diagnostic for multiple
hereditary exostoses (MHE). MHE is
an autosomal-dominant disorder with greater than 95% penetrance. It is associated
with mutations of the EXT1 or EXT2 genes. In addition to multiple osteochondromas,
affected individuals typically exhibit short stature and angular deformities of the
long bones. The Preferred Response to Question # 40 is 2.
29
246
2011 Musculoskeletal Tumors Self-Assessment Exam by Dr.Dhahirortho

Question 41 A 57-year-old woman has a right proximal humerus lesion that was
originally discovered when a chest radiograph was obtained. The right shoulder is
asymptomatic. An AP radiograph of the right shoulder is seen in Figure 41. What is
the next step in management?

1- MRI
2- CT
3- Bone scan
4- Biopsy
5- Observation

DISCUSSION: The radiograph shows a benign-appearing lesion with stippled


calcification. There is no cortical erosion. Equally important is the fact that the
patient is asymptomatic. These findings are typical of enchondroma. No further
work-up is indicated for this patient except for observation with serial radiographs.
If the patient has symptoms, further cross-sectional imaging may be necessary to
assess the aggressiveness of the lesion. A bone scan will differentiate monostotic
disease from polyostotic disease and the relative uptake may help assess activity.
Biopsy is usually not helpful in distinguishing between enchondromas and low-grade
chondrosarcomas. The Preferred Response to Question # 41 is 5.

Question 42 Giant cell tumor of the tendon sheath is histologically most closely
related to which of the following?
1- Desmoplastic fibroma
2- Dermatofibroma
3- Pigmented villonodular synovitis (PVNS)
4- Myositis ossificans
5- Epithelial inclusion cyst

DISCUSSION: A giant cell tumor of the tendon sheath consists of multinucleated


giant cells, polygonal mononuclear cells, and histiocytes (may contain abundant
hemosiderin or lipid). This same cell population is seen in PVNS in addition to
hemosiderin. Desmoplastic fibroma is composed of dense and irregularly arranged
collagen bundles with infrequent fibroblasts. Dermatofibroma is composed of
nodular cellular proliferation consisting of short intersecting fascicles of fibroblastic
cells in a loose crisscross or storiform pattern. Myositis ossificans is characterized by
zonal proliferation with central fibroblasts and peripheral osteoblast-rimmed bone
30
247
2011 Musculoskeletal Tumors Self-Assessment Exam by Dr.Dhahirortho

trabeculae. Epithelial inclusion cysts are filled with keratin from desquamation of
the hyperkeratotic, stratified squamous epithelial cells that line the cysts.
The Preferred Response to Question # 42 is 3.

Question 43 Figures 43a through 43d show the radiograph, bone scan, CT scan, and
biopsy specimen of a 64-year-old woman who reports increasing right hip pain for
the past 6 months. Treatment of this lesion consists of which of the following?

1- Surgery alone
2- Chemotherapy alone
3- Radiation therapy alone
4- Surgery and chemotherapy
5- Surgery and radiation therapy

DISCUSSION: The radiograph and the CT scan show an expansile, destructive lesion
of the right acetabulum with stippled calcification. The bone scan shows increased
uptake in the area of the lesion. The biopsy specimen reveals hypercellular cartilage,
confirming the diagnosis of conventional chondrosarcoma. Treatment consists of
surgery alone. In this patient, an internal hemipelvectomy with wide margins would
be appropriate. Chondrosarcomas are resistant to both chemotherapy and radiation
therapy. The Preferred Response to Question # 43 is 1.

Question 44 Figure 44a shows the lateral radiograph of a 28-year-old-man who has
had shin pain for the past 2 years. Sagittal and axial MRI scans are shown in
Figures 44b and 44c. A biopsy specimen is shown in Figure 44d. What is the best
treatment for this lesion?
1- Observation
2- Curettage and grafting
3- External beam radiation alone
4- Wide resection alone
5- Chemotherapy and wide excision
31
248
2011 Musculoskeletal Tumors Self-Assessment Exam by Dr.Dhahirortho

DISCUSSION: The imaging showing a bubbly lesion in the anterior cortex of the tibia
and the histology showing epithelial nests are consistent with adamantinoma. The
best treatment for this lesion is wide resection without adjuvant treatments.
Histology demonstrating epithelial cells in a fibrous stroma confirm the diagnosis,
and differentiate it from osteofibrous dysplasia, which follows the same anatomic
distribution and has an overlapping radiologic appearance. Observation, curettage,
and external beam radiation are not indicated for adamantinoma. There is no role
for chemotherapy in this low-grade lesion. The Preferred Response # 44 is 4.

Question 45 An 18-year-old woman has had knee pain for the past 6 months. A
radiograph and biopsy specimen are shown in Figures 45a and 45b. What is the
most appropriate treatment option?
1- Wide resection and reconstruction
2- Radiofrequency ablation
3- Radiation therapy
4- Extended curettage with use of adjuvants
5- Observation

DISCUSSION: The radiograph and biopsy


specimen are characteristic for giant cell
tumor of bone. The standard of care involves extended curettage, use of adjuvants
to lower the risk of local recurrence, and filling of the void with bone graft and/or
polymethylmethacrylate. Multiple adjuvants are acceptable and vary based on
surgeon training and preference. These include phenol, liquid nitrogen, sterile
water, and argon beam coagulation. Wide resection and skeletal reconstruction
32
249
2011 Musculoskeletal Tumors Self-Assessment Exam by Dr.Dhahirortho

should be reserved for extensive bone destruction where salvaging the involved
bone is not feasible. Radiation therapy should be used with extreme caution
because of the risk of secondary sarcomatous degeneration. In multiply recurrent
lesions and more central, hard to access lesions, radiation therapy sometimes can
play a role in the management of giant cell tumors. The Preferred Re# 45 is 4.

Question 46 A 50-year-old woman with a recent diagnosis of multiple myeloma is


being evaluated for left hip pain and some difficulty with ambulation. She is still
receiving her initial systemic management and has extensive skeletal involvement
found on her initial skeletal survey and other staging modalities. Once referred for
orthopaedic evaluation, an AP radiograph of her pelvis was obtained and is shown
in Figure 46. What is the most appropriate surgical option for this patient?

1- Radical resection of the proximal third of the femur and tumor prosthetic
reconstruction
2- Cemented hemiarthroplasty or total hip arthroplasty
3- Intramedullary nailing
4- Percutaneous screw fixation
5- Hip screw and side plate fixation

DISCUSSION: The goal of surgical management of myeloma is the same as it is for


bone metastases - to relieve pain and restore function by achieving local tumor
control and immediate mechanical stability. Pathologic femoral neck fractures that
require surgical intervention should always be treated with removal and
hemiarthroplasty or total hip arthroplasty. Bone cement augments structural
stability and enables the patient to withstand the stress of immediate motion and
function. Internal fixation options are less attractive because they typically do not
allow immediate full weight bearing and would require the fracture to heal with the
harmful effects of radiation and chemotherapy interfering with the healing process.
Radical resection of the proximal femur and the tendinous attachments is not
necessary for this patient and would increase the morbidity of the procedures. The
Preferred Response to Question # 46 is 2.

33
250
2011 Musculoskeletal Tumors Self-Assessment Exam by Dr.Dhahirortho

Question 47 A 10-year-old boy reports increasing left knee pain. A radiograph, MRI
scan, and biopsy specimen are shown in Figures 47a through 47c. What is the most
likely diagnosis?
1- Osteoblastoma
2- Chondroblastoma
3- Giant cell tumor
4- Langerhans cell histiocytosis
5- Aneurysmal bone cyst

DISCUSSION: Chondroblastomas are painful benign tumors that typically occur in


patients between the ages of 10 and 25. They almost always occur in an epiphysis or
an apophysis. Although they are benign, the MRI scan frequently demonstrates
surrounding edema as seen in this patient. The biopsy specimen shows
multinucleated giant cells as well as chondroblasts. The chondroblasts are polygonal
cells with well-defined cytoplasmic borders and indented nuclei. Osteoblastoma is a
bone-forming lesion with interlacing osteoid and fibrovascular stroma similar to
osteoid osteoma. Osteoblastoma and osteoid osteoma are differentiated more by
size and clinical presentation than histology. Giant cell tumor is composed of
multinucleated giant cells with stromal cells sharing similar nuclear morphology.
Langerhans cell histiocytosis is a common, usually self-limiting process with a
variable radiographic appearance. The lesion is composed of plump histiocytes,
eosinophils, and variable inflammatory cells. Aneurysmal bone cysts are not typical
in the epiphysis and are composed of lakes of bloods surrounded by variable
amounts of fibrous stroma containing giant cells, histiocytes, and hemosiderin. The
Preferred Response to Question # 47 is 2.

Question 48 Figure 48 shows the AP and lateral radiographs of a 12-year-old boy


with knee pain after a fall. Previous to the fall he denies any history of pain. What
is the most likely diagnosis?
1- Unicameral bone cyst
2- Chondroblastoma
3- Fibrous dysplasia
4- Nonossifying fibroma
5- Osteosarcoma

DISCUSSION: The patient has a nonossifying


fibroma. Nonossifying fibroma is classically
an eccentric, lytic lesion with a thin sclerotic
34
251
2011 Musculoskeletal Tumors Self-Assessment Exam by Dr.Dhahirortho

border in the metaphysis. Unicameral bone cyst is typically central with variable
surrounding sclerosis. Chondroblastoma is a lytic lesion in the epiphysis. Fibrous
dysplasia is typically a central lytic lesion in the metaphysis or diaphysis with a
ground glass appearance. Osteosarcoma is usually a mixed lytic/blastic lesion with
bone destruction and formation, without sclerotic margins. Prefe Res# 48 is 4.

Question 49 The diagnosis of gout can be made either by the presence of


tophaceous deposits in the skin or bursae of the extremities or by the presence of
which of the following?
1- Elevated urine pH
2- Elevated serum uric acid
3- Calcium pyrophosphate crystals in the synovial fluid
4- Monosodium urate crystals in the synovial fluid
5- Elevated serum phosphate

DISCUSSION: Gout is an inflammatory arthritis caused by the presence of


monosodium urate crystals in the joint. It is characterized acutely by a painful joint
that remits after 1 to 2 weeks and recurs periodically. The diagnosis of gout can be
made by confirming the presence of monosodium urate crystals in the joint fluid
aspirated from the inflamed joint. Patients with gout may also have tophaceous
deposits within the skin or bursae of the extremities. Elevated urine pH, serum uric
acid, and serum phosphate can all be associated with numerous conditions and are
not specific to gout. Calcium pyrophosphate crystals are associated with
chondrocalcinosis (pseudogout). The Preferred Response to Question # 49 is 4.

Question 50 A 29-year-old man reports pain in the right little finger. The pain is
worse at night. Symptoms were completely relieved with naproxen; however, the
patient is no longer able to continue naproxen secondary to gastrointestinal
problems. A radiograph is seen in Figure 50a and a CT scan is seen in Figure 50b.
What is the best treatment option for this patient?
1- Curettage
2- Observation
3- Acetaminophen
4- Wide resection
5- Radiofrequency ablation

35
252
2011 Musculoskeletal Tumors Self-Assessment Exam by Dr.Dhahirortho

DISCUSSION: The radiograph and the CT scan demonstrate a sclerotic lesion with a
central nidus consistent with osteoid osteoma. The patient's pain characteristics
(worse at night and relieved by anti-inflammatory drugs) are also typical of osteoid
osteoma. The best treatment for this patient would be curettage with a power burr
which is associated with less than 10% local recurrence. Observation is not a good
option for this patient because he is experiencing pain. Studies have demonstrated
high levels of cyclooxygenases and prostaglandins in the tumor osteoblasts. This
might explain why many patients receive dramatic relief with anti-inflammatory
medications. Acetaminophen would not be expected to provide adequate pain
relief. Whereas radiofrequency ablation is currently the most common method
employed to treat osteoid osteomas, this procedure is contraindicated for lesions in
the digits because of the risk of thermal necrosis of the overlying skin as well as the
digital neurovascular structures. Wide resection would be overly aggressive for this
benign lesion. The Preferred Response to Question # 50 is 1.

Question 51A 57-year-old man was treated for an upper extremity high-grade soft-
issue sarcoma 9 months ago with resection and postoperative radiation therapy.
At his restaging visit, a new solitary lesion is seen on the CT chest scan shown in
Figure 51. What is the most effective treatment of this new identified lesion?
1- Wedge resection
2- Pneumonectomy
3- Chemotherapy
4- Radiation therapy
5- Palliative care

DISCUSSION: For patients with soft-tissue sarcoma, the lungs are the most common
site of metastatic disease. Although pulmonary metastases most commonly arise
from primary tumors in the extremities, they may arise from almost any primary site
or histology. Resection of metastatic disease is the single most important factor that
determines outcome in these patients. Long-term survival is possible in selected
patients, particularly when recurrent pulmonary disease is resected. Surgical
excision of lung metastases from soft-tissue sarcomas is well accepted and should
be considered as a first line of treatment if preoperative evaluation indicates that
complete resection of the metastases is possible. With an isolated lesion, a wedge
resection or lobectomy would be adequate for controlling disease. Pneumonectomy
could be used for more extensive disease. Further investigation is needed before

36
253
2011 Musculoskeletal Tumors Self-Assessment Exam by Dr.Dhahirortho

chemotherapy can be recommended as additional therapy. Radiation therapy is not


typically used for pulmonary metastasis from soft-tissue sarcomas. Pre Re# 51 is 1.

Question 52 Figures 52a and 52b show the radiographs of a 30-year-old-man who
had Ewing's sarcoma at the age of 10 treated with radiation therapy for local
control. What is the most likely diagnosis?
1- Recurrent Ewing's sarcoma
2- Osteomyelitis
3- Radiation-induced sarcoma
4- Metastatic adenocarcinoma
5- Bone infarct

DISCUSSION: The most likely


diagnosis is radiation-induced
sarcoma, which typically occur years
after the radiation is received. The
radiographs show a lytic lesion in the area of previously treated Ewing's sarcoma.
Occasionally Ewing's sarcoma is treated with radiation therapy, typically in locations
where limb salvage is not feasible. Recurrent Ewing's sarcoma would be unusual 20
years after the primary tumor, bone infarct is not purely radiolucent,
adenocarcinoma is rare in patients younger than age 40, and osteomyelitis would be
an unusual diagnosis in this clinical setting. The Preferred Response # 52 is 3.

Question 53 Based on the radiograph, post-gadolinium enhanced MRI scan, and


histologic study shown in Figures 53a through 53c, what is the most likely
diagnosis?

37
254
2011 Musculoskeletal Tumors Self-Assessment Exam by Dr.Dhahirortho

1- Undifferentiated pleomorphic sarcoma


2- Synovial sarcoma
3- Liposarcoma
4- Fibromatosis
5- Epithelioid sarcoma

DISCUSSION: Fibromatosis more frequently occurs in women, with a peak incidence


between puberty and age 40 years. On MRI, the tumor is isointense to hypointense
with skeletal muscle on both pulse sequences but varies depending on the amount
of collagenous stroma; it enhances markedly with gadolinium administration as seen
in Figure 53b. Histologically, this tumor is composed of fibroblasts and
myofibroblasts in a collagenous matrix with low mitotic activity. Although the tumor
recurs frequently (due to its infiltrative growth pattern) despite wide local surgery, it
does not metastasize. Chemotherapy using low-dose methotrexate and vinblastine
has been reported to be beneficial in treating this tumor. Radiographically, with a
few exceptions, it is very difficult to distinguish one soft-tissue tumor from another.
Histologically, undifferentiated pleomorphic sarcoma contains spindle malignant
cells that have a storiform pattern. Synovial sarcomas contain spindle cells arranged
in short intersecting fascicles; they may also have pseudoglandular areas in the
biphasic form of disease. Epithelioid sarcoma contains spindled and epithelioid cells
arranged in nodules with necrosis and hemorrhage (pseudoangiosarcomatous
appearance). The Preferred Response to Question # 53 is 4.

Question 54The radiographs, bone scan, and MRI scans of a 10-year-old girl are
seen in Figures 54a through 54f. A biopsy specimen is seen in Figure 54g. Which of
the following represents the treatment option with the best prognosis?

1- Wide resection without adjuvant


2- Methotrexate-based chemotherapy with radiation therapy
3- Methotrexate-based chemotherapy with wide resection
4- Vincristine, doxorubicin, cyclophosphamide, and dactinomycin-based
chemotherapy with wide resection, with or without radiation therapy
5- Vincristine, doxorubicin, cyclophosphamide, and dactinomycin-based
chemotherapy

38
255
2011 Musculoskeletal Tumors Self-Assessment Exam by Dr.Dhahirortho

DISCUSSION: The patient has Ewing's sarcoma. This tumor is best treated with
neoadjuvant chemotherapy consisting of a regimen based around vincristine,
doxorubicin, cyclophosphamide, and dactinomycin in combination with wide
resection or amputation. Radiation therapy may have a role for local disease control
with close surgical margins, surgically inaccessible sites, or in the presence of
advanced disease. Osteosarcoma is treated with methotrexate-based chemotherapy
and wide resection or amputation. The Preferred Response to Question # 54 is 4.

Question 55 A 32-year-old woman has had hip pain for 8 months. Initially, the pain
was present with activity but has now progressed to pain also at rest. An AP
radiograph of the pelvis, CT scan, and MRI scan are seen in Figures 55a through
55c. Figures 55d and 55e show the biopsy specimens. What is the most
appropriate treatment?

39
256
2011 Musculoskeletal Tumors Self-Assessment Exam by Dr.Dhahirortho

1- Observation
2- Curetting and bone grafting
3- Chemotherapy followed by wide excision
4- Radiotherapy
5- Radical resection

DISCUSSION: Chondromyxoid fibroma is a rare tumor and is most often found in the
proximal tibia. Other common sites include the collective bones of the foot and
ankle as well as the pelvis. The radiograph demonstrates a radiolucent abnormality
in the right ilium that is partially obscured by bowel gas. The CT scan reveals a
destructive, low attenuation abnormality with a rim of bone on the periphery and
subtle internal matrix. The coronal T2-weighted MRI scan shows a hyperintense
signal abnormality with a lobular growth pattern; the transition between the lesion
and normal bone is distinct. Low power H&E stained tissue shows a biphasic
histologic pattern with hypercellular spindle cells surrounding a relatively
hypocellular area. High power reveals very characteristic stellate cells. Treatment for
this condition is intralesional (curetting and bone grafting). Local recurrence is
reported in 20% to 25% of cases. Chondrosarcoma is in the differential in this case
and would require wide local excision. Chondroblastoma is benign, albeit sometimes
aggressive and does not respond to adjuvant therapies such as chemotherapy or
radiation therapy. The Preferred Response to Question # 55 is 2.

Question 56 A 72-year-old woman is evaluated for sacrococcygeal pain sustained


after a twisting injury. Radiographic and MRI evaluation confirms the presence of
a nondisplaced fracture at the sacrococcygeal junction. Over a 3-week period, the
pain has gotten significantly better. No additional lesions or injuries are noted.
Laboratory studies show a serum calcium level of 8.8 mg/dL (normal 8.6-10.3
mg/dL) and a 25-OH Vitamin D level of 14 ng/mL (normal 30-80 ng/mL). What is
the most appropriate treatment for this patient?
1- Expectant observation
2- Calcium supplementation
3- High dose vitamin D supplementation
4- Bisphosphonate therapy
5- Surgical fixation of the sacrococcygeal fracture

DISCUSSION: Chronic Vitamin D deficiency leads to problems with bone health and
has been shown to increase the risk of falls in the elderly. Appropriate
supplementation of Vitamin D has been shown to decrease this risk. Conversion in
40
257
2011 Musculoskeletal Tumors Self-Assessment Exam by Dr.Dhahirortho

the skin decreases with age and may be nearly nonexistent in darkly pigmented
individuals. Vitamin D3 is the preferred form for supplementation, but D2 is the
form most available by prescription in the US. Hypervitaminosis D is rare and very
high doses can be tolerated without significant concern for toxicity. Because the
patient has sustained one insufficiency fracture, she is at risk for insufficiency
fractures in other skeletal locations, rendering expectant observation insufficient.
Her serum calcium is normal, and with a low Vitamin D level, calcium utilization in
her system would be inadequate. Bisphosphonate therapy in addition to calcium
and vitamin D supplementation may provide a good long-term solution, but should
not be instituted until the bone mineral imbalance has been adequately corrected.
Surgical fixation of this fracture is not indicated, particularly in lieu of improving
symptoms.
The Preferred Response to Question # 56 is 3.

Question 57 Figures 57a through 57c show the radiograph, MRI scan, and
photomicrograph of a 13-year-old boy who reports increasing left groin pain. What
is the most appropriate method of treatment of this lesion?

1- Surgery alone
2- Radiation therapy alone
3- Chemotherapy alone
4- Chemotherapy and surgery
5- Surgery and radiation therapy

DISCUSSION: The radiograph demonstrates an expansile lesion of the left superior


pubic ramus. The MRI scan demonstrates multiple fluid-fluid levels consistent with
an aneurysmal bone cyst. The photomicrograph demonstrates cavernous
hemorrhagic tissue with a benign-appearing spindle cell stroma and giant cells, thus
confirming the diagnosis of aneurysmal bone cyst. Surgical management is the
recommended treatment for this lesion. Most aneurysmal bone cysts are amenable
to curettage and bone grafting. Lesions in expendable bones can be treated with
resection. Embolization may decrease intraoperative blood loss when treating large
lesions of the pelvis. Radiation therapy has been reported to be effective but is
seldom used secondary to the risk of malignant transformation as well as the risk of
damage to reproductive organs, physes, and the spinal cord. Chemotherapy has no
role in the treatment of this lesion.
The Preferred Response to Question # 57 is 1.
41
258
2011 Musculoskeletal Tumors Self-Assessment Exam by Dr.Dhahirortho

Question 58 Figures 58a and 58b show the radiographs of an otherwise healthy 64-
year-old man who has had right groin pain for the past 3 months. What is the next
most appropriate step in management?
1- Biopsy
2- Observation
3- Radiation therapy
4- Prophylactic fixation of the femur
5- Bone scan, CT scan of the
chest/abdomen/pelvis, laboratory studies

DISCUSSION: The radiographs show a


radiolucent lesion in the proximal femur. In a patient older than age 40 years, a new
painful bone lesion most likely represents metastatic carcinoma or multiple
myeloma even if the patient does not have a known history of cancer. The diagnosis
must be firmly established and the patient should be staged prior to initiating
treatment. Imaging studies should be completed prior to proceeding with a biopsy.
This patient should undergo a bone scan to look for other lesions. A CT scan of the
chest, abdomen, and pelvis should be performed to look for a primary tumor as well
as other sites of metastases. (Lung or kidney would be the most common primary
sites for a patient who presents with metastases of unknown origin.) Serum and
urine protein electrophoresis should be obtained to look for multiple myeloma. The
Preferred Response # 58 is 5.

Question 59 A 23-year-old woman who noted 1 day of thigh pain after jogging now
reports persistent thigh swelling and can feel a mass. The radiograph, CT scan, and
MRI scans are shown in Figures 59a through 59d. What is the most likely
diagnosis?

42
259
2011 Musculoskeletal Tumors Self-Assessment Exam by Dr.Dhahirortho

1- Soft-tissue sarcoma
2- Pseudoaneurysm
3- Abscess
4- Osteosarcoma
5- Myositis ossificans

DISCUSSION: Myositis ossificans (MO) is a reparative lesion that is distinguished by


the presence of metaplastic bone formation. The important entity that MO must be
distinguished from is extraskeletal osteosarcoma. Extraskeletal osteosarcoma
usually occurs in older patients and lacks the zonation phenomenon of MO. Myositis
ossificans develops a well-defined ossified rim, maturing peripherally more than
centrally, where osteosarcomas do the opposite. Histologically the two may be
confusing; therefore, clinical and radiographic evaluation is a critical factor in the
diagnosis. Soft-tissue sarcoma, pseudoaneurysm, and abcess do not have zonal
ossification patterns. The Preferred Response to Question # 59 is 5.

Question 60 A 33-year-old man has a painless mass in the anterior thigh. Selected
sequences of MRI scans are shown in Figure 60a (T1) and Figure 60b (T2 fat
saturated). Biopsy photomicrographs are shown in Figure 60c (low-power) and 60d
(high-power). Cytogenetics of the specimen shows a 12:16 translocation. What is
the most likely diagnosis?
1- Well-differentiated liposarcoma
2- Myxoid liposarcoma
3- Round cell liposarcoma
4- Fibrolipomatous hamartoma
5- Intramuscular lipoma

DISCUSSION: The T1-weighted MRI scan shows a heterogeneous fatty tumor within
the thigh musculature. The amount of heterogeneity and enhancement are more
suggestive of malignancy, but are not diagnostic. The histology shows lipoblasts and
primitive mesenchymal cells within a stroma myxomatous tissue with a delicate
plexiform capillary network. The cytogenetics demonstrates the characteristic
translocation for a myxoid liposarcoma. Treatment is usually consists of wide local
excision with or without radiation therapy. Chemotherapy may be considered if
there is a significant (25% or greater) round cell component.Preferred Resp# 60 is 2.

43
260
2011 Musculoskeletal Tumors Self-Assessment Exam by Dr.Dhahirortho

Question 61 A 59-year-old man who works as a laborer has had left hip pain for
the past 12 months. He reports some worsening with activity and occasional pain
that wakes him from sleep. He rates his baseline pain as 3 out of 10. He denies any
fevers, malaise, or other systemic symptoms. A radiograph, CT scan, and biopsy
photomicrographs are shown in Figures 61a through 61e. Appropriate treatment
of this lesion would entail which of the following?

1- Chemotherapy and wide resection


2- Radiation therapy alone
3- Extended curettage and radiation therapy
4- Wide surgical resection alone
5- Observation with repeat scans in 3 months

DISCUSSION: This is a classic presentation of chondrosarcoma of the pelvis. Wide


surgical resection is the treatment of choice. Though chemotherapy may be offered
for patients with dedifferentiated chondrosarcoma, the histology of this lesion is
consistent with an intermediate to high-grade myxoid chondrosarcoma.
Chondrosarcomas are radioresistant, and should be aggressively treated with
surgery. Extended intralesional resection of low-grade chondrosarcoma is
acceptable in the appendicular skeleton; it is inappropriate in patients with
intermediate or high-grade lesions and lesions that occur in the pelvis and other
axial sites. The Preferred Response to Question # 61 is 4.

Question 62 Figures 62a through 62c show the radiographs of a 40-year-old man
who works as a heavy laborer and has hand pain after sustaining a minor injury.
What is the most appropriate treatment for this patient?
1- Biopsy
2- Ray resection
3- Protective splinting followed by observation after fracture healing
4- Protective splinting followed by curettage and bone grafting after fracture
healing
5- Acute open reduction and internal fixation combined with curettage and bone
grafting

44
261
2011 Musculoskeletal Tumors Self-Assessment Exam by Dr.Dhahirortho

DISCUSSION: The radiographs are consistent with an enchondroma with pathologic


fracture. Allowing the fracture to heal and then proceeding with curettage and bone
grafting allows a more limited procedure with satisfactory results compared with
acute surgical treatment. Hand enchondromas that have fractured should generally
be treated surgically because repeated fractures will typically occur, particularly in a
person who works as a laborer. MRI will not add to the diagnostic work-up and the
presence of a fracture may confuse the interpretation. Biopsy is not necessary
unless aggressive bony changes are seen. Ray resection is generally reserved for
bone and soft-tissue sarcomas or other primary malignancies involving the digit. The
Preferred Response to Question # 62 is 4.

Question 63 A 47-year-old woman with a history of breast cancer and a recent


diagnosis of lung carcinoma with multiple painful biopsy-proven skeletal
metastasis now reports progressive weight-bearing pain in her left thigh. Current
treatment consists of chemotherapy for the lung mass and radiation therapy. An
AP radiograph of the left femur is shown in Figure 63. What is the best option for
treatment of her thigh pain?
1- Bisphosphonate therapy
2- Resection and megaprosthetic reconstruction
3- Prophylactic rodding
4- Radiation therapy
5- Chemotherapy

DISCUSSION: Predicting which patients will sustain a


fracture with metastatic bone disease is a difficult
clinical decision. Mirel's rating system is a useful clinical
tool but must be used cautiously. Lower extremity
peritrochanteric, large, radiolucent, and painful lesions

45
262
2011 Musculoskeletal Tumors Self-Assessment Exam by Dr.Dhahirortho

are at highest risk for fracture. The ultimate goal is palliative in nature and designed
to limit pain, minimize time in the hospital, and improve the quality of life in these
patients with limited survival time. If a high risk lesion is treated with radiation
therapy first, the bone will become weaker before starting to regain structural
integrity and the fracture risk increases. Timing of prophylactic fixation, radiation
therapy, and chemotherapy requires a multidisciplinary team of doctors working
collaboratively. Bisphosphonates should be included in the medical treatment for
most patients with metastatic bone disease to lower the risk of further skeletal
complications. They are not indicated for acute treatment of impending fractures.
Resection of a metastatic lesion is reserved for patients in which internal fixation
devices will not have adequate bone stock present to allow stabilization and
immediate weight bearing or in patients with selected isolated metastases, such as
those caused by renal carcinoma. Radiation therapy and chemotherapy have already
failed to control progressive bone destruction and pain in this patient.

The Preferred Response to Question # 63 is 3.

Question 64 A 45-year-old man has an enlarging, painless soft-tissue mass in his


arm. An MRI scan shows a 15 x 8 x 6 cm enhancing, heterogenous deep mass.
Biopsy shows a sarcoma. What is the next most appropriate imaging study to
obtain?

1- PET scan
2- Bone scan
3- Chest CT
4- Abdomen CT
5- MRI of the chest

DISCUSSION: Staging takes into consideration the histologic grade, size, depth, and
presence of metastasis of a tumor. Locoregional lymph node spread is uncommon
for most sarcomas and is evaluated by palpation; suspicious areas require further
imaging to assess. The chest is the most common area for soft-tissue sarcomas to
metastasize and should be considered for imaging prior to biopsy to assist with the
staging of the patient. In some specific subtypes (myxoid liposarcoma), bone scans
and abdominal imaging are important to obtain to exclude metastases. The role for
PET scans in patients with soft-tissue sarcomas is still being defined.

The Preferred Response to Question # 64 is 3.


46
263
2011 Musculoskeletal Tumors Self-Assessment Exam by Dr.Dhahirortho

Question 65 A 20-year-old collegiate field hockey player has had diminished knee
flexion for the past 3 years. Lately she has noted right knee pain that requires her
to stop playing. Radiographs are shown in Figures 65a and 65b. What is the next
best step in management?

1- Radiation therapy
2- Surgical biopsy
3- Chemotherapy
4- Knee arthroscopy
5- Bisphosphonates

DISCUSSION: The lesion has the characteristic features of a parosteal osteosarcoma,


including its surface location on the posterior distal femur, decreased range of
motion, and pain. Biopsy is necessary to confirm a diagnosis before recommending
treatment. Chemotherapy and radical resection may be indicated in dedifferentiated
disease, but should be considered only after a biopsy and appropriate staging have
been completed. There is no role for knee arthroscopy or bisphosphonates.
The Preferred Response to Question # 65 is 2.

66.Which of the following genes is involved in the etiology of aneurysmal bone


cyst?
1- ASPL 2- ATF1 3- EWS 4- PAX3 5- USP6

DISCUSSION: Aneurysmal bone cyst was recently shown definitively to be a


neoplasm driven by upregulation of the ubiquitin-specific protease USP6 (Tre2) gene
on 17p13 when combined by translocation with a promoter pairing. The most
commonly described translocation is t(16;17)(q22;p13) which results in juxtaposition
of the promoter region CDH11 on 16q22. In the past, all aneurysmal bone cysts were
thought to be reactive in nature. Whereas primary aneurysmal bone cysts are now
known to be neoplasms, secondary aneurysmal bone cysts are not because no
translocation has been identified in them. ASPL is involved in alveolar soft part
sarcoma (tX;17)(TFE3-ASPL). ATF1 is involved in clear cell chondrosarcoma. EWS is a
gene on chromosome 22 involved in several relevant musculoskeletal oncology
conditions, including Ewing's sarcoma (t11;22)(EWS-FLI1), desmoplastic small round
cell tumor (t11;22)(EWS-WT1), extraskeletal myxoid chondrosarcoma (t9;22)(EWS-
CHN), and clear cell chondrosarcoma (t12;22)(EWS-ATF1). PAX3 is involved in
alveolar rhabdomyosarcoma (t2;13)(PAX3-FKHR). The Preferred Respo# 66 is 5.
47
264
2011 Musculoskeletal Tumors Self-Assessment Exam by Dr.Dhahirortho

Question 67 A 39-year-old man has multiple bone lesions in his right leg, bowing
deformity, and limb-length inequality. Radiographs are shown in Figures 67a
through 67c. A biopsy specimen is shown in Figure 67d. When counseling the
patient regarding his diagnosis, how would you explain the inheritance pattern for
his disease?

1- Autosomal dominant
2- Autosomal dominant with variable penetrance
3- Autosomal recessive
4- Sex-linked recessive
5- No known inheritance pattern

DISCUSSION: The patient's radiographs show multiple lytic bone lesions with some
femoral deformity. The radiographic differential would include fibrous dysplasia,
enchondromatosis (Ollier's disease), eosinophilic granulomatosis, metastatic
disease, multiple myeloma, hyperparathyroidism (brown tumors), and infection. The
pathology demonstrates relatively acellular cartilaginous tissue, supporting the
diagnosis of Ollier's disease. Commonly, patients with enchondromatosis have
problems with shortened and/or bowing of their affected limbs. Additionally, unlike
solitary enchondromas which have a very low risk of malignant transformation (<
1%), patients with enchondromatosis have a significantly higher risk of malignant
transformation, up to 25%. Despite the disease and lesions being present from birth,
there is no described inheritance pattern associated with the disease.Pre Re# 67 is 5.

Question 68 A 10-year-old boy has a 5-month history of pain in the left thigh and
knee that has increased in severity such that he is currently unable to walk
secondary to pain. He has a large fusiform swelling about the distal femur. A
radiograph of both knees is shown in Figure 68a, and MRI scans are shown in
Figures 68b through 68d. After complete staging, a biopsy is performed and a
specimen is shown in Figure 68e. What is the most likely diagnosis?
48
265
2011 Musculoskeletal Tumors Self-Assessment Exam by Dr.Dhahirortho

1- Osteomyelitis
2- Stress fracture
3- Osteoblastoma
4- Osteosarcoma
5- Ewing's sarcoma

DISCUSSION: The patient's clinical presentation is one of progressive pain. His


radiographs show a destructive, bone-producing lesion that has extended outside
the bone into the soft tissues. The pathology shows the production of osteoid by
malignant spindle stromal cells. This is consistent with osteosarcoma. There is no
inflammatory component of the histology to suggest osteomyelitis. Whereas
osteoblastoma can be confused with osteosarcoma, the histology of malignant
stromal cells as well as the radiographic findings are classic for osteosarcoma.
Ewing's sarcoma can be associated with a significant soft-tissue mass, as seen in the
MRI scan; however, histology would demonstrate a small blue round-cell tumor. The
Preferred Response to Question # 68 is 4.

Question 69 A 65-year-old woman has a bone mineral density t-score of -2.0. She
has no identifiable secondary cause of bone loss. In addition to calcium and
vitamin D supplementation, this patient should be considered for
1- calcitonin.
2- hormone replacement therapy.
3- teriparatide.
4- bisphosphonate therapy.
5- observation.

DISCUSSION: A patient with a bone mineral density t-score of between -1.0 and -2.5
should be considered for osteoporosis prevention therapy. This would consist of
calcium (1,200 to 1,500 mg daily), vitamin D (800 to 1000 IU daily), and a
bisphosphonate. Calcitonin and teriparatide are used to treat established
osteoporosis (t-score of -2.5 or lower). Observation is not recommended with a t-
49
266
2011 Musculoskeletal Tumors Self-Assessment Exam by Dr.Dhahirortho

score of less than -1.0. Hormone replacement therapy is associated with an


increased risk of cardiovascular disease and breast cancer. Pref Re # 69 is 4.

Question 70 Figures 70a and 70b show the radiograph and MRI scan of a 66-year-
old man who has fatigue, weight loss, and muscle weakness. Examination reveals
marked pain and discomfort in the left mid leg. Biopsy specimens are shown in
Figures 70c and 70d. What is the most likely diagnosis?

1- Mastocytosis
2- Multiple myeloma
3- Hyperparathyroidism
4- Metastatic carcinoma
5- Multicentric giant cell tumor

DISCUSSION: The signs and symptoms of hyperparathyroidism are similar to those in


patients with diffuse skeletal metastases. Serum markers are very helpful in making
the diagnosis. In this patient, the radiograph shows multiple lesions in the tibia and
proximal fibula that have a variable appearance. For example the mid-tibial lesion is
radiolucent and slightly expansile whereas the more proximal tibial lesions are
radiodense. The proximal fibula lesion is mixed (radiolucent/radiodense). These
findings would be very uncommon in patients with myeloma, metastatic disease, or
multicentric giant cell tumor. The histopathology shows a bland fibrous stroma with
multiple multinucleated giant cells. On higher power, the stromal cells are spindled
and the giant cells are relatively small in contrast to giant cell tumor where the giant
cells are larger and the stromal cells are more rounded with nuclei that closely
resemble those in the giant cells. There is blood extravasation (stromal hemorrhage)
and hemosiderin deposition. The constellation of findings is most consistent with
brown tumors due to hyperparathyroidism (secondary to a parathyroid adenoma in
this patient). The Preferred Response to Question # 70 is 3.
50
267
2011 Musculoskeletal Tumors Self-Assessment Exam by Dr.Dhahirortho

Question 71 A 76-year-old woman has a painful, snapping mass at the inferior


angle of the scapula. She has no history of trauma to the area. MRI scans are
shown in Figures 71a through 71c. A biopsy specimen is shown in Figure 71d. What
is the most appropriate treatment at this time?

1- Observation
2- Excision
3- Radiation therapy alone
4- Chemotherapy followed by wide excision
5- Local steroid injection

DISCUSSION: Elastofibroma is a benign, likely inflammatory mass that typically


occurs deep to the inferior angle of the scapula. It may be bilateral and may be
symptomatic with either pain or mechanical symptoms related to its location. The
recommended treatment for symptomatic lesions is excision. Small or asymptomatic
lesions may also be observed. Biopsy to confirm the diagnosis and to exclude
sarcoma seems prudent. Neither radiation therapy or chemotherapy are indicated
for this benign tumor. Local steroid injection plays no role in the treatment of
benign elastofibroma. The Preferred Response to Question # 71 is 2.

Question 72 Figures 72a and 72b show the radiograph and biopsy specimen of a
92-year-old woman who reports increasing right shoulder pain. What is the most
likely diagnosis?

1- Enchondroma
2- Juxtacortical chondroma
3- Conventional chondrosarcoma
4- Dedifferentiated chondrosarcoma
5- Mesenchymal chondrosarcoma

51
268
2011 Musculoskeletal Tumors Self-Assessment Exam by Dr.Dhahirortho

DISCUSSION: The radiograph shows a lesion of the proximal humerus with stippled
calcification suggestive of a cartilaginous neoplasm. The lesion has eroded through
the cortex, and therefore, most likely represents a chondrosarcoma. The biopsy
specimen reveals a high-grade spindle cell sarcoma adjacent to low-grade cartilage
and is thus diagnostic of dedifferentiated chondrosarcoma. An enchondroma is a
benign cartilaginous tumor contained entirely within the medullary cavity. A
juxtacortical chondroma is a benign cartilage tumor on the surface of a bone.
Conventional chondrosarcoma could present a radiographic appearance similar to
this case; however, it would not contain a spindle cell component. Mesenchymal
chondrosarcoma typically has small round blue cells and vascular proliferation with
a hemangiopericytomatous pattern.
The Preferred Response to Question # 72 is 4.

Question 73 An otherwise healthy 30-year-old woman with no history of


malignancy reports a 1-year history of right hip and low back pain. A radiograph,
CT scan, and MRI scan are shown in Figures 73a through 73c. A biopsy specimen is
shown in Figure 73d. What is the most likely diagnosis?

1- Aneurysmal bone cyst


2- Hemangioma of bone
3- Sarcoid
4- Paget's disease of bone
5- Chondrosarcoma

DISCUSSION: The patient has a lytic lesion in her right ilium with coarsened
trabeculae. The radiographic differential for such a lesion would include aneurysmal
bone cyst, hemangioma of bone, sarcoid, Paget's disease of bone, and fibrous
dysplasia. The photomicrograph of the biopsy demonstrates small vascular channels
lined with a single layer of endothelial cells, supporting the diagnosis of
hemangioma of bone. There are no giant cells typically seen in an aneurysmal bone
cyst and the early phases of Paget's disease. There are no granulomas to suggest
sarcoid, and the histopathologic features are not consisent with cartilage. Most
patients with hemangiomas require no treatment. Lesions causing symptoms are
best treated with intralesional excision.

The Preferred Response # 73 is 2.

52
269
2011 Musculoskeletal Tumors Self-Assessment Exam by Dr.Dhahirortho

Question 74 A 56-year-old woman has a 5-month history of a rapidly growing mass


in the posteromedial aspect of the right leg. A clinical photograph, MRI scan, and
biopsy specimen are shown in Figures 74a through 74c. What is the most
appropriate treatment for this patient?

1- Observation
2- Wide resection alone
3- Radiation therapy alone
4- Wide resection and radiation therapy
5- Debridement and antibiotics

DISCUSSION: The clinical photograph shows an aggressive tumor growing through


the skin (fungating). The axial T1-weighted MRI scan with contrast shows a large
mass in the posterior and proximal leg with heterogeneous enhancement and a
large nonenhancing center that suggests tumor necrosis, two poor prognostic
factors in disease outcome. Finally, the histopathology suggests an undifferentiated
pleomorphic spindle cell neoplasm. This is a high-grade sarcoma that requires both
radiation therapy and wide excision to local disease control. The histopathology is
characteristic of a high-grade sarcoma, not a local infection. The role of
chemotherapy for most adult soft-tissue sarcomas remains controversial. The
Preferred Response to Question # 74 is 4.

Question 75 An otherwise healthy 12-year-old boy has ankle pain after being
kicked while playing soccer. Radiographs are shown in Figures 75a and 75b.
Examination reveals tenderness to palpation, but not with weight bearing. He had
no pain preceding the incident. What is the next most appropriate step in
management?
1- Observation
2- Curettage and grafting
3- Wide surgical resection
4- Whole body bone scan
5- Chemotherapy
53
270
2011 Musculoskeletal Tumors Self-Assessment Exam by Dr.Dhahirortho

DISCUSSION: The lesion shown is a nonossifying fibroma. With no pain preceding the
traumatic episode and painless weight bearing, the lesion does not appear at risk for
fracture. Simple observation with repeat radiographs at a time interval (3 to 6
months) to document stability is sufficient. Surgical intervention is unnecessary
because the risk of fracture is low and the natural history is one of spontaneous
regression during adolescence. The lesion is benign; therefore, chemotherapy is not
indicated. While a bone scan may provide some useful information, it is unnecessary
for the diagnosis and adds little to management decisions. Pre Resp # 75 is 1.

Question 76 A 31-year-old patient has had a left medial elbow mass for 1 month.
The mass has been increasing in size and has now become very painful and
erythematous. MRI scans are shown in Figures 76a and 76b. Laboratory studies
show an erythrocyte sedimentation rate of 49 mm/h (normal 0 to 20 mm/h) and
C-reactive protein level of 23 mg/L (normal 0 to 0.3 mg/L). Histology showed
lymphoid tissue and multiple necrotizing granulomas. What organism is
responsible for this clinical picture?
1- Borrelia burgdorferi
2- Trichophyton tonsurans
3- Bartonella henselae
4- Mycobacterium avium
5- Corynebacterium minutissimum

DISCUSSION: Cat scratch disease


(CSD) is an important diagnosis for
the orthopaedic surgeon to consider in the differential diagnosis of soft-tissue
masses adjacent to epitrochlear or cervical lymph nodes. It is a soft-tissue tumor
simulator and a high index of suspicion is necessary in all patients with upper
extremity or head and neck adenopathy and a history of cat exposure. Although
generally not required for diagnosis, cross-sectional imaging will reveal a mass with
surrounding edema in an area of lymphatic drainage. A peripheral blood sample can
be tested for Bartonella henselae - the offending organism with this diagnosis.
Classically the histology of these lesions when biopsied will show multiple
necrotizing granulomas. Mycobacterium avium is the only other organism that
would demonstrate a granulomatous reaction and the location is classic for CSD.
Borrelia burgdorferi is associated with Lyme disease. Mycobacterium avium may be
a source of immunocompromised infections in HIV patients. Trichophyton tonsurans
and corynebacterium minutissimum are not associated with orthopaedic diseases.
The Preferred Response to Question # 76 is 3.
54
271
2011 Musculoskeletal Tumors Self-Assessment Exam by Dr.Dhahirortho

Question 77 A 45-year-old woman has a painful mass in the dorsum of the right
wrist. It is firm and nontender to palpation. She states it has slowly gotten bigger
over the past 3 years. You suspect a dorsal wrist ganglion. What is the most
definitive way to confirm this diagnosis?
1- Observe it for 1 year to see if it changes dramatically in size.
2- Obtain a gadolinium enhanced MRI scan.
3- Obtain radiographs, looking for scapholunate joint degenerative changes.
4- Perform a needle aspiration and send the aspirate for cytologic examination.
5- Apply direct firm manual pressure over the mass to see if it can be ruptured.

DISCUSSION: Dorsal wrist ganglions are synovial cysts that arise most frequently
from the scapholunate joint. They often extend between the extensor digitorum
communis and extensor pollicis longus tendons at the wrist. Aspiration of the cyst is
both oncologically safe if done appropriately and also the easiest way to definitively
confirm the diagnosis. Clear, yellow viscous fluid/gel is most often aspirated.
Cytologic evaluation is mandatory to exclude myxoid neoplasms. Because the lesion
has been present for 3 years, further observation is not warranted. The classic
presentation, physical examination findings, and location make MRI and radiographs
unnecessary. Manual rupture of the mass is not recommended. Pref Res # 77 is 4.

Question 78 A 68-year-old woman has had progressive pain in the right thigh for
the past several months. She has a history of hypertension, treated with
hydrochlorothiazide and osteoporosis treated with alendronate for 10 years. At
this point, she is virtually wheelchair bound. Radiographs are shown in Figures 78a
and 78b. Additional studies show no signs of systemic disease. What is the most
likely etiology of her condition?

1- Prolonged use of bisphosphonates


2- Use of calcium-wasting diuretics
3- Occult metastatic cancer
4- Vitamin D-resistant rickets
5- Disuse osteopenia

DISCUSSION: The patient has been on


alendronate for 10 years and has
evidence of a proximal diaphyseal
fatigue fracture. These have been
associated with long-term use of
55
272
2011 Musculoskeletal Tumors Self-Assessment Exam by Dr.Dhahirortho

bisphosphonates. Staging studies have failed to show systemic disease, and while
metastasis with an unidentifiable primary does occur, it would be unlikely to present
with this radiographic appearance, now recognized to be classic for stress fractures
associated with chronic bisphosphonate usage. Hydrochlorothiazide does not cause
calcium wasting. Vitamin D-resistant rickets would be a long-standing event and
would present much earlier in life, often with pronounced deformities. Whereas the
patient's progression to intolerance of weight bearing likely has led to some degree
of disuse osteopenia, the underlying problem is the long-term bisphosphonate
exposure. The Preferred Response to Question # 78 is 1.

Question 79 A 32-year-old man reports pain, and examination reveals swelling and
tenderness about the knee and distal femur. A radiograph and an MRI scan are
shown in Figures 79a and 79b. A bone scan is shown in Figure 79c, and a biopsy
specimen is shown in Figure 79d. Cytogenetic analysis of the biopsy specimen
failed to show an 11:22 chromosomal translocation. What is the most likely
diagnosis?

1- Ewing's sarcoma
2- Bacterial osteomyelitis
3- Tuberculous osteomyelitis
4- Lymphoma of bone
5- Osteosarcoma

DISCUSSION: The biopsy specimen reveals sheets of blue cells and therefore falls
within the differential of blue cell tumors. The majority of Ewing's sarcomas will
have the 11:22 translocation present. The pathology does not reveal the mixed cell
inflammatory process seen in osteomyelitis nor does it show the caseating
granulomas of tuberculosis. There are no findings of any osteoid being produced by
malignant-appearing spindle cells typical of osteosarcoma. Malignant lymphomas of
bone must be considered within the differential for sclerotic lesions of bone with
soft-tissue masses as in this patient. The Preferred Response to Question # 79 is 4.
56
273
2011 Musculoskeletal Tumors Self-Assessment Exam by Dr.Dhahirortho

Question 80 An 18-year-old woman has pain and swelling in her lateral hindfoot.
On examination, she has significant pain with resisted eversion of the foot. MRI
scans are seen in Figures 80a (T1) and Figure 80b (non-fat T2), and a biopsy
specimen is seen in Figure 80c. What is the most appropriate treatment for this
patient?

1- Marginal excision
2- Transtibial amputation
3- Injection of a radioisotope
4- Chemotherapy and radiation therapy
5- Wide local excision and radiation therapy

DISCUSSION: The lesion has a stalk that originates in the subtalar joint, fills the sinus
tarsi, and effaces the peroneal tendons. Soft-tissue masses that are periarticular
should arouse suspicion for synovial sarcoma. Unlike synovial sarcoma however, this
lesion was hypointense on both T1 and T2 MRI pulse-weighted sequences because
of the large amounts of hemosiderin deposition, characteristic of pigmented
villonodular synovitis. Furthermore, intra-articular synovial sarcomas are very rare.
Radioisotopes are not very effective for pigmented villonodular synovitis in the foot
and ankle because contiguous involvement of multiple joints is not uncommon.
Furthermore, skin necrosis can occur with extravasation of radioisotope into the soft
tissue. For this reason, complete (marginal) excision is recommended. For recurrent
tumors, combined surgery and external beam irradiation has been advocated by
some investigators. This is a benign tumor; therefore, aggressive surgical procedures
(amputation and wide excision) are not appropriate. Radiation therapy may be a
consideration in patients with recurrent and destructive disease.
The Preferred Response to Question # 80 is 1.

57
274
2011 Musculoskeletal Tumors Self-Assessment Exam by Dr.Dhahirortho

Question 81 A 26-year-old man has a 1-year history of worsening heel pain. There
is no history of penetrating injuries to the heel. A radiograph, bone scan, and MRI
scan are shown in Figures 81a through 81c, and biopsy specimens are shown in
Figures 81d and 81e. Management should consist of which of the following?

1- Curettage with or without local adjuvants and bone grafting


2- Injection of corticosteroids
3- Irrigation, debridement, and antibiotic treatment
4- Wide resection
5- Chemotherapy and wide resection

DISCUSSION: With an epiphyseal bone lesion, the radiographic differential would


consist of chondroblastoma, Brodie's abscess, or giant cell tumor of bone. Because
of the location in the calcaneus, consideration should also be given for an epidermal
inclusion cyst if there is a history of penetrating trauma. The biopsy specimens show
a very cellular and vascular lesion with mononuclear chondroblasts with grooved
nuclei ("coffee-bean nuclei"). There are some osteoblast-like giant cells. The
surrounding chondroid matrix stains pink and has scant areas of fine mineralization
outlining the stromal cells ("chicken-wire"). This is characteristic of a
chondroblastoma. The preferred treatment for a chondroblastoma in a
nonexpendable bone is intralesional curettage with or without local adjuvants
(phenol, liquid nitrogen, argon beam coagulation) and bone grafting (or cement).
Some very aggressive chondroblastomas with soft-tissue extension, particularly
those in the pelvis, may require en-bloc resection. Preferred Response # 81 is 1.

Question 82 An 11-year-old boy has a 6-month history of groin pain and a limp. A
radiograph is shown in Figure 82a and a biopsy specimen is shown in Figure 82b.
What is the etiology of the lesion?
1- Viral infection
2- G(s) alpha mutation
3- t(11;22)
4- t(X;18)
5- Posttraumatic
58
275
2011 Musculoskeletal Tumors Self-Assessment Exam by Dr.Dhahirortho

DISCUSSION: Fibrous dysplasia is a common benign skeletal lesion that may involve
one bone (monostotic) or multiple bones (polyostotic) and occurs throughout the
skeleton with a predilection for the long bones, ribs, and craniofacial bones. The
etiology of fibrous dysplasia has been linked to an activating mutation in the gene
that encodes the alpha subunit of stimulatory G protein (G(s)alpha) located at
20q13.2-13.3. The etiology for Paget's disease of bone is still unknown but growing
evidence shows a possible link to a viral infection. t(11;22) is most commonly seen
with Ewing's sarcoma and t(X;18) with synovial sarcoma. Preferred Respo# 82 is 2.

Question 83 What is the most common complication of total knee arthroplasty in


patients with Paget disease?
1- Pain
2- Joint arthrosis
3- Pathologic fracture
4- Malalignment
5- Neurologic compromise

DISCUSSION: Paget disease affects 3% to 4% of the population in the US older than


age 40 years. Urinary hydroxyproline and alkaline phosphatase are elevated in the
lytic phase of disease. Insertion mutations in the TNFRSF11A gene encoding RANK
are identified. The most common complication of total knee arthroplasty in patients
with Paget disease is malalignment. The most frequent complication of hip surgery is
hemorrhage. Joint arthrosis, pathologic fracture, and rarely neurologic compromise
are sequelae of the disease process itself. The Preferred Response # 83 is 4.

Question 84 A 56-year-old woman has an 8 x 6 x 5 cm intramuscular anterior thigh


soft-tissue mass. Biopsy shows a grade 3 undifferentiated pleomorphic sarcoma. A
CT of the chest reveals no evidence of other lesions. According to the American
Joint Commission on Cancer (AJCC) staging system, what is the stage of this
tumor?
1- II-T1a 2- II-T1b 3- II-T2a 4- III 5- IV

DISCUSSION: The AJCC staging system for soft-tissue sarcomas is based on tumor
grade, size, depth, and the presence of metastases. Stage I tumors are low grade.
Stage II tumors are high grade. Grade is considered high grade for G3 or G4 tumors
on a four-tier grading system and for G2 or G3 on a three-tier grading system. Size is
designated by T1 for a size of less than or equal to 5.0 cm or T2 for a size of greater
than 5.0 cm in maximal dimension. An 'a' or 'b' designation immediately follows the
59
276
2011 Musculoskeletal Tumors Self-Assessment Exam by Dr.Dhahirortho

size designation to distinguish between superficial (a) and deep (b) tumors. The
"deep" designation applies to tumors that involve or are deep to the fascia. Stage III
tumors are high grade, deep, and large (eg, T2b). Stage IV tumors include either N1
(nodal) or M1 (distant) metastases regardless of grade. Hence, stage II-T1a would
refer to a high grade small superficial sarcoma. Stage II-T1b is a high grade small
deep sarcoma. Stage II-T2a is a high grade large superficial tumor. The tumor
presented in this case is high grade (G3 or G4), large (>5.0 cm), intramuscular (deep
to the fascia), and without metastases. Hence, it is a stage III tumor.

The Preferred Response to Question # 84 is 4.

Question 85 Based on the radiographic findings and biopsy specimen shown in


Figures 85a and 85b, what is the most likely diagnosis?
1- Ewing's sarcoma
2- Osteofibrous dysplasia
3- Chondromyxoid fibroma
4- Osteoid osteoma
5- Adamantinoma

DISCUSSION: Adamantinoma
is a low-grade malignant tumor with epithelial differentiation (cytokeratin positive).
It can occur in other bones but is most common in the tibia. Cytokeratin is positive in
most cases. Tumor metastases occur in up to 20% of patients. There is an
association between adamantinoma and osteofibrous dysplasia. Ewing's sarcoma is
a non-matrix producing medullary-based tumor associated with an aggressive
periosteal reaction. Histologically it is a small round blue cell tumor. Osteofibrous
dysplasia (OFD) is radiographically very similar to adamantinoma because both are
cortically based and most common in the tibia. Histologically OFD is composed of
bone trabeculae arranged as "chinese letters" with prominent osteoblastic rimming.
There are no nests of epithelioid cells. Chondromyxoid fibroma is common in the
anterior proximal tibia. It is medullary-based with subtle mineralization and causes
thinning of the corticies. Osteoid osteoma is also most commonly periosteally or
cortically based but is generally small (< 1cm) and has a sclerotic border.

The Preferred Response to Question # 85 is 5.

60
277
2011 Musculoskeletal Tumors Self-Assessment Exam by Dr.Dhahirortho

Question 86 Figures 86a and 86b show the radiograph and biopsy specimen of a
16-year-old boy who reports increasing right foot pain. What is the most likely
diagnosis?
1- Enchondroma
2- Giant cell tumor
3- Chondrosarcoma
4- Chondromyxoid fibroma
5- Nonossifying fibroma

DISCUSSION: The radiograph


shows a benign-appearing bubbly radiolucent lesion with a narrow zone of transition
and a thin rim of surrounding reactive bone. The biopsy specimen demonstrates
spindle cells with a myxoid cartilaginous matrix. The diagnosis of the lesion is a
chondromyxoid fibroma. This rare, benign tumor is usually painful and can be locally
aggressive. Enchondroma is typically associated with calcifications and a bland
hyaline cartilage on histology. Giant cell tumors are typically radiolucent without the
bubbly appearance seen here and composed of multinucleated giant cells with
stromal cells having similar nuclear morphology. Chondrosarcoma is uncommon in
the hands and feet. Nonossifying fibromas can have a similar radiographic
appearance but are histologically distinguished by a storiform pattern of bland
fibroblasts with scattered giant cells. The Preferred Response to Question # 86 is 4.

Question 87 Figures 87a through 87c show the AP radiograph and coronal and
axial MRI scans of a 50-year-old woman who has had right shoulder pain with
overhead activity for the past 6 months. What is the most appropriate treatment
of this lesion?

1- Observation with serial radiographs


2- Biopsy
3- Curettage and grafting
4- Wide resection
5- Chemotherapy followed by wide resection
61
278
2011 Musculoskeletal Tumors Self-Assessment Exam by Dr.Dhahirortho

DISCUSSION: The images show a mineralized lesion in the metaphysis of the


proximal humerus, with no cortical disruption or endosteal scalloping. The
mineralization pattern is suggestive of a hyaline cartilage neoplasm. There is no soft-
tissue mass on the MRI scans. This is consistent with a benign enchondroma.
Observation is the treatment of choice for the lesion. Biopsy is not indicated and
would be difficult to interpret. The other treatments listed are for low- or high-grade
chondrosarcomas. Other explanations for the patient's shoulder pain, such as
rotator cuff pathology or impingement, should be sought and treated.
The Preferred Response to Question # 87 is 1.

Question 88 A 51-year-old woman has shoulder pain after a minor fall. A


radiograph, MRI scan, and bone scan are seen in Figures 88a through 88c. Biopsy
specimens are seen in Figures 88d and 88e. What is the most likely diagnosis?

1- Osteosarcoma
2- Enchondroma
3- Fibrous dysplasia
4- Chondrosarcoma
5- Chondromyxoid fibroma

DISCUSSION: The radiograph shows an expansile, calcified tumor of the proximal


humerus. The MRI scan shows an associated soft-tissue mass and the bone scan
demonstrates increased uptake, although these findings could be associated with a
pathologic fracture. Pathology is consistent with chondrosarcoma. Enchondroma
could present with a similar pattern but should generally be smaller, less expansile,
and have no soft-tissue extension; it is also less likely to be associated with a
pathologic fracture. Additionally, the pathology would show more benign-appearing
cartilage. Chondromyxoid fibroma is more typically eccentric and is associated with
radiologic and pathologic findings consisting of benign chondroid, fibrous, and
myxoid elements. The radiograph and pathology are not consistent with
osteosarcoma or fibrous dysplasia.
The Preferred Response to Question # 88 is 4.

62
279
2011 Musculoskeletal Tumors Self-Assessment Exam by Dr.Dhahirortho

Question 89 Based on the lesion seen in Figure 89a and the biopsy specimen seen
in Figure 89b, what is the most likely diagnosis?

1- Schwannoma
2- Nodular fasciitis
3- Lipoma
4- Hemangioma
5- Synovial sarcoma

DISCUSSION: The most reliable sign in


diagnosing schwannoma is the
percussion sign (radiating pain in the
affected nerve distribution on percussion of the lesion). Although sensory deficits
are reported, motor weakness is rare. The sagittal MRI scan demonstrates both the
string (attenuation of the nerve above and below the tumor) and target signs
(circular low signal area within the mass). The biopsy specimen shows the palisading
nuclei (Verocay bodies) typically seen in this disease. Nodular fasciitis is a benign
tumor that arises in the deep fascia. It is fairly homogeneous and hyperintense on T2
pulse-weighted images and isointense to hypointense on corresponding T1 images.
Lipoma is homogeneous bright on both T1 and T2 pulse-weighted images.
Hemangioma is usually intramuscular and heterogeneous because of vessel
thrombosis and mineralization and adjacent interspersed fat. Synovial sarcoma is
more often periarticular and located within muscle. It too can be very
heterogeneous because of its cellularity, intralesional necrosis, collagenization, and
mineralization. The Preferred Response # 89 is 1.

Question 90 What syndrome is characterized by polyostotic fibrous dysplasia, cafe-


au-lait spots with serrated borders, endocrine abnormalities, and unilaterality?
1- Jaffe-Campanacci syndrome
2- Hunter's syndrome
3- Maffucci's syndrome
4- Multiple hereditary exostoses
5- McCune-Albright syndrome

DISCUSSION: This is a description for McCune-Albright syndrome. Jaffe-Campanacci


syndrome is characterized by multiple nonossifying fibromas, cafe-au-lait spots, and
the absence of neurofibromas. Hunter's syndrome is a mucopolysaccharidosis and
lysosomal storage disease that affects all large joints with pain and decreased range
63
280
2011 Musculoskeletal Tumors Self-Assessment Exam by Dr.Dhahirortho

of motion. Maffucci's syndrome is associated with multiple hemangiomas and


enchondromas. Multiple hereditary exostosis is characterized by the presence of
multiple exostoses and is not associated with skin lesions or endocrine
abnormalities. The Preferred Response to Question # 90 is 5.

Question 91 A 25-year-old man has had low back pain for the past 4 months. An
AP pelvis radiograph, CT scan, MRI scan, and biopsy specimen are shown in Figures
91a through 91d. What is the most likely diagnosis?

1- Giant cell tumor


2- Chordoma
3- Osteosarcoma
4- Hemangioma
5- Rectal adenocarcinoma

DISCUSSION: Imaging shows a lesion in the sacrum, with extension out of the bone.
Histology shows multinucleated giant cells with deposits of hemosiderin. This is
consistent with giant cell tumor. Chordoma commonly occurs in the sacrum as a
midline lesion of notocord remnants, but the histology would show physaliferous
cells. Osteosarcoma occasionally occurs in the sacrum, but histology would show
malignant cells and osteoid. Hemangiomas in the spine are typically small,
intraosseus lesions with vertical striations. Rectal adenocarcinoma would originate
anterior to the sacrum and show a glandular pattern on biopsy.Pre Res # 91 is 1.

Question 92 An 11-year-old boy reports pain after throwing a ball in gym class. He
denies prior pain in the arm. Radiographs of the humerus are shown in Figures 92a
and 92b. What is the next most appropriate step in management?

1- Staging studies (MRI, bone scan, and chest CT)


2- Open biopsy
3- Sling and closed fracture care
4- Curettage and bone grafting
5- Open reduction and internal fixation

64
281
2011 Musculoskeletal Tumors Self-Assessment Exam by Dr.Dhahirortho

DISCUSSION: The radiographs reveal a fracture through a lesion of the proximal


humerus. The central lucency without any obvious matrix and thinning of the
cortical bone are typical findings of an unicameral bone cyst. The proximal humerus
is the most common site for this lesion. Typically these fractures should be allowed
to heal prior to treatment, which may include, classically, aspiration and injection of
corticosteroid. Recently other treatment options, including the injection of bone
marrow aspirate or bone graft substitutes, have been reported, with open curettage
and bone grafting reserved only for recurrent or nonresponding cysts. The Preferred
Response to Question # 92 is 3.

Question 93 Figures 93a through 93d show the MRI scan, CT scans, and a biopsy
specimen of a 23-year-old woman who reports left-sided back pain that travels
around the lateral aspect of the thigh and down the lateral aspect of the left leg.
The pain is relieved with anti-inflammatory medications. It is not associated with
weight bearing. What is the most likely diagnosis?

1- Osteosarcoma
2- Osteoblastoma
3- Giant cell tumor
4- Aneurysmal bone cyst
5- Stress fracture

DISCUSSION: Osteoblastoma is a bone-forming lesion characterized by a mixed lytic-


blastic lesion of variable size greater than 2 centimeters. Histology shows osteoid
formation with benign rimming osteoblasts and an intervening highly vascular
stroma. Osteosarcoma can look similar radiographically but histology typically shows
malignant spindle cells interspersed in osteoid. Giant cell tumor histology shows
multiple multinucleated giant cells in a background stroma of stromal cells with
similar nuclei. Aneurysmal bone cyst typically has lakes of blood surrounded by a
thin benign fibrous membrane. Stress fracture does not typically have a large lucent
central lesion on radiographs. The Preferred Response to Question # 93 is 2.
65
282
2011 Musculoskeletal Tumors Self-Assessment Exam by Dr.Dhahirortho

Question 94 What is the primary problem in rickets osteomalacia?


1- Defect in the zone of proliferation within the physis
2- Defect in type I collagen
3- Defect in the ext-1 gene
4- Low level of calcium
5- Production of dysplastic fibrous bone

DISCUSSION: Rickets is a disorder of bones in children that results from decreased


calcium available in the blood resulting in poor mineralization of bone that can lead
to fractures and deformity. The most common cause of rickets is from vitamin D
deficiency but it can also be caused by poor nutrition or gastrointestinal disease that
results in poor calcium absorption such as celiac disease or severe diarrhea from
other causes. Rickets is not primarily a physeal disorder. Osteogenesis imperfecta is
caused by a defect in type I collagen. A defect in the ext-1 gene is often seen in
patients with multiple hereditary exostoses. Fibrous dysplasia also can result in bone
deformity and fractures due to production of dysplastic fibrous bone but is not
caused by calcium or vitamin D deficiency. The Preferred Response # 94 is 4.

Question 95 A 42-year-old man has pain and swelling about his knee that has been
worsening over the last 6 months. Examination shows an effusion, bogginess in
the suprapatellar pouch, and a decrease in range of motion when compared to the
contralateral knee. Radiographs are shown in Figures 95a and 95b and a sagittal
MRI scan is shown in Figure 95c. A biopsy specimen is shown in Figure 95d. What is
the most appropriate treatment for this lesion to minimize the risk of local
recurrence?

1- Observation
2- Intra-articular injection of methylprednisolone
3- Limited arthroscopic synovectomy
4- Open anterior and posterior synovectomy
5- Intra-articular injection of radioisotope
66
283
2011 Musculoskeletal Tumors Self-Assessment Exam by Dr.Dhahirortho

DISCUSSION: The clinical presentation, radiographic studies, and biopsy specimen


are all consistent with pigmented villonodular tenosynovitis (PVNS). Most agree that
the lowest chance of recurrence for diffuse PVNS of the knee is with open anterior
and posterior synovectomy. However, this can be associated with significant
morbidity. Arthroscopic synovectomy, while associated with a higher recurrence
rate, is preferred by some due to lower morbidity. Regardless of the aggressiveness
of synovectomy, the recurrence rate of diffuse PVNS remains significant. The cause
of PVNS is controversial, but most believe it is a non-neoplastic inflammatory
process that results in intra-articular synovial thickening, bloody effusion, pain, and
even intraosseous erosion. Intra-articular radioisotopes have been removed from
the market because of the risk of hematomyelopoetic diseases. Observation alone
will likely lead to disease progression and joint destruction. Intra-articular steroids
will not halt the progression of PVNS and, therefore have no role in its treatment.
The Preferred Response to Question # 95 is 4.

Question 96 Acral bone metastases (to the hands and feet) are most likely the
result of a primary solid organ tumor in which of the followings structures?
1- Liver 2- Lung 3- Breast 4- Thyroid 5- Prostate

DISCUSSION: Lung cancer is the most likely solid organ cancer to metastasize distal
to the elbow and the knees. The most common sites for metastatic disease are the
vertebral bodies, pelvis, and long bones. Genitourinary tumors may also metastasize
to distal sites. The Preferred Response to Question # 96 is 2.

Question 97 A 13-year-old girl has had a painless thigh mass for the past 3 months.
A biopsy is performed and chromosome analysis reveals a t(X;18)(p11:q11)
translocation (involving the genes SYT-SSX). What is the most likely diagnosis?
1- Liposarcoma
2- Synovial sarcoma
3- Myxoid chondrosarcoma
4- Gastrointestinal stromal tumor
5- Malignant fibrous histiocytoma

DISCUSSION: Although synovial sarcoma accounts for 6% of all soft-tissue sarcomas,


it represents 16% of soft-tissue sarcomas in adolescents and young adults, making it
and rhabdomyosarcoma (which accounts for 15%) the most common soft-tissue
sarcomas in this age group. The (X;18)(p11:q11) translocation is characteristic of
synovial sarcoma. Myxoid chondrosarcomas often demonstrate a t(9;22)
67
284
2011 Musculoskeletal Tumors Self-Assessment Exam by Dr.Dhahirortho

translocation. Well-differentiated liposarcomas and malignant fibrous histiocytomas


may demonstrate a ring form of chromosome 12. Gastrointestinal stromal tumors
may exhibit monosomies of chromosomes 14 and 22. Preferred Respon # 97 is 2.

Question 98 Figures 98a through 98c show the radiograph, MRI scan, and biopsy
specimen of a 13-year-old girl who reports increasing right ankle pain for the past
2 months. Work-up reveals no other lesions. What is the most appropriate
treatment for this patient?

1- Surgery alone
2- Radiation therapy alone
3- Chemotherapy alone
4- Surgery and radiation therapy
5- Surgery and chemotherapy, with or without radiation therapy

DISCUSSION: The imaging studies show a permeative destructive lesion of the distal
fibula with periosteal reaction. The biopsy specimen demonstrates a small blue cell
tumor consistent with Ewing's sarcoma. The most appropriate treatment for this
patient would be a combination of surgery and chemotherapy. Local control of
Ewing's sarcoma is usually achieved by surgery with wide margins. In some cases,
radiation therapy can be used for local control if wide resection would be associated
with unacceptable morbidity. Chemotherapy, however, is required in all cases to
treat systemic disease even if no metastases are revealed on the initial work-up.

The Preferred Response to Question # 98 is 5.

68
285
2011 Musculoskeletal Tumors Self-Assessment Exam by Dr.Dhahirortho

Question 99 When compared with postoperative external beam radiation therapy


for the treatment of soft-tissue sarcomas, preoperative radiation therapy is
associated with which of the following?
1- Higher wound complications
2- Higher doses of radiation
3- Larger volumes of tissue irradiated
4- Worse overall survival
5- Worse functional results
DISCUSSION: Preoperative radiation therapy requires a lower dose of radiation
(5,000 cGy versus 6,600 cGy) and lower volume of tissue, with no difference in
survival and a trend toward better functional outcome, compared with
postoperative radiation. Preoperative radiation is associated with a significantly
higher wound complication rate (35% versus 17%). Preferred Response # 99 is 1.

Question 100 A 43-year-old woman has a pathologic right acetabular fracture seen
in Figure 100. Laboratory studies are unremarkable but a bone scan shows
multiple skeletal areas with increased activity, and a CT scan of the
chest/abdomen/pelvis shows some visceral involvement and also a right breast
mass, suspicious for a primary lesion. What is the next most appropriate step in
management?
1- PET scan
2- Biopsy
3- Referral to medical oncologist
4- Total hip arthroplasty with fixation of the acetabular fracture
5- Hospice referral
DISCUSSION: The next most appropriate step in management is to
proceed with a biopsy of the most accessible site. This can be done
by the surgeon or by an interventional radiologist trained in core biopsy techniques.
It is imperative to make a pathologic diagnosis prior to proceeding with any further
medical, surgical, or radiation treatments. A histologic diagnosis at this point is the
only way a medical oncologist can have a meaningful discussion with the patient
about their disease, its natural history, and ultimately discuss treatment options and
prognosis. Further imaging at this point only delays the time to histologic evaluation.
Consideration of surgical stabilization can be delayed until a diagnosis is established
and a multidisciplinary approach is initiated. This fracture can be treated at least
temporarily with nonsurgical protected weight bearing with a walker or crutches.
Hospice may soon serve a useful role but a diagnosis must first be rendered and a
limited life expectancy anticipated. Pre Res # 100 is 2
69
286
Online 2011 Anatomy-Imaging Self-Assessment Examination by Dr.Dhahirortho

1
287
Online 2011 Anatomy-Imaging Self-Assessment Examination by Dr.Dhahirortho

2
288
Online 2011 Anatomy-Imaging Self-Assessment Examination by Dr.Dhahirortho

Q 1 1a 1b 1c 1d 1e A 43-year-old female factory worker has had a 6-month history of right


plantar and lateral foot pain. She has pain with weight bearing and has difficulty standing at
work. Management consisting of physical therapy, time off of work, and fracture boot
immobilization has failed to provide relief. She is overweight and, as a result of the pain, cannot
exercise to lose weight; thus she is getting worse instead of better. Examination reveals that the
foot is not grossly swollen but is diffusely tender over the lateral, plantar, and medial hindfoot.
The alignment is normal and the posterior calf muscles are mildly tight. A lateral radiograph is
shown in Figure 1a and MRI scans are shown in Figures 1b through 1e. These findings are most
consistent with which of the following?

1- Painful os trigonum
2- Complex regional pain syndrome
3- Stress fracture of the anterior aspect of the calcaneus
4- Hematogenous osteomyelitis
5- Plantar fasciitis

DISCUSSION: The studies are most consistent with a stress fracture or insufficiency fracture of the
anterior portion of the calcaneus. The radiograph shows normal findings. There is increased signal
involving the inferior anterior aspect of the calcaneus on the T2-weighted images (Figures 1c
through 1e), which is consistent with edema. There is also an abnormal trabecular pattern within
this region with changes on the T1 and T2 images consistent with a stress or insufficiency fracture
of the calcaneus. Whereas there is some increased signal from the os trigonum and the origin of
the plantar fascia, these diagnoses are inconsistent with her symptoms. The MRI findings of
osteomyelitis (decreased T1 signal and increased T2 signal) with secondary soft-tissue findings of
adjacent soft-tissue ulcers, cellulitis, phlegmon, abscess, sinus tracts, or cortical bone destruction
are not present. Complex regional pain syndrome has a wide spectrum of findings on MRI and is
usually much more diffuse.

The Preferred Response to Question # 1 is 3.


3
289
Online 2011 Anatomy-Imaging Self-Assessment Examination by Dr.Dhahirortho

Question2 2a 2b 2c A 36-year-old man reports pain and fullness in the medial arm just proximal
to the epicondyle. He denies trauma to the arm but noted some soreness following an arm
wrestling match a few months ago. Figures 2a through 2c show the radiographs and a CT scan.
What is the most likely diagnosis?

1- Extraskeletal
osteosarcoma
2- Parosteal osteosarcoma
3- Osteochondroma
4- Myositis ossificans
5- Synovial cell sarcoma

DISCUSSION: The radiographs reveal a mature lesion that is ossified, not calcified, in the soft
tissues of the arm, which is juxtaposed to the bone but not sessile ("stuck on") on the cortex nor
does the lesion share the cortex. The CT scan reveals a soft-tissue lesion that is mineralized at the
periphery, not centrally. This combination of features (smooth, ossified soft-tissue mass with
mineral density at the periphery, known as the Zonation Phenomenon of Ackerman) strongly
suggests myositis ossificans as the diagnosis. Osteosarcoma would mature in the central areas, not
the periphery, and would not have this smooth appearance. Parosteal osteosarcoma would be
attached to the cortex (this is not a good location for that diagnosis), osteochondroma would
share the bony cortex, and synovial sarcoma generally presents with whispy, irregular calcification
within the mass. The images do not support any of these other diagnoses. The Preferred Response
to Question # 2 is 4.

3 Which of the following vascular structures provides the most significant secondary
contribution to the blood supply of the femoral head?

1- Lateral femoral circumflex artery


2- Superior gluteal artery
3- Inferior gluteal artery
4- Circumflex iliac artery
5- Superficial femoral artery

DISCUSSION: The superior and inferior gluteal arteries form a vascular network posterior to the
hip. Whereas both arteries provide blood supply to the acetabulum, the inferior gluteal artery
frequently anastomoses directly into the deep branch of the medial femoral circumflex artery and
in a minority of patients has been shown to be the dominant blood supply into the femoral head.
Pref Respo is 3.

4
290
Online 2011 Anatomy-Imaging Self-Assessment Examination by Dr.Dhahirortho

4 Figure 4 shows the radiograph of a 65-year-old patient who is undergoing right total knee
arthroplasty. After performing bone resections for a posterior cruciate-substituting femoral
component, you note that both the flexion and extension gaps are tight in the lateral
compartment. Which of the following structures should be released
first?
1- Medial collateral ligament
2- Lateral collateral ligament
3- Posterolateral capsule
4- Iliotibial band
5- Popliteus tendon

DISCUSSION: For patients with tightness in both flexion and


extension, the lateral collateral ligament should be released before
the other lateral structures. Soft-tissue balancing after correction of a
valgus deformity is performed to equalize varus and valgus laxity in both flexion and extension. If it
is released after the other stabilizers (iliotibial band, posterolateral capsule, or popliteus tendon),
the magnitude of these corrections will be more significant. This may lead to a greater flexion-
extension asymmetry if an isolated extension or flexion release has been performed first. Release
of the posterior cruciate ligament has the greatest impact on lateral knee balance, so the decision
to use a cruciate retaining or substituting implant ideally should be made before proceeding with
release of the lateral structures. An isolated flexion contracture is typically approached with a
posterolateral capsule release. An isolated extension contracture may be treated with either a
release of the iliotibial band or popliteus tendon. However, authors recently have indicated a
preference for retention of the popliteus tendon. The Preferred Response to Question # 4 is 2.

5 Figure 5 shows the MRI scan of a 35-year-old woman with shoulder pain. What is the most
likely diagnosis?

1- Superior labral tear


2- Partial articular surface supraspinatus tear
3- Partial bursal surface supraspinatus tear
4- Full-thickness supraspinatus tear
5- Internal impingement

DISCUSSION: The MRI scan shows a coronal oblique view of a partial-


thickness bursal surface supraspinatus tear. Partial-thickness rotator cuff tears are a common
cause of shoulder pain, with articular-sided tears two to three times more common than bursal-
sided tears. The articular surface of the supraspinatus is intact in this image; therefore, no partial
articular or full-thickness tear exists. The superior labrum is seen at the top of the glenoid with no
tear. Internal impingement affects the articular surface of the rotator cuff in abduction and
external rotation, causing labral tearing and partial articular-sided rotator cuff tears. The Preferred
Response to Question # 5 is 3.

5
291
Online 2011 Anatomy-Imaging Self-Assessment Examination by Dr.Dhahirortho

6 Figure 6 shows a sagittal oblique MRI scan of a right shoulder. The asterisk indicates what
anatomic structure?

1- Subscapularis
2- Supraspinatus
3- Infraspinatus
4- Teres minor
5- Teres major

DISCUSSION: The asterisk indicates the subscapularis. The sagittal


oblique MRI image shown is taken at the level of the coracoid, which
allows orientation of the anterior and posterior aspects of the shoulder. Evaluation of the rotator
cuff in this plane is important to determine the amount of muscle atrophy and fatty infiltration,
which may determine prognosis for rotator cuff repair. The supraspinatus is superior to the
glenoid and the infraspinatus; teres minor and teres major are posterior. Prefe Res is 1.

7 .7a 7b 7c A 52-year-old woman has had progressive shoulder pain for the past 18 months. She
has pain at night that awakens her from sleep, and a constant ache in her shoulder that has
required narcotics for pain control. She has a history of fracture following an automobile
accident 15 years prior that healed without incident. Figures 7a through 7c show the radiograph,
bone scan, and T2-weighted coronal MRI scan of the proximal humerus. Based on this
information, what is the most likely diagnosis?

1- Progressive bone infarct


2- Osteosarcoma
3- Ewing's sarcoma
4- Chondrosarcoma
5- Fibrous dysplasia

DISCUSSION: The images reveal a


mineralized lesion of the proximal
humerus that has the classic
appearance of a chondroid lesion with
the "rings and arcs" densities typically
seen on a radiograph. The bone scan
shows increased uptake in this area, consistent with an active lesion. The MRI scan confirms a
lobular lesion that is bright on the T2-weighted scan and fills the bone. This, in combination with
the history and age of the patient, is strongly suggestive of a malignant chondroid lesion, or
chondrosarcoma. Whereas a bone infarct has mineral density on radiographs, it tends to be more
"smoke up the chimney" or wispy in appearance, and is usually not hot on bone scan and not
symptomatic. Ewing's lesions are destructive lytic lesions without the mineralization as seen here.
Fibrous dysplasia similarly lacks this mineralization pattern, but has the "ground glass" or "hazy"

6
292
Online 2011 Anatomy-Imaging Self-Assessment Examination by Dr.Dhahirortho

appearance and is rarely symptomatic. Osteosarcoma typically appears in younger patients and
has a generally more
destructive appearance with "cloud-like" mineralization from bone formation, as compared with
the "rings and arcs" mineralization seen in chondroid lesions.Prefer Response to Question # 7 is 4.

8 8a 8b A 35-year-old woman is involved in a head-on collision while driving. Initial radiographs


are shown in Figures 8a and 8b. Injury to what vessel increases the risk for osteonecrosis of the
injured bone?

1- Dorsalis pedis artery


2- Perforating peroneal artery
3- Lateral tarsal artery
4- Artery of the tarsal canal
5- Artery of the tarsal sinus

DISCUSSION: The patient has a Hawkins type


III talar neck fracture-dislocation with a risk
of osteonecrosis ranging from 69% to 100%.
Anatomic studies have shown that the artery
of the tarsal canal supplies the lateral two
thirds of the talar body. The other vessels listed provide no significant contribution to the talus.
The Preferred Response to Question # 8 is 4.

9 When performing hip arthroscopy, the hip should be placed in neutral to slight internal
rotation to protect which of the following structures?

1- Femoral nerve
2- Lateral femoral cutaneous nerve
3- Ascending lateral femoral circumflex artery
4- Ascending medial femoral circumflex artery
5- Sciatic nerve

DISCUSSION: The sciatic nerve is at greatest risk for injury during hip arthroscopy with placement
of a posterolateral (posterior paratrochanteric portal). It can be within 3 cm of this portal.
Advancing the trocar with the hip in neutral to slight internal rotation helps to protect the sciatic
nerve from iatrogenic injury. The two structures in closest proximity with placement of
arthroscopy portals are the lateral femoral cutaneous nerve (anterior portal) and the ascending
branch of the lateral femoral circumflex artery (mid-anterior portal). The femoral nerve and medial
femoral circumflex arteries are located medial to these anterior portals. Rotation of the hip has
not been associated with increased risk of injury to any of these additional structures.

The Preferred Response to Question # 9 is 5.

7
293
Online 2011 Anatomy-Imaging Self-Assessment Examination by Dr.Dhahirortho

10 The finding at L3 in the sagittal CT scan shown in Figure 10 is characteristic of which of the
following conditions?
1- Paget's disease
2- Hemangioma
3- Giant cell tumor
4- Metastatic disease
5- Chordoma

DISCUSSION: The L3 vertebral body shows the classic vertical


striations of a vertebral hemangioma. On axial images, these
would appear as a collection of bony spots as they are seen end-
on. Paget's disease may have areas of sclerotic trabeculae but the
vertebrae appear square and enlarged with thickened cortex and
demonstrate no organization in the mineral density. Giant cell
tumors are more lytic and locally destructive. Metastatic disease can be lytic or sclerotic but does
not organize in this pattern. Similarly, chordoma does not have this organization. Chordomas more
commonly occur in the sacrum, but may occur in the lumbar vertebrae. Prefer Res is 2.

11 A 28-year-old man has decreased finger proximal interphalangeal (PIP) joint range of motion
after open reduction and internal fixation of a proximal phalanx fracture with the use of a side
plate. Examination shows greater passive PIP joint flexion with metatarsophalangeal (MP) joint
extension, than when the MP joint is flexed. This finding demonstrates contracture/scarring of
which of the following structures?
1- Flexor tendons
2- Extensor tendon
3- Oblique retinacular ligament
4- Intrinsic muscles
5- PIP joint

DISCUSSION: In phalanx fractures treated with a plate and open reduction and internal fixation,
adhesions commonly develop between the fracture, hardware, and extensor system. This example
demonstrates extrinsic tightness. The flexor tendons usually are not scarred in this type of surgical
approach. The oblique retinacular ligament is near the distal interphalangeal joint and would not
significantly impact the PIP joint. The intrinsics are less affected by this scarring than the extrinsics,
resulting in different exam results (improved PIP motion with MP flexion). PIP joint stiffness would
be constant regardless of the position of the MP joint. The Preferred Response to Questi # 11 is 2.

12 12a Figure 12a shows a cross section of the pelvis at the level of the greater trochanters.
What structure is marked with the arrow?
1- Adductor magnus
2- Obturator internus
3- Obturator externus
4- Pectineus
5- Adductor brevis

8
294
Online 2011 Anatomy-Imaging Self-Assessment Examination by Dr.Dhahirortho

DISCUSSION: In Figure 12b, the arrow marks the obturator internus muscle which projects
posteriorly and banks around the ischium, inserting on the posterior aspect of the proximal
femur, just below the piriformis. The other structures are labeled. The obturator externus is more
anterior and is seen anterior to the ischium. The adductor magnus is not seen in this image, and is
more distal. The adductor longus is just starting to appear anteriorly (with the adductor brevis just
posterior), and the pectineus is seen posterior and just deep to the femoral vessels. The Preferred
Response to Ques# 12 is 2.

13 13a 13b 13c The MRI scans shown in Figures 13a through 13c show findings that are classic
and, in combination on the MRI sequences, are pathognomonic for what diagnosis?

1- Lymphoma
2- Primary epidural abscess
3- Degenerative disk disease
4- Diskitis
5- Early ankylosing spondylitis

DISCUSSION: The sagittal MRI sequences show findings that are classic for diskitis. The T2 image
(Figure 13a) has a bright signal appearance within the disk space (free water) consistent with pus.
On the T1 image (Figure 13b), the disk and vertebral bony margins appear dark with uniform signal
across the disk that results in loss of the distinction between disk and vertebral body. Lastly, on T1,
fat suppressed with gadolinium (Figure 13c), the abscess noted on T2 is now dark with a
surrounding rim of enhancement (hypervascularity) that includes the adjacent vertebral bodies.
Although an advanced degenerative disk can
appear with a fluid signal within the disk space, the surrounding hypervascularity or obliteration of
the distinct margins of the disk is not expected. The Preferred Response to Question # 13 is 4.

14 Figure 14 shows the view looking forward from the posterosuperior portal during shoulder
arthroscopy. The structure highlighted by the asterick is innervated by which of the following
nerves?
1- Suprascapular
2- Subscapular
3- Radial
4- Median
5- Axillary

DISCUSSION: The structure shown in the arthroscopic image


is the tendon of the subscapularis muscle. The subscapularis
muscle is innervated by the subscapular nerve. The
subscapularis is not innervated by the other nerves listed.
The Preferred Response to Question # 14 is 2.

9
295
Online 2011 Anatomy-Imaging Self-Assessment Examination by Dr.Dhahirortho

15 During establishment of an anterior portal for hip arthroscopy, what structure is at greatest
risk for injury?
1- Lateral femoral cutaneous nerve
2- Femoral nerve
3- Femoral artery
4- Superior gluteal nerve
5- Sciatic nerve

DISCUSSION: The anterior portal for hip arthroscopy is approximately 6 cm distal to the anterior
superior iliac spine, penetrating the muscle belly of the sartorius and the rectus femoris before
entering through the anterior capsule. The lateral femoral cutaneous nerve is divided into three or
more branches at the level of this portal and may be injured during portal placement. The femoral
nerve and artery are more medial and at less risk. The superior gluteal and sciatic nerves are
posterior and not at risk with an anterior portal. The Preferred Response to Question # 15 is 1.

16 Following fixation of a comminuted both-bone forearm fracture, the patient has weakness of
the flexor pollicis longus and flexor digitorum profundus to the index finger. Which of the
following structures has most likely been injured?

1- Anterior interosseous nerve


2- Posterior interosseous nerve
3- Radial nerve
4- Ulnar nerve
5- Lateral antebrachial cutaneous nerve

DISCUSSION: The anterior interosseous nerve innervates the flexor pollicis longus (FPL) and the
flexor digitorum profundus (FDP) to the index finger. It branches posteriorly from the median
nerve deep to the pronator teres where it is susceptible to injury. The posterior interosseous
nerve, radial nerve, or ulnar nerve do not innervate the FPL or FDP to the index finger. The lateral
antebrachial cutaneous nerve is a sensory nerve. The Preferred Response to Question # 16 is 1.

17 During an anterior retroperitoneal approach to the lumbar spine, what nerve is encountered
lying on the anteromedial surface of the psoas muscle?
1- Genitofemoral
2- Ilioinguinal
3- Femoral
4- Lateral femoral cutaneous
5- Iliohypogastric

DISCUSSION: The genitofemoral nerve arises from the L1 and L2 roots and then emerges through
the psoas between the third and fourth lumbar vertebrae from where it runs along the surface of
the psoas. The ilioinguinal, lateral femoral cutaneous, and the iliohypogastric nerves all arise from
upper lumbar roots but remain posterior to the psoas and then run along the inner surface of the
quadratus lumborum and iliacus muscles. The femoral nerve runs posterior to the psoas muscle in

10
296
Online 2011 Anatomy-Imaging Self-Assessment Examination by Dr.Dhahirortho

the retroperitoneum before wrapping around laterally to ultimately lie on the anterior surface of
the iliopsoas muscles distally as it exits the pelvis. The Preferred Response to Question # 17 is 1.

18 The most common neurologic injury following an anterior cervical diskectomy and fusion
(ACDF) is injury to which of the following structures?
1- Recurrent laryngeal nerve
2- Superior laryngeal nerve
3- C5 root
4- Spinal cord
5- Sympathetic chain

DISCUSSION: The most common neurologic injury in ACDF is injury to the recurrent laryngeal
nerve. It is most vulnerable on the right because it crosses from lateral to midline more cephalad
in the incision after it passes under the subclavian artery; conversely, on the left the course is
more caudal because it passes under the aortic arch, a more caudal structure. The superior
laryngeal nerve runs along with the superior thyroid artery in the upper cervical spine, putting it at
risk during surgical procedures on the upper cervical spine which are less commonly performed. A
C5 root palsy more commonly occurs as a result of multilevel posterior decompressive procedures,
possibly because of its short transverse take-off from the cord. The sympathetic chain lies on top
of the longus colli and can be injured if retractors are not placed under the longus colli muscle. The
Preferred Response to Question # 18 is 1.

19a 19b A 15-year-old girl injured her shoulder in a fall while riding her bicycle. She reports a
mild ache over the latter aspect of the shoulder, present since the accident, but denies any prior
shoulder symptoms of any kind. AP and lateral radiographs shown in Figures 19a and 19b reveal
a lesion in the proximal humerus. What is the most likely diagnosis?

1- Osteoblastoma
2- Aneurysmal bone cyst
3- Enchondroma
4- Osteochondroma
5- Nonossifying fibroma

DISCUSSION: This scenario represents the common


"serendipitous" finding of benign chondroid lesions.
The radiographs demonstrate the classic "rings and
arcs" calcification of an enchondroma, in a commonly
presenting location (proximal humerus). The lesion is generally centrally located, and may have a
well-defined lucent appearance, typically in the metaphysis of the bones. The other lesions listed
do not have the typical calcification seen in these chondroid lesions. The mineral density in an
osteoblastoma is more sclerotic and the lesion is often destructive. An aneurysmal bone cyst is
purely lytic and generally expansile. Osteochondroma is an exophytic lesion, protruding outside
the bone. Nonossifying fibroma is an eccentric, well-demarcated lesion with no mineral density.
The Preferred Response to Question # 19 is 3.

11
297
Online 2011 Anatomy-Imaging Self-Assessment Examination by Dr.Dhahirortho

20 A 24-year-old man has a deep knife wound across the dorsal aspect of his wrist, transecting
all of his wrist and finger extensor tendons. How does the surgeon determine which of the
proximal tendon stumps in the fourth dorsal compartment is the extensor indicis proprius?

1- The tendon runs in a separate compartment.


2- The tendon has a more circular cross section.
3- The tendon has the most distal muscle belly.
4- The tendon is the most radial and superficial.
5- The tendon has two separate slips.

DISCUSSION: The extensor indicis proprius tendon is deep and ulnar to the extensor digitorum
communis (EDC) tendons in the fourth dorsal compartment. It is a single tendon and there is no
subcompartment. It has no distinguishing characteristics other than it has a more distal muscle
belly in comparison to the EDC tendons. Response #1 is incorrect because the extensor indicis
proprius and extensor digitorum communis run in the same compartment. Both tendons have a
rather flat cross section and cannot be distinguished by this method. Responses #4 and #5 are
incorrect as the tendon is not superficial, nor does it have two separate slips. Pre Resp # 20 is 3.

21 Figure 21 shows a coronal T1-weighted MRI scan of the knee. The arrow indicates what
anatomic structure?
1- Posterior cruciate ligament: anterolateral bundle
2- Posterior cruciate ligament: posteromedial bundle
3- Meniscofemoral ligament
4- Popliteus
5- Oblique popliteal ligament

DISCUSSION: The arrow is pointing to the meniscofemoral ligament.


The meniscofemoral ligament connects the posterior horn of the
lateral meniscus to the intercondylar wall of the medial femoral
condyle. The ligament of Humphrey passes anterior to the posterior
cruciate ligament (PCL), whereas the ligament of Wrisberg passes posterior to the PCL. One or the
other has been identified in 71% to 100% of cadaver knees, with the ligament of Wrisberg being
more common. The PCL is shown inferior to the indicated structure. The popliteus and oblique
popliteal ligament are not visualized in this image. Preferred Response to Question # 21 is 3.

22 A 20-year-old man has a dorsal metacarpophalangeal dislocation of the index finger. Multiple
attempts to reduce the dislocation in the emergency department have not been successful.
What structure is most likely preventing the joint from being reduced?
1- First dorsal interosseous
2- Radial collateral ligament
3- Ulnar collateral ligament
4- Natatory ligament
5- Volar plate

12
298
Online 2011 Anatomy-Imaging Self-Assessment Examination by Dr.Dhahirortho

DISCUSSION: The volar plate is the structure that usually prevents the finger metacarpophalangeal
joint from reducing. Blockage by the first dorsal interosseous is not a common reason for an
irreducible metacarpophalangeal joint dislocation. None of the other structures commonly
prevent metacarpophalangeal joint reduction. The Preferred Response to Question # 22 is 5.

23 23a 23b A 15-year-old girl is referred to your office by her primary care physician who is
concerned about a "shadow on the bone" noted when office radiographs were obtained
following a minor soccer accident. The patient denies any history of knee pain, and has been
fully active without any restrictions. Examination is consistent with a minor sprain but otherwise
is unremarkable. The lesion is shown in Figures 23a and 23b. What is the most likely diagnosis?

1- Nonossifying fibroma
2- Giant cell tumor
3- Chronic infection
4- Osteofibrous dysplasia
5- Enchondroma

DISCUSSION: The AP and lateral radiographs reveal a


lytic, eccentric, well-marginated (mildly sclerotic)
lesion that is minimally expansile of the cortex (on
the lateral image). This is the classic appearance of a
nonossifying fibroma, which most commonly appears
in the metaphyseal region of the lower extremity long bones, particularly around the knee, and in
a young patient population. The lesion may also have a "bubbly" appearance, which is not
demonstrated in this particular case. Unless the lesion is large, or accompanied by a pathologic
fracture, they are generally incidental findings, as in this patient. Giant cell tumor is a more
destructive lesion that is typically subchondral in location generally in somewhat older patients.
Osteofibrous dysplasia, which may have a somewhat similar appearance, is almost always
diaphyseal in location and typically involves the anterior cortex. Enchondroma is not typically
eccentric, and while it may have a lytic appearance, usually demonstrates mineral density in the
lesion, and is not generally sclerotic at the margin. Infection may have an extremely variable
radiographic appearance, but would typically appear more aggressive and present with underlying
symptoms. The Preferred Response to Question # 23 is 1.

24 A 38-year-old man reports a 6-month history of pain in his left wrist. He denies any injury and
is otherwise healthy. An MRI scan is shown in Figure 24. What is the recommended treatment?

1- Radial shortening osteotomy


2- Lunate excision with tendon interposition
3- Lunate implant arthroplasty
4- Ulnar shortening osteotomy
5- Total wrist arthrodesis

13
299
Online 2011 Anatomy-Imaging Self-Assessment Examination by Dr.Dhahirortho

DISCUSSION: The MRI scan shows avascularity (decreased signal intensity on T1-weighted image)
of the lunate in an ulnar minus wrist, consistent with Kienbock's disease. No degenerative changes
are seen in the carpus. Of the choices listed, radial shortening osteotomy is the treatment of
choice. This procedure provides an extra-articular approach to treatment. The other options could
be considered in more advanced cases or if joint deterioration/destruction was noted. Lunate
excision with tendon interposition and lunate implant arthroplasty are rarely used at this time. An
ulnar shortening osteotomy could make the problem worse by increasing the contact forces
between the radius and lunate. A limited intercarpal fusion is usually used prior to resorting to
total wrist arthrodesis. The Preferred Response to Question # 24 is 1.

25 A patient undergoes open surgical dislocation of the hip to address femoroacetabular


impingement. During which stage of the surgical approach is the blood supply to the femoral
head at greatest risk?

1- Release of the piriformis tendon


2- Release of the anteroinferior capsule
3- Release of the posterosuperior capsule
4- Trochanteric osteotomy lateral to the piriformis
5- Anterior dislocation of the femoral head

DISCUSSION: Extended anteroinferior capsular release at or below the level of the lesser
trochanter may place the medial femoral circumflex artery at risk of direct injury. During a surgical
dislocation procedure, a trochanteric osteotomy is performed lateral to the piriformis insertion to
decrease the risk of vascular injury during dislocation. This preserves the piriformis insertion,
which is not released, and protects the ascending branch of the medial femoral circumflex artery
(ramus profunda) as it enters the capsule and courses superiorly to penetrate the femoral neck.
The posterosuperior capsule is safe from direct injury to the ramus profunda and the medial
femoral circumflex artery. Several published studies have demonstrated that this surgical
dislocation technique can allow anterior hip dislocation with minimal risk to femoral head
vascularity or osteonecrosis. The Preferred Response to Question # 25 is 2.

26 Figure 26 shows an axial T1-weighted MRI scan of the foot of a 13-year-old boy. The three-
pronged structure indicated by the arrow shows which of the following?

1- Insertion of the peroneus longus tendon


2- Insertion of the posterior tibial tendon
3- Flexor digitorum longus tendons
4- Inferior calcaneonavicular (spring) ligament
5- Branches of the posterior tibial artery

DISCUSSION: The arrow is pointing to the spring ligament. The image shown is
a transverse cut through the hindfoot below the subtalar joint and shows the
calcaneus, cuboid, inferior navicular, the cuneiforms, and the surrounding soft
tissues. The spring ligament spans between the navicular surface anteriorly and the middle

14
300
Online 2011 Anatomy-Imaging Self-Assessment Examination by Dr.Dhahirortho

calcaneal articular surface posteriorly. The flexor digitorum longus and posterior tibial tendons and
the posterior tibial artery are more proximal and medial. The peroneus longus is a lateral structure
that crosses from lateral to medial along the plantar foot as it crosses the cuboid and lies lateral to
the calcaneus at this level. The Preferred Response to Question # 26 is 4.

27 Which of the following surgical approaches to the knee has the greatest potential for
denervation of the quadriceps muscle?

1- Subvastus
2- Midvastus
3- Quadriceps sparing
4- Median parapatellar
5- Tibial tubercle osteotomy

DISCUSSION: Several studies have demonstrated excellent functional results and recovery after
total knee arthroplasty (TKA) with a variety of minimally invasive approaches; however, studies
have demonstrated abnormal electromyographic (EMG) studies in a significant number of TKAs
performed using a midvastus exposure. Patients whose vastus medialis intervals were developed
bluntly were significantly more likely to fully recover normal EMG activity than if the intervals were
developed with sharp dissection.
The Preferred Response to Question # 27 is 2.

28 The radiographic findings shown at the C5-6 disk above the C6-7 fusion in Figure 28 are most
commonly associated with what part of the surgical technique?

1- A needle placed there for radiographic confirmation of the level


2- Excessive periosteal stripping of the upper fused level
3- Dissection of the longus colli extending cephalad to that disk
4- Screw penetration of the upper end plate
5- The upper end of the plate being in close proximity to the adjacent
disk

DISCUSSION: When the upper border of the plate is located in close


proximity to the cephalad adjacent disk, there is a higher incidence of
osteophyte formation. The clinical implications of this are not yet
understood. Screw penetration or needle puncture may influence the
degenerative process at that disk, but this would manifest itself more as
narrowing and end plate changes as opposed to an osteophyte forming
along the anterior annulus. The role of the longus colli and periosteal dissection are not fully
elucidated but are less commonly associated with this finding.

The Preferred Response to Question # 28 is 5.

15
301
Online 2011 Anatomy-Imaging Self-Assessment Examination by Dr.Dhahirortho

29 29a 29b 29c 29d A 13-year-old boy has had a 12-month history of stiffness with worsening
right hindfoot and ankle pain. Examination reveals normal ankle motion but there is decreased
subtalar motion. Radiographs are shown in Figures 29a and 29b and MRI scans are shown in
Figures 29c and 29d. What is the most likely diagnosis?

1- Calcaneonavicular coalition
2- Talocalcaneal coalition
3- Osteochondroma of the talar head
4- Early inflammatory arthropathy
5- Talonavicular coalition

DISCUSSION: The patient's history and studies are consistent with a talocalcaneal coalition. The
lateral radiograph shows talar beaking and a positive "C" sign; however, the axial view does not
show the classic sloped medial facet that can be seen with a bony talocalcaneal coalition. The MRI
views are consistent with a fibrous coalition of the medial aspect of the posterior facet with
subchondral edema. There are no signs of any other coalitions. Whereas the talar beak is large and
the studies show the medullary canal is in continuity with the lesion, the other findings are more
consistent with a coalition than an osteochondroma, which would also tend to have an irregular
appearing surface. There is no periarticular osteopenia or hyperemia consistent with an
inflammatory arthropathy. The Preferred Response to Question # 29 is 2.

30 30a 30b A 22-year-old man sustained a buckling injury of the right knee while wake boarding.
Figure 30a shows a T1-weighted MRI scan of the knee, and Figure 30b shows an arthroscopic
view of the knee from an inferolateral viewing portal. What is the most likely diagnosis?

1- Midsubstance anterior cruciate ligament rupture


2- Tibial avulsion of the anterior cruciate ligament
3- Midsubstance posterior cruciate ligament rupture
4- Tibial avulsion of the posterior cruciate ligament
5- Displaced bucket handle meniscus tear

DISCUSSION: The MRI scan shows an avulsion of the


anterior cruciate ligament, with a small fragment of
bone that has been described by Meyers and McKeever in three different
fracture patterns. The arthroscopic view confirms a bony avulsion of the tibial spine and not a
midsubstance tear. Type I fractures are nondisplaced or have minimal displacement of the anterior
16
302
Online 2011 Anatomy-Imaging Self-Assessment Examination by Dr.Dhahirortho

margin. Type II fractures have superior displacement of their anterior aspect with an intact
posterior hinge. Type III fractures are completely displaced. Open or arthroscopic reduction and
internal fixation is recommended for type II and type III fractures that do not respond to closed
reduction. The images do not show injury to the posterior cruciate ligament or menisci. The
Preferred Response to Question # 30 is 2.

31 During a lateral approach to the left ankle of a 69-year-old woman with a displaced
bimalleolar fracture, the structure labeled with an arrow in Figure 31 is encountered. Which of
the following is an accurate statement concerning this structure?

1- Proximally, this nerve innervates the muscles of the anterior compartment.


2- Proximally, this nerve innervates the muscles of the lateral compartment.
3- This is strictly a sensory nerve to the lateral foot.
4- This nerve innervates the extensor digitorum brevis muscle.
5- This nerve supplies sensation to the dorsal aspect of the first interspace.

DISCUSSION: The structure shown is the superficial peroneal nerve, also known as the superficial
fibular nerve. It is a branch of the common peroneal nerve after it crosses the fibular head and
resides in the lateral compartment of the leg to supply the peroneus longus and brevis muscles. It
terminates as the intermediate and medial dorsal cutaneous nerves of the foot supplying the skin
of the dorsum of the foot and toes except for the first interspace
which is innervated by the deep peroneal nerve. It pierces the crural fascia approximately 10 cm to
12 cm proximal to the tip of the fibula and is at risk during the lateral approach to the ankle. The
sural nerve supplies the sensation to the lateral foot. The deep peroneal nerve innervates the
anterior compartment muscles and the extensor digitorum brevis and extensor hallucis brevis
muscles and supplies sensation to the dorsal first web space. The Preferred Response # 31 is 2.

32 32a 32b 32c 32d A 58-year-old woman has left knee pain. She states the pain is modest, but
there is some swelling and pain with increasing activity, and is alleviated with rest. She denies
any history of trauma, and denies any known history of arthritis. Examination is remarkable only
for some modest pain with full flexion. Figures 32a through 32d show the lateral radiograph,
sagittal T1-, T1-gadolinium, and T2-weighted MRI scans respectively. Based on the history and
the images, what is the most likely diagnosis?

17
303
Online 2011 Anatomy-Imaging Self-Assessment Examination by Dr.Dhahirortho

1- Infection
2- Osteoarthritis
3- Inflammatory arthritis
4- Pigmented villonodular synovitis
5- Synovial osteochondromatosis

DISCUSSION: The lateral radiograph is not particularly remarkable, except that it does not show
any arthritic or erosive changes. There are no mineralized changes seen in the joint. The T1-
weighted MRI scan shows low signal, lobular lesions that are in the posterior joint
and into the proximal tibia. These lesions show some minimal enhancement with gadolinium, and
are somewhat bright ("wet") on the T2-weighted image. This lobular, invasive appearance in the
posterior knee is most consistent with pigmented villonodular synovitis. The lack of mineral
density in the joint and the invasive nature of the demonstrated lesion into the bone make
synovial osteochondromatosis very unlikely. The images show no evidence of inflammatory,
infectious, or arthritic changes. The Preferred Response to Question # 32 is 4.

33 The MRI scan shown in Figure 33 reveals the sequelae of an acute traumatic anteroinferior
shoulder dislocation. The image reveals the typical separation of what two commonly injured
structures?
1- Anteroinferior labrum from the bony glenoid
2- Anteroinferior labrum from the cartilaginous surface of the glenoid
3- Biceps tendon from its origin on the supraglenoid tubercle
4- Anterior capsule from the proximal humerus
5- Posteroinferior labrum from the bony glenoid

DISCUSSION: The MRI scan reveals the sequelae of an anteroinferior


dislocation, specifically separation of the anteroinferior labrum from the bony glenoid. The
separation does not classically occur only at the cartilage-labral junction, but extends to the bony
surface of the medial glenoid neck. Separation of the biceps tendon from its origin on the
supraglenoid tubercle (SLAP lesion) or separation of the anterior capsule with the proximal
humerus (HAGL lesion) may occur but are not the most common sequelae and are not
demonstrated in this MRI image. Anteroinferior shoulder dislocations normally do not affect the
posterior labral structures. In their landmark study, Rowe and associates noted that this
demonstrated lesion was the most common lesion, present in 85% of their series. P R to # 33 is 1.

34 34a 34b 34c 34d 34e A 39-year-old man reports a 2-year history of increasing right anterior
ankle pain. He denies trauma, steroid use, or heavy drinking. He states that he has chronic pain
that worsens when he walks on irregular surfaces. Treatment consisting of two prior ankle
arthrotomies for the debridement of anterior loose bodies has not alleviated his symptoms.
Selective local injections show his symptoms are limited to the talonavicular and subtalar joints.
A lateral radiograph, CT scans, and MRI scans are shown in Figures 34a through 34e. What is the
most likely diagnosis?

18
304
Online 2011 Anatomy-Imaging Self-Assessment Examination by Dr.Dhahirortho

1-Advanced degenerative changes of late osteonecrosis of the talar head


2-Talar neck fracture with posttraumatic arthrosis
3-Pigmented villonodular synovitis
4-Chronic hematogenous osteomyelitis
5-Nonunion of an intra-articular talar head fracture

DISCUSSION: The patient has talonavicular and subtalar symptoms from advanced degenerative
changes secondary to chronic osteonecrosis of the talar head; this is a rare condition. The MRI
scans, with decreased intensity on T1 and increased intensity on the T2 with a dark serpiginous
line, are consistent with osteonecrosis of the talar head. The radiograph and CT scans show
significant degenerative changes of the talonavicular and subtalar joints. The studies shown are
not consistent with a talar neck or head fracture. The characteristic erosions and hemosiderin
deposition consistent with pigmented villonodular synovitis are not seen. There are no fluid
collections or bony destruction consistent with chronic osteomyelitis. The PR to Question # 34 is 1.

35 Normal thumb flexor tendon kinematics are restored by repairing which of the following
pulleys when the A-2 is intact?
1- Av-2
2- Av-1
3- Oblique or A-1
4- A-3
5- Palmar aponeurotic
DISCUSSION: When the A-2 pulley remains intact, dividing either the A-1 or the oblique pulley will
not alter thumb mechanical efficiency or joint angular displacement. If both the oblique pulley and
A-1 pulley are cut, significant bow stringing will occur. Studies showed that repair or
reconstruction of either the oblique pulley or the A-1 pulley after injury will restore thumb
kinematics as long as the A-2 pulley is intact. The Preferred Response to Question # 35 is 3.

36 36a 36b 36c An active 45-year-old man sustained an acute traumatic anteroinferior
dislocation. MRI scans and an arthroscopic view are shown in Figures 36a through 36c. The
lesion represents compressive injury to which of the following structures?
1- Greater tuberosity
2- Lesser tuberosity
3- Posterosuperior humeral head
4- Superior glenoid
5- Central portion of the humeral head
19
305
Online 2011 Anatomy-Imaging Self-Assessment Examination by Dr.Dhahirortho

DISCUSSION: During an anteroinferior dislocation, the posterosuperior portion of the humeral


head impacts the inferior rim of the glenoid, resulting in an impaction injury. This lesion is
classically referred to as a Hill-Sachs lesion. The Preferred Response to Question # 36 is 3.

37 .During the posterolateral approach to the hip, the sciatic nerve is most frequently identified
passing between which of the following structures?
1- Obturator internus and superior gemellus
2- Obturator internus and inferior gemellus
3- Piriformis and superior gemellus
4- Piriformis and gluteus minimus
5- Inferior gemellus and obturator externus

DISCUSSION: In most (> 80%) patients, the sciatic nerve lies anterior to the piriformis as it exits the
pelvis through the greater sciatic notch and then runs through the interval between the piriformis
and the superior gemellus to continue its course posterior to the remainder of the short external
rotators. Other variations include passing superior to or piercing the piriformis. The Preferred
Response to Question # 37 is 3.

38a Figure 38a shows the cross-sectional anatomy of the proximal thigh. What structure is
indicated by the arrow?
1- Adductor magnus
2- Adductor longus
3- Adductor brevis
4- Sartorius
5- Gracilis

DISCUSSION: In Figure 38b, the arrow marks the adductor longus,


which lies just deep to the superficial femoral artery. The
adductor magnus is the larger, more posterior muscle on this
cross-sectional image. The adductor brevis is located between the adductor longus and magnus,
deep to the deep femoral vessels. The sartorius is more superficial and covers the superficial
femoral vessels. The gracilis is medial and more superficial. The Preferred Response # 38 is 2.

39 .Extended exposure of the posteromedial aspect of the knee can be obtained using the
interval between the medial border of the gastrocnemius and the posterior border of the
semimembranosus tendon. Further exposure of the posteromedial tibial surface or the posterior
cruciate ligament (PCL) fossa requires dissection of what structure?

1- Popliteus
2- Plantaris
3- Semitendinosus
4- Gracilis
5- Soleus

20
306
Online 2011 Anatomy-Imaging Self-Assessment Examination by Dr.Dhahirortho

DISCUSSION: Further exposure of the tibial surface or PCL insertion requires subperiosteal
elevation of the popliteus muscle off the posterior tibia. The extended posteromedial approach of
the knee may be used for meniscal repair, open reduction and internal fixation of tibial plateau
fractures or PCL tibial avulsion fractures, PCL tibial inlay reconstruction, Baker's cyst excision, and
posterior capsular release. Superficially, the saphenous nerve and vein are at risk. The interval
between the medial border of the gastrocnemius and the posterior border of the
semimembranosus tendon allows adequate exposure of the posteromedial joint capsule for
inside-out meniscus repair. Care must be taken to avoid injury to branches of the popliteal artery,
and the inferior medial genicular artery frequently can be spared. The plantaris, gracilis, and
semitendinosus are superficial to this dissection. The soleus is well distal. The Preferred Response
to Question # 39 is 1.

40 A 15-year-old male football player reports chronic left foot and ankle pain. A CT scan is shown
in Figure 40. The arrow points to what structure?

1- Posterior malleolus
2- Accessory navicular
3- Tarsal coalition
4- Os trigonum fragment
5- Anterior loose body

DISCUSSION: The CT scan reveals an axial image of the left ankle, showing the talus, medial
malleolus, lateral malleolus, and a bony ossicle off of the posterior talus that is referred to as an os
trigonum. The os trigonum varies in size and shape and develops as a secondary ossification center
and may or may not fuse to the lateral tubercle of the talus. It may become symptomatic in
athletes who participate in sports with frequent hyper plantar flexion of the ankle. The posterior
malleolus is part of the distal tibia. The accessory navicular is a medial structure within the
posterior tibial tendon and is further distal in location. Tarsal coalitions are also distal to this level,
between the posterior tarsal bones. The arrow points to a posterior structure; thus, it could not be
considered an anterior loose body. The Preferred Response to Question # 40 is 4.

41 41a 41b 41c A 61-year-old man who reports left hip pain is seen in the emergency
department. Figure 41a shows a radiograph obtained at that time. Ten months later, he reports
excruciating left hip pain with ambulation. He notes that the pain has markedly worsened over
the past several weeks. Figures 41b and 41c show a current radiograph and a coronal inversion
recovery MRI scan of the pelvis. What is the most likely diagnosis?

1- Infection of the hip


2- Fracture of the hip
3- Osteoarthritis of the hip
4- Osteonecrosis of the hip
5- Rheumatoid arthritis of the hip

21
307
Online 2011 Anatomy-Imaging Self-Assessment Examination by Dr.Dhahirortho

DISCUSSION: The initial radiograph shows subtle flattening of the left femoral head, suggestive of
osteonecrosis but without significant subchondral sclerosis. Figure 41b shows marked collapse in
the left head over the intervening 10 months, and the MRI scan reveals collapse, significant edema
in the head, and low signal intensity in the superior segment, all suggestive of osteonecrosis. Note
that the right hip shows MRI changes, suggesting bilateral disease in this patient. The Preferred
Response to Question # 41 is 4.

42 42a 42b 42c A 37-year-old man reports a 6-month history of a slowly enlarging mass in the
right medial thigh that has recently become painful. He denies any history of trauma.
Examination reveals the lesion is firm and deep-seated, and moderately tender to palpation.
Figures 42a through 42c show a T1-weighted axial MRI scan, a gadolinium fat saturation axial
MRI scan, and a sagittal T2-weighted MRI scan, respectively. Based on the MRI characteristics,
what is the most likely diagnosis?

1- Synovial cell sarcoma


2- Extra-skeletal chondrosarcoma
3- High-grade spindle cell sarcoma
4- Liposarcoma
5- Rhabdomyosarcoma

DISCUSSION: The T1-weighted scan reveals a lesion that contains a large amount of fat signal
(bright), suggesting a fatty tumor of some kind. The gadolinium image similarly has large areas that
are dark from the fat saturation, which reinforces that the lesion contains fat, although there are
some areas of enhancement within the lesion that suggest more than just simple fat is present.
The sagittal T2-weighted image confirms the presence of edematous tissue within the mass,
suggesting that this lesion is most likely a liposarcoma, one of the most common soft-tissue
sarcomas. The other lesions listed do not have any distinguishing MRI characteristics. Synovial cell
sarcomas are often cystic in nature and do not contain significant amounts of fat. The Preferred
Response to Question # 42 is 4.

43 A 21-year-old man has had progressive knee pain and has been limping for the past several
weeks. Examination reveals decreased motion, pain, swelling, and marked tenderness over the
lateral aspect of the knee. An AP radiograph is shown in Figure 43. Given the radiographic
appearance of the lesion, what is the most likely diagnosis?
1- Fibrous dysplasia
2- Giant cell tumor of bone
3- Enchondroma
4- Osteoid osteoma
5- Osteochondroma
22
308
Online 2011 Anatomy-Imaging Self-Assessment Examination by Dr.Dhahirortho

DISCUSSION: The radiograph shows a lytic, eccentric, expansile lesion of the distal femur that
extends to the subchondral surface and has a pathologic fracture. There is no surrounding
sclerosis. This is a classic appearance of a giant cell tumor of bone, and the knee is the most
common presenting location. None of the other lesions listed have these same characteristics, and
typically do not appear in a subchondral location. Chondroblastoma (not a choice) may have a
similar appearance, and typically appears in younger patients with open physes. The Preferred
Response to Question # 43 is 2.

44 Which of the following constitutes a positive intrinsic tightness test?


1- Decreased proximal interphalangeal joint flexion with extension of the metacarpophalangeal
joint
2- Decreased proximal interphalangeal joint flexion with flexion of the metacarpophalangeal
joint
3- Normal proximal interphalangeal joint flexion with flexion of the metacarpophalangeal joint
4- Normal proximal interphalangeal joint flexion with extension of the metacarpophalangeal
joint
5- Increased proximal interphalangeal joint flexion with extension of the metacarpophalangeal
joint

DISCUSSION: Extension of the metacarpophalangeal joint places tension on the intrinsic


contribution to the extensor system via the lateral bands. Contracture of the intrinsics decreases
the flexion at the proximal interphalangeal joint with the metacarpophalangeal joint placed in
extension past neutral. The Preferred Response to Question # 44 is 1.

45 45a 45b An otherwise healthy 68-year-old man has thoracic pain with radiation along his
chest wall. His pain began a few weeks ago and is constant. He denies any neurologic symptoms.
AP and lateral thoracic radiographs are shown in Figures 45a and 45b. What is the next most
appropriate step in management?

1- Order physical therapy and schedule follow-up to assess


response
2- Refer to a pain clinic for intercostal nerve blocks
3- Obtain complete blood count, erythrocyte
sedimentation rate, and C-reactive protein for suspicion of
infection
4- Provide reassurance and schedule follow-up if the pain
persists beyond 6 weeks
5- Obtain an MRI scan with gadolinium of the thoracic
spine

DISCUSSION: The next most appropriate step in management should be to obtain an MRI scan
because the AP image has a missing pedicle on the left side in the upper thoracic spine; this is
known as a "winking owl" sign and is the result of pedicle destruction from neoplastic disease,

23
309
Online 2011 Anatomy-Imaging Self-Assessment Examination by Dr.Dhahirortho

most commonly metastatic in this age group. Although physical therapy and observation can be
the initial management for a few weeks of pain, this radiographic finding warrants immediate
further imaging. Infection more commonly destroys the disk and works its way into the vertebral
body; in this patient the disks are well preserved as seen in Figure 45c. The Preferred Response to
Question # 45 is 5.

46 A 17-year-old boy is shot in the left side of the neck at the C5-6 level and sustains an
incomplete spinal cord injury that is called a Brown-Sequard syndrome. Which of the following
best describes the expected deficits?

1- Profound bilateral wrist extensor, finger flexor, and intrinsic weakness with good preservation
of lower extremity motor function
2- Severe bilateral upper and lower extremity weakness, pain and temperature sensory deficit
but preservation of deep pressure and proprioception
3- Weakness of the right upper and lower extremity with diminished pain and temperature
sensation on the left side of the body
4- Left wrist extensor weakness and numbness along the radial border of the left forearm
extending into the thumb and index finger
5- Weakness of the left upper and lower extremity with diminished pain and temperature
sensation on the right side of the body

DISCUSSION: Brown-Sequard syndrome is an incomplete spinal cord injury that involves damage
unilaterally to the cord, most commonly from penetrating trauma. The motors fibers of the cord
decussate within the brainstem so the motor deficit is ipsilateral to the injury; whereas, the pain
and temperature fibers cross midline immediately on entering the cord so that the sensory deficit
is contralateral to the injury. This patient was shot in the left side, thus he would have weakness of
the left upper and lower extremity with diminished pain and temperature sensation on the right
side of the body. Response 3 describes
opposite symptoms that would result from a right-sided injury. Response 1 describes a central
syndrome with greater upper than lower extremity involvement. Response 2 is an anterior cord
syndrome with only preservation of the posterior columns of the cord. Response 4 describes a C6
root injury. The Preferred Response to Question # 46 is 5.

47 A digastric (flip) trochanteric osteotomy is performed for hip exposure to perform a surgical
dislocation of the hip. Where should the posterosuperior aspect of the osteotomy exit the
femoral neck?
1- Anterior to the posterior insertion of the gluteus medius
2- Between the piriformis and gluteus medius
3- Between the piriformis and gluteus minimus
4- Between the piriformis and superior gemellus
5- Between the superior gemellus and obturator internus

DISCUSSION: The technique for the trochanteric osteotomy during surgical dislocation procedures
should be performed with a maximal thickness of 1.5 cm. The interval between the gluteus medius

24
310
Online 2011 Anatomy-Imaging Self-Assessment Examination by Dr.Dhahirortho

and piriformis is not developed prior to the osteotomy. At the most proximal extent of the
osteotomy, the saw should exit anterior to the posterior insertion of the gluteus medius to ensure
the osteotomy does not inadvertently penetrate into the short external rotators, thus preserving
the deep branch of the medial femoral circumflex artery. Preferred Response to Question # 47 is 1.

48 An 18-year-old man sustains a twisting injury to the left knee while playing football. An MRI
scan is shown in Figure 48. What is the most likely diagnosis?

1- Anterior cruciate ligament rupture


2- Posterior cruciate ligament rupture
3- Medial meniscus tear
4- Lateral meniscus tear
5- Osteochondral lesion

DISCUSSION: The MRI scan shows a displaced, bucket-handle


lateral meniscus tear. The sagittal view shows the typical
"large anterior horn" sign, or "double meniscus" sign in which the displaced bucket-handle
fragment appears just anterior to the native anterior horn of the lateral meniscus. The presence of
the fibula on the sagittal view confirms this as the lateral compartment. The image is lateral and
the cruciate ligaments are not visualized. The articular cartilage shown does not demonstrate an
osteochondral lesion Preferred Response # 48 is 4.

49 49a 49b An 85-year-old man reports diffuse pelvic and back pain that has progressed over the
past 6 months. He also notes that he is chronically fatigued and is unable to get comfortable in
any position. Figures 49a and 49b show a bone scan and a pelvic CT scan. On the basis of the
history and the appearance of the studies, what is the most likely diagnosis??

1- Metastatic renal cell carcinoma


2- Multiple myeloma
3- Metastatic thyroid carcinoma
4- Metastatic prostate carcinoma
5- Metastatic lung carcinoma

DISCUSSION: The bone scan shows multiple areas of


bony activity, which would suggest metastatic disease or multiple myeloma in this age group.
(Myeloma may not be hot on bone scan, but certainly can present on a bone scan in this fashion).
The CT scan reveals multiple blastic lesions in the bone, which is typical of metastatic prostate
cancer. Myeloma, renal carcinoma, and thyroid carcinoma would present with lytic lesions. Lung
metastases are more commonly lytic lesions, presenting as blastic lesions approximately 25% to
30% of the time.
The Preferred Response to Question # 49 is 4.

25
311
Online 2011 Anatomy-Imaging Self-Assessment Examination by Dr.Dhahirortho

50 When planning the incision for an anterior approach to the cervical spine, what external
landmark is easily palpable that would correspond most closely to the C6 vertebral level?
1- Upper border of the thyroid cartilage
2- Cricoid cartilage
3- Hyoid bone
4- Lower border of the thyroid cartilage
5- Carotid tubercle

DISCUSSION: The most reliable palpable external landmark for C6 is the cricoid cartilage. The
carotid tubercle also corresponds to the C6 level but is not always palpable externally and is
generally used as an internal landmark once the dissection has begun. The hyoid aligns with C3.
The upper border of the thyroid cartilage identifies C4, and the lower border identifies C5. Some
recent evidence includes the use of the angle of the mandible as a reliable landmark, but this has
not been widely adopted. The Preferred Response to Question # 50 is 2.

51 During an anterior approach to the shoulder, what is the most likely arterial structure to be
encountered in the superior extent of the deltopectoral interval (just distal to the anterior edge
of the clavicle)?

1- Acromial branch of the thoracoacromial artery


2- Axillary artery
3- Arcuate artery
4- Suprascapular artery
5- Subclavian artery

DISCUSSION: The acromial branch of the thoracoacromial artery sits in the proximal interval
between the anterior deltoid and the pectoralis major and is likely to be encountered when
the proximal plane between these two muscles is dissected to the anterior edge of the clavicle.
The axillary artery runs inferior to the humeral head. The arcuate artery runs in the intertubercular
groove. The suprascapular artery runs superior to the clavicle and deep to the trapezius. The
subclavian artery is medial to the coracoid and should not be encountered in the deltopectoral
interval. It is notable that the acromial branch of the thoracoacromial artery is responsible for the
bleeding encountered in release of the coracoacromial ligament.
The Preferred Response to Question # 51 is 1.

52a 52b 52c Figures 52a through 52c show the axial, coronal, and sagittal T2-weighted MRI scans
respectively of a knee. The highlighted structure represents what anatomic finding?

1- Lateral meniscus tear


2- Medial meniscus tear
3- Anterior meniscofemoral ligament
4- Ligamentum mucosum (infrapatellar plica)
5- Transverse meniscal ligament

26
312
Online 2011 Anatomy-Imaging Self-Assessment Examination by Dr.Dhahirortho

DISCUSSION: The images show a transverse meniscal ligament, which connects the anterior horns
of the medial and lateral menisci. On sagittal images, the interface of this structure with the
anterior horn of the lateral meniscus often simulates a tear. Following this structure over several
successive images is helpful in identifying it as a normal structure. There is no abnormal signal
within the menisci to suggest a tear. A meniscofemoral ligament is a posterior structure. A
ligamentum mucosum or infrapatellar plica is best seen on the sagittal image and runs from the
intercondylar notch to the anterior fat pad. The Preferred Response to Question # 52 is 5.

53 Which of the following best characterizes the injury shown in Figure 53?

1- Stable tear drop extension injury


2- Facet dislocation
3- Unstable flexion compression injury
4- Stable axial load injury
5- Burst fracture

DISCUSSION: The injury shown is a flexion compression injury also known as


"tear drop" fracture. It is characterized by the large anteroinferior fragment
off the vertebral body and the retrolisthesis seen in this image. It is considered an unstable injury
and should be distinguished from the more stable and minor extension tear drop avulsion where
there is no vertebral malalignment and the anteroinferior fracture is a small avulsion of the
annulus attachment. Other axial load injuries can be stable but have more of a compression or
even burst pattern with loss of body height rather than the anteroinferior fragment. The
radiograph does not demonstrate facet malalignment that would be seen with a facet dislocation.
The Preferred Response to Question # 53 is 3.

54a 54b A 28-year-old man reports an episode of buckling and giving-way of his right knee.
Figure 54a and 54b show a radiograph and sagittal MRI scan. What is the most likely diagnosis?

1- Anterior cruciate ligament rupture


2- Posterior cruciate ligament rupture
3- Quadriceps tendon rupture
4- Patellar tendon rupture
5- Displaced bucket-handle meniscus tear

DISCUSSION: The radiograph shows patella alta consistent with a rupture of the patellar tendon.
The MRI scan confirms disruption of the patellar tendon from the inferior pole of the patella. The
cruciate ligaments are not visualized in this image, and would not result in patella alta. Quadriceps
tendon rupture would result in patella baja. There is no evidence of meniscal tearing on these
images. The Preferred Response to Question # 54 is 4.

27
313
Online 2011 Anatomy-Imaging Self-Assessment Examination by Dr.Dhahirortho

55 Which of the following arteries is the pedicle supply to the lateral arm flap?

1- Radial recurrent
2- Profunda brachii
3- Interosseous recurrent
4- Anterior radial collateral
5- Posterior radial collateral

DISCUSSION: The posterior radial collateral artery provides the vascular supply to the lateral arm
flap. The radial collateral artery travels with the radial nerve in the spiral groove until both
penetrate the lateral intermuscular septum. It then divides into the anterior and posterior radial
collateral arteries. The posterior branch passes posterior to the lateral intermuscular septum.

The Preferred Response to Question # 55 is 5.

56a 56b A 29-year-old woman has had a 6-month history of chronic left anterolateral ankle pain
after sustaining an inversion ankle sprain while playing soccer. Management consisting of rest,
nonsteroidal anti-inflammatory drugs, immobilization, a cortisone injection, and 2 months of
physical therapy has failed to allow her to return to her previous level of activities. Examination
reveals good strength, motion, and ligamentous stability, with anterolateral ankle tenderness.
Radiographs are normal. During an anterolateral approach to the left ankle, the structure
labeled with the arrow in Figure 56a is noted to be impinging on the anterolateral dome of the
talus and is removed as shown in Figure 56b. Removal of this structure will most likely result in
which of the following?

1- Alleviation of her symptoms


2- Destabilization of the syndesmosis
3- Increase the anterior drawer but not influence the
talar tilt
4- Increase the talar tilt but not influence the anterior drawer
5- Have no effect on her symptoms or her ankle instability

DISCUSSION:The structure shown is the inferior portion of the anteroinferior tibiofibular ligament,
often referred to as "Bassett's ligament." It was described by Bassett and associates in 1990 as a
separate distal fascicle of the anteroinferior tibiofibular ligament that is present in most human
ankles and can be a cause of talar impingement, abrasion of the articular cartilage, and pain in the
anterior aspect of the ankle. In their series, an inversion injury to the ankle was followed by
chronic anterior ankle pain. The thickened distal fascicle was resected without loss of stability of
the ankle and all symptoms were eliminated or markedly improved after resection of the fascicle.

The Preferred Response to Question # 56 is 1.

28
314
Online 2011 Anatomy-Imaging Self-Assessment Examination by Dr.Dhahirortho

57 Which of the following radiographic parameters is most predictive of a poor result following
multilevel fusion surgery for adult degenerative scoliosis?
1- An L5-S1 degenerative disk left out of the fusion
2- Coronal imbalance
3- Residual scoliosis of greater than 25 degrees
4- Residual foraminal stenosis
5- Sagittal imbalance

DISCUSSION: Sagittal imbalance appears to be the greatest predictor of a poor surgical outcome in
multilevel fusions for adult scoliosis. Coronal imbalance is better tolerated as long as it is not
excessive. The amount of residual scoliosis does not seem to play a role as
long as overall balance is achieved. The issue of including the L5-S1 level in long fusions remains
debatable, and some residual foraminal stenosis can be tolerated, particularly when included
within the stabilized/fused segments. The Preferred Response to Question # 57 is 5.

58 Following a posterior approach to the radius (dorsal Thompson), the patient is unable to
extend his thumb and index finger at the metacarpophalangeal joint. He has sensation to the
radial forearm and dorsal thumb and can extend his wrist but with radial deviation. What nerve
was injured?
1- Radial
2- Posterior interosseous
3- Anterior interosseous
4- Median
5- Musculocutaneous

DISCUSSION: During a posterior approach (dorsal Thompson) to the radius, the posterior
interosseous nerve (PIN) should be identified and/or protected. Pronation of the forearm will aid
in protection of the PIN. The radial nerve splits into the PIN and the superficial branch of the radial
nerve (SBRN) proximal to the extent of this approach. Preservation of sensation in the distribution
of the SBRN and intact wrist extension with radial deviation locates the injury distal to the
SBRN/PIN split (extensor carpi radialis brevis palsy with intact extensor carpi radialis longus). The
median nerve and its branch and the anterior interosseous nerve are not encountered in this
approach. The musculocutaneous nerve is not observed during this approach. The Preferred
Response to Question # 58 is 2.

59 During a posterolateral exposure of the knee, the fascial intervals between the iliotibial band
and the biceps femoris tendon are incised. What vascular structure is at most risk during this
exposure?
1- Peroneal artery
2- Lateral sural artery
3- Superior lateral genicular artery
4- Inferior lateral genicular artery
5- Posterior tibial recurrent artery

29
315
Online 2011 Anatomy-Imaging Self-Assessment Examination by Dr.Dhahirortho

DISCUSSION: Exposure of the posterolateral aspect of the knee uses the fascial intervals between
the iliotibial band and the biceps femoris tendon distally and the short head of the biceps femoris
slightly more proximally. The inferior lateral genicular artery may be encountered during this
surgical approach or with aggressive arthroscopic meniscal debridement that penetrates the joint
capsule. The superior lateral genicular artery is found well above this interval. The lateral sural
artery is superior and posterior. The peroneal and posterior tibial recurrent arteries are well distal.
The Preferred Response to Question # 59 is 4.

60 A boutonniere deformity is treated with distal extensor tenotomy. What structures allow for
active extension at the distal interphalangeal (DIP) joint after tenotomy?
1- Lateral bands
2- Sagittal bands
3- Central slip
4- Oblique retinacular ligament
5- A healed but lengthened terminal extensor tendon

DISCUSSION: Hyperextension of the DIP joint from a boutonniere deformity can be treated by the
Dolphin tenotomy that divides the terminal extensor mechanism. Near normal extension of the
DIP joint is the result of the intact oblique retinacular ligament of Landsmeer. Lateral bands are at
the level of the proximal interphalangeal (PIP) joint. Sagittal bands are at the level of the
metacarpophalangeal joint and are responsible for maintaining centralization of the extensor
tendon at that level. The central slip extends the PIP joint. The corrective effect is immediate and
is not determined by a healed extensor tendon at that level. The Preferred Response # 60 is 4.

61 61a Figure 61a shows the cross-sectional anatomy of the pelvis at the level of the femoral
heads. What structure is marked by the arrow?
1- Rectus femoris
2- Sartorius
3- Iliacus
4- Obturator externus
5- Tensor fascia lata
DISCUSSION: In Figure 61b, the structure marked is the iliacus muscle, which joins with the psoas
(the psoas tendon is immediately medial to the muscle) to form the iliopsoas that will then insert
on the lesser trochanter. The sartorius is more superficial and anterior. The rectus femoris is just
anterior and slightly lateral to the iliacus. The tensor is more lateral and superficial. The obturator
externus is medial and deep to the pectineus. The Preferred Response to Question # 61 is 3.

62.A 41-year-old man has a severe posttraumatic elbow contracture. The surgeon chooses to
approach laterally. This exposure to the anterior elbow capsule exploits what anatomic interval?
1- Anconeus and anterior surface of the humerus
2- Brachioradialis and extensor carpi radialis longus
3- Brachioradialis and brachialis
4- Extensor carpi radialis longus and extensor carpi radialis brevis
5- Brachioradialis/extensor carpi radialis longus and anterior surface of the humerus

30
316
Online 2011 Anatomy-Imaging Self-Assessment Examination by Dr.Dhahirortho

DISCUSSION: A modified Kocher incision is used to approach the lateral elbow. Skin flaps are then
elevated and the anterior capsule exposed by elevating the brachioradialis and extensor carpi
radialis longus off the anterior supracondylar ridge. Further medial exposure is achieved by
elevating the brachialis anteriorly. The Preferred Response to Question # 62 is 5.

63 An 83-year-old patient has had a 6-month history of right groin pain. History reveals that hip
arthroplasty was performed 14 years ago. An AP radiograph is shown in Figure 63. Preoperative
evaluation reveals no evidence of infection. Which of the following studies is the next most
appropriate step in evaluation?
1- Technitium-99 bone scan
2- MRI scan 3- Full-length radiographs of the femur
4- Judet radiographs
5- Dynamic fluoroscopy

DISCUSSION: The patient has a single radiograph for interpretation. To


thoroughly evaluate the symptom of groin pain, additional radiographic
views are appropriate. Judet views will show the integrity of the posterior column. Cross table
lateral radiographs do not provide adequate visualization of the acetabulum for osteolysis. CT
would most definitively show the extent of osteolyis, but it is not one of the options. MRI would
have too much metal artifact. A bone scan could suggest loosening, but initial radiographic studies
are more appropriate to obtain first in assessment. Dynamic fluoroscopy is not normally used in
the assessment of implant failure. The Preferred Response to Question # 63 is 4.

64a 64b 64c A 57-year-old man is evaluated for what he reports as a lifetime of chronic left ankle
pain and deformity. He is a community ambulatory and walks with a cane. Radiographs are
shown in Figures 64a through 64c. What is the most likely cause of his condition?

1- Chronic ankle instability from polio at age 12 years


2- A bacterial infection at age 8 years
3- Gouty arthritis
4- An untreated talus fracture
5- A significant neonatal talocalcaneal coalition

DISCUSSION: The patient has a ball-and-socket ankle that is a result of a congenital or early
developmental deformity such as a significant subtalar coalition. It is the result of significant early
31
317
Online 2011 Anatomy-Imaging Self-Assessment Examination by Dr.Dhahirortho

abnormal biomechanics of the subtalar joint or transverse tarsal joint that cause increased stress
on the developing ankle and induce secondary changes. Because neonates have very little
ossification around the ankle, a congenital ball-and-socket ankle is not readily diagnosed until the
child is much older, making the differentiation between congenital and developmental somewhat
difficult. It usually develops by about age 5 years. Conditions that present later in life, such as polio
at age 12 years or a bacterial infection, are not likely to produce a ball-and-socket ankle. Most
tarsal coalitions do not present until later in life and usually only produce secondary changes like
talar beaking. Significant neonatal infections cause more bone and articular destruction. Gout
presents too late to cause a developmental deformity. An untreated talus fracture will cause
hindfoot arthrosis and deformities. The Preferred Response to Question # 64 is 5.

65 A 34-year-old woman who underwent release of her first dorsal compartment at the wrist for
de Quervain's tenosynovitis 3 months ago continues to report radial-sided wrist pain and
tenderness similar to what she had prior to surgery. Examination appears classic for de
Quervain's with a positive Finkelstein's test and continued pain with palpation over the first
dorsal compartment. What is the likely source of her continued pain?
1- Tendon subluxation
2- Intersection syndrome
3- Injury to the dorsal radial sensory nerve
4- Tendon injury to the abductor pollicis longus (APL) tendon
5- Unreleased extensor pollicis brevis (EPB) tendon

DISCUSSION: Persistant pain after first dorsal compartment release is often the result of failure to
release all potential septations or compartments. It has been found that 24% to 34% of wrists have
a separate compartment involving the EPB or APL. If each tendon is not identified, an incomplete
release can result, causing continued symptoms. Intersection syndrome is more proximal, pain is
not over the first dorsal compartment. Radial sensory nerve injury would not result in a positive
Finkelstein's test. Tendon subluxation and tendon injury usually do not cause pain over the first
dorsal compartment with palpation. The Preferred Response to Question # 65 is 5.

66 When using an anterior exposure of the acetabulum during minimally invasive two-incision
total hip arthroplasty, the deep approach to the acetabulum is accomplished through the
interval between which of the following structures?
1- Gluteus medius and gluteus minimus
2- Gluteus medius and tensor fascia lata
3- Gluteus medius and gluteus maximus
4- Rectus femoris and iliopsoas
5- Rectus femoris and tensor fascia lata

DISCUSSION: During a two-incision minimally invasive surgical approach, the inferior aspect of the
classic anterior (Smith Peterson) interval is used. The deep interval is between the tensor fascia
lata and the rectus femoris. The superficial interval is between the tensor fascia lata and the

32
318
Online 2011 Anatomy-Imaging Self-Assessment Examination by Dr.Dhahirortho

sartorius. The gluteus medius is proximal to the intervals used for the anterior approach in a two-
incision technique. The Preferred Response to Question # 66 is 5.

67 67a 67b 67c 67d A 20-year-old woman has progressive severe heel pain, swelling, and
difficulty with shoe wear. A mass has been present for several weeks, and the pain awakens her
from sleep and requires narcotics for symptomatic control. Figures 67a through 67d show the
lateral radiograph, CT scan, and coronal T1- and T2-weighted MRI scans, respectively. What is
the most likely diagnosis?

1- Enchondroma
2- Giant cell tumor
3- Osteosarcoma
4- Metastatic carcinoma
5- Osteoblastoma

DISCUSSION: The radiograph reveals a blastic-appearing lesion within the body of the calcaneus.
The CT scan confirms the presence of a blastic lesion within the bone, and shows extension into
the soft tissues with mineral density (bone formation) in the lateral aspect of the heel. The MRI
scans confirm that the lesion extends outside the bone with a lobular-appearing soft-tissue mass
with low T1-weighted and intermediate T2-weighted signal, both of which show the low signal
intensity associated with bone formation. This is most characteristic of a bone-forming lesion that
is behaving in an aggressive fashion, and represents an osteosarcoma of the calcaneus. Metastatic
carcinoma is highly unlikely in this age and location, and would not generally present with mineral
density in the soft tissue. Giant cell tumor, while it may extend outside the bone, is not a blastic
lesion. Osteoblastoma, while blastic and expansile, does not generally present with soft-tissue
invasion. This lesion does not have the appearance of an enchondroma, which would be contained
within the bone (no soft-tissue extension) and demonstrate "rings and arcs" mineral density on
imaging. The Preferred Response to Question # 67 is 3.

68 Figure 68 shows the view from a posterosuperior shoulder arthroscopic portal. The muscle
associated with the tendinous structure shown is innervated by what nerve?

1- Axillary 2- Median 3- Musculocutaneous 4- Radial


5- Ulnar
33
319
Online 2011 Anatomy-Imaging Self-Assessment Examination by Dr.Dhahirortho

DISCUSSION: The structure shown in the arthroscopic image is the tendon of the long head of the
biceps, originating from the supraglenoid tubercle. The biceps brachii muscle is innervated by the
musculocutaneous nerve. The long head of the biceps brachii is not innervated by the axillary,
median, radial, or ulnar nerves.

The Preferred Response to Question # 68 is 3.

69 During an anterior approach to the hip, what structure has the greatest potential for injury?

1- Femoral nerve
2- Femoral artery
3- Femoral vein
4- Lateral femoral cutaneous nerve
5- Medial femoral circumflex artery

DISCUSSION: The anterior (Smith-Peterson) approach to the hip develops the superficial interval
between the tensor fascia lata (TFL) and sartorius and the deep interval between the gluteus
medius and rectus femoris. The lateral femoral cutaneous nerve penetrates the fascia overlying
the interval between the TFL and sartorius approximately 1 cm distal to the anterior superior iliac
spine. Identifying the interval between the TFL and sartorius distally can be helpful in preventing
injury to the lateral femoral cutaneous nerve, which is the structure at greatest risk for injury
during an anterior approach to the hip. The femoral artery, vein, and nerve are medial to the
approach. The medial femoral circumflex artery runs posterior to the femoral neck. The ascending
branch of the lateral femoral circumflex artery is routinely encountered during this approach, but
is not one of the options.
The Preferred Response to Question # 69 is 4.

70 Figure 70 shows the arthroscopic view of a right knee from an inferolateral viewing portal.
The probe is touching what anatomic structure?

1- Posterior cruciate ligament, anterolateral bundle


2- Posterior cruciate ligament, posterolateral bundle
3- Posterior cruciate ligament, anteromedial bundle
4- Posterior cruciate ligament, posteromedial bundle
5- Posterior meniscofemoral ligament

DISCUSSION: The posterior cruciate ligament consists of two functional bundles: anterolateral and
posteromedial. The probe is in contact with the anterolateral bundle, which becomes tighter in
knee flexion. Knowledge of this anatomic and functional difference from the posteromedial bundle
is important when considering anatomic reconstruction of the posterior cruciate ligament. The
posterior meniscofemoral ligament connects the posterior horn of the lateral meniscus to the
intercondylar wall of the medial femoral condyle and cannot be visualized because it is posterior
to the structures shown. The Preferred Response to Question # 70 is 1.

34
320
Online 2011 Anatomy-Imaging Self-Assessment Examination by Dr.Dhahirortho

71 After completion of bone preparation during a total knee arthroplasty, the lateral
compartment is tight in both flexion and extension. At what point during the release is the
peroneal nerve at greatest risk for injury?
1- Release of the posterior capsule
2- Release of the posterolateral capsule with the knee extended
3- Release of the posterolateral capsule with the knee flexed
4- Release of the iliotibial band with the knee extended
5- Release of the iliotibial band with the knee flexed

DISCUSSION: The peroneal nerve traverses the proximal aspect of the knee joint in the interval
between the biceps femoris and lateral gastrocnemius. The lateral gastrocnemius muscle provides
some protection for the peroneal nerve. Cadaveric studies have suggested that the peroneal nerve
can be as close as 7 mm to 9 mm from the posterolateral corner with the knee in extension, where
it is at greatest risk for injury. The iliotibial band is anterior to the course of the peroneal nerve at
the joint line. The Preferred Response to Question # 71 is 2.

72 The radial forearm free flap has a vascular pedicle that passes between which of the
following muscles?
1- Flexor carpi radialis and brachioradialis
2- Flexor carpi radialis and pronator teres
3- Brachioradialis and pronator teres
4- Brachioradialis and abductor pollicis longus
5- Brachioradialis and palmaris longus

DISCUSSION: The radial artery is the pedicle for this free flap. The artery bifurcates from the
brachial artery and exits between the muscle bellies of the flexor carpi radialis and brachioradialis,
after which it courses superficial to the flexor digitorum superficialis muscle. The Preferred
Response to Question # 72 is 1.

73 73a 73b Figures 73a and 73b show the AP and lateral radiographs of the left humerus of a 19-
year-old woman with an incidental finding of this lesion on a chest radiograph. The patient
denies any pain or loss of function in the arm, and is fully active with no restrictions. Based on
the radiographic appearance of this lesion, what is the most likely diagnosis?

1- Chondrosarcoma
2- Enchondroma
3- Fibrous dysplasia
4- Unicameral bone cyst
5- Bone infarct

DISCUSSION: The radiographs demonstrate a


metaphyseal/diaphyseal lesion of the proximal
humerus that is centrally located, slightly expansile,
and with prominent cortical thinning. The lesion is

35
321
Online 2011 Anatomy-Imaging Self-Assessment Examination by Dr.Dhahirortho

sharply demarcated from the remaining normal-appearing bone. There is a "ground glass"
appearance to the lesion, and the lesion appears to fill the long bone over the affected length. This
is a common presentation for fibrous dysplasia. Chondrosarcoma would
be uncommon in this age, and would be more destructive, with "rings and arcs" mineralization.
Enchondroma, similarly would have "rings and arcs" mineralization and is not expansile nor would
it demonstrate such significant endosteal scalloping over a long area of the bone. Unicameral bone
cyst, while possibly of this size, would not have the same "ground glass" appearance and would
have more rounded edges at the margin. Bone infarcts are central, have a whispy, "smoke up the
chimney" mineralization, and are not expansile.
The Preferred Response to Question # 73 is 3.

74 A 20-year-old man sustains a burst fracture at L1. Examination reveals 3/5 weakness of
bilateral ankle plantar flexion and dorsiflexion, and 4/5 quadriceps strength. He is unable to void
spontaneously and has diminished rectal tone. Which of the following would best describe the
neurologic deficit?

1- Conus medullaris injury


2- Incomplete spinal cord injury
3- Cauda equina injury
4- Central cord syndrome
5- Anterior cord syndrome

DISCUSSION: The tip of the spinal cord usually ends at the L1-2 disk level, thus a neurologic injury
from fracture at the L1 level would damage the conus medullaris and have a mixed spinal cord and
nerve root (upper and lower neuron) picture as in this patient. Cauda equina injury would be
present at L2 or lower, and spinal cord level injury typically above T12.
The Preferred Response Question # 74 is 1.

75 The peroneal division of the sciatic nerve innervates which of the following muscles in the
thigh?

1- Long head of the biceps femoris


2- Short head of the biceps femoris
3- Semimembranosus
4- Semitendinosus
5- Lateral head of the gastrocnemius

DISCUSSION: The tibial division of the sciatic nerve provides innervations to all of the hamstring
muscles in the thigh with the exception of the short head of the biceps femoris which receives its
innervations from the common peroneal branch of the sciatic nerve. Both heads of the
gastrocnemius muscle are innervated by the tibial division of the sciatic nerve. The Preferred
Response to Question # 75 is 2.

36
322
Online 2011 Anatomy-Imaging Self-Assessment Examination by Dr.Dhahirortho

76a 76b What anatomic structure is marked with an asterisk in Figure 76a in the posterior
arthroscopic view of a left shoulder subacromial space and with the arrow in the sagittal oblique
MRI scan in Figure 76b?

1- Coracoacromial ligament
2- Coracohumeral ligament
3- Conoid ligament
4- Trapezoid ligament
5- Acromioclavicular ligament

DISCUSSION: The coracoacromial ligament extends from the coracoid process inferiorly to the
acromion superiorly. It forms a portion of the osseous outlet, and thickening, hypertrophy, or
calcification of the ligament may result in impingement of the anterior portion of the rotator cuff.
The coracohumeral ligament originates from the base and lateral border of the coracoid and
inserts on the greater tuberosity. The conoid and trapezoid ligaments form the coracoclavicular
ligaments. The acromioclavicular ligament travels from the acromion to the clavicle.P R # 76 is 1.

77 A 58-year-old man has had groin pain for the past 3 months. The patient reports pain with
ambulation and at rest. Examination reveals an antalgic gait and range of motion is mildly
restricted. He denies any history of trauma, or steroid or alcohol abuse. Radiographs are normal.
An MRI scan is shown in Figure 77. What is the most appropriate management?
1- Protected weight bearing and anti-inflammatory drugs
2- Total hip arthroplasty
3- Intraosseous steroid injection
4- A vascularized fibula graft to the femoral head
5- Core decompression of the femoral head

The patient has transient osteoporosis of the hip. The MRI findings
show highly increased signal through the entire femoral head and neck that is diagnostic of
transient osteoporosis of the femoral head.á This entity is usually a self-limited condition that is
most frequently seen in women in the third trimester of pregnancy and in men in the sixth decade
of life.á Transient osteoporosis is best treated nonsurgically with protected weight bearing and
anti-inflammatory drugs. The natural history is that of self-resolution. A vascularized fibula graft to
the femoral head and core decompression of the femoral head each have a described role in
treating osteonecrosis (not transient osteoporosis) depending on the stagingáof the disease. Total
hip arthroplasty indications include end-stage osteonecrosis of the hip as well as osteoarthritis.
Steroid injections are generally reserved for simple cysts of bone. The Preferred Respon# 77 is 1.

78 What osseous ridge separates the femoral attachments of the anteromedial and
posterolateral bundles of the anterior cruciate ligament?
1- Lateral intercondylar ridge
2- Lateral bifurcate ridge
3- Lateral interfemoral ridge
4- Lateral interfascicular ridge 5- Lateral cruciate ridge

37
323
Online 2011 Anatomy-Imaging Self-Assessment Examination by Dr.Dhahirortho

DISCUSSION: The anterior cruciate ligament is composed of the anteromedial and posterolateral
bundles. The lateral bifurcate ridge is nearly perpendicular to the lateral intercondylar ridge and
separates the anteromedial and posterolateral bundles from one another. The femoral insertion of
the anterior cruciate ligament does not extend beyond the lateral intercondylar ridge (or
resident's ridge) anteriorly (or superiorly with the knee in 90
degrees of flexion). The lateral interfemoral, interfascicular, or cruciate ridges are not accepted
nomenclature. The Preferred Response to Question # 78 is 2.

79a Figure 79a shows the cross-section image of the mid thigh. What structure is marked by the
arrow?
1- Biceps femoris
2- Adductor magnus
3- Semitendinosus
4- Gracilis
5- Semimembranosus

DISCUSSION: In Figure 79b the arrow marks the semimembranosus muscle, which is more medial
than the semitendinosus muscle at this level, which is seen more lateral. The biceps femoris is
more lateral still, and the adductor magnus is medial and deep to the marked semimembranosus.
The gracilis is superficial to the adductor magnus muscle and is the most medial structure. The
Preferred Response to Question # 79 is 5.

80 80a 80b 80c 80d 80e Figure 80a shows an arthroscopic view from an infralateral portal of a
right knee. Figure 80b shows a coronal MRI scan, and Figures 80c through 80e show consecutive
sagittal images of the knee. The images show what anatomic finding?

1- Loose body
2- Discoid lateral meniscus
3- Transverse meniscal ligament
4- Displaced lateral meniscus tear
5- Displaced medial meniscus tear

DISCUSSION: The arthroscopic view and the coronal MRI scan show a discoid lateral meniscus
covering almost the entire lateral tibial plateau. The sagittal views show a contiguous meniscus or
"bow tie" sign on three consecutive images, pathognomonic for a discoid meniscus. Lateral discoid
menisci are much more common than medial. There is no evidence of abnormal signal to indicate
meniscal tearing. A transverse meniscal ligament is best seen anterior to the anterior horn of the
lateral meniscus on multiple views. There is no evidence of a loose body on the arthroscopic or
MRI images. The Preferred Response to Question # 80 is 2.

38
324
Online 2011 Anatomy-Imaging Self-Assessment Examination by Dr.Dhahirortho

81 An L3 radiculopathy is best differentiated from a femoral neuropathy by testing what


muscle?
1- Quadriceps 2- Adductor longus 3- Iliacus 4- Sartorius 5- Psoas

DISCUSSION: The major differential diagnosis in patients with a femoral neuropathy is a lumbar
radiculopathy. Patients with femoral nerve (L2-4) lesions may be distinguished from L2 or L3
radiculopathy by testing adduction (an obturator nerve-innervated function). All muscles listed
above, with the exception of the adductor longus, are innervated by the femoral nerve. The
Preferred Response to Question # 81 is 2.

82 82a 82b 82c A 20-year-old woman reports mild fullness and occasional aching over the left
hip; the symptoms are worse with activities and better with rest. She denies trauma to the area.
She states the fullness has been present as long as she can remember. Examination reveals a
deep fullness anterior to the hip joint, and mild tenderness at the extreme of hip flexion. Figure
82a through 82c show an AP radiograph, a CT scan, and a T2-weighted MRI scan. Based on these
imaging studies, what is the most likely diagnosis?

1- Osteochondroma
2- Myositis ossificans traumatica
3- Infection
4- Osteosarcoma
5- Osteoblastoma

DISCUSSION: The radiograph shows a mineralized lesion that is poorly defined around the
anterosuperior aspect of the acetabulum. It is common for pelvic radiographs to lack detail, but
the radiograph does reveal some kind of bone forming process. The CT scan, which provides far
greater detail of the complex bony anatomy around the hip, reveals a bony lesion that projects
outward from the pelvis and appears to share the cortex with the pelvic bone. On the MRI scan,
there is a small cartilage cap (bright on the T2-weighted image) but no surrounding edema and no
soft-tissue mass suggestive of an aggressive process. Any cartilage cap larger than 1 to 2 cm is
concerning for a secondary chondrosarcoma. These findings are most consistent with an
osteochondroma. Myositis ossificans traumatica would not share the bony cortex with the pelvis
and is generally not so lobular in appearance. The lack of significant edema or bone destruction
rules against infection, osteosarcoma, and osteoblastoma.
The Preferred Response to Question # 82 is 1.

39
325
Online 2011 Anatomy-Imaging Self-Assessment Examination by Dr.Dhahirortho

83 The saphenous nerve is most at risk with which of the following ankle arthroscopy portals?
1- Anteromedial
2- Anterolateral
3- Anterocentral
4- Medial midline
5- Posterolateral

DISCUSSION: The saphenous nerve travels along the distal medial tibia and ankle and is most at
risk from the anteromedial portal which is medial to the tibialis anterior tendon. The anterolateral
portal is lateral to the peroneus tertius tendon and puts the superficial
peroneal nerve at risk. The anterocentral portal is between the tendons of the extensor digitorum
longus and puts the deep peroneal nerve at risk. The medial midline portal is between the
extensor hallucis longus tendon and the tibialis anterior tendon and puts the deep and superficial
peroneal nerves at risk; it should be well lateral to the saphenous nerve. The posterolateral portal
is lateral to the Achilles tendon and puts the sural nerve at risk. The
Preferred Response to Question # 83 is 1.

84 84a 84b 84c 84d A 62-year-old man returns for evaluation of a painless total knee
arthroplasty 6 months after his surgery. He notes recurrent, mild knee effusions. His initial
postoperative radiographs are shown in Figures 84a and 84b. His current radiographs are shown
in Figures 84c and 84d. What is the next step in evaluation of this patient?

1- Clinical and radiographic follow-up in 3 months


2- CT scan
3- Laboratory testing
4- Stress radiographs
5- Technetium-99 bone scan

DISCUSSION: The development of a progressive radiolucency within the first year following knee
arthroplasty surgery is concerning for infection. Infection work-up should include laboratory
testing for erthrocyte sedimentation rate and C-reactive protein levels. A joint
aspiration should be strongly considered, especially if the laboratory studies are elevated. A CT
scan would be appropriate to assess component rotation for patellar instability, but does not
40
326
Online 2011 Anatomy-Imaging Self-Assessment Examination by Dr.Dhahirortho

benefit evaluation of this patient. Stress radiographs could be useful in confirming clinical
instability noted on examination, but early component loosening is the clinical concern for this
patient. A bone scan would be expected to show activity at 6 months after surgery and would not
add useful information to the work-up. The Preferred Response to Question # 84 is 3.

85 A 59-year-old woman seen in the emergency department reports the rapid onset of pain in
the left lower quadrant of her abdomen radiating to the anterior thigh that began about 4 to 5
hours ago. She also notes that now her left knee is buckling, causing her to fall to the ground.
History reveals that the woman is an alcoholic and takes warfarin for atrial fibrillation.
Examination reveals 1/5 quadriceps strength and pain on hip flexion against resistance. What is
a likely pathoanatomic cause for her problem that should be rapidly evaluated to prevent
permanent damage?

1- A left posterolateral L3-4 disk herniation


2- A spontaneous bleed into the iliopsoas sheath
3- A spontaneous epidural hematoma at L3-4
4- A pseudoaneurysm of the femoral artery at the femoral canal
5- A left far lateral L4-5 disk herniation

DISCUSSION: The femoral nerve runs within the fascial sheath of the iliopsoas muscle and a bleed
into the muscle can occur with excessive anticoagulation, creating a compartment-like syndrome.
Initially, it causes pain; however, gradual loss of motor function of the femoral nerve typically
occurs unless the iliopsoas fascia is released and the hematoma is evacuated. A posterolateral L3-4
disk herniation, far lateral L4-5 disk herniation, and epidural hematoma can all cause pain and
weakness, but are not associated with abdominal complaints. Additionally, this patient has "psoas
signs" as demonstrated by pain on hip flexion against
resistance, indicating psoas tendon irritability. A pseudoaneurysm of the femoral artery would be
palpable and a much less likely cause of acute femoral nerve palsy. The Preferred Respo # 85 is 2.

86 A sagittal MRI scan of the hindfoot and ankle is shown in Figure 86. The arrow points to what
structure?
1- Posterior tibial artery
2- Peroneal artery
3- Flexor hallucis longus
4- Posterior tibial nerve
5- Calcaneal artery

DISCUSSION: The MRI scan shows a medial sagittal cut through


the hindfoot and ankle with the medial malleolus and posterior
tibial tendon in the center for orientation. The arrow points to the
posterior tibial artery that lies in this plane. Blood vessels can be
visualized by MRI because flowing blood produces little signal and they appear as low intensity
tubular structures. The peroneal artery is a much more lateral structure. The flexor hallucis longus
is near this location but should be straighter and darker and it does not fork or split in this

41
327
Online 2011 Anatomy-Imaging Self-Assessment Examination by Dr.Dhahirortho

location. The posterior tibial nerve is in this location but does not have this low signal intensity
appearance. The calcaneal artery is a branch off of the posterior tibial artery and is more distal and
medial. The Preferred Response to Question # 86 is 1.

87 Contracture or tightness of the triangular ligament of the finger is associated with which of
the following conditions?
1- Sagittal band insufficiency
2- Volar subluxation of the lateral bands
3- Swan-neck deformity
4- Volar plate contracture
5- Boutonniere deformity

DISCUSSION: Swan-neck deformity may result from contracture/tightness of the triangular


ligament. Anatomically, the triangular ligament is on the dorsal aspect at the base of the middle
phalanx just distal to the central slip. It keeps the lateral bands dorsal. With a boutonniere
deformity, the lateral bands move volar to the central axis resulting in a flexion deformity of the
proximal interphalangeal joint and extension of the distal interphalangeal joint. Sagittal band
insufficiency results in subluxation of the extensor tendon(s) at the metacarpophalangeal joint
level. Volar plate contracture will not cause swan-neck deformity. The Preferred Respon # 87 is 3.

88 An axial MRI scan of an ankle is shown in Figure 88. The arrow indicates what tendinous
structure?
1- Posterior tibial
2- Peroneus longus
3- Peroneus brevis
4- Flexor digitorum longus
5- Flexor hallucis longus

DISCUSSION: The image shown is a transverse cut of the left ankle


that shows the distal tibia metaphysis and the proximal medial
malleolus, the lateral malleolus, and the surrounding soft tissues.
The structure highlighted by the arrow is the flexor hallucis longus tendon that travels along the
posterior tibia, just anterior to the Achilles tendon. The peroneal tendons lie behind the fibula, and
the posterior tibial and flexor digitorum longus tendons travel posterior to the medial malleolus.
The Preferred Response to Question # 88 is 5.

89 89a 89b 89c A 47-year-old man reports a 12-week history of pain and swelling of his right
hindfoot and ankle. Examination reveals a significant limp with swelling and tenderness over the
distal Achilles tendon. He also has weak plantar flexion strength and squeezing of his calf
produces only a small amount of ankle plantar flexion that is much less than his asymptomatic
contralateral ankle. He reports suffering an Achilles tendon rupture some years ago that was
treated in a cast. A radiograph obtained at that time is shown in Figure 89a. He was sent for
physical therapy and did well except for a mild persistent limp. He then returned 1 year later

42
328
Online 2011 Anatomy-Imaging Self-Assessment Examination by Dr.Dhahirortho

with similar complaints and with a history of a fall 3 months earlier. A current radiograph and
MRI scan are shown in Figures 89b and 89c. What is the most likely diagnosis?

1-Heterotopic ossification of the proximal Achilles tendon


2-An untreated acute rupture of the midsubstance of the Achilles tendon
3-Multiple gouty crystalline deposits
4-Chronic Achilles tendon avulsion (sleeve) rupture
5-Rupture of the Achilles tendon at the musculotendinous junction

DISCUSSION: The patient has a symptomatic chronic Achilles tendon avulsion (sleeve) rupture. The
radiographs show movement of the calcified/ossified tendon away from the insertion and the MRI
scan reveals a chronic avulsion of the Achilles tendon from the insertion site. The heterotopic
ossification shown is of the distal Achilles tendon. The rupture shown is at the insertion site and
not midsubstance or at the musculotendinous junction. The studies are not consistent with the
uric acid deposition of gout. The Preferred Response to Question # 89 is 4.

90 During an anterior retroperitoneal approach to the L4-5 disk, the iliac vessels must be
mobilized. The dissection is carried out along the lateral edge of the vessels so they can be
retracted medially across the midline. What structure that tethers the common iliac vein must
be identified and taken down for safe and adequate mobilization?
1- Ureter
2- Genitofemoral nerve
3- Internal femoral artery
4- Iliolumbar vein
5- Central sacral vein

DISCUSSION: The iliolumbar vein is the only branch off the common iliac vein. It is located at about
the L5 level and is easily avulsed if not identified and ligated during this mobilization. The ureter
runs over the iliac vessels but is easily mobilized with the peritoneum and does not tether the iliac
vein. The internal femoral artery has a more distal takeoff and does not interfere with the amount
of mobility needed to get to the anterior surface of L4-5. The central sacral vessels come out of the
iliac bifurcation (more distal) and are ligated to gain access to the L5-S1 disk space as dissection
occurs between the common iliacs. The Preferred Response to Question # 90 is 4.

43
329
Online 2011 Anatomy-Imaging Self-Assessment Examination by Dr.Dhahirortho

91 91a 91b 91c A 65-year-old woman who underwent right knee arthroplasty 12 years ago
reports that she has had knee pain for the past year. Examination reveals that knee range of
motion is from 0 degrees to 100 degrees. A standing AP radiograph obtained 3 years ago is
shown in Figure 91a. Recent radiographs are shown in Figures 91b and 91c. Laboratory study
findings include an erythrocyte sedimentation rate of 9 mm/h and a C-reactive protein level of
0.3 mg/L. What is the most likely cause of her knee pain?

1- Infection
2- Ligamentous instability
3- Polyethylene wear
4- Extensor mechanism dysfunction
5- Technical error during the total knee arthroplasty

DISCUSSION: The radiographs show a change in femoral component position with proximal
migration and a change in alignment from the initial near anatomic to a more varus position.
Polyethylene wear is the most common contributor to both focal osteolysis and component
loosening at long-term follow-up. The knee performed well for 12 years with good initial
alignment, so a technical factor at the index surgery would not explain the development of
loosening. Laboratory findings are not consistent with infection. There is no clinical information in
the history that would suggest that the patient has either instability or poor function in the
extensor mechanism. The Preferred Response to Question # 91 is 3.

92 A 24-year-old man sustained a bilateral C5-6 facet dislocation in a car accident and was
intubated at the scene. He remains sedated in the intensive care unit so the clinical neurologic
examination is limited. What MRI finding would most likely predict a complete spinal cord
injury?
1- 4-mm rostral caudal cord edema
2- Disruption of the anterior longitudinal ligament
3- Edema in the soft tissue anterior to the spine
4- Diffuse cord edema
5- 6-mm cord hematoma

DISCUSSION: The MRI finding that most consistently corresponds with a complete spinal injury is a
hematoma within the cord. Cord edema can predict a poor prognosis if it is more

44
330
Online 2011 Anatomy-Imaging Self-Assessment Examination by Dr.Dhahirortho

extensive but is not considered as consistent a finding. Ligamentous injury about the neck can
indicate musculoskeletal instability but it does not in and of itself indicate the presence or predict
the severity of spinal cord injury. Likewise, soft-tissue edema anterior to the spine may indicate
musculoskeletal injury but does not offer specific information regarding the presence or absence
of cord injury. The Preferred Response to Question # 92 is 5.

93 Figure 93 shows an arthroscopic view of a left shoulder (posterior portal, beach chair
position). The asterisk indicates what anatomic structure?
1- Subscapularis
2- Superior glenohumeral ligament
3- Middle glenohumeral ligament
4- Anterior labrum
5- Biceps tendon

DISCUSSION: The arthroscopic image shows the anterior structures of the glenohumeral joint from
a posterior portal. The asterisk indicates the middle glenohumeral ligament. Whereas there is
significant variability in its appearance, the classic arrangement appears as a folded thickening in
the anterior capsule that crosses the subscapularis tendon at a 45-degree angle to insert on the
anterior superior neck of the glenoid, on the labrum, or just medial to it. The subscapularis is seen
anterior to the middle glenohumeral ligament. The superior glenohumeral ligament and biceps
tendon are not visible in this image. The Preferred Response to Question # 93 is 3.

94 94a 94b 94c A 31-year-old woman reports right shin pain that is constant in nature, not
associated with activity, and periodically awakens her from sleep at night. The patient states
that nonsteroidal anti-inflammatory drugs help alleviate the pain. Radiographs and a CT scan are
shown in Figures 94a through 94c. What is the most likely diagnosis?

1- Osteoid osteoma
2- Fibrous dysplasia
3- Stress fracture
4- Enchondroma
5- Osteoma

DISCUSSION: The radiographs


reveal a nonaggressive-
appearing lytic lesion of the
tibial cortex that lacks the
usual reactive sclerosis
(cortical and periosteal
reaction) that is often seen
with osteoid osteoma, but is not a uniform finding. The CT scan clearly demonstrates an
intracortical lucent nidus with a small amount of mineralization. This appearance, along with the

45
331
Online 2011 Anatomy-Imaging Self-Assessment Examination by Dr.Dhahirortho

history of pain relief with the use of nonsteroidal anti-inflammatory drugs, is strongly suggestive of
osteoid osteoma. Osteomas are always present as a dense bony mass and are usually juxtacortical.
The other choices listed would not have this type of history or imaging appearance. The Preferred
Response to Question # 94 is 1.

95 What is the most common site of posterior interosseous nerve compression in radial tunnel
syndrome?
1- Fibrous bands superficial to the radiocapitellar joint
2- Radial recurrent artery branches (leash of Henry)
3- Fibrous edge of the supinator (arcade of Frohse)
4- Distal edge of the supinator
5- Tendinous margin of the extensor carpi radialis brevis (ECRB)

DISCUSSION: The five compression sites described in radial tunnel syndrome are: the distal edge of
the supinator; fibrous bands superficial to the radiocapitellar joint; tendinous margin of the
extensor carpi radialis brevis (ECRB); radial recurrent artery (leash of Henry); and the most
common site of compression, the fibrous edge of the supinator (arcade of Frohse). The tendinous
portion of the supinator is next to the bone and does not compress the posterior interosseous
nerve. The Preferred Response to Question # 95 is 3.

96 What are the two most important pulleys to preserve/reconstruct during flexor tendon
surgery?
1- A1 and A2
2- A2 and C3
3- A2 and A3
4- A2 and A4
5- A3 and C4

DISCUSSION: If the flexor tendon sheath is nonfunctional or a surgical approach to the flexor
tendons is being performed, the minimum preservation/reconstruction should include the A2 and
A4 pulleys. Ideally, the surgeon should reconstruct a pulley both proximal and distal to each joint
to minimize bowstringing and maximize excursion. The Preferred Response to Question # 96 is 4.

97 When releasing a proximal interphalangeal (PIP) joint flexion contracture, the check rein
ligaments are released first, followed by which of the following structures?

1- Proper collateral ligament from the proximal phalanx


2- Proper collateral ligament from the middle phalanx
3- Extensor tendon
4- Dorsal capsule
5- Accessory collateral ligament and volar plate

46
332
Online 2011 Anatomy-Imaging Self-Assessment Examination by Dr.Dhahirortho

DISCUSSION: When releasing a PIP joint flexion constracture, each step should be followed by an
attempt to extend the PIP joint. If there is no passive extension, then the next stage is performed.
The steps for a volar approach PIP flexion contracture release are as follows: retract the flexor
tendons after appropriate pulley takedown; release check rein ligaments; then accessory collateral
ligament and volar plate; and finally the proper collateral ligament is then released off the
proximal phalanx. Extensor tenolysis only needs to be performed if there is no active extension.
The Preferred Response to Question # 97 is 5.

98 98a 98b 98c 98d A 35-year-old woman reports the insidious onset of shoulder pain for the
past several weeks. Figures 98a through 98d show the radiograph and MRI scans of the
shoulder. What is the most likely diagnosis?

1- Impingement syndrome
2- Adhesive capsulitis
3- Partial-thickness rotator cuff tear
4- Full-thickness rotator cuff tear
5- Calcific tendinitis

DISCUSSION: Calcific tendinitis of the rotator cuff is a common disorder of unknown etiology. It
typically affects women more often than men, and usually involves the supraspinatus and/or the
infraspinatus. The radiograph shows the typical calcific deposition. MRI sequences show the
typical globular area of low signal intensity abnormality located in the supraspinatus tendon. There
is no significant acromial spurring to indicate impingement syndrome, and no evidence of partial-
or full-thickness rotator cuff tearing. Adhesive capsulitis is a clinical diagnosis that has no
consistent radiographic findings. The Preferred Response to Question # 98 is 5.

99 While performing a medial approach to the hip, the superficial dissection takes place
between the gracilis and the adductor longus muscles. The deeper dissection takes place
between what two muscles?
1- Adductor longus and adductor brevis
2- Adductor longus and pectineus
3- Adductor brevis and adductor magnus
4- Adductor magnus and semimembranosus
5- Pectineus and iliopsoas

47
333
Online 2011 Anatomy-Imaging Self-Assessment Examination by Dr.Dhahirortho

DISCUSSION: The deep dissection in the medial approach to the hip takes place in the interval
between the adductor brevis and the adductor magnus. Whereas the tissue planes open more
easily between the adductor longus and the adductor brevis, that interval takes the surgeon onto
the profundus vessels and is more difficult to expose proximally. The longus and pectineus interval
is too anterior and also puts the profundus at risk. The pectineus and iliopsoas interval is too
anterior. The adductor magnus and semimembranosus interval is too posterior. The Preferred
Response to Question # 99 is 3.

100 A 34-year-old woman reports pain, swelling, and loss of knee motion that has been present
for more than 3 years. Examination reveals fullness around the knee, a joint effusion, limited
knee flexion, and tenderness to deep posterior palpation. Figure 100 shows a lateral radiograph
of the knee. What is the most likely diagnosis?
1- Chondrosarcoma
2- Lymphoma of bone
3- Aggressive fibrous dysplasia
4- Parosteal osteosarcoma
5- Metastatic carcinoma

DISCUSSION: The lateral radiograph reveals a mineralized lesion that is lobulated and appears to
be stuck onto the metaphyseal surface of the bone over a broad area. This is the common
appearance of a parosteal osteosarcoma, which presents in the posterior aspect of the distal
femur approximately 75% of the time. Note that this mineral density is rather "cloud-like,"
suggestive of ossification. None of the other lesions listed have the classic, surface appearance of
this tumor, which is more common in women than in men, and generally presents in the third and
fourth decades. Whereas surface chondrosarcoma is a recognized entity, it tends to be more
destructive of the cortex and radiographically demonstrates "rings and arcs" calcifications. The
Preferred Response to Question # 100 is 4.

48
334

You might also like